You are on page 1of 281

Visit: www.civilstap.

com
For Any Query Mail us: hello@civilstap.com or call us at - (+91)-8146207241
0
emphasis. Where shall this word be inserted if the
UPSC EPFO APFC - 2015 emphasis is to be
(i) On the recipient
Q1. What is the indication out of the sentence: ‘I (ii) On the uniqueness of the item given
gave him a piece of my mind’? (iii) On the giver
(a) Appreciation Respectively (only one at a time)?
(b) Learning (a) (i) Between 3-4; (ii) After 6; (iii) Between 2-3
(c) Greeting (b) (i) Between 3-4; (ii) Between 5-6; (iii) Before 1
(d) Scolding (c) (i) Between 4-5; (ii) Between 3-4; (iii) After 1
(d) (i) Between 2-3; (ii) Between 4-5; (iii) Between
Q2. What is the meaning of the expression: 'Blue 1-2
blood'?
(a) Polluted industrial waste water Q8. What is the meaning of the term 'didactic'?
(b) Sap of teak wood (a) Intended to be inspirational
(c) An aristocrat (b) Teaching a moral lesson
(d) A costly object (c) Received as comical
(d) Sharing an informative experience
Q3. ‘He was hoist by his own petard' refers to
(a) He had problems as a result of his own plans to Q9. Consider the statement: The message of
hurt others peace and brotherhood permeated the address
(b) He was high up on the pole by the Chief Guest. Which of the following is
(c) He would usually run away from trouble meant by 'permeate' in this statement?
(d) He was indifferent to his surroundings (a) To advocate
(b) To spread all over
Q4. What is Ballad? (c) To anchor and stabilize
(a) A novel (d) To leave a permanent impression
(b) A historical narration
(c) A popular story or folktale in verse 10. Arrange the following to form a
(d) Musical comedy grammatically correct sentence:
1. Einstein was
Q5. Plagiarism means 2. although a great scientist
(a) There was an epidemic of plague in the area 3. weak in Arithmetic
(b) It is a sort of political philosophy 4. right from his school days
(c) It indicates a happy community spirit like in Select the correct answer using the codes given
playing Holi below:
(d) It is presenting the work of someone else as (a) 4, 1, 3 and 2
one's own (b) 2, 1, 3 and 4
(c) 4, 3, 1 and 2
Q6. Rivalry between the two clans has become (d) 2, 3, 1 and 4
water under the bridge means.
(a) The rivalry continues Q11. Besides resistance to pests, what are the
(b) It has become a thing of the past other prospects for which plants have been
(c) It connects the two clans genetically engineered?
(d) It is not forgotten 1. To enable them to withstand drought
2. To increase the nutritive value of the produce
Q7. Consider the sentence: 3. To enable them to grow and do photosynthesis
The teacher gave me the book. in spaceships and space stations
1 2 3 4 5 6 4. To increase their shelf life
where the words are numbered for convenience Select the correct answer using the codes given
of reference. Consider also the insertion of a single below:
word 'only' into this sentence to indicate a desired (a) 1, 2 and 3 only
(b) 3 and 4 only
1|P a g e W W W . E D U T A P . C O . I N QUERY? HELLO@EDUTAP.CO.IN / 8146207241
(c) 1, 2 and 4 only 1. A Generation represents the number of
(d) 1, 2, 3 and 4 subscribers; higher Generation has more
subscribers.
Q12. Acid rain is due to 2. 2G technology has two standards CDMA and
(a) Sulphur dioxide pollution GSM.
(b) Carbon monoxide pollution 3. 2G technology has CDMA standard and 3G has
(c) Pesticide pollution GSM standard.
(d) Dust particles in the atmosphere Which of the above statements is/are correct?
(a) 1 and 3 only
Q13. DNA fingerprinting is a technique used for (b) 1 only
the detection of (c) 3 only
(a) Alzheimer's disease (d) 2 only
(b) Disputed parentage
(c) AIDS Q18. Consider the following statements for
(d) Yellow fever Nano-technology:
1. It is the technology of creating materials and
Q14. In raising an object to a given height by devices atom-by-atom.
means of an inclined plane, as compared with 2. Physical properties change at the Nanometer
raising the object vertically, there is a reduction scale.
in 3. Chemical properties change at the nanometer
(a) Force to be applied scale.
(b) Work required Which of the above statements are correct?
(c) Distance covered (a) 1 and 2 only
(d) Friction force (b) 1 and 3 only
(c) 2 and 3 only
Q15. A sinusoidal transverse wave is travelling on (d) 1, 2 and 3
a string. Any point on the string moves in
(a) SHM with the same angular frequency as that Q19. Consider an industry with the following
of the wave features:
(b) SHM with a different frequency than that of • Budgeted monthly fixed cost = Rs.2,20,000
the wave • Normal monthly output= 12000 per standard
(c) Uniform circular motion with the same angular labour hour
speed as that of the wave • Standard variable overhead rate = Rs.25 per
(d) Uniform circular motion with a different labour hour
angular speed than that of the wave
What would be the total factory overhead rate?
Q16. Consider the following statements on (a) 40.33 per labour hour
Global Positioning System (GPS): (b) 41.67 per labour hour
1. GPS allows accurate time-stamping on ATM (c) 42.67 per labour hour
transactions. (d) 43.33 per labour hour
2. GPS relies on a set of satellites for transferring
signals worldwide. Q20. Data regarding inventory of a particular
Which of the above statements is/are correct? item of usage in the production activities of an
(a) 1 only organization are the quantity in stock is 1500
(b) 2 only units and the value of this stock is Rs.1,27,500.
(c) Both 1 and 2 (This works out to an average unit cost of 85.)
(d) Neither 1 nor 2 During the ensuing year X. an additional 300 units
are purchased at a unit cost of Rs.95.
Q17. Consider the following statements: Cellular Consumption in production processes during the
technology evolves in stages called Generation year X has been 600 units.
(G), where

2|P a g e W W W . E D U T A P . C O . I N QUERY? HELLO@EDUTAP.CO.IN / 8146207241


Working by the First-In-First-Out basis, the value (a) Duplex
of the residual inventory of the item at the end of (b) Half Duplex
the year X will be (c) Full Duplex
(a) Rs.1,00,000 (d) Simplex
(b) Rs.1,02,500
(c) Rs.1,05,000 Q26. A collection of programs that controls how
(d) Rs.1,07,500 the computer system runs and processes
information is called as:
Q21. Which of the following are the typical (a) Compiler
differences between the private insurance (b) Operating System
programmes and the social insurance (c) Linker
programmes? (d) Assembler
1. Adequacy versus Equity
2. Voluntary versus Mandatory Participation Q27. SMPS is the acronym for
3. Contractual versus Statutory Rights (a) Store Mode Power Supply
4. Funding (b) Single Mode Power Supply
Select the correct answer using the codes given (c) Switched Mode Power Supply
below: (d) Start Mode Power Supply
(a) 1, 2 and 3 only
(b) 1, 2 and 4 only Q28. USB is the acronym for
(c) 3 and 4 only (a) Uniform Service Broadcasting
(d) 1, 2, 3 and 4 (b) Unique Solution Bus
(c) Universal Serial Bus
Q22. The devices that work with computer (d) Universal Service Broadcasting
systems as soon as they are connected are
described as: Q29. The 'Cloud Computing technology refers to
(a) Hot Swapping (a) A set of algorithms that solves problems using
(b) Bay Swap fuzzy logic
(c) Plug-N-Play (b) Many computers that are interconnected
(d) USB Swapping through wireless networks and satellites
(c) A distributed computer architecture that
Q23. Which one of the following software provides software, infrastructure and
applications would be the most appropriate for platforms just as required by
performing numerical and statistical applications/users
calculations? (d) A futuristic technology that will use clouds to
(a) Database perform computing
(b) Spreadsheet
(c) Graphics package Q30. Which of the following is not an absolute
(d) Document processor measure of dispersion?
(a) Range
Q24. LAN, WAN and MAN are computer networks (b) Mean Deviation
covering different areas. Their first alphabets L, (c) Quartile Deviation
W and M respectively stand for (d) Coefficient of Variation
(a) Local, World and Middle
(b) Long, Wireless and Metropolitan Q31. The mean and standard deviation of a set of
(c) Local, Wide and Metropolitan 16 non-zero positive numbers in an observation
(d) Least, Wireless and Maximum are 26 and 3-5 respectively. The mean and
standard deviation of another set of 24 non-zero
Q25. The method of communication in which positive numbers without changing the
transmission can take place in both directions, circumstances of both sets of observations, are
but happens only in one direction at a time, is 29 and 3, respectively. The mean and standard
called
3|P a g e W W W . E D U T A P . C O . I N QUERY? HELLO@EDUTAP.CO.IN / 8146207241
deviation of their combined set of observations (a) 525
will respectively be (b) 725
(a) 27.8 and 3.21 (c) 850
(b) 26.2 and 3.32
(d) 900
(c) 27.8 and 3.32
(d) 26.2 and 3.21
Q36. A certain sequence of integers is
constructed as follows:
Q32. What is the perimeter of the figure shows
Consider 0 and 1 as the first two numbers. The
below?
next, i.e. the third number is constructed by their
sums, i.e. 1. This process of constructing the next
number by the sum of the last two constructed
numbers continues. Taking these numbers 0, 1, 1
as the first, second and third numbers in the
sequence, what will be the 7th and 10th numbers,
AB=x, CD=x+1, EF=x+2, GH = x+3, BC=DE = FG =HI respectively?
=y (a) 6 and 30
(a) 44 cm (b) 7 and 33
(b) 48 cm (c) 8 and 34
(c) 54 cm (d) 10 and 39
(d) 58 cm
Q37. A firm is purchasing two items, both on
Q33. In a medium-size township, the trend of credit on the same day. The credit term offered
annual immigration is an addition of 20% of the for the first item is 2 1/2/10, 1/20, net 30; and 2
population as it was at the beginning; also 15% of the credit term offered for the second item is 3/5,
the population as it was at the beginning is 2/15, net 30. The declared purchase cost of item
estimated to relocate elsewhere every year. If 1 is Rs.60,000 and that of item 2 Rs.1,40,000. If
the current population is 80000, what is the likely both credits can be settled on the 14th day, what
population three years hence? will be the total amount to be paid out?
(a) 90000 (a) Rs.1,97,200
(b) 91200 (b) Rs.1,97,500
(c) 92000 (c) Rs.1,96,600
(d) 92610 (d) Rs.1,98,400
Q34. There are two circles of radii r1 and r2 (r1 < Q38. There are 20 girls and 30 boys in a class, and
r2). The area of the bigger circle is 693/2 cm². The their respective average marks are found to be 55
difference of cm their 2 circumferences is 22 cm. and 58. The average marks of the entire class are
What is the sum of the diameters of the two (a) 565
circles? (b) 566
(a) 17.5 cm (c) 567
(b) 22 cm (d) 568
(c) 28.5 cm
(d) 35 cm Q39. There are four identically sized and shaped
balls in a box, with only its top open. Each ball is
Q35. Numbers a1, a2, a3, a4, a5, …, a24 are in of a different colour, these being Green, Red,
arithmetic progression and a1+a5+a10+a15+a20+a24 White and Blue, only one ball of each colour.
= 225. The value of a1+a2+a3+a15+…a23+a24 is Without looking into the box, one ball is

4|P a g e W W W . E D U T A P . C O . I N QUERY? HELLO@EDUTAP.CO.IN / 8146207241


randomly picked out and its colour is noted; then (a) 10
it is returned to the box. What are the chances (b) 15
that, in two successive draws, one may get the (c) 20
white ball and the red ball in whichever order? (d) 25
(a) 1/16
(b) 1/8 Q44. A man buys apples at a certain price per
(c) 1/4 dozen and sells them at 8 times that price per
(d) 1/2 hundred. What percentage does he gain or lose?
(a) 4% profit
Q40. A cylindrical closed tank contains 36m cubic (b) 6% profit
metres of water, and is filled to half of its (c) 4% loss
capacity. When the cylindrical tank is placed (d) 6% loss
upright on its circular base on level ground, the
height of the water in the tank is 4 metres But Q45. For which time intervals, is the percentage
when this tank is placed on its side on level rise of population the same for the following
ground, what will be the height of the surface of data?
the water above the ground?
(a) 9 metres
(b) 6 metres
(c) 3 metres
(d) 1 metre.

Q41. In a race, the first four winners are to be


awarded points. Each winner's points must be 5
more than that of the next position winner. Total
(a) 1970-80 and 1980-90
sum of the points to be awarded is 50. What will
(b) 1980-90 and 1990-2000
be the points for the third position winner?
(c) 2000-2010 and 1990-2000
(a) 30
(d) 1980-90 and 2000-2010
(b) 20
(c) 10
Q46. Four quantities are such that their
(d) 5
arithmetic mean (A.M.) is the same as the A.M. of
the first three quantities. The fourth quantity is
Q42. A rectangular garden is to be twice as long
(a) Sum of the first three quantities
as its width. If 360 m of fencing including gates
(b) A.M. of the first three quantities
will totally enclose this garden, what is the length
(c) (Sum of the first three quantities)/4
of the garden?
(d) (Sum of the first three quantities)/2
(a) 120 m
(b) 130 m
Q47. If the difference of two numbers is greater
(c) 140 m
than the sum of the numbers, then
(d) 150 m
(a) Both the numbers are negative
(b) Exactly one of the numbers is negative
Q43. In a chess tournament, each of the six
(c) At least one of the numbers is negative
players will play with every other player exactly
(d) None of the above
once. What is the number of matches that will be
played during the tournament?

5|P a g e W W W . E D U T A P . C O . I N QUERY? HELLO@EDUTAP.CO.IN / 8146207241


Q48. A palindrome is a number which reads the (c) 44 m
same from left as well as from right, for example, (d) 45 m
23732. What is the number of palindromes
between 10 and 1010? Q54. In an office, 40% of the employees are men
(a) 101 and the rest women. Half of the employees are
(b) 100 tall and half short. If 10% of the employees. men
(c) 99 and short, and are 40 employees are women and
(d) 90 tall, the number of tall men employees is
(a) 60
Q49. In writing all the integers from 1 to 300, how (b) 50
many times is the digit 1 used? (c) 40
(a) 160 (d) 30
(b) 140
(c) 120 Q55. A provisions shop-owner is found to mix 25
(d) 110 kg of rice worth Rs.32/kg and 20 kg of rice worth
Rs.35/kg and the mixed rice is sold at 15% profit.
Q50. Consider the sequential integers 27 to 93, What is the selling price of mixed rice.
both included in the sequence. The arithmetic (a) 35.40/kg
average of these numbers will be (b) 38.33/kg
(a) 61-5 (c) 36.50/kg
(b) 61 (d) 37.42/kg
(c) 60-5
(d) 60 Q56. The original lay of a rectangular plot ABCD
on open ground is 80 m long along AB, and 60 m
Q51. Walking at 3/4th of his usual speed, a man wide along BC. Concreted pathways are intended
reaches his office 20 minutes late. What is the to be laid on the inside of the plot all around the
time taken by him to reach the office at his usual sides. The pathways along BC and DA are each 4
speed? m wide. The pathways along AB and DC will
(a) 80 minutes mutually be of equal widths such that the un-
(b) 70 minutes concreted internal plot will measure three-fourth
(c) 60 minutes of the original area of the plot ABCD. What will
(d) 50 minutes be the width of each of these pathways along AB
Q52. A and B run a 1 km race. A gives B a start of and DC?
50 m and still beats him by 15 seconds. If A runs (a) 3 m
at 8 km/h, what is the speed of B? (b) 4 m
(a) 4.4 km/h (c) 5 m
(b) 5.4 km/h (d) 6 m
(c) 6.4 km/h
(d) 7.4 km/h Q57. At a dinner party, every two guests used a
bowl of rice between them, every three guests
Q53. In a race of 1 km, A can beat B by 40 m and used a bowl of dal among them and every four
B can beat C by 50 m. With how much distance guests used a bowl of curd among them. There
can A beat C in a 0-5 km race? are altogether 65 bowls. What is the number of
(a) 42 m guests present at the party?
(b) 43 m (a) 90

6|P a g e W W W . E D U T A P . C O . I N QUERY? HELLO@EDUTAP.CO.IN / 8146207241


(b) 80 Q62. The Right to Free and Compulsory Education
(c) 70 Act, 2009 provides for education to every child. in
(d) 60 the age group
(a) 5 – 13 years
(b) 8 – 16 years
Q58. If 5 men or 9 women can finish a piece of (c) 7 – 15 years
work in 19 days, 3 men and 6 women will done (d) 6 – 14 years
same work in
(a) 12 days Q63. If the Prime Minister of India is a member of
(b) 13 days the Rajya Sabha
(c) 14 days (a) He can make statements only in the Rajya
Sabha
(d) 15 days
(b) He has to become a member of the Lok Sabha
within six months
Q59. If the radius of a circle is reduced by 50%, its (c) He will not be able to speak on the budget in
area will be reduced by the Lok Sabha
(a) 30% (d) He will not be able to vote in his Favour
(b) 50%
(c) 60% Q64. Which one of the following formed the basic
premise for developing the Indian Constitution?
(d) 75%
(a) The Government of India Act, 1935
(b) The U.S. Constitution
Q60. In an examination paper where maximum (c) The British Constitution
marks are 500, A got 10% marks less than B, B got (d) The UN Charter
25% marks more than C, and C got 20% marks less
than D. If A got 360 marks, what marks did D get? Q65. The Malimath Committee (2003) looked at
ways to reform the
(a) 65%
(a) Educational System in India
(b) 70% (b) Criminal Justice System in India
(c) 75% (c) Copyright Laws in India
(d) 80% (d) Public-Private Partnership in India

Q61. Under the Constitution of India, which of Q66. The Parliament can make any law for whole
the following statements are correct? or any part of India for implementing
1. The Constitution is supreme. international treaties
2. There is a clear division of powers between the (a) With the consent of all the States
Union and the State Governments. (b) With the consent of majority of the States
3. Amendments to the Constitution have to (c) With the consent of the States: concerned
follow the prescribed procedure. (d) Without the consent of any State
4. The Union Parliament and the State
Legislatures are sovereign. Q67. Statement (I): Speaker of the Lok Sabha
5. The Preamble to the Constitution cannot be appoints the Chairman of the Public Accounts
invoked to determine the ambit of Committee.
Fundamental Rights. Statement (II): Members of Parliament, eminent
Select the correct answer using the codes given persons from industry and trade are the members
below: of the Public Accounts Committee.
(a) 1, 2, 3, 4 and 5 Codes:
(b) 2, 3 and 4 only (a) Both Statement (I) and Statement (II) are
(c) 1, 4 and 5 only individually true, and Statement (II) is the
(d) 1, 2 and 3 only correct explanation of Statement (1)

7|P a g e W W W . E D U T A P . C O . I N QUERY? HELLO@EDUTAP.CO.IN / 8146207241


(b) Both Statement (I) and Statement (II) are 1. The ancient Indian Ganas were oligarchies.
individually true but Statement (II) is NOT the 2. The ancient Indian Ganas were non-
correct explanation of Statement (1) monarchical states.
(c) Statement (I) is true but Statement (II) is false 3. Magadha, Kosala, Vatsa and Avanti were the
(d) Statement (I) is false, but Statement (II) is True most powerful States of that era.
4. The procedures of the Buddhist monastic
Q68. What is the chronological sequence of the order (Sangha) patterned on the Sangha
following events? politics.
1. First Battle of Panipat Select the correct answer using the codes given
2. Vietnam War below:
3. French Revolution (a) 1, 2 and 3 only
4. First Gulf War (b) 1, 2, 3 and 4
5. World War I (c) 1, 2 and 4 only
Select the correct answer using the codes given (d) 3 and 4 only
below:
(a) 1, 5, 3, 2 and 4 Q72. What is the chronological sequence of the
(b) 3, 1, 5, 4 and 2 following developments?
(c) 3, 1, 4, 5 and 2 1. Decline in the export of Indian Cotton
(d) 1, 3, 5, 2 and 4 2. Cotton boom in India
3. Civil War in America
Q69. Which of the following statements are Select the correct answer using the codes given
correct in respect of 'Stridhana"? below:
1. Various types of movable property gifted to a (a) 1, 3 and 2
woman on various occasions during her (b) 2, 3 and 1
lifetime. (c) 1, 2 and 3
2. Various types of movable property gifted to a (d) 2, 1 and 3
woman at the time of her first marriage.
3. On the woman's death, this wealth could be Q73. The Problem of Rupee' was whose D. Sc
inherited by her children and husband. Thesis
4. On the woman's death, this wealth could be (a) Sri Aurobindo
inherited by her natural legally adopted (b) Dr. Rajendra Prasad
children, with none of the sons-in-law, having (c) Pt. Jawaharlal Nehru
any claim on any part thereof. (d) Dr. Bhimrao Ambedkar
Select the correct answer using the codes given
below: Q74. Which of the following industries were first
(a) 2 and 3 only established during the British Rule in India?
(b) 2 and 4 only (a) Cotton Textile Industry and Jute Industry
(c) 1 and 4 only (b) Jute Industry and Iron & Steel Industry
(d) 1 and 3 only (c) Cotton Textile Industry and Chemical Industry
(d) Jute Industry and Chemical Industry
Q70. The Khilafat Committee accepted Mahatma
Gandhi's suggestion for a non-violent non- Q75. Which of the following were the main
cooperation protest against the Government in aspects of the Gandhi-Irwin Pact of 1931?
its meeting at 1. Suspension of the Civil Disobedience
(a) Bombay Movement
(b) Nagpur 2. To take part in the then forthcoming Shimla
(c) Allahabad Conference
(d) Kanpur 3. Pay taxes for the period of the Civil
Disobedience Movement
Q71. Which of the following statements reflect 4. Release of Political Prisoners
the distinctive features about 6th century BCE Select the correct answer using the codes given
Ganas or Sanghas? below:
8|P a g e W W W . E D U T A P . C O . I N QUERY? HELLO@EDUTAP.CO.IN / 8146207241
(a) 1 and 3 only (c) Yamuna, Ghaghara, Gandak, Son, and Kosi
(b) 2 and 3 only (d) Ghaghara, Yamuna, Kosi, Gandak and Son
(c) 1 and 4 only
(d) 2 and 4 only Q82. In the study of pollution, SPM refers to
(a) Sulphur Phosphorus Matter
Q76. A branch of scholarship that flourished in (b) Sulphur Particulate Matter
Europe, particularly from the fifteenth to the (c) Solid Particulate Matter
eighteenth centuries, and was devoted to the (d) Suspended Particulate Matter
appreciation, chronicling and classification of
historic and non-organic relics, monuments and Q83. What is know India Programme (KIP)?
old texts refers to (a) A programme for children aged 12-
(a) Archaeology 15years from rural areas to make them
(b) History aware of different aspects of India
(c) Architecture (b) A programme that facilitates visits of
(d) Antiquarianism rural teachers to different parts of the
country
Q77. Which of the following is not true of the (c) An out-reach programme of the Ministry
Western Ghats? of External Affairs to build closer ties with
(a) UNESCO declared 39 places in the Western India's neighbourhood
Ghats as 'World Heritage Sites' in 2012 (d) A programme for diaspora youth to
(b) It is one of the eight hottest hotspots of promote awareness among them about
biological diversity in the world different aspects of India.
(c) It has two reserves and thirteen national parks
(d) ONGC surveys have recently found huge oil Q84. Which of the following are incorporated in
reserves the Government approved National Telecom
Policy, 2012?
Q78. Ayodhya is located at the bank of which 1. Broadband for all with a minimum
river? download speed of two megabits per
(a) Falgu second
(b) Ganga 2. India's rural tele-density to be improved
(c) Sarayu from 39% to 70% in the next five years
(d) Yamuna 3. Roaming charges shall be scrapped
Select the correct answer using the codes
Q79. The term 'Carbon footprint' means given below:
(a) A region which is rich in coal mines (a) 1 and 2 only
(b) The amount of reduction in the emission of (b) 1 and 3 only
CO₂ by a country (c) 2 and 3 only
(c) The use of Carbon in manufacturing industries (d) 1, 2 and 3
(d) The amount of greenhouse gases produced by
our day-to-day activities. Q.85 Which of the following is one of the
Millennium Development Goals fixed by the
Q80. Green Revolution in India was expeditiously UNO?
feasible in zones of (a) Having extreme poverty and halting the
(a) High rainfall spread of HIV/AIDS
(b) Moderate rainfall (b) Develop a new variety of foodgrains
(c) Assured irrigation which can ensure bumper crops
(d) Higher population density (c) To ensure that population does not
increase in least developed countries.
Q81. What is the correct sequence of tributaries (d) Develop procedures towards adequate
of the river Ganga from West to East? and timely forewarning on hurricanes and
(a) Yamuna, Ghaghara, Son, Gandak and Kosi tsunamis
(b) Ghaghara, Yamuna, Gandak, Kosi and Son
9|P a g e W W W . E D U T A P . C O . I N QUERY? HELLO@EDUTAP.CO.IN / 8146207241
Q.86 Amongst the following, who is the latest (c) 2 and 3 only
recipient of the "Bharat Ratna' award? (d) 1 and 2 only
(a) Sachin Tendulkar
(b) Atal Bihari Vajpayee Q.91 The Large Hadron Collider (LHC) used in the
(c) Amartya Sen CERN project is a particle accelerator located on
(d) A.P.J. Abdul Kalam the border between
(a) Spain and France
Q.87 The Malimath Committee (2003) looked at (b) France and Germany
ways to reform the (c) Spain and Switzerland
(a) Educational System in India (d) France and Switzerland
(b) Criminal Justice System in India
(c) Copyright Laws in India Q92. Special Economic Zones (SEZ) are developed
(d) Public-Private Partnership in India. to
(a) Generate additional economic activity
Q.88 The Rangarajan Committee on throughout the country
disinvestment of shares in Public Sector (b) Beautify suburban areas
Enterprises suggested that (c) Upgrade the facilities in the countryside
1. The percentage of equity to be divested (d) Promote investment from domestic and
56 should be no more than 49% for foreign sources
industries explicitly reserved for the
public sector and it should be either 74% Q93. Unbalanced' Growth is hypothesized when
or 100% for others. (a) Expansion can take place simultaneously on
2. Year-wise targets of disinvestment several growth routes
should be maintained. (b) Supply of labour is fixed
Which of the above statements is/are (c) Supply of capital is unlimited
correct? (d) Active sectors need to, and do energize
(a) 1 only sluggish sectors
(b) 2 only
(c) Both 1 and 2 Q94. Transportation plays a great role in any
(d) Neither 1 nor 2 economy. Which of the following modes of
transportation has the highest elasticity?
Q.89 What was the position held by Dr. S. (a) Air
Radhakrishnan, before he became the Vice- (b) Rail
President of India? (c) Road
(a) Ambassador to the U.S.A. (d) Water
(b) Chairman, U.G.C.
(c) Chairman, Planning Commission Q95. How does an expansionary monetary policy
(d) Ambassador to the Soviet Union affect the rate of interest and level of income?
(a) Raises the level of income but lowers the rate
Q.90 Which of the following trends in FDI inflows of interest
are correct? (b) Raises the rate of interest but lowers the level
1. In 2003 04, the FDI Equity inflow of income
percentage growth was negative. (c) Raises both, the rate of interest and the level
2. From 2004-05 to 2007-08, the FDI inflows of income
were very high and positive. (d) Lowers both, the rate of interest and the level
3. In 2008-09, the FDI inflows were positive, of income
but had decreased relative to the
previous year. Q96. When the Government ownership in Public
Select the correct answer using the codes Sector Undertakings is diluted, it is called
given below: (a) Privatization
(a) 1 and 3 only (b) Public-Private Partnership
(b) 1, 2 and 3 (c) Disinvestment
10 | P a g e W W W . E D U T A P . C O . I N QUERY? HELLO@EDUTAP.CO.IN / 8146207241
(d) Deflation d) Each country maintains its own set of tariffs
and quotas on imports from outside countries
Q97. Consider the following items:
1. Consumer goods and services Q101. The term 'Dear Money' refers to
2. Gross private domestic investment a) Low rate of interest on housing loans
3. Goods and services produced by the b) Value of money at the recession stage
Government c) High rate of interest
4. Net income from abroad d) Savings gained due to decrease in rate of
interest on housing loans
Which of the above items are included in GNP?
(a) 1, 2 and 3 only Q102. Which of the following are the roles of the
(b) 1, 2 and 4 only Finance Commission in India?
(c) 3 and 4 only 1. The distribution of money collected through
(d) 1, 2, 3 and 4 taxes
2. Evaluation of Centrally sponsored schemes
Q98. Which of the following statement best 3. Evolve principles based on which funds are
describes the content of the theory of allotted among States
distribution? 4. To develop Five Year Plans
a) The distribution of income among different Select the correct answer using the codes given
individuals in the economy below:
b) The distribution of income between the a) 1 and 4 only
Centre and the State Governments b) 1 and 3 only
c) The principle of just distribution of wealth and c) 2 and 4 only
income d) 2 and 3 only
d) The distribution of income between the
owners of factor resources Q103. Which of the following statements are
correct regarding the International Monetary
Q99. The demand for a factor of production is Fund (IMF)?
said to be derived demand because 1. India is a founder member of the IMF
1. It is a function of the profitability of an 2. India's quota in the IMF is more than 4 percent
enterprise 3. Finance Minister is represented ex-officio on
2. It depends on the supply of complementary the IMF
factors 4. IMF conducts regular review of India's
3. It stems from the demand for the final economic status under Article IV
product Select the correct answer using the codes given
4. It arises out of means being scarce in relation below:
to wants a) 1, 2 and 3 only
Which of the above reasons is/are correct? b) 1, 3 and 4 only
a) 1 only c) 2 and 4 only
b) 3 only d) 1, 2, 3 and 4
c) 2 and 4 only
d) 1, 2, 3 and 4 Q104. Statement (1): The effects of an income tax
on consumption, saving and investment are micro
Q100. Whenever countries set up a Free Trade effects
Area, they abolish all restrictions on trade among Statement (II): Income tax is an example of direct
themselves and tax.
a) They establish a common external tariff on Codes:
imports from outside countries (a) Both Statement (I) and Statement (II) are
b) They abolish all restrictions on imports from individually true and Statement (II) is the
outside countries correct explanation of Statement (I)
c) They abolish all restrictions on imports from
other Free Trade Areas
11 | P a g e W W W . E D U T A P . C O . I N QUERY? HELLO@EDUTAP.CO.IN / 8146207241
(b) Both Statement (I) and Statement (II) are a) 1 and 2 only
individually true but Statement (II) is not the b) 1 and 3 only
correct explanation of Statement (I) c) 2 and 3 only
(c) Statement (I) is true but Statement (II) is false d) 1, 2 and 3
(d) Statement (I) is false but Statement (II) is true
Q109. Consider the following programmes:
Q105. Statement (1): Trade between two 1. Credit Linked Capital Subsidy Scheme
countries takes place on account of differences in 2. Micro Finance Programme
costs. 3. National Manufacturing Competitiveness
Statement (II): Different countries have different Programme
factor endowments 4. Cluster Development Programme
Codes: What is common in the above programmes?
(a) Both Statement (I) and Statement (II) are a) They are related to improving agriculture
individually true and Statement (II) is the b) They are programmes related to Micro, Small
correct explanation of Statement (I) and Medium Enterprises
(b) Both Statement (I) and Statement (II) are c) They are programmes to improve large scale
individually true but Statement (II) is not the industries
correct explanation of Statement (I) d) They are programmes to improve the
(c) Statement (I) is true but Statement (II) is false traditional cottage industries
(d) Statement (I) is false but Statement (II) is true
Q110. The Maternity Benefit Act, 1961 (M.B. Act)
Q106. Statement (1): Foreign investment may provides for how many weeks' wages during the
affect a country's export performance. maternity period?
Statement (II): Inflow of foreign exchange may a) 11 weeks
cause appreciation of local currency leading to a b) 12 weeks
rise in the price of export commodities. c) 13 weeks
Codes: d) 14 weeks
(a) Both Statement (I) and Statement (II) are
individually true and Statement (II) is the Q111. Which of the following are the instruments
correct explanation of Statement (I) of providing social security in India?
(b) Both Statement (I) and Statement (II) are 1. Income Tax
individually true but Statement (II) is not the 2. Employees' Provident Fund
correct explanation of Statement (I) 3. General Sales Tax
(c) Statement (I) is true but Statement (II) is false 4. LIC
(d) Statement (I) is false but Statement (II) is true 5. National Pension Scheme
6. Postal Provident Fund
Q107. The famous 'Giri' approach in Industrial Select the correct answer using the codes given
Relations in India espouses the cause of below:
(a) Adjudication a) 1, 2, 3 and 4
(b) Compulsory Collective Bargaining b) 2, 3, 4 and 5
(c) Conciliation c) 2, 4, 5 and 6
(d) Arbitration d) 3, 4, 5 and 6

Q108. What are the disadvantages of Provident Q112. Consider the following statements in
Fund Scheme? respect of Atal Pension Yojana:
1. Money is inadequate for risks occurring early 1. Beneficiary must be in the age group of 18 to
in working life. 40 years.
2. Inflation erodes the real value of savings. 2. Beneficiary will receive the pension only after
3. It generates forced saving that can be used to he attains the age of 60 years.
finance national development plans. 3. After the death of a beneficiary, his spouse
Select the correct answer using the codes given continues to receive the pension.
below: 4. No nominee of the beneficiary is permitted.
12 | P a g e W W W . E D U T A P . C O . I N QUERY? HELLO@EDUTAP.CO.IN / 8146207241
Which of the above statements are correct? Q116. Consider the following statements
a) 3 and 4 only regarding the Pradhan Mantri Jeevan Jyoti Bima
b) 1, 3 and 4 only Yojana:
c) 1, 2 and 3 only 1. It is applicable to all adults above the age
d) 1, 2, 3 and 4 group of 18 years.
2. The premium is deducted from the account
Q113. Consider the following statements holder's bank account through 'auto debit
regarding the Pradhan Mantri Suraksha Bima facility'.
Yojana: 3. The life insurance worth is decided by the
1. It is applicable for all bank account holders up account holder and he has to pay the annual
to the age of 60 years. premium accordingly.
2. It is a life insurance cover 4. The life insurance amount is given to the
3. It is an accident insurance cover. family after the death of the subscriber.
4. The insurance covers death and permanent Which of the above statements are correct?
disability due to accident. a) 1 and 3 only
Which of the above statements are correct? b) 1 and 4 only
a) 1 and 2 only c) 2 and 4 only
b) 3 and 4 only d) 2 and 3 only
c) 2 and 3 only
d) 1 and 4 only Q117. SPAM in a system (e-mail) is:
(a) A message distributed indiscriminately
Q114. Social Security may provide cash benefits (b) A search engine
tom persons faced with (c) An activity of the user
1. Sickness and disability (d) A command initiated by the sender
2. Unemployment
3. Crop failure Q118. What is Apartheid?
4. Loss of the marital partner (a) An international organization of peace
Select the correct answer using the codes given (b) A medical term
below: (c) A trade charter
a) 1, 2 and 3 only (d) A policy of racial segregation
b) 1, 2 and 4 only
c) 3 and 4 only Q119. Consider the following statements about
d) 1, 2, 3 and 4 All India Radio:
1. It is governed by the Prasar Bharati Board.
Q115. Employees State Insurance Act, 1948 2. It was so named in 1936.
covers factors like 3. It does not provide DTH services.
1. Factories and establishments with 10 or more 4.FM Rainbow and FM Gold are its subsidiary
employees. channels.
2. Provision of comprehensive medical care to Which of the above statements are correct?
employees and their families. (a) 1, 2 and 3 only
3. Provision of cash benefits during sickness and (b) 3 and 4 only
maternity. (c) 1, 2 and 4 only
4. Monthly payments in case of death or (d) 1, 2, 3 and 4
disablement.
Which of the above statements are correct? Q120. Which of the following are the functions of
a) 1, 2 and 3 only Foreign Investment Promotion Board (FIPB)?
b) 1, 2 and 4 only 1. To ensure expeditious clearance of the
c) 3 and 4 only proposals for foreign investment.
d) 1, 2, 3 and 4 2. Τo review periodically the implementation of
the proposals cleared by the Board

13 | P a g e W W W . E D U T A P . C O . I N QUERY? HELLO@EDUTAP.CO.IN / 8146207241


3. To undertake all other activities for promoting
and facilitating FDI as considered necessary from
time to time
4. To interact with the FIPC being constituted
separately by the Ministry of Industry
Which of the above statements are correct?
(a) 1, 2 and 3 only
(b) 1, 2 and 4 only
(c) 1, 2, 3 and 4
(d) 3 and 4 only

14 | P a g e W W W . E D U T A P . C O . I N QUERY? HELLO@EDUTAP.CO.IN / 8146207241


Visit: www.civilstap.com
For Any Query Mail us: hello@civilstap.com or call us at - (+91)-8146207241
0
ruined, or defeated by one's own action, device, or
SOLUTIONS – PREVIOUS YEAR plot that was intended to harm another; having
QUESTIONS fallen victim to one's own trap or schemes’. Option
(a) is the most suitable answer here.

UPSC EPFO APFC - 2015 Q4. What is Ballad?


(a) A novel
Q1. What is the indication out of the sentence: ‘I (b) A historical narration
gave him a piece of my mind’? (c) A popular story or folktale in verse
(a) Appreciation (d) Musical comedy
(b) Learning Answer: C
(c) Greeting Explanation: A ‘ballad’ is a poem or song narrating
(d) Scolding a story in short stanzas. Traditional ballads are
Answer: D typically of unknown authorship, having been
Explanation: The expression, ‘To give someone a passed on orally from one generation to the next.
piece of your mind’ means to speak to someone in Therefore, option (c) is the correct answer. A
an angry way. Therefore, the answer here is ‘novel’ is a fictitious prose narrative of book
‘scolding’ which means the act of speaking angrily length, typically representing character and action
to someone because you disapprove of their with some degree of realism. A ‘Historical
behaviour. Hence, option (d) is the correct answer. Narrative’ or Narrative history is the practice of
‘Appreciation’ means recognition and enjoyment writing history in a story-based form. A ‘musical
of the good qualities of someone or something. comedy’ is a type of play or film that has singing
‘Learning’ means the acquisition of knowledge or and dancing as part of the story and that is
skills through study, experience, or being taught. humorous and entertaining,
‘Greeting’ means a polite word or sign of welcome
or recognition. Q5. Plagiarism means
(a) There was an epidemic of plague in the area
Q2. What is the meaning of the expression: 'Blue (b) It is a sort of political philosophy
blood'? (c) It indicates a happy community spirit like in
(a) Polluted industrial waste water playing Holi
(b) Sap of teak wood (d) It is presenting the work of someone else as
(c) An aristocrat one's own
(d) A costly object Answer: D
Answer: C Explanation: The term ‘plagiarism’ means the
Explanation: ‘Blue blood’ is an idiomatic practice of taking someone else's work or ideas
expression which means ‘A member of a wealthy, and passing them off as one's own, which is given
upper-class family or ancestry’ and ‘aristocrat’ in option (d). Therefore, option (d) is the correct
means ‘ someone whose family has a high social answer.
rank, especially someone who has a title’.
Therefore, ‘an aristocrat’ is the correct meaning of Q6. Rivalry between the two clans has become
the expression ‘blue blood’. Hence, option (c) is water under the bridge means.
the correct answer. (a) The rivalry continues
(b) It has become a thing of the past
Q3. ‘He was hoist by his own petard' refers to (c) It connects the two clans
(a) He had problems as a result of his own plans to (d) It is not forgotten
hurt others Answer: B
(b) He was high up on the pole Explanation: The idiom ‘water under the bridge’
(c) He would usually run away from trouble means problems that someone has had in the past
(d) He was indifferent to his surroundings that they do not worry about because they
Answer: A happened a long time ago and cannot now be
Explanation: ‘He was hoist by his own petard’ is an changed. Option (b) is the correct answer here as
idiomatic expression which means ‘Injured, it gives the right meaning of the expression.
1|P a g e W W W . E D U T A P . C O . I N QUERY? HELLO@EDUTAP.CO.IN / 8146207241
Q7. Consider the sentence: spread all over. Hence, option (b) is the correct
The teacher gave me the book. answer.
1 2 3 4 5 6
where the words are numbered for convenience Q10. Arrange the following to form a
of reference. Consider also the insertion of a single grammatically correct sentence:
word 'only' into this sentence to indicate a desired 1. Einstein was
emphasis. Where shall this word be inserted if the 2. although a great scientist
emphasis is to be 3. weak in Arithmetic
(i) On the recipient 4. right from his school days
(ii) On the uniqueness of the item given Select the correct answer using the codes given
(iii) On the giver below:
Respectively (only one at a time)? (a) 4, 1, 3 and 2
(a) (i) Between 3-4; (ii) After 6; (iii) Between 2-3 (b) 2, 1, 3 and 4
(b) (i) Between 3-4; (ii) Between 5-6; (iii) Before 1 (c) 4, 3, 1 and 2
(c) (i) Between 4-5; (ii) Between 3-4; (iii) After 1 (d) 2, 3, 1 and 4
(d) (i) Between 2-3; (ii) Between 4-5; (iii) Between Answer: B
1-2 Explanation: A sentence should be arranged in a
Answer: B Subject+ verb+ object form. The subject can be
Explanation: (i)the recipient is ‘me’, therefore it noun, pronoun, gerund, or clause. Here, the
will be “only me” i.e. between 3-4. sentence will start with either 1 or 2 (noun and
(ii) the uniqueness of the item given is ‘book’, clause). Therefore, option (a) and (c) will be
therefore it will be “the only book” i.e. between 5- eliminated. Option (b) will make a grammatically
6. correct sentence after rearrangement i.e. 2134.
(iii) the giver is the ‘teacher’, therefore it will be Hence, option (b) is the correct answer.
“only the teacher” i.e. before 1.
Hence, option (b) is the correct answer here. Q11. Besides resistance to pests, what are the
other prospects for which plants have been
Q8. What is the meaning of the term 'didactic'? genetically engineered?
(a) Intended to be inspirational 1. To enable them to withstand drought
(b) Teaching a moral lesson 2. To increase the nutritive value of the produce
(c) Received as comical 3. To enable them to grow and do photosynthesis
(d) Sharing an informative experience in spaceships and space stations
Answer: B 4. To increase their shelf life
Explanation: ‘Didactic’ means ‘intended to teach, Select the correct answer using the codes given
particularly in having moral instruction as an below:
ulterior motive’. Therefore, option (b) is the (a) 1, 2 and 3 only
correct answer. (b) 3 and 4 only
(c) 1, 2 and 4 only
Q9. Consider the statement: The message of (d) 1, 2, 3 and 4
peace and brotherhood permeated the address Answer: D
by the Chief Guest. Which of the following is Explanation:
meant by 'permeate' in this statement? Plant genetic engineering, also known as plant
(a) To advocate genetic modification or manipulation, is the key
(b) To spread all over that opens up the doors for introducing crops with
(c) To anchor and stabilize valuable traits to produce plants that require
(d) To leave a permanent impression fewer pesticides, fungicides, or fertilizers, and can
Answer: B be more resistant to stress conditions. The
Explanation: The term ‘permeate’ means ‘to applications of Plant Genetic Engineering are:
penetrate, pass through, and often become • Some benefits of genetic engineering in
widespread throughout something’. Option (b) agriculture are increased crop yields, reduced
gives the right meaning of ‘permeate’ i.e. to costs for food or drug production, reduced
need for pesticides, enhanced nutrient
2|P a g e W W W . E D U T A P . C O . I N QUERY? HELLO@EDUTAP.CO.IN / 8146207241
composition and food quality, resistance to Q14. In raising an object to a given height by
pests and disease, greater food security, and means of an inclined plane, as compared with
medical benefits to the world's growing raising the object vertically, there is a reduction
population. in
• Drought-resistance and increasing shelf life (a) Force to be applied
are some of the other applications of genetic (b) Work required
modification in plants. (c) Distance covered
• Genetic modification to enable growth of (d) Friction force
plants in space is being done on the Answer: A
International Space Station. Hence option D is Explanation:
correct. Inclined plane is a simple machine consisting of a
sloping surface, used for raising heavy bodies.
Q12. Acid rain is due to • The force required to move an object up the
(a) Sulphur dioxide pollution incline is less than the weight being raised.
(b) Carbon monoxide pollution • It reduces the force to be applied in
(c) Pesticide pollution comparison to lifting the load vertically. Hence
(d) Dust particles in the atmosphere option A is correct.
Answer: A • It cannot be said that the work done is less as
Explanation: this depends upon the distance travelled.
Acid rain results when sulphur dioxide (SO2) and
nitrogen oxides (NOX) are emitted into the Q15. A sinusoidal transverse wave is travelling on
atmosphere and transported by wind and air a string. Any point on the string moves in
currents. Hence option A is correct. (a) SHM with the same angular frequency as that
• The SO2 and NOX react with water, oxygen and of the wave
other chemicals to form sulfuric and nitric (b) SHM with a different frequency than that of
acids. These then mix with water and other the wave
materials before falling to the ground. (c) Uniform circular motion with the same angular
speed as that of the wave
Q13. DNA fingerprinting is a technique used for (d) Uniform circular motion with a different
the detection of angular speed than that of the wave
(a) Alzheimer's disease Answer: A
(b) Disputed parentage Explanation:
(c) AIDS A sinusoidal transverse wave travelling on a string
(d) Yellow fever has any point on the string moving with the same
Answer: B angular frequency as that of the wave.
Explanation: • This is the basic condition empirically observed
DNA fingerprinting is a laboratory technique used which allows equations of phase, phase
to determine the probable identity of a person difference etc. to be derived. Hence statement
based on the nucleotide sequences of certain A is correct and statement B is incorrect.
regions of human DNA that are unique to • The particle in such a wave moves in SHM and
individuals. Some of the applications of DNA not in uniform circular motion. Hence
Fingerprinting are: statements C and D are incorrect.
• DNA analysis in forensic tests.
• Can be used to establish paternity tests. Q16. Consider the following statements on
Hence option B is correct. Global Positioning System (GPS):
• In criminal investigations. 1. GPS allows accurate time-stamping on ATM
• To determine the frequency of specific genes transactions.
in a population which gives rise to diversity. 2. GPS relies on a set of satellites for transferring
• Can be used to trace the role of genetic drift in signals worldwide.
evolution. Which of the above statements is/are correct?
(a) 1 only
(b) 2 only
3|P a g e W W W . E D U T A P . C O . I N QUERY? HELLO@EDUTAP.CO.IN / 8146207241
(c) Both 1 and 2 2. Physical properties change at the nanometer
(d) Neither 1 nor 2 scale.
Answer: C 3. Chemical properties change at the nanometer
Explanation: scale.
The Global Positioning System, originally Navstar Which of the above statements are correct?
GPS, is a satellite-based radionavigation system (a) 1 and 2 only
owned by the United States government and (b) 1 and 3 only
operated by the United States Space Force. (c) 2 and 3 only
• Major financial institutions use GPS to obtain (d) 1, 2 and 3
precise time for setting internal clocks used to Answer: D
create financial transaction timestamps. Explanation:
Hence statement 1 is correct. Nanotechnology refers to the branch of science
• In the United States’ Navstar GPS, 24 main and engineering devoted to designing, producing,
satellites in 6 orbits circle Earth every 12 and using structures, devices, and systems by
hours. Hence statement 2 is correct. manipulating atoms and molecules at nanoscale,
i.e. having one or more dimensions of the order of
Q17. Consider the following statements: Cellular 100 nanometres (100 millionth of a millimetre) or
technology evolves in stages called Generation less. Hence statement 1 is correct.
(G), where • Thus, Nanotechnology is not simply working at
1. A Generation represents the number of ever smaller dimensions; rather, working at
subscribers; higher Generation has more the nanoscale enables scientists to utilize the
subscribers. unique physical, chemical, mechanical, and
2. 2G technology has two standards CDMA and optical properties of materials that naturally
GSM. occur at that scale. Hence statements 2 and 3
3. 2G technology has CDMA standard and 3G has are correct.
GSM standard.
Which of the above statements is/are correct? Q19. Consider an industry with the following
(a) 1 and 3 only features:
(b) 1 only • Budgeted monthly fixed cost = Rs.2,20,000
(c) 3 only • Normal monthly output= 12000 per standard
(d) 2 only labour hour
Answer: D • Standard variable overhead rate = Rs.25 per
Explanation: labour hour
The generation of cellular technology provides for:
• Increased speed and change in technology What would be the total factory overhead rate?
used to achieve that speed. Hence statement (a) 40.33 per labour hour
1 is incorrect. (b) 41.67 per labour hour
• 1G refers to the first generation of wireless (c) 42.67 per labour hour
telephone technology. These are the analog (d) 43.33 per labour hour
telecommunication standards that were Answer: D
introduced in 1979. Explanation
• The main difference between the two mobile • The factory overhead is the total of all costs
telephone systems (1G and 2G), is that the incurred to maintain and run the production
radio signals used by 1G network are analog, facility or factory.
while 2G networks are digital. Its examples • To compute the overhead rate, divide your
include CDMA and GSM. Hence statement 2 is monthly overhead costs by your total
correct. Hence option D is correct. monthly sales.
• Monthly sales are directly given in the
Q18. Consider the following statements for question, and it stands for 12000 units
Nano-technology: • Monthly Overhead cost = Fixed Cost +
1. It is the technology of creating materials and Variable cost. Whereas fixed cost is directly
devices atom-by-atom. given in the question, which is Rs. 2,20,000
4|P a g e W W W . E D U T A P . C O . I N QUERY? HELLO@EDUTAP.CO.IN / 8146207241
• Variable cost = Standard variable overhead Already 1500 85 1,27,500
rate * monthly output. Which will be, Present
12000*25 = 3,00,000. inventory
• Thereby total cost equals to 2,20,000 + Year X 300 95 28,500
3,00,000 = 5,20,000 Total 1,56,000
• Total factory overhead rate = Total Overhead Value of
cost/total monthly sales. inventory
• Total factory overhead rate = 5,20,000/12000
= 43.33. • According to the question, company is
• Hence the correct answer will be option D. consuming 600 units and as per FIFO, the
assets purchased or acquired first are disposed
Q20. Data regarding inventory of a particular first. Which means we will utilise the already
item of usage in the production activities of an present inventory first.
organization are the quantity in stock is 1500 • The following table depicts the inventory
units and the value of this stock is Rs.1,27,500. usage by the year end (after consumption)
(This works out to an average unit cost of 85.)
During the ensuing year X. an additional 300 units Timeline Number Unit Total
are purchased at a unit cost of Rs.95. of Units Price of Inventory
Consumption in production processes during the (A) the value
year X has been 600 units. product (A*B)
(B)
Working by the First-In-First-Out basis, the value Already 900 85 76,500
of the residual inventory of the item at the end of Present (1500-
the year X will be inventory 600)
(a) Rs.1,00,000 Year X 300 95 28,500
(b) Rs.1,02,500 Total 1,05,000
(c) Rs.1,05,000 Value of
(d) Rs.1,07,500 inventory
Answer: C
Explanation • Thereby at the end of the year, total inventory
• First In, First Out (FIFO) is an accounting value will be 1,05,000.
method in which assets purchased or acquired
first are disposed of first. Q21. Which of the following are the typical
• According to the question, the firm already differences between the private insurance
has the inventory of 1500 units, which is programmes and the social insurance
valued at 1,27,500 and thereby the unit price programmes?
of these 1500 units comes out to be 85. 1. Adequacy versus Equity
• According to the question, in the year X, 2. Voluntary versus Mandatory Participation
company purchased 300 extra units at a price 3. Contractual versus Statutory Rights
of 95, thereby the total inventory value of 4. Funding
these 300 units will be 28,500 Select the correct answer using the codes given
• Following table depicts the inventory usage below:
data (a) 1, 2 and 3 only
(b) 1, 2 and 4 only
Timeline Number Unit Total (c) 3 and 4 only
of Units Price of Inventory (d) 1, 2, 3 and 4
(A) the value Answer: D
product (A*B) Explanation:
(B) • Typical differences between private
insurance programs and social insurance
programs include:
5|P a g e W W W . E D U T A P . C O . I N QUERY? HELLO@EDUTAP.CO.IN / 8146207241
• Equity versus Adequacy: Private insurance hardware changes with minimal intervention
programs are generally designed with greater by the user.
emphasis on equity between individual • A user can add and remove devices without
purchasers of coverage, while social having to do manual configuration, and
insurance programs generally place a greater without knowledge of computer hardware. For
emphasis on the social adequacy of benefits example, a user can dock a portable computer
for all participants. Therefore statement 1 is and use the docking station keyboard, mouse,
correct. and monitor without making manual
• Voluntary versus Mandatory Participation: configuration changes
Participation in private insurance programs is
often voluntary, individuals usually have a Q23. Which one of the following software
choice of insurers. Participation in social applications would be the most appropriate for
insurance programs is generally mandatory. performing numerical and statistical
Therefore statement 2 is correct. calculations?
• Contractual versus Statutory Rights: The right (a) Database
to benefits in a private insurance program is (b) Spreadsheet
contractual, based on an insurance contract. (c) Graphics package
However, Social insurance programs are not (d) Document processor
generally based on a contract, but rather on a Answer: B
statute, and the right to benefits is thus Explanation:
statutory rather than contractual. The • A spreadsheet software tool is a platform
provisions of the program can be changed if designed to analyse, organize, and store
the statute is modified. Therefore statement 3 information in a table with rows and columns.
is correct. • Today’s spreadsheets are able to perform
• Public health insurance plans are run by a basic math functions and have built-in
country’s government. Whereas Private health capabilities to carry out financial accounting
insurance plans are run by individual work and statistical tasks.
insurance companies. The cost of your • Three of the most common uses of
monthly premium depends heavily on the spreadsheets include: producing financial
types of services your plan will pay for. budgets, sorting and storing data, and
Depending on the type of plan you choose, you creating charts and graphs
may pay a great deal more for private health
insurance than public. Therefore statement 4
is correct. Q24. LAN, WAN and MAN are computer networks
Hence the correct answer will be option D. covering different areas. Their first alphabets L,
W and M respectively stand for
Q22. The devices that work with computer (a) Local, World and Middle
systems as soon as they are connected are (b) Long, Wireless and Metropolitan
described as: (c) Local, Wide and Metropolitan
(a) Hot Swapping (d) Least, Wireless and Maximum
(b) Bay Swap Answer: C
(c) Plug-N-Play Explanation:
(d) USB Swapping A computer network is mainly of four types:
Answer: C • Personal Area Network (PAN) - Personal Area
Explanation: Network is a network arranged within an
• Plug and Play (PnP) is a technology that allows individual person, typically within a range of 10
the operating system to detect and configure meters.
internal and external peripherals as well as • Local Area Network (LAN) - Local Area
most adapters. Network is a group of computers connected to
• Plug and Play (PnP) is the part of Windows that each other in a small area such as building,
enables a computer system to adapt to office

6|P a g e W W W . E D U T A P . C O . I N QUERY? HELLO@EDUTAP.CO.IN / 8146207241


• Metropolitan Area Network (MAN) - A computer hardware resources and provides
Metropolitan Area Network is a network that common services for computer programs.
covers a larger geographic area by • An operating system is a type of software
interconnecting a different LAN to form a without which you cannot operate or run a
larger network computer. It acts as an intermediary or
• Wide Area Network (WAN) - A Wide Area translation system between computer
Network is a network that extends over a large hardware and application programs installed
geographical area such as states or countries. on the computer.
It is quite bigger network than the LAN • In other words, you cannot directly use
computer programs with computer hardware
Q25. The method of communication in which without having a medium to establish a
transmission can take place in both directions, connection between them.
but happens only in one direction at a time, is
called Q27. SMPS is the acronym for
(a) Duplex (a) Store Mode Power Supply
(b) Half Duplex (b) Single Mode Power Supply
(c) Full Duplex (c) Switched Mode Power Supply
(d) Simplex (d) Start Mode Power Supply
Answer: B Answer: C
Explanation: Explanation:
The Transmission mode is divided into three • SMPS is short for switched-mode power
categories: supply.
• Simplex mode - In Simplex mode, the • SMPS is a power supply that uses a switching
communication is unidirectional, i.e., the data regulator to control and stabilize the output
flow in one direction. A device can only send voltage by switching the load current on and
the data but cannot receive it or it can receive off.
the data but cannot send the data • These power supplies offer a greater power
• Half-duplex mode - In a Half-duplex channel, conversion and reduce the overall power loss.
direction can be reversed, i.e., the station can
transmit and receive the data as well. Q28. USB is the acronym for
Messages flow in both the directions, but not (a) Uniform Service Broadcasting
at the same time. The entire bandwidth of the (b) Unique Solution Bus
communication channel is utilized in one (c) Universal Serial Bus
direction at a time. (d) Universal Service Broadcasting
• Full-duplex mode - In Full duplex mode, the Answer: C
communication is bi-directional, i.e., the data Explanation:
flow in both the directions. Both the stations • USB is short for Universal Serial Bus.
can send and receive the message • The Universal Serial Bus standard has been
simultaneously extremely successful.
• USB ports and cables are used to connect
Q26. A collection of programs that controls how hardware such as printers, scanners,
the computer system runs and processes keyboards, mice, flash drives, external hard
information is called as: drives, joysticks, cameras, monitors, and
(a) Compiler more to computers of all kinds, including
(b) Operating System desktops, tablets, laptops, netbooks, etc
(c) Linker
(d) Assembler Q29. The 'Cloud Computing technology’ refers to
Answer: B (a) A set of algorithms that solves problems using
Explanation fuzzy logic
• An operating system (OS) is basically a (b) Many computers that are interconnected
collection of software that manages through wireless networks and satellites

7|P a g e W W W . E D U T A P . C O . I N QUERY? HELLO@EDUTAP.CO.IN / 8146207241


(c) A distributed computer architecture that (c) 27.8 and 3.52
provides software, infrastructure and (d) 26.2 and 3.21
platforms just as required by Answer: C
applications/users Explanation
(d) A futuristic technology that will use clouds to As per the question
perform computing Dataset 1:
Answer: C • Total observations = 16
Explanation: • Mean = 26
• Cloud computing is the on-demand • Standard Deviation = 3.5
availability of computer system resources, Dataset 2:
especially data storage (cloud storage) and • Total observations = 24
computing power, without direct active • Mean = 29
management by the user. • Standard Deviation = 3
• Large clouds often have functions distributed
over multiple locations, each location being a
data centre.
A Cloud is a type of parallel and distributed system
consisting of a collection of interconnected and
virtualized computers that are dynamically
provisioned and presented as one or more
unified computing resources based on service- • X1 = mean of dataset 1 = 26
level agreements established through negotiation • X2 = mean of dataset 2 = 29
between the service provider and consumers. • N1 = total number of observations in dataset
1 = 16
Q30. Which of the following is not an absolute • N2= total number of observations in dataset
measure of dispersion? 2 = 24
(a) Range • So, combined mean (X12) = (26*16 + 29*24)/
(b) Mean Deviation (16+24)
(c) Quartile Deviation • = (416 + 696) /40 = 27.8
(d) Coefficient of Variation
Answer: D Calculating Combined Standard Deviation by the
Explanation: below mentioned formula
• We can differentiate between relative
dispersion and absolute dispersion by
checking whether the statistic involves units.
• The coefficient of variation (relative standard
deviation) is a statistical measure of the
dispersion of data points around the mean.
• The metric is commonly used to compare the
data dispersion between distinct series of data

Q31. The mean and standard deviation of a set of


16 non-zero positive numbers in an observation
are 26 and 3-5 respectively. The mean and • D1 = 27.8 – 26 = 1.8
standard deviation of another set of 24 non-zero • D2 = 27.8 – 29 = -1.2
positive numbers without changing the
circumstances of both sets of observations, are
29 and 3, respectively. The mean and standard
deviation of their combined set of observations
will respectively be
(a) 27.8 and 3.21
(b) 26.2 and 3.32
8|P a g e W W W . E D U T A P . C O . I N QUERY? HELLO@EDUTAP.CO.IN / 8146207241
• Now, Combined SD = = 44cm

Q33. In a medium-sized township, the trend of


annual immigration is an addition of 20% of the
population as it was at the beginning; also 15% of
the population as it was at the beginning is
estimated to relocate elsewhere every year. If
the current population is 80000, what is the likely
population three years hence?
(a) 90000
(b) 91200
(c) 92000
(d) 92610
Answer: C
Explanation:
Let initially population = P = 8000
Annual immigration = in addition of 20% of initial
population = 0.2P
15% relocate to elsewhere = 15% of P = 0.15P
So, three years hence population will be
Therefore, the correct answer will be option C = (P + 3* (0.2P – 0.15P))
= 1.15P
Q32. What is the perimeter of the figure shown
below? = 1.15*80000
= 92000

Q34. There are two circles of radii r1 and r2 (r1 <


r2). The area of the bigger circle is 693/2 cm². The
difference of their 2 circumferences is 22 cm.
What is the sum of the diameters of the two
AB=x, CD = x+1, EF = x+2, GH = x+3, BC = DE = FG circles?
=HI = y (a) 17.5 cm
(a) 44 cm (b) 22 cm
(b) 48 cm (c) 28.5 cm
(c) 54 cm (d) 35 cm
(d) 58 cm Answer: D
Answer: A Explanation:
Explanation: Area of bigger circle = 693/2 cm2 = π(r2)2
From figure it is clear, (693/2)*(7/22)= (r2)2
AB + CD + EF + GH = JI 21/2 = r2
Similarly, BC + DE + FG + HI = AJ Diameter (D2) = 21cm
So, Perimeter = AB + BC + CD + DE + EF + FG + GH
+ HI + IJ + JA The difference in circumference = 2 π r2 – 2 π r1 =
= (AB + CD + EF + GH) + ( BC + DE + FG + HI) + IJ + 22cm
JA = r2 – r1 = 7/2
= 12 + 10 + 12 + 10 By putting the values,

9|P a g e W W W . E D U T A P . C O . I N QUERY? HELLO@EDUTAP.CO.IN / 8146207241


So, r1 = 7cm This type of number series is known as Fibonacci
So, Diameter of a smaller circle(D1) = 14cm Series
0, 1, 1, 2, 3, 5, 8, 13, 21, 34, ….
Now, So, the 7th term is 8
Sum of diameters = 21 + 14 = 35cm 10th term is 34

Q35. Numbers a1, a2, a3, a4, a5, …, a24 are in Q37. A firm is purchasing two items, both on
arithmetic progression and a1+a5+a10+a15+a20+a24 credit on the same day. The credit term offered
= 225. The value of a1+a2+a3+a15+…a23+a24 is for the first item is (2 ½)/10, 1/20, net 30; and the
(a) 525 credit term offered for the second item is 3/5,
(b) 725 2/15, net 30. The declared purchase cost of item
(c) 850 1 is Rs.60,000 and that of item 2 Rs.1,40,000. If
(d) 900 both credits can be settled on the 14th day, what
Answer: D will be the total amount to be paid out?
Explanation: (a) Rs.1,97,200
Let say a1 = a, a2 = a+d, an = a + (n-1)*d (b) Rs.1,97,500
Now, (c) Rs.1,96,600
a1 + a5 + a10 + a15 + a20 + a24 = 225 (d) Rs.1,98,400
a + a+4d + a+9d + a+14d + a+19d + a+23d =225 Answer: C
6a + 69d = 225 Explanation:
3*(2a + 23d) =225 Purchase cost of item 1 = 60000
2A + 23d = 225/3 Now, If paid in 10 days so, discount = (5/2)%
Now, If paid within 20 days then 1% discount.
Sum of 24 terms = Sn = (n/2) *(2a + (n-1)*d) Max limit 30 days
S24 = (24/2) *(2a + (24-1)*d) SO, he paid on 14th day
S24 = 12*(2a + 23d) So, discount applicable = 1%
S24 = 12*225/3 So, remaining price of item 1= 60000 – 1%of 60000
S24 = 900 = 59400

Q36. A certain sequence of integers is Purchase cost of item 2 = 140000


constructed as follows: Now, If paid in 5 days so, discount = 3%
Consider 0 and 1 as the first two numbers. The If paid within 15 days then 2% discount.
next, i.e. the third number is constructed by their Max limit 30 days
sums, i.e. 1. This process of constructing the next SO, he paid on the 14th day
number by the sum of the last two constructed So, discount applicable = 2%
numbers continues. Taking these numbers 0, 1, 1 So, remaining price of item 2 = 140000 – 2% of
as the first, second and third numbers in the 140000 = 137200
sequence, what will be the 7th and 10th
numbers, respectively? So, total amount to be paid = 59400 + 137200 =
(a) 6 and 30 196600
(b) 7 and 33
(c) 8 and 34 Q38. There are 20 girls and 30 boys in a class, and
(d) 10 and 39 their respective average marks are found to be 55
Answer: C and 58. The average marks of the entire class are
Explanation: (a) 565

10 | P a g e W W W . E D U T A P . C O . I N QUERY? HELLO@EDUTAP.CO.IN / 8146207241


(b) 566 ground, what will be the height of the surface of
(c) 567 the water above the ground?
(d) 568 (a) 9 metres
Answer: D (b) 6 metres
Explanation: (c) 3 metres
Average marks of girls = 55, total girls = 20. So, (d) 1 metre.
total marks of girls = 55*20 = 1100 Answer: C
Average marks of boys = 58, total boys = 30. So, Explanation:
total marks of boys = 58*30 = 1740 Since the tank is half full when placed upright then
naturally it'll also be half full when placed on its
So, the average marks of the entire class = (1100+ side, so the level of the water (when placed that
1740)/ 20+30 way) will be half of the diameter, so, r.
= 2840/50
= 568 Now, given that Vwater = π∗r2∗Hwater
Vwater = π∗r2∗Hwater
Q39. There are four identically sized and shaped 36π = πr2∗4
balls in a box, with only its top open. Each ball is r=3
of a different colour, these being Green, Red,
White and Blue, with only one ball of each colour. Q41. In a race, the first four winners are to be
Without looking into the box, one ball is awarded points. Each winner's points must be 5
randomly picked out and its colour is noted; then more than that of the next position winner. Total
it is returned to the box. What are the chances sum of the points to be awarded is 50. What will
that, in two successive draws, one may get the be the points for the third position winner?
white ball and the red ball in whichever order? (a) 30
(a) 1/16 (b) 20
(b) 1/8 (c) 10
(c) 1/4 (d) 5
(d) 1/2 Answer: C
Answer: B Explanation:
Explanation: First position winner award = x points
Now 2 balls are drawn one by one with Second position winner awarded = x-5 points
replacement- chance of 1 being white and another Third position winner awarded = x-10 points
being red Fourth position winner awarded = x-15 points
= Either white first and red at 2nd draw or Red first Total points award = 50
and white at 2nd draw So, x + x-5 + x-10 + x-15 = 50
= (1/4) *(1/4) + (1/4) *(1/4) = 4x = 80
= 1/8 X = 20
Now, third position winner awarded = x-10 points
Q40. A cylindrical closed tank contains 36m cubic = 20-10 = 10 points
metres of water, and is filled to half of its
capacity. When the cylindrical tank is placed Q42. A rectangular garden is to be twice as long
upright on its circular base on level ground, the as its width. If 360 m of fencing including gates
height of the water in the tank is 4 metres But will totally enclose this garden, what is the length
when this tank is placed on its side on level of the garden?
(a) 120 m

11 | P a g e W W W . E D U T A P . C O . I N QUERY? HELLO@EDUTAP.CO.IN / 8146207241


(b) 130 m
(c) 140 m Q45. For which time intervals, is the percentage
(d) 150 m rise of the population the same for the following
Answer: A data?
Explanation:
Let the width of the rectangular garden be x
So, its length will be 2x

So, its perimeter will be = 2 (L + W) = 2 (x +2x) = 6x


That is given to be equal to 360 m.
Thus, 6x = 360 → x = 60m.
So, the length of the garden will be 2x = 120

(a) 1970-80 and 1980-90


Q43. In a chess tournament, each of the six
(b) 1980-90 and 1990-2000
players will play with every other player exactly
(c) 2000-2010 and 1990-2000
once. What is the number of matches that will be
(d) 1980-90 and 2000-2010
played during the tournament?
Answer: B
(a) 10
Explanation:
(b) 15
For years 1970-80, the population increase =
(c) 20
(50000-40000/40000) *100 = 25%
(d) 25
For years 1980-90, the population increase =
Answer: B
(60000-50000/50000) *100 = 20%
Explanation:
For the years 1990-2000, the population increase
Every team plays with another team exactly once
= (72000-60000/60000) *100 = 20%
= nC2 = 6C2 = 15
So, it is the same for 1980-90 and 1990-2010

Q44. A man buys apples at a certain price per


Q46. Four quantities are such that their
dozen and sells them at 8 times that price per
arithmetic mean (A.M.) is the same as the A.M. of
hundred. What percentage does he gain or lose?
the first three quantities. The fourth quantity is
(a) 4% profit
(a) Sum of the first three quantities
(b) 6% profit
(b) A.M. of the first three quantities
(c) 4% loss
(c) (Sum of the first three quantities)/4
(d) 6% loss
(d) (Sum of the first three quantities)/2
Answer: C
Answer: B
Explanation:
Explanation:
Let a man purchase 1dozen (12) apples for Rs.12
A.M of 4 terms = A.M of the first 3 terms (given)
CP of 1 dozen apple = Rs. 12
Let’s say 4 terms are a, b, c, and d
CP of 1 apple = Rs. 1
A.M of 4 terms = (a + b + c + d)/4
A.M of first 3 terms = (a + b + c)/3
Sold them 8 times per 100=12*8/100 i.e. 0.96
Now,
SP of 1 apple = 0.96
According to the given condition
(a + b + c + d)/4 = (a + b + c)/3
Loss =1–0.96 i.e. 0.04
3*(a + b + c + d) = 4*(a + b + c)
Loss % = 0.04*100/1 i.e. 4%
3d = a + b + c
Loss % = 4%
d = (a + b + c)/3
12 | P a g e W W W . E D U T A P . C O . I N QUERY? HELLO@EDUTAP.CO.IN / 8146207241
So, d = A.M of first three terms
Now consider 3-digit palindromes from 100 to
Q47. If the difference between two numbers is 200.
greater than the sum of the numbers, then They are 101, 111, 121,.....191. These are 10 in
(a) Both the numbers are negative number.
(b) Exactly one of the numbers is negative
(c) At least one of the numbers is negative In the same way, from
(d) None of the above 200-299: 10 palindromes
Answer: C 300-399: 10
Explanation: 400-499: 10
Let’s say 2 numbers are = a, b 500-599: 10
Difference of the number = a – b 600-699: 10
Sum of the numbers = a + b 700-799: 10
According to the question: 800-899: 10
a–b>a+b 900-999: 10
0 > b.
So, b is negative. The last palindrome would be 1001.
Now,
one no. is definitely negative, but we can't So we got 100 palindromes in total between 10
definitely say whether the other no. is negative or and 1010.
not.
For e.g., a=1, b= -2 then 1-(-2) > 1+(-2) = 3>-1 which Q49. In writing all the integers from 1 to 300, how
is true many times is the digit 1 used?
(a) 160
another e.g., a=-1, b=-2 then -1-(-2) > -1+(-2) = 1>- (b) 140
1 which is also true (c) 120
(d) 110
and one no. could be 0 also. Answer:
Explanation: A
Q48. A palindrome is a number that reads the From 1 to 9, 1 will come once.
same from the left as well as from the right, for From 10 to 99, 1 can come (1*10C1) + (9C1*1) = 19
example, 23732. What is the number of From 100 to 199, “1 can come at hundred’s place
palindromes between 10 and 1010? where tens place and unit place can have any
(a) 101 number” or “1 comes at hundred and tens place
(b) 100 and unit place can have any number” or “1 comes
(c) 99 at hundred and unit place and tens place can have
(d) 90 any number” = (1*10C1*10C1) + (1*1*10C1) +
Answer: B (1*10C1*1) = 120
Explanation: From 200 to 299, 2 comes at hundreds place and 1
We can easily solve this problem using comes at tens place and unit place can have any
permutations. digit or 2 comes at hundreds place and 1 at unit
place and tens place can have any number=
First take the two-digit palindromes. (1*1*10C1) + (1*10C1*1) = 20
They would be 11,22,33,..... and 99. They are 9 in
number.

13 | P a g e W W W . E D U T A P . C O . I N QUERY? HELLO@EDUTAP.CO.IN / 8146207241


So, total times 1 digit used = 1 + 19 + 120 + 20 = T = 60 minutes
160
Q52. A and B run a 1 km race. A gives B a start of
Q50. Consider the sequential integers 27 to 93, 50 m and still beats him by 15 seconds. If A runs
both included in the sequence. The arithmetic at 8 km/h, what is the speed of B?
average of these numbers will be (a) 4.4 km/h
(a) 61.5 (b) 5.4 km/h
(b) 61 (c) 6.4 km/h
(c) 60.5 (d) 7.4 km/h
(d) 60 Answer: D
Answer: D Explanation:
Explanation: Let a & b denote the running speeds (in m/s) of A
Sum formula = (n/2)*(2a + (n-1)*d) where n= & B respectively in a 1km (= 1000m) race.
numbers, a= first term, d= difference. It is mentioned that when A gives B a 50m start, A
wins by 15 seconds. So we get the following
Sum of integers from 27 to 93(both included) = equation,
Sum of integers from (1 to 93) – Sum of integers
from 1 to 26. Speed of A = 8km/hr = 8*1000/60*60 = 20/9 m/s
= {(93/2)*(2*1 + (93-1)*1)} – {(26/2)*(2*1 + (26- Time is taken by A to finish the race = 1000/(20/9)
1)*1)} Time = 450 second
= {47*93} – {13*27}
= 4371 – 351 Time taken by B to finish the race = 450 + 15 = 465
= 4020 seconds.
Average of sum for numbers from 27 to 93. (total So, speed of B = (1000 -50)/465
numbers = 93- 27 + 1= 67) = 4020/67 = 60. = 950/465 m/s
So, in km/hr = (950/465) *(18/5)
Q51. Walking at 3/4th of his usual speed, a man = 7.35km/hr ~ 7.4km/hr
reaches his office 20 minutes late. What is the
time taken by him to reach the office at his usual Q53. In a race of 1 km, A can beat B by 40 m and
speed? B can beat C by 50 m. With how much distance
(a) 80 minutes can A beat C in a 0-5 km race?
(b) 70 minutes (a) 42 m
(c) 60 minutes (b) 43 m
(d) 50 minutes (c) 44 m
Answer: C (d) 45 m
Explanation: Answer: D
Let’s say usual speed = S Explanation:
Walking at 3/4th of usual speed reaches 20 minutes Ratio of distance covered by A and B = 1000: 960 =
late i.e., T+20 25: 24
Speed = Distance/Time Ratio of distance covered by B and C = 1000: 950=
(3/4)S = D/(T+20) 20:19
After solving,
So, distance is the same in both cases So, ratio of A: B: C = 125: 120: 114
So, S*T = (3/4)S*(T+20)
4T = 3T + 60 For 1 km difference of A and C is 9 units

14 | P a g e W W W . E D U T A P . C O . I N QUERY? HELLO@EDUTAP.CO.IN / 8146207241


So, in 5km race, A beat C by 9*5 = 45m wide along BC. Concreted pathways are intended
to be laid on the inside of the plot all around the
Q54. In an office, 40% of the employees are men sides. The pathways along BC and DA are each 4
and the rest women. Half of the employees are m wide. The pathways along AB and DC will
tall and half short. If 10% of the employees. men mutually be of equal widths such that the un-
and short, and are 40 employees are women and concreted internal plot will measure three-fourth
tall, the number of tall men employees is of the original area of the plot ABCD. What will
(a) 60 be the width of each of these pathways along AB
(b) 50 and DC?
(c) 40 (a) 3 m
(d) 30 (b) 4 m
Answer: A (c) 5 m
Explanation: (d) 6 m
Total employees = 100x Answer: C
Men = 40% = 40x Explanation:
Women = 60x Area of ABCD = 80*60
Tall = 50x Width of pathway along breadth = 4cm
Short = 50x Width of path way along length = x cm
10% employees men and short = 10x New Area = (3/4)*old area
So, men and tall = 40x – 10x =30x (80 – 2*4)*(60-2x) = (3/4)*80*60
Women and tall = 40 72*(60-2x) = 45*80
540 – 18x = 450
Tall = women tall + men tall 90 = 18x
50x = 40 + 30x X= 5cm
20x = 40
X= 2 Q57. At a dinner party, every two guests used a
Now, Men and tall = 30x = 30*2 = 60 bowl of rice between them, every three guests
used a bowl of dal among them and every four
Q55. A provisions shop-owner is found to mix 25 guests used a bowl of curd among them. There
kg of rice worth Rs.32/kg and 20 kg of rice worth are altogether 65 bowls. What is the number of
Rs.35/kg and the mixed rice is sold at 15% profit. guests present at the party?
What is the selling price of mixed rice. (a) 90
(a) 35.40/kg (b) 80
(b) 38.33/kg (c) 70
(c) 36.50/kg (d) 60
(d) 37.42/kg Answer: D
Answer: B Explanation:
Explanation: Let the total number of guests be x. Then,
CP = 25*32 + 20*35 = 1500
Profit required = 15% Number of bowls of rice = x/2
SP = 1.15(1500) =1725
So, SP of each kg = 1725/45 = 38.33/kg Number of bowls of dal = x/3

Q56. The original lay of a rectangular plot ABCD Number of bowls of meat = x/4
on open ground is 80 m long along AB, and 60 m

15 | P a g e W W W . E D U T A P . C O . I N QUERY? HELLO@EDUTAP.CO.IN / 8146207241


x/2 + x/3 + x/4 = 65 Area = πr2

6x+4x+3x /12 = 65 Now, radius reduced by 50%. So, new radius = r/2
So, new area = π (r/2)2
⟹x=60
So, area reduced by = {(πr2 - π (r/2)2)/ πr2} *100
Therefore, 60 guests were present at the party = (3/4) *100
= 75%
Q58. If 5 men or 9 women can finish a piece of
work in 19 days, 3 men and 6 women will done Q60. In an examination paper where maximum
same work in marks are 500, A got 10% marks less than B, B got
(a) 12 days 25% marks more than C, and C got 20% marks less
(b) 13 days than D. If A got 360 marks, what marks did D get?
(c) 14 days (a) 65%
(d) 15 days (b) 70%
Answer:D (c) 75%
Explanation: (d) 80%
M1 * D1/W1 = M2*D2/W2 Answer: D
Where M1 = working men. D = day, w = work Explanation:
It is given that 5 men or 9 women can do the job in Max marks = 500
19 days. A got 360 marks
So, B got 360/0.9 = 400
C = 400/1.25 = 320
So, 5 men=9 women
D gets = 320/0.8 = 400
So, D = (400/500)*100 = 80%
1 man = (9/5)Women
3 men = 3 ( 9/5) Women Q61. Under the Constitution of India, which of
the following statements are correct?
So, work done = 3 men+6 women = 3*( 9/5) + 6 1. The Constitution is supreme.
Women = 57/5 2. There is a clear division of powers between the
Union and the State Governments.
Now,
3. Amendments to the Constitution have to
M1 * D1/W1 = M2*D2/W2 follow the prescribed procedure.
Let D2 = a 4. The Union Parliament and the State
Also W1 = W2 Legislatures are sovereign.
9*19 = (57/5)*a 5. The Preamble to the Constitution cannot be
So, a = 15 days invoked to determine the ambit of
Fundamental Rights.
Select the correct answer using the codes given
Q59. If the radius of a circle is reduced by 50%, its
below:
area will be reduced by (a) 1, 2, 3, 4 and 5
(a) 30% (b) 2, 3 and 4 only
(b) 50% (c) 1, 4 and 5 only
(c) 60% (d) 1, 2 and 3 only
(d) 75% Answer: D
Answer: D Explanation:
• Both Indian Parliament and State legislatures
Explanation:
are not sovereign and their powers are
Let radius of circle be ‘r’. restricted by:
16 | P a g e W W W . E D U T A P . C O . I N QUERY? HELLO@EDUTAP.CO.IN / 8146207241
- Supremacy of the Constitution of India. Member of Parliament has the right to vote ONLY
- Judicial Review power of the Supreme in the house of which he is member.
Court and High Courts.
- Federal nature of Indian Constitution. Q64. Which one of the following formed the basic
Hence statement 4 is not correct. Thus the premise for developing the Indian Constitution?
correct answer is D. (a) The Government of India Act, 1935
• The Indian Constitution is the fundamental (b) The U.S. Constitution
and Supreme law of the land in our country. (c) The British Constitution
- It has defined the authority and jurisdiction (d) The UN Charter
of all the three organs of the Union Answer: A
government and the nature of Explanation:
interrelationship between them. • The structural part of the Constitution is, to a
• The Constitution divided the powers between large extent, derived from the Government of
the Centre and the states in terms of the Union India Act of 1935 and this act forms the basic
List, State List and Concurrent List in the premise for the development of our
Seventh Schedule. Constitution.
• Preamble is non-justiciable in nature, that is,
its provisions are not enforceable in courts of Q65. The Malimath Committee (2003) looked at
law. It cannot be invoked to determine the ways to reform the
ambit of Fundamental Rights as it is neither a (a) Educational System in India
source of power to legislature nor a (b) Criminal Justice System in India
prohibition upon the powers of legislature. (c) Copyright Laws in India
(d) Public-Private Partnership in India
Q62. The Right to Free and Compulsory Education Answer: B
Act, 2009 provides for education to every child. in Explanation:
the age group Malimath Committee was headed by Justice V.S.
(a) 5 – 13 years Malimath, former Chief Justice of the Karnataka
(b) 8 – 16 years and Kerala High Courts. This Committee began its
(c) 7 – 15 years work in 2000 when it was constituted by the Home
(d) 6 – 14 years Ministry.
Answer: D • The task of examining the fundamental
Explanation: principles of criminal law to restore
Article 21 A declares that the State shall provide confidence in the criminal justice system.
free and compulsory education to all children of • This involved reviewing the Code of Criminal
the age of six to fourteen years in such a manner Procedure (CrPC), 1973, the Indian Evidence
as the State may determine. Act, 1872, and the Indian Penal Code (IPC),
• In pursuance of Article 21A, the Parliament 1860.
enacted the Right of Children to Free and
Compulsory Education (RTE) Act, 2009. Q66. The Parliament can make any law for whole
or any part of India for implementing
Q63. If the Prime Minister of India is a member of international treaties
the Rajya Sabha (a) With the consent of all the States
(a) He can make statements only in the Rajya (b) With the consent of majority of the States
Sabha (c) With the consent of the States: concerned
(b) He has to become a member of the Lok Sabha (d) Without the consent of any State
within six months Answer: D
(c) He will not be able to speak on the budget in Explanation:
the Lok Sabha • The Parliament alone can make laws on any
(d) He will not be able to vote in his Favour matter in the State List for implementing the
Answer: D international treaties, agreements or
Explanation: conventions without the consent of any state.
This provision enables the Central government
17 | P a g e W W W . E D U T A P . C O . I N QUERY? HELLO@EDUTAP.CO.IN / 8146207241
to fulfil its international obligations and • The Vietnam War was a conflict in Vietnam
commitments. and USA from 1 November 1955 to the fall of
Saigon on 30 April 1975.
Q67. Statement (I): Speaker of the Lok Sabha • The French Revolution was a period of radical
appoints the Chairman of the Public Accounts political and societal change in France that
Committee. began with the Estates General of 1789 and
Statement (II): Members of Parliament, eminent ended with the formation of the French
persons from industry and trade are the members Consulate in November 1799.
of the Public Accounts Committee. • World War I was a major global conflict that
Codes: began on 28 July 1914 and ended on 11
(a) Both Statement (I) and Statement (II) are November 1918.
individually true, and Statement (II) is the • The First Persian Gulf War, also known as
correct explanation of Statement (1) the Gulf War, 1991, was an armed conflict
(b) Both Statement (I) and Statement (II) are between Iraq and a coalition of 39 nations
individually true but Statement (II) is NOT the including the United States, Britain, Egypt,
correct explanation of Statement (1) France, and Saudi Arabia
(c) Statement (I) is true but Statement (II) is false
(d) Statement (I) is false, but Statement (II) is True Q69. Which of the following statements are
Answer: C correct in respect of 'Stridhana"?
Explanation: 1. Various types of movable property gifted to a
• The chairman of the Public Account committee woman on various occasions during her
is appointed from amongst its members by the lifetime.
Speaker of Lok Sabha. Hence statement I is 2. Various types of movable property gifted to a
true. woman at the time of her first marriage.
The members of Public Account Committee are 3. On the woman's death, this wealth could be
elected by the Parliament every year from inherited by her children and husband.
amongst its members according to the principle of 4. On the woman's death, this wealth could be
proportional representation by means of the inherited by her natural legally adopted
single transferable vote. So it has Member of children, with none of the sons-in-law, having
Parliament as its members only. Hence statement any claim on any part thereof.
II is false. Select the correct answer using the codes given
below:
Q68. What is the chronological sequence of the (a) 2 and 3 only
following events? (b) 2 and 4 only
1. First Battle of Panipat (c) 1 and 4 only
2. Vietnam War (d) 1 and 3 only
3. French Revolution Answer: B
4. First Gulf War Explanation:
5. World War I The ancient texts like Manusmritis prescribe the
Select the correct answer using the codes given wealth to be inherited by the sons and the women
below: could not claim a share of these resources.
(a) 1, 5, 3, 2 and 4 • However, women were allowed to retain
(b) 3, 1, 5, 4 and 2 Stridhana, which included the gifts they
(c) 3, 1, 4, 5 and 2 received on the occasion of their marriage.
(d) 1, 3, 5, 2 and 4 • This could be inherited by their children,
Answer: D without the husband having any claim on it.
Explanation: • At the same time, the Manusmriti warned
• The first Battle of Panipat, on 20 April 1526, women against hoarding family property, or
was fought between the invading forces of even their own valuables, without the
Babur and the Lodi dynasty. husband’s permission.

18 | P a g e W W W . E D U T A P . C O . I N QUERY? HELLO@EDUTAP.CO.IN / 8146207241


Q70. The Khilafat Committee accepted Mahatma mahajanapadas and established the first
Gandhi's suggestion for a non-violent non- Indian empire.
cooperation protest against the Government in • The sangha originated in the group of disciples
its meeting at who renounced the worldly life to wander with
(a) Bombay the Buddha and listen to his teachings.
(b) Nagpur Hence Option A is the correct answer.
(c) Allahabad
(d) Kanpur Q72. What is the chronological sequence of the
Answer: C following developments?
Explanation: 1. Decline in the export of Indian Cotton
On 9 June 1920, the Khilafat committee at 2. Cotton boom in India
Allahabad unanimously accepted the suggestion 3. Civil War in America
of non-cooperation and asked Gandhiji to lead the Select the correct answer using the codes given
movement. below:
(a) 1, 3 and 2
Q71. Which of the following statements reflect (b) 2, 3 and 1
the distinctive features about 6th century BCE (c) 3, 2 and 1
Ganas or Sanghas? (d) 2, 1 and 3
1. The ancient Indian Ganas were oligarchies. Answer: C
2. The ancient Indian Ganas were non- Explanation:
monarchical states. • When the American Civil war broke out in
3. Magadha, Kosala, Vatsa and Avanti were the 1861, a wave of panic spread in Britain. As the
most powerful States of that era. cotton prices soared export merchants in
4. The procedures of the Buddhist monastic Bombay were keen to meet the British
order (Sangha) patterned on the Sangha demand.
politics. • They gave advances to the Urban sahukars
Select the correct answer using the codes given who in turn extended credit to rural money
below: lenders which promised to secure the produce.
(a) 1, 2 and 3 only • While the American crisis continued, cotton
(b) 1, 2, 3 and 4 production in the Bombay Deccan expanded.
(c) 1, 2 and 4 only • By 1862 over 90% of the cotton imports into
(d) 3 and 4 only Britain were coming from India.
Answer: A • As the civil war ended, cotton production in
Explanation: America revived and Indian cotton exports
The proto states of the Gangetic region were declined.
known as janapadas and comprised chiefdoms,
republics and small kingdoms. Sixteen Q73. The Problem of Rupee' was whose D. Sc
mahajanapadas find mention in the early texts. Thesis
• There were also gana - sanghas or oligarchies, (a) Sri Aurobindo
which were centred on clans. (b) Dr. Rajendra Prasad
• These kingdoms did not come under the single (c) Pt. Jawaharlal Nehru
decision-making authority of a king but (d) Dr. Bhimrao Ambedkar
decisions were taken on a collective basis by Answer: D
the heads of the different clans together. Explanation:
There were also smaller kingdoms such as 'The Problem of the Rupee' was the title of the
Kosala and Kasi. thesis that Dr. B. R. Ambedkar submitted to the
• Among the 16 mahajanapadas, Kasi was London School of Economics for which he was
initially powerful. However, Kosala became awarded his doctorate in 1923.
dominant later. A power struggle broke out
between Magadha, Kosala, Vatsa and Avanti. Q74. Which of the following industries were first
Eventually Magadha emerged as the dominant established during the British Rule in India?
(a) Cotton Textile Industry and Jute Industry
19 | P a g e W W W . E D U T A P . C O . I N QUERY? HELLO@EDUTAP.CO.IN / 8146207241
(b) Jute Industry and Iron & Steel Industry appreciation, chronicling and classification of
(c) Cotton Textile Industry and Chemical Industry historic and non-organic relics, monuments and
(d) Jute Industry and Chemical Industry old texts refers to
Answer: A (a) Archaeology
Explanation: (b) History
Cotton and Jute are the sets of industries were (c) Architecture
among first modern industries in India. (d) Antiquarianism
• The emergence of the cotton textile Answer: A
industry started with the introduction of the Archaeology or archeology is the scientific study of
first machinery in Kolkata in the year 1818 at human activity through the recovery and analysis
fort Gloster. Followed by this, another major of material culture. The archaeological record
event is the installation of machinery in consists of artifacts, architecture, biofacts or
Mumbai in the year 1854 by Nanabhai Davar. ecofacts, sites, and cultural landscapes.
• The first jute mill was established at Rishra, on
the River Hooghly near Calcutta in 1855 when Q77. Which of the following is not true of the
Mr. George Acland brought jute spinning Western Ghats?
machinery from Dundee. (a) UNESCO declared 39 places in the Western
Ghats as 'World Heritage Sites' in 2012
Q75. Which of the following were the main (b) It is one of the eight hottest hotspots of
aspects of the Gandhi-Irwin Pact of 1931? biological diversity in the world
1. Suspension of the Civil Disobedience (c) It has two reserves and thirteen national parks
Movement (d) ONGC surveys have recently found huge oil
2. To take part in the then forthcoming Shimla reserves.
Conference Answer: D
3. Pay taxes for the period of the Civil • ONGC has not found any reserves of oil or
Disobedience Movement natural gas in the Western ghats.
4. Release of Political Prisoners • Rest all the statements are true.
Select the correct answer using the codes given
below: Q78. Ayodhya is located at the bank of which
(a) 1 and 3 only river?
(b) 2 and 3 only (a) Falgu
(c) 1 and 4 only (b) Ganga
(d) 2 and 4 only (c) Sarayu
Answer: C (d) Yamuna
Explanation: Answer: C
Gandhi-Irwin Pact is the name given to a political Ayodhya is located on the banks of Sarayu river.
agreement concluded by Mahatma Gandhi and
Lord Irwin, the then Viceroy of India, on 5th March Q79. The term 'Carbon footprint' means
1931. As per this pact: (a) A region which is rich in coal mines
• The Indian National Congress (INC) agreed to (b) The amount of reduction in the emission of
take part in the Round Table Conference. CO₂ by a country
• The INC would stop the civil disobedience (c) The use of Carbon in manufacturing industries
movement. (d) The amount of greenhouse gases produced by
• Withdrawal of all ordinances that curbed the our day-to-day activities.
activities of the Congress. Answer: D
• Government agreed to release all political A carbon footprint is the total amount of
prisoners to save those involved in violence. greenhouse gases (including carbon dioxide and
Hence C is the correct answer. methane) that are generated by our actions.

Q76. A branch of scholarship that flourished in Q80. Green Revolution in India was expeditiously
Europe, particularly from the fifteenth to the feasible in zones of
eighteenth centuries, and was devoted to the (a) High rainfall
20 | P a g e W W W . E D U T A P . C O . I N QUERY? HELLO@EDUTAP.CO.IN / 8146207241
(b) Moderate rainfall (d) A programme for diaspora youth to
(c) Assured irrigation promote awareness among them about
(d) Higher population density different aspects of India.
Answer: C Answer: D
Green revolution in India was feasible in the zones Explanation
where irrigation coverage was good. Thus, • About Know India Programme
regions of Punjab, Haryana & Western Uttar o It is organized by the Ministry of
Pradesh were benefited in the Green Revolution. External Affairs
o It is an important initiative of the
Q81. What is the correct sequence of tributaries Government of India with an aim to
of the river Ganga from West to East? engage and make the students and
(a) Yamuna, Ghaghara, Son, Gandak and Kosi young professionals of Indian
(b) Ghaghara, Yamuna, Gandak, Kosi and Son Diaspora, in the age group of 18 to
(c) Yamuna, Ghaghara, Gandak, Son, and Kosi 30years.
(d) Ghaghara, Yamuna, Kosi, Gandak and Son o It engages a sense of connect with
Answer: A their motherland, to be motivated
and inspired by transformational
changes taking place in India and to
give them an exposure to various
aspects of contemporary India’s
forms of art, heritage and culture.

Q84. Which of the following are incorporated in


the Government approved National Telecom
Policy, 2012?
1. Broadband for all with a minimum
download speed of two megabits per
second
2. India's rural tele-density to be improved
Q82. In the study of pollution, SPM refers to from 39% to 70% in the next five years
(a) Sulphur Phosphorus Matter 3. Roaming charges shall be scrapped
(b) Sulphur Particulate Matter Select the correct answer using the codes
(c) Solid Particulate Matter given below:
(d) Suspended Particulate Matter (a) 1 and 2 only
Answer: D (b) 1 and 3 only
Particulates – also known as atmospheric aerosol (c) 2 and 3 only
particles, atmospheric particulate matter, (d) 1, 2 and 3
particulate matter (PM) or suspended particulate Answer: D
matter (SPM) – are microscopic particles of solid Explanation
or liquid matter suspended in the air. • About National Telecom Policy 2012
o Strive to create One Nation - One
Q83. What is Know India Programme (KIP)? License across services and service
(a) A programme for children aged 12- areas.
15years from rural areas to make them o Achieve One Nation - Full Mobile
aware of different aspects of India Number Portability and work
(b) A programme that facilitates visits of towards One Nation - Free
rural teachers to different parts of the Roaming.
country o Increase rural teledensity from the
(c) An out-reach programme of the Ministry current level of around 39 to 70 by
of External Affairs to build closer ties with the year 2017 and 100 by the year
India's neighbourhood 2020.
21 | P a g e W W W . E D U T A P . C O . I N QUERY? HELLO@EDUTAP.CO.IN / 8146207241
o To recognise telecom, including o Ensure environmental
broadband connectivity as a basic sustainability
necessity like education and health o Global partnership for
and work towards ‘Right to development
Broadband’. • The SDGs were set up in 2015 by the
o Provide affordable and reliable United Nations General Assembly and are
broadband-on-demand by the year intended to be achieved by 2030
2015 and to achieve 175 million
broadband connections by the year
2017 and 600 million by the year
2020 at minimum 2 Mbps
download speed and making
available higher speeds of at least
100 Mbps on demand.
o Provide high speed and high quality
broadband access to all village
panchayats through a combination
of technologies by the year 2014
and progressively to all villages and
habitations by 2020. .
Q.86 Amongst the following, who is the latest
recipient of the "Bharat Ratna' award?
Q.85 Which of the following is one of the (a) Sachin Tendulkar
Millennium Development Goals fixed by the (b) Atal Bihari Vajpayee
UNO? (c) Amartya Sen
(a) Having extreme poverty and halting the (d) A.P.J. Abdul Kalam
spread of HIV/AIDS Answer: B
(b) Develop a new variety of foodgrains Explanation
which can ensure bumper crops • Atal Bihari Vajpayee was conferred the
(c) To ensure that population does not Bharat Ratana Award in 2015
increase in least developed countries. • After him following persons have received
(d) Develop procedures towards adequate the Bharat Ratana in 2019: Pranab
and timely forewarning on hurricanes and Mukherjee, Nanaji Deshmukh and Bhupen
tsunamis Hazarika
Answer: D
Explanation Q.87 The Malimath Committee (2003) looked at
• The Millennium Development Goals were ways to reform the
targeted from 2000-2015 realizing these (a) Educational System in India
values around the world by 2015 and (b) Criminal Justice System in India
served as the focus for UN work (c) Copyright Laws in India
throughout the period: (d) Public-Private Partnership in India.
o Eradicate extreme poverty and Answer: A
hunger Explanation
o Achieve universal primary • About Malimath Committee
education • It’s objective was to suggest the need to re-
o Promote gender equality and write the Code of Criminal Procedure
empower women (CrPC), the Indian Penal Code (IPC) or the
o Reduce child mortality Indian Evidence Act to fulfil the aspiration
o Improve maternal health of citizens and to accommodate these with
o Combat HIV/AIDS, malaria and changing nature of the crime.
other diseases

22 | P a g e W W W . E D U T A P . C O . I N QUERY? HELLO@EDUTAP.CO.IN / 8146207241


• It suggested the setting up of a State • It suggested four modes of disinvestment
Security Commission to insulate the police viz. Trade sale, Strategic Sale, Offer of
from political pressure. shares and Closure or sale of Assets.
• It was/ given the mandate to review only • But the then Government did not take any
the Code of Criminal Procedure (CrPC), decision on the Committee's
1973. recommendations..

Q.88 The Rangarajan Committee on


disinvestment of shares in Public Sector Q.89 What was the position held by Dr. S.
Enterprises suggested that Radhakrishnan, before he became the Vice-
1. The percentage of equity to be divested President of India?
56 should be no more than 49% for (a) Ambassador to the U.S.A.
industries explicitly reserved for the (b) Chairman, U.G.C.
public sector and it should be either 74% (c) Chairman, Planning Commission
or 100% for others. (d) Ambassador to the Soviet Union
2. Year-wise targets of disinvestment Answer: D
should be maintained. Explanation
Which of the above statements is/are • Dr S. Radhakrishnana was serving as Indian
correct? ambassador from 1949 to 1952 to the Soviet
(a) 1 only Union.
(b) 2 only • He was elected as Vice President in 1952.
(c) Both 1 and 2
(d) Neither 1 nor 2 Q.90 Which of the following trends in FDI inflows
Answer: A are correct?
Explanation 1. In 2003 04, the FDI Equity inflow
• The Rangarajan Committee of 1993 was percentage growth was negative.
constituted by the government for making 2. From 2004-05 to 2007-08, the FDI inflows
recommendations in context with the were very high and positive.
disinvestment. The committee said that 3. In 2008-09, the FDI inflows were positive,
• Disinvesting up to 49% of PSEs equity for but had decreased relative to the
industries explicitly reserved for the public previous year.
sector and over 74% in other industries. Select the correct answer using the codes
• The units to be disinvested should be given below:
identified and disinvestment could be (a) 1 and 3 only
made upto any level, except in defence and (b) 1, 2 and 3
atomic energy where the government (c) 2 and 3 only
should retain the majority holding in (d) 1 and 2 only
equity. Answer: As the question is based on latest trends,
• Disinvestment should be a transparent students are advised to prepare the latest trends
process duly protecting the right of the related to important parameters (Like GPD,
workers. Inflation, FDI etc.) for the examination.
• An autonomous body for the smooth
functioning and monitoring of the Q.91 The Large Hadron Collider (LHC) used in the
disinvestment programme should be CERN project is a particle accelerator located on
established. This recommendation led to the border between
the Disinvestment Commission in 1996 as (a) Spain and France
an advisory body having a full time (b) France and Germany
chairman and four part-time members. (c) Spain and Switzerland
The Commission was required to advise (d) France and Switzerland
the government on the extent, made, Answer: D
timing and princing of disinvestment. Explanation

23 | P a g e W W W . E D U T A P . C O . I N QUERY? HELLO@EDUTAP.CO.IN / 8146207241


• The Large Hadron Collider (LHC) is the (a) Air
most powerful particle accelerator ever (b) Rail
built. (c) Road
• It is located on the Franco-Swiss border (d) Water
near Geneva, Switzerland. Answer: C
Explanation:
Q92. Special Economic Zones (SEZ) are developed • The extent of responsiveness of demand with
to change in the price is not always the same.
(a) Generate additional economic activity The demand for a product can be elastic or
throughout the country inelastic, depending on the rate of change in
(b) Beautify suburban areas the demand with respect to change in price of
(c) Upgrade the facilities in the countryside a product. Elastic demand is the one when the
(d) Promote investment from domestic and response of demand is greater with a small
foreign sources proportionate change in the price. On the
Answer: D other hand, inelastic demand is the one when
Explanation: there is relatively a less change in the demand
• Special Economic Zones (SEZ) are developed to with a greater change in the price.
create additional economic activity, to boost • Therefore, in terms of transportation, road
the export of goods and services, to generate transportation has the highest elasticity as it
employment, to boost domestic and whenever there is change in fare of road
foreign investments, to develop infrastructure transportation, its demand will change
facilities. Hence D is the right answer. drastically and people may adopt to other
modes in case of increase in fare. On the other
Q93. Unbalanced' Growth is hypothesized when
hand, if fare of road transportation decreases
(a) Expansion can take place simultaneously on
then people will more prefer it. Hence, C is
several growth routes
correct.
(b) Supply of labour is fixed
(c) Supply of capital is unlimited
Q95. How does an expansionary monetary policy
(d) Active sectors need to, and do energize
affect the rate of interest and level of income?
sluggish sectors
(a) Raises the level of income but lowers the rate
Answer: D
of interest
Explanation:
(b) Raises the rate of interest but lowers the level
• Balanced growth aims at harmony,
of income
consistency and equilibrium whereas
(c) Raises both, the rate of interest and the level
unbalanced growth suggests the creation of
of income
disharmony, inconsistency and
(d) Lowers both, the rate of interest and the level
disequilibrium. The implementation of
of income
balanced growth requires huge amount of
Answer: A
capital. On the other hand, unbalanced
Explanation:
growth requires less amount of capital,
• Expansionary monetary policy works
making investment in only leading sectors.
by expanding the money supply faster than
Balanced growth is long term strategy
usual or lowering short-term interest rates. It
because the development of all the sectors of
is enacted by central banks and comes about
economy is possible only in long run period.
through open market operations, reserve
But the unbalanced growth is a short-term
requirements, and setting interest rates.
strategy as the development of few leading
• Expansionary monetary policy aims to
sectors is possible in short span of period.
increase aggregate demand and economic
Hence, D is the right answer.
growth in the economy. It involves cutting
interest rates or increasing the money supply
Q94. Transportation plays a great role in any
to boost economic activity. It could also be
economy. Which of the following modes of
termed a ‘loosening of monetary policy’. It is
transportation has the highest elasticity?

24 | P a g e W W W . E D U T A P . C O . I N QUERY? HELLO@EDUTAP.CO.IN / 8146207241


the opposite of ‘tight’ monetary policy. included. The value of GNP is calculated on
Hence, A is the right answer. the basis of GDP.
• GNP = GDP + X – M. Where, X = income of the
Q96. When the Government ownership in Public people of a country who are living outside of
Sector Undertakings is diluted, it is called the Country. M = income of the foreigners in
(a) Privatization a country.
(b) Public-Private Partnership • Hence, D is the right answer.
(c) Disinvestment
Q98. Which of the following statement best
(d) Deflation
describes the content of the theory of
Answer: C
distribution?
Explanation:
a) The distribution of income among different
• Privatisation refers to the sale of the individuals in the economy
government’s majority stake, or the whole b) The distribution of income between the
enterprise, to private investors. In case of Centre and the State Governments
privatisation, the government doesn’t hold c) The principle of just distribution of wealth and
the resulting control and ownership. income
• A public-private partnership (PPP) is a long- d) The distribution of income between the
term contract between a private party and a owners of factor resources
government agency for the provision of a Answer: D
public asset or service, in which the private Explanation:
party assumes major risk and management • Distribution refers to the way total output,
responsibilities. income, or wealth is distributed among the
• Disinvestment means the government or an factors of production such as labour, land, and
organisation is liquidating or selling its stake in capital. Hence, D is the right answer.
a company. But it will be less than 50% and the
government or the organisation will still be in Q99. The demand for a factor of production is
the saddle, calling the shots. said to be derived demand because
• Deflation is when consumer and asset prices 1. It is a function of the profitability of an
decrease over time, and purchasing power enterprise
increases. 2. It depends on the supply of complementary
• Hence, C is the right answer. factors
3. It stems from the demand for the final
Q97. Consider the following items: product
1. Consumer goods and services 4. It arises out of means being scarce in relation
2. Gross private domestic investment to wants
3. Goods and services produced by the Which of the above reasons is/are correct?
Government a) 1 only
4. Net income from abroad b) 3 only
Which of the above items are included in GNP? c) 2 and 4 only
(a) 1, 2 and 3 only d) 1, 2, 3 and 4
(b) 1, 2 and 4 only Answer: B
(c) 3 and 4 only Explanation:
(d) 1, 2, 3 and 4 • In essence, the demand for, say, a factor of
Answer: D production by a firm is dependent on the
Explanation: demand by consumers for the product
• GNP is the value of all final goods and services produced by the firm. The term was first
produced by the residents of a country in a introduced by Alfred Marshall in his Principles
financial year of Economics in 1890. Demand for all factors
• While Calculating GNP, income of foreigners of production is considered as derived
in a country is excluded but income of people demand. Hence, B is the right answer.
who are living outside of that country is

25 | P a g e W W W . E D U T A P . C O . I N QUERY? HELLO@EDUTAP.CO.IN / 8146207241


Q100. Whenever countries set up a Free Trade • The Finance Commission helps in maintaining
Area, they abolish all restrictions on trade among fiscal federalism in India by performing
themselves and following functions:
a) They establish a common external tariff on ➢ The distribution between the Union and
imports from outside countries the States of the net proceeds of taxes
b) They abolish all restrictions on imports from which are to be divided between them
outside countries and the allocation between the States of
c) They abolish all restrictions on imports from the respective shares of such proceeds.
other Free Trade Areas ➢ Determination of principles and
d) Each country maintains its own set of tariffs quantum of grants-in-aid to States which
and quotas on imports from outside countries are in need of such assistance.
Answer: A ➢ Measures needed to augment the
Explanation: Consolidated Fund of a State to
• Whenever countries set up a Free Trade Area, supplement the resources of the
they abolish all restrictions on trade among Panchayats and Municipalities in the
themselves and they establish a common State on the basis of the
external tariff on imports from outside recommendations made by the Finance
countries. Hence, A is the right answer. Commission of the State. Hence,
statement 1 and 3 are correct.
Q101. The term 'Dear Money' refers to ➢ Evaluation of Centrally sponsored
a) Low rate of interest on housing loans schemes are not a responsibility of
b) Value of money at the recession stage Finance Commission. Hence, statement 2
c) High rate of interest is incorrect.
d) Savings gained due to decrease in rate of ➢ Developing five years plan was the
interest on housing loans responsibility of erstwhile Planning
Answer: C Commission. Hence, statement 4 is
Explanation: incorrect.
• Dear money refers to hard-to-borrow funds ➢ Therefore, B is the right answer.
created by a high-interest rate environment,
Q103. Which of the following statements are
making money more expensive to obtain.
correct regarding the International Monetary
When central banks enact tight monetary
Fund (IMF)?
policy, interest rates go up, encouraging
saving and discouraging lending or 1. India is a founder member of the IMF
investment. Hence C is the right answer. 2. India's quota in the IMF is more than 4 percent
3. Finance Minister is represented ex-officio on
Q102. Which of the following are the roles of the the IMF
4. IMF conducts regular review of India's
Finance Commission in India?
economic status under Article IV
1. The distribution of money collected through
Select the correct answer using the codes given
taxes
below:
2. Evaluation of Centrally sponsored schemes
a) 1, 2 and 3 only
3. Evolve principles based on which funds are
b) 1, 3 and 4 only
allotted among States
c) 2 and 4 only
4. To develop Five Year Plans
d) 1, 2, 3 and 4
Select the correct answer using the codes given
Answer: B
below:
Explanation:
a) 1 and 4 only
• India is a founder member of the IMF. Hence,
b) 1 and 3 only
statement 1 is correct.
c) 2 and 4 only
• India's quota in the IMF in 2022 is around 2.75
d) 2 and 3 only
percent. Hence, statement 2 is incorrect.
Answer: B
• Finance Minister is represented ex-officio on
Explanation:
the IMF. Hence, statement 3 is correct.
26 | P a g e W W W . E D U T A P . C O . I N QUERY? HELLO@EDUTAP.CO.IN / 8146207241
• Country surveillance is an ongoing process Statement (II): Different countries have different
that culminates in regular (usually annual) factor endowments
comprehensive consultations with individual Codes:
member countries, with discussions in (a) Both Statement (I) and Statement (II) are
between as needed. The consultations are individually true and Statement (II) is the
known as "Article IV consultations" because correct explanation of Statement (I)
they are required by Article IV of the IMF's (b) Both Statement (I) and Statement (II) are
Articles of Agreement. During an Article IV individually true but Statement (II) is not the
consultation, an IMF team of economists visits correct explanation of Statement (I)
a country to assess economic and financial (c) Statement (I) is true but Statement (II) is false
developments and discuss the country's (d) Statement (I) is false but Statement (II) is true
economic and financial policies with Answer: A
government and central bank officials. IMF Explanation:
staff missions also often meet with • Trade between two countries takes place on
parliamentarians and representatives of account of differences in costs. The reason is
business, labor unions, and civil society. that different countries have different factor
Hence, statement 4 is correct. endowments. Hence, both the statements
• Therefore, B is the right answer. are correct and also Statement (II) is the
correct explanation of Statement (I).
Q104. Statement (I): The effects of an income tax • Therefore, A is the right answer.
on consumption, saving and investment are micro
effects Q106. Statement (1): Foreign investment may
Statement (II): Income tax is an example of direct affect a country's export performance.
tax. Statement (II): Inflow of foreign exchange may
Codes: cause appreciation of local currency leading to a
(a) Both Statement (I) and Statement (II) are rise in the price of export commodities.
individually true and Statement (II) is the Codes:
correct explanation of Statement (I) (a) Both Statement (I) and Statement (II) are
(b) Both Statement (I) and Statement (II) are individually true and Statement (II) is the
individually true but Statement (II) is not the correct explanation of Statement (I)
correct explanation of Statement (I) (b) Both Statement (I) and Statement (II) are
(c) Statement (I) is true but Statement (II) is false individually true but Statement (II) is not the
(d) Statement (I) is false but Statement (II) is true correct explanation of Statement (I)
Answer: B (c) Statement (I) is true but Statement (II) is false
Explanation: (d) Statement (I) is false but Statement (II) is true
• The effects of an income tax on consumption, Answer: A
saving and investment are micro effects. Explanation:
Hence, statement I is correct. • Foreign investment may affect a country's
• In the case of direct tax, the burden can't be export performance as inflow of foreign
shifted by the taxpayer to someone else. exchange may cause appreciation of local
These are largely taxes on income or currency leading to a rise in the price of export
wealth. Income tax, corporation tax, property commodities. Hence, both the statements
tax, inheritance tax and gift tax are examples are correct and also Statement (II) is the
of direct tax. Hence, statement II is correct. correct explanation of Statement (I).
• Both statement I and II are correct but Therefore, A is the right answer.
Statement (II) is not the correct explanation of
Statement (I) Q107. The famous 'Giri' approach in Industrial
• Therefore, B is the right answer. Relations in India espouses the cause of
(a) Adjudication
Q105. Statement (1): Trade between two (b) Compulsory Collective Bargaining
countries takes place on account of differences in (c) Conciliation
costs. (d) Arbitration
27 | P a g e W W W . E D U T A P . C O . I N QUERY? HELLO@EDUTAP.CO.IN / 8146207241
Answer: D • Inflation is a big challenge for the schemes like
Explanation: the Provident Fund etc., since it erodes the
• According to Shri V.V. Giri, the late President real value of savings. Hence, statement 2 is
of India, collective bargaining’s and mutual correct.
negotiations between management and • Provident Fund Scheme does not generate
labour should be used to settle industrial forced saving that can be used to finance
disputes. He suggested that there should be national development plans. Hence, C is
bipartite machinery in every industry and incorrect.
every unit of the industry to settle differences • Therefore, A is the right answer.
from time-to-time with active encouragement
of Government. Q109. Consider the following programmes:
• Outside interference should not encroach 1. Credit Linked Capital Subsidy Scheme
upon industrial peace. Giri’s stress was on 2. Micro Finance Programme
voluntary efforts of the management and the 3. National Manufacturing Competitiveness
trade unions to wind up their differences, Programme
through voluntary arbitration. He was against 4. Cluster Development Programme
compulsory adjudication which cuts the very What is common in the above programmes?
root of the trade union movement. He a) They are related to improving agriculture
advocated collective bargaining for securing b) They are programmes related to Micro, Small
industrial peace. and Medium Enterprises
• Thus, the Giri Approach to employer- c) They are programmes to improve large scale
employee relations implies the industries
encouragement of mutual settlement of d) They are programmes to improve the
disputes, collective bargaining and voluntary traditional cottage industries
arbitration. The essence of this approach is Answer: B
internal settlement in preference to Explanation:
compulsion from outside and voluntary • The above-mentioned programmes are
arbitration and collective bargaining rather related to MSMEs. Hence, B is the right
than compulsory arbitration. answer.
• Hence, D is the right answer.
Q110. The Maternity Benefit Act, 1961 (M.B. Act)
Q108. What are the disadvantages of Provident provides for how many weeks' wages during the
Fund Scheme? maternity period?
1. Money is inadequate for risks occurring early a) 11 weeks
in working life. b) 12 weeks
2. Inflation erodes the real value of savings. c) 13 weeks
3. It generates forced saving that can be used to d) 14 weeks
finance national development plans. Answer: None of the above
Select the correct answer using the codes given Explanation:
below: • Maximum period for which any woman be
a) 1 and 2 only entitled to maternity benefit shall be 26
b) 1 and 3 only weeks of which not more than 8 weeks shall
c) 2 and 3 only precede the date of her expected delivery.
d) 1, 2 and 3 • Earlier the limit was 12 weeks which has been
Answer: A extended to 26 weeks.
Explanation:
• One of the main disadvantages of Provident Q111. Which of the following are the instruments
Fund Scheme is that Money is inadequate for of providing social security in India?
risks occurring early in working life since the 1. Income Tax
corpus collected during early working period 2. Employees' Provident Fund
is small. Hence, statement 1 is correct. 3. General Sales Tax
4. LIC
28 | P a g e W W W . E D U T A P . C O . I N QUERY? HELLO@EDUTAP.CO.IN / 8146207241
5. National Pension Scheme 4. The insurance covers death and permanent
6. Postal Provident Fund disability due to accident.
Select the correct answer using the codes given Which of the above statements are correct?
below: a) 1 and 2 only
a) 1, 2, 3 and 4 b) 3 and 4 only
b) 2, 3, 4 and 5 c) 2 and 3 only
c) 2, 4, 5 and 6 d) 1 and 4 only
d) 3, 4, 5 and 6 Answer: B
Answer: C Explanation:
Explanation: • The Scheme is available to people in the age
• Income tax, Sales tax are not the instruments group 18 to 70 years with a bank account who
of social security. These are taxes imposed on give their consent to join / enable auto-debit
the people. Rest all are the instruments of on or before 31st May for the coverage period
social security. Hence, C is the right answer. 1st June to 31st May on an annual renewal
basis. Hence, statement 1 is incorrect.
Q112. Consider the following statements in • The risk coverage under the scheme isRs.2
respect of Atal Pension Yojana: lakh for accidental death and full disability and
1. Beneficiary must be in the age group of 18 to Rs. 1 lakh for partial disability. Therefore, it is
40 years. not a life insurance cover. Hence, statement 2
2. Beneficiary will receive the pension only after is incorrect and statement 3 is correct.
he attains the age of 60 years. • The insurance covers death and permanent
3. After the death of a beneficiary, his spouse disability due to accident. Hence, statement 4
continues to receive the pension. is correct.
4. No nominee of the beneficiary is permitted. • Therefore, B is the right answer.
Which of the above statements are correct?
a) 3 and 4 only Q114. Social Security may provide cash benefits
b) 1, 3 and 4 only to persons faced with
c) 1, 2 and 3 only 1. Sickness and disability
d) 1, 2, 3 and 4 2. Unemployment
Answer: C 3. Crop failure
Explanation: 4. Loss of the marital partner
• Under Atal Pension Yojana, beneficiary must Select the correct answer using the codes given
be in the age group of 18 to 40 years. Hence, below:
statement 1 is correct. a) 1, 2 and 3 only
• Beneficiary will receive the pension only after b) 1, 2 and 4 only
he attains the age of 60 years. Hence, c) 3 and 4 only
statement 2 is correct. d) 1, 2, 3 and 4
• After the death of a beneficiary, his spouse Answer: D
continues to receive the pension. Hence, Explanation:
statement 3 is correct. • Social security may provide to persons faced
• Beneficiary can nominate a nominee for with sickness and disability, unemployment,
getting the benefits after his death. Hence, crop failure, loss of the marital partner etc.
statement 4 is incorrect. Hence D is the right answer.
• Therefore, C is the right answer.
Q115. Employees State Insurance Act, 1948
Q113. Consider the following statements covers factors like
regarding the Pradhan Mantri Suraksha Bima • Factories and establishments with 10 or more
Yojana: employees.
1. It is applicable for all bank account holders up • Provision of comprehensive medical care to
to the age of 60 years. employees and their families.
2. It is a life insurance cover • Provision of cash benefits during sickness and
3. It is an accident insurance cover. maternity.
29 | P a g e W W W . E D U T A P . C O . I N QUERY? HELLO@EDUTAP.CO.IN / 8146207241
• Monthly payments in case of death or • The life insurance amount is given to the
disablement. family after the death of the
Which of the above statements are correct? subscriber. Hence, statement 4 is correct.
a) 1, 2 and 3 only Hence, C is the right answer.
b) 1, 2 and 4 only
c) 3 and 4 only Q117. SPAM in a system (e-mail) is:
d) 1, 2, 3 and 4 (a) A message distributed indiscriminately
Answer: D (b) A search engine
Explanation: (c) An activity of the user
• Employees State Insurance Act, 1948 covers (d) A command initiated by the sender
factors like factories and establishments with Answer: A
10 or more employees, provision of Explanation
comprehensive medical care to employees • Spam is any form of unrequested
and their families, provision of cash benefits communication sent in bulk (Unsolicited Bulk
during sickness and maternity, monthly Email, or UBE).
payments in case of death or disablement etc. • Its most frequent form is a commercial email
Therefore, D is the right answer. sent to many addresses (Unsolicited
Commercial Email, or UCE), but “spamming”
Q116. Consider the following statements is also possible via instant messages, texts
regarding the Pradhan Mantri Jeevan Jyoti Bima (SMS), social media or even voicemail.
Yojana: • Spam can be used to spread computer viruses,
1. It is applicable to all adults above the age trojan horses or other malicious software.
group of 18 years. The objective may be identity theft, or worse
2. The premium is deducted from the account (e.g., advance fee fraud). Some spam attempts
holder's bank account through 'auto debit to capitalize on human greed, while some
facility'. attempts to take advantage of the victims'
3. The life insurance worth is decided by the inexperience with computer technology to
account holder and he has to pay the annual trick them.
premium accordingly.
4. The life insurance amount is given to the Q118. What is Apartheid?
family after the death of the subscriber. (a) An international organization of peace
Which of the above statements are correct? (b) A medical term
a) 1 and 3 only (c) A trade charter
b) 1 and 4 only (d) A policy of racial segregation
c) 2 and 4 only Answer: D
d) 2 and 3 only Explanation
Answer: C Apartheid is the policy that governed relations
Explanation: between South Africa’s white minority and non-
• The PMJJBY is available to people in the age white majority for much of the latter half of the
group of 18 to 50 years having a bank account 20th century, sanctioning racial segregation and
who give their consent to join / enable auto- political and economic discrimination against non-
debit. Aadhar would be the primary KYC for whites.
the bank account. Hence, statement 1 is
incorrect and statement 2 is correct. Q119. Consider the following statements about
• The life cover of Rs. 2 lakhs shall be for the All India Radio:
one-year period stretching from 1st June to 1. It is governed by the Prasar Bharati Board.
31st May and will be renewable. Risk coverage 2. It was so named in 1936.
under this scheme is for Rs. 2 Lakh in case of 3. It does not provide DTH services.
death of the insured, due to any reason. 4.FM Rainbow and FM Gold are its subsidiary
Therefore, it is pre-decided and not fixed by channels.
the account holder. Hence, statement 3 is Which of the above statements are correct?
incorrect. (a) 1, 2 and 3 only
30 | P a g e W W W . E D U T A P . C O . I N QUERY? HELLO@EDUTAP.CO.IN / 8146207241
(b) 3 and 4 only • The extant FDI Policy, Press Notes and other
(c) 1, 2 and 4 only related notified guidelines formulated by
(d) 1, 2, 3 and 4 Department for Promotion of Industry and
Answer: C Internal Trade (DPIIT) in the Ministry of
Explanation Commerce and Industry are the bases of the
• In April 1930, the Indian Broadcasting Service, FIPB decisions.
under the Department of Industries and • In the process of making recommendations,
Labour, commenced its operations on an the FIPB provides significant inputs for FDI
experimental basis. Lionel Fielden was policy-making.
appointed the first Controller of Broadcasting The following are the functions of FIPB:
in August 1935. • Quickly approve FDI proposals. Hence
• In the following month Akashvani Mysore, a statement 1 is correct.
private radio station was set up. On June 8, • Review FDI policies and help set up
1936, the Indian State Broadcasting Service transparent guidelines that encourage FDI into
became All India Radio. It is governed by the different sectors. Hence statement 3 is
Prasar Bharati Board. correct.
• In 2004, the DTH service of Doordarshan and • To examine the implementation of the various
AIR was started. proposals that it had approved. Hence
• AIR operates at present 18 FM stereo statement 2 is correct.
channels, called AIR FM Rainbow, targeting • To encourage FDI into the country by
the urban audience in a refreshing style of establishing contracts with international
presentation. Four more FM channels called, companies and inviting them to make
AIR FM Gold, broadcast composite news and investments in India.
entertainment programmes from Delhi, • To communicate with the government and
Kolkata, Chennai and Mumbai. industry to augment the inflow of FDI into the
• Hence statements 1, 2 and 4 are correct. country.
Hence C is the correct answer. • To identify other sectors that need FDI.
• To interact with the FIPC being constituted
Q120. Which of the following are the functions of separately by the Ministry of Industry. Hence
Foreign Investment Promotion Board (FIPB)? statement 4 is correct.
1. To ensure expeditious clearance of the
proposals for foreign investment. Subsequent to abolition of the Foreign Investment
2. Τo review periodically the implementation of Promotion Board (FIPB) by the Government in
the proposals cleared by the Board 2017, the work of granting government approval
3. To undertake all other activities for promoting for foreign investment under the extant FDI Policy
and facilitating FDI as considered necessary from and FEMA Regulations, has been entrusted to the
time to time concerned Administrative Ministries /
4. To interact with the FIPC being constituted Departments.
separately by the Ministry of Industry
Which of the above statements are correct?
(a) 1, 2 and 3 only
(b) 1, 2 and 4 only
(c) 1, 2, 3 and 4
(d) 3 and 4 only
Answer: C
Explanation
The Foreign Investment Promotion Board (FIPB),
housed in the Department of Economic Affairs,
Ministry of Finance, is an inter-ministerial body,
responsible for processing of FDI proposals and
making recommendations for Government
approval.
31 | P a g e W W W . E D U T A P . C O . I N QUERY? HELLO@EDUTAP.CO.IN / 8146207241
Visit: www.civilstap.com
For Any Query Mail us: hello@civilstap.com or call us at - (+91)-8146207241
0
(d) High specific resistance and high melting point.
UPSC EPFO APFC – 2012
Q6. Bancassurance is :
Q1. For calculating Body Mass Index (BMI), (a) An insurance scheme to insure bank deposits
weight of the person (in kg) is divided by the: (b) An insurance scheme exclusively for bank
(a) Square of the weight (in kg) employees
(b) Square of the height (in meters) (c) A composite financial service offering both
(c) Square root of the height (in meters) bank and insurance products
(d) Vitamins intake (d) A bank deposit scheme exclusively for
employees of insurance companies
Directions: Each of the next One (2) item consists
of two statements, one labeled as the 'Statement Q7. The word FTP stands for:
(I) and the other as 'Statement (II)'. You are to (a) File Transit Provision
examine these two statements carefully and select (b) File Translate Protocol
the answers to these items using the codes given (c) File Transfer Provision
below: (d) File Transfer Protocol

Codes: Q8. RAM stands for:


(a) Both Statement (I) and Statement (II) are (a) Random Access Memory
individually true, and Statement (II) is the (b) Read Access Memory
correct explanation of Statement (1) (c) Random Attribute Memory
(b) Both Statement (I) and Statement (II) are (d) Random Applicable Memory
individually true but Statement (II) is NOT the
correct explanation of Statement (1) Q9. Which of the following is also known as brain
(c) Statement (I) is true but Statement (II) is false of computer?
(d) Statement (I) is false, but Statement (II) is True (a) Monitor
(b) Arithmetic Logic Unit (ALU)
Q2. Statement (I): In order to produce electric (c) Control Unit
power from a geothermal reservoir, temperature (d) Central Processing Unit (CPU)
above 1800 is required.
Statement (II): To drive steam turbines, high Q10. A technique in which data is written to two
temperature steam is used. duplicate disks simultaneously, is called as _____.
(a) Mirroring
Q3. Gun metal is an alloy of (b) Multiplexing
(a) Copper, Tin and Zinc (c) Duplicating
(b) Aluminium, Tin and Steel (d) Copying
(c) Copper, Steel and Zinc
(d) Aluminium, Tin and Zinc Q11. The term 'e-Waste' refers to :
(a) The files that are deleted and enter the
Q4. If the electrical resistance of a typical 'Waste-bin' folder in a computer
substance suddenly drops to zero, then the (b) The temporary files, folders, links etc. that are
substance is called : rarely used in a computer
(a) Semiconductor (c) The electronic products such as mobiles, PCs
(b) Conductor etc. that are disposed off after their useful life
(c) Superconductor (d) A portal that offers services for collecting
(d) Super semiconductor household waste

Q5. The material used for electric fuse is an alloy Q12. CAD stands for:
of tin and lead. This alloy should have: (a) Computer Aided Design
(a) High specific resistance and low melting point. (b) Computer Application in Design
(b) Low specific resistance and high melting point. (c) Coded Algorithm in Design
(c) Low specific resistance and low melting point. (d) Design
1|P a g e W W W . E D U T A P . C O . I N QUERY? HELLO@EDUTAP.CO.IN / 8146207241
(d) 54 km/hour
Q13. Three sets of data on comparable situations
are available as under: Q18. An iron rod of 1 cm diameter and 8 cm
length is drawn into a wire of 18 m length with
Set No. of Mean Standard uniform thickness. The thickness of the wire
No. data Value Deviation would be:
1 9 8 1.6 (a) 1/21 cm
2 12 7 12 (b) 1/18 cm
3 15 9 1.4 (c) 1/15 cm
(d) 1/12 cm
Considering all the data sets together, the overall
mean value would be: Q19. A 60 m long train travels at a uniform
(a) 7.24 speed of 72 km/hour. It passes non-stop along
(b) 7.66 the 600 m platform of a wayside station. What is
(c) 8.08 the elapsed time for the train to entirely clear
(d) 8.50 the platform?
Q14. If 20% of P= 30% of Q = 1/6 of R, then P: Q: (a) 30 seconds
R is: (b) 31 seconds
(a) 2 : 3 : 16 (c) 32 seconds
(b) 3 : 2 : 16 (d) 33 seconds
(c) 10 : 15 : 18
(d) 15 : 10 : 18
Q20. Train A is 75 m long and travels at a
Q15. A tree increases annually by 1/8th
of its uniform speed of 54 km/hour. Train B is 125 m
height. What will be its height after 2 years, if it long and travels at a uniform speed of 36
stands today 64 cm high? km/hour in the direction opposite to that of
(a) 72 cm Train A. If these trains are crossing at a double-
(b) 74 cm track stretch, what is the time taken for the two
(c) 81 cm
trains to fully clear each other?
(d) 85 cm
(a) 10 seconds
Q16. The sides of a triangle GHL are GH = 65 m, (b) 8 seconds
HL75 m and LG = 80 m. What is the area of this (c) 7.2 seconds
triangle?
(d) 6.6 seconds
(a) 2100 m²
(b) 2160 m² Q21. Consider the following statements in
(c) 2200 m² respect of the Comptroller and Auditor General
(d) 2280 m² (CAG):
1. Reports on the accounts of the States are
Q17. A train travels at a certain average speed submitted to the President who shall cause
for a distance of 63 km. Thereafter it travels a these to be placed before the Parliament.
2. CAG is appointed by the President of India and
distance of 72 km with an average speed of 6
can be removed only on grounds and
km/hour more than the original speed. Total procedure similar to those of a Supreme Court
time taken to complete the journey is 3 hours. Judge.
What is the original average speed of the train? 3. The form in which accounts of the Centre and
(a) 36 km/hour States are to be kept is prescribed by CAG.
(b) 42 km/hour Which of these statements are correct?
(c) 48 km/hour (a) 1, 2 and 3

2|P a g e W W W . E D U T A P . C O . I N QUERY? HELLO@EDUTAP.CO.IN / 8146207241


(b) 1 and 2 only (d) Presidential order in accordance with the
(c) 2 and 3 only Constitution of India
(d) 1 and 3 only
Q26. The Seventh schedule of the Constitution of
Q22. Statement (I): Every State in India has a High India contains provisions regarding:
Court in its (State’s) territory. (a) Scheduled Languages
Statement (II): As per Constitution of India, each (b) Oaths and affirmations
State is provided to have a high Court. (c) Administration of Tribal areas
Codes: (d) Union, State, Concurrent Lists
(a) Both Statement (I) and Statement (II) are
individually true, and Statement (II) is the Q27. The President of India is elected by the
correct explanation of Statement (1) members of an electoral college consisting of
(b) Both Statement (I) and Statement (II) are elected members of:
individually true but Statement (II) is NOT the 1. Both Houses of Parliament
correct explanation of Statement (1) 2. The Legislative Assemblies of the States
(c) Statement (I) is true but Statement (II) is false 3. Union Council of Ministers
(d) Statement (I) is false, but Statement (II) is True Codes:
(a) 1 only
Q23. Statement (I): The Directive Principles of (b) 2 only
State Policy enshrined in the constitution, aim at (c) 1 and 2
providing the social and economic base of a (d) 2 and 3
genuine democracy.
Statement (II): The Directive Principles are merely Q28. The responsibility of preparation of
directives which the government has to keep in electoral rolls in India rests with:
mind while framing policy and is not enforceable (a) The Parliament
through courts. (b) The Local Administration
Codes: (c) The Election Commission
(a) Both Statement (I) and Statement (II) are (d) The Returning Officer
individually true, and Statement (II) is the
correct explanation of Statement (1) Q29. At the time of becoming the Prime Minister
(b) Both Statement (I) and Statement (II) are of our country one:
individually true but Statement (II) is NOT the (a) Must be a member of one of the Houses of the
correct explanation of Statement (1) Parliament. If not, must become a member of
(c) Statement (I) is true but Statement (II) is false one of the Houses within Six months.
(d) Statement (I) is false, but Statement (II) is True (b) Need not necessarily be a member of one of
the Houses of Parliament but must get elected
Q24. Which one of the following Institutions was as a member of Lok Sabha within Six months.
NOT set up by the Constitution of India? (c) Must be a member of one of the Houses of
(a) Union Public Service Commission Parliament prior to appointment.
(b) Election Commission (d) Must be a member of Rajya Sabha.
(c) Planning Commission
(d) Union Judiciary Q30. Match List-I with List-II and select the
correct answer using the code given below the
Q25. The following provision, "Protection of lists:
monuments and places and objects of national List-I (Commission)
importance" is a: A. Second Administrative Reforms Commission
(a) Fundamental Right guaranteed by the B. Second Commission on Centre State relations
Constitution of India C. Thirteenth Finance Commission
(b) Directive Principle of State Policy of the D. National Commission for the Review of the
Constitution of India working of the Constitution
(c) Fundamental Duty laid down by the
Constitution of India List-II (Chairperson)
3|P a g e W W W . E D U T A P . C O . I N QUERY? HELLO@EDUTAP.CO.IN / 8146207241
1. Vijay Kelkar (a) 1 only
2. Justice M.N. Venkatachaliah (b) 2 only
3. Justice Madan Mohan Punchi (c) Both 1 and 2
4. M. Veerappa Moily (d) Neither 1 nor 2
Codes:
A B C D Q35. Which is the correct meaning of the term
(a) 4 3 1 2 'rule of primogeniture?
(b) 2 3 1 4 (a) Favourite son inheriting his father's estate
(c) 4 1 3 2 (b) Eldest son inheriting his father's estate
(d) 2 1 3 4 (c) Division of the inheritance amongst all the
sons
Q31. What is common to Sardar Hukum Singh, (d) Division of the inheritance amongst all the
Jagjivan Ram, Inderjit Gupta, Somnath Chatterjee sons and daughters
and Seth Govind Das?
(a) They were Speakers of the Lok Sabha Q36. Which of the following would be the most
(b) They were Deputy Speakers of the Lok Sabha useful source of evidence for research about the
(c) They were Pro-tem Speakers of the Lok Sabha agrarian history of the sixteenth and early
(d) None of the Above seventeenth centuries?
(a) Accounts of contemporary travellers
Q32. The purpose of Adjournment motion in our (b) Individual revenue records
Parliament is: (c) Chronicles and documents from the Mughal
(a) To allow a discussion on a definite matter of court
urgent public importance. (d) Records of the East India company
(b) To let opposition members, collect
information from concerned Ministers. Q37. Which of the following is a correct
(c) To allow a reduction of specific amount in the statement about both the Mughal and Ottoman
demand of grant. empires in the sixteenth century?
(d) To postpone the proceedings to check the (a) Both of these empires had powerful navies
inappropriate or the violent behaviour on the that engaged European navies.
part of some members. (b) Both of these empires expanded through the
use of gunpowder weapons and extensive
Q33. Consider the following statements bureaucracies.
regarding limitations on the authority of the (c) Both of these empires gave little monetary
Indian Parliament: support to artistic and cultural endeavours.
1. Most of the important laws can be introduced (d) In both of these empires the majority of the
in the Parliament with the prior consent of the people were Muslims.
President.
2. Parliament has to operate within the Q38. Consider the following:
jurisdiction earmarked by the Constitution. 1. Rowlatt Act movement
Which of these statements is/are correct? 2. Kheda movement
(a) Both 1 and 2 3. Champaran
(b) Neither 1 nor 2 4. Ahmedabad mill strike
(c) 1 only What is the correct chronological order of the
(d) 2 only above-mentioned Gandhian movements?
(a) 3, 4, 2 and 1
Q34. Consider the following statements: (b) 2, 3, 1 and 4
1. By the early nineteenth century British carried (c) 3, 2, 4 and 1
out detailed surveys. (d) 2, 1, 3 and 4
2. The effort was to know the topography, the
soil quality, the flora, the fauna, the logical Q39. Which one of the following sets of industries
histories and the cropping pattern. were among first modern industries in India?
Which of these statements is/are correct? (a) Tea, Cotton and Sugar
4|P a g e W W W . E D U T A P . C O . I N QUERY? HELLO@EDUTAP.CO.IN / 8146207241
(b) Jute, Silk and Cotton 2. Most of the thermal plants in our country are
(c) Cotton, Jute and Coal Mines coal based and they are unable to get
(d) Sugar, Silk and Steels sufficient coal supply from our coal mines.
3. Steel plants require large quantities of coking
Q40. Which term is used to describe the spread coal which we do not have, coking coal is to be
of Buddhism from India through Southeast Asia? imported from other countries.
(a) Social mobility Select the correct answer using the codes given
(b) Cultural diffusion below:
(c) Ethnocentrism (a) 1, 2 and 3
(d) Interdependence (b) 2 and 3 only
(c) 1 and 2 only
Q41. Consider the following statements about (d) 1 and 3 only
heritage :
1. Heritage is that which has been or may be Q45. A cultivator has about two hectares rain fed
inherited. land without irrigation facilities. Which type of
2. Heritage is anything given or received to be a crops would he like grow?
proper possession. 1. Jowar
Which of these statements is/are correct? 2. Cotton
(a) 1 only 3. Arhar
(b) 2 only 4. Potatoes
(c) Both 1 and 2 Select the correct answer using the codes given
(d) Neither 1 nor 2 below:
(a) 1, 2, 3 and 4
Q42. Consider the following statements (b) 1, 2 and 3 only
regarding human development: (c) 2 and 4 only
1. Skills may be harnessed in the society through (d) 1 and 3 only
participations of NGOs.
2. To fill the internal gaps in the system, reviews Q46. Match List-1 with List-II and select the
and international experiences can prove to be correct answer using the code given below the
helpful. lists:
Which of these statements is/are correct? List-I List-II
(a) 1 only A. Buxa Tiger Reserve 1. Rajasthan
(b) 2 only B. Periyar Tiger 2. Gujarat
(c) Neither 1 nor 2 Reserve
(d) Both 1 and 2 C. Sariska National 3. West Bengal
Park
Q43. Biome, largest recognizable assemblage of D. Wild Ass Sanctuary 4. Kerala
animals and plants on the Earth, is controlled Code:
mainly by: A B C D
(a) Biological activity (a) 2 1 4 3
(b) Landforms (b) 3 4 1 2
(c) Climate (c) 2 4 1 3
(d) Soil (d) 3 1 4 2

Q44. Despite having huge coal reserve in our Q47. Which one of the following statements is
country, why do we import millions of tons of true about red soil?
coal? (a) It is rich in humus.
1. It is policy of Govt. of India to save its own coal (b) It is rich in potash.
reserve for future use and import now from (c) It is rich in iron compounds.
other countries for present use. (d) It is derived from volcanic origin.

5|P a g e W W W . E D U T A P . C O . I N QUERY? HELLO@EDUTAP.CO.IN / 8146207241


Q48. "Kyoto Protocol', an agreement signed by (d) Joint farming must be practiced with
various countries, is associated with respect to all lands of the society.
(a) International trade
(b) Deep sea oil and mineral exploration Q.53 Three States which have Human
(c) Clean environment and climate change Development Index (HDI) higher than China are:
(d) Building common food stock to save human (a) Kerala, Maharashtra and Madhya
being from any natural disaster Pradesh
(b) Kerala, Punjab and Maharashtra
Q49. Which of the following can be threats to the (c) Maharashtra, Madhya Pradesh and
biodiversity of a geographical area? (d) Kerala, Madhya Pradesh and Gujarat
1. Global warming
2. Fragmentation of habitats Q.54 Match List-I with List-II and select the
3. Invasion of alien species correct answer using the code given below the
4. Promotion of vegetarianism lists:
(a) 1, 2 and 3 only List-I (Chairperson)
(b) 1, 2 and 4 only A. D.M. Dharmadhikari
(c) 3 and 4 only B. Anil Kakodkar
(d) 1, 2, 3 and 4 C. B.N. Srikrishna
D. Adi Godrej
Q50. Vultures, which were very common in our List-II (Issue)
countryside a few years ago, are rarely seen 1. National Corporate Governance Policy
nowadays. This is attributed to one of the 2. Air India Indian Airlines merger
following: 3. Indian Railways High Level Safety Review
(a) Destruction of their nesting sites by new Committee
invasive species 4. Financial Sector Legislative Reforms
(b) A drug used by cattle owners for treating their Commission
diseased cattle Code:
(c) Scarcity of food available to them A B C D
(d) A widespread, persistent and fatal disease (a) 1 3 4 2
amongst them (b) 2 3 4 1
(c) 1 4 3 2
Q51. NABARD means: (d) 2 4 3 1
(a) National Bank for Agriculture and Rural
Development Q.55 The newly introduced rupee symbol is based
(b) National Agricultural Bank and Rural on:
Development (a) Roman and Greek script
(c) National Agricultural Board and Rural (b) Persian and Dravidian script
Development (c) Roman and Devanagari script
(d) National Board for Agricultural and Rural (d) Greek and Devanagari script
Area Development
Q.56 The regional project by name 'CASA-1000*
Q52. Which of the following is NOT a (Central Asia-South Asia-1000), funded by World
recommendation of the National Cooperative Bank, pertains to the field of:
Farming Advisory Board for putting cooperative (a) Eradication of HIV AIDS
farming on a sounder footing? (b) Roads and Buildings
(a) Every Society should have a definite (c) Electricity
programme for total pooling of lands. (d) Eradication of illiteracy
(b) Financial assistance should be given only
for purchase of new land. Q57. Who won the Golden Boot Award in the
(c) State Governments should give priority to recently concluded UEFA Euro 2012?
revitalization of the existing societies. (a) Fernando Torres (Spain)
(b) Mario Balotelli (Italy)
6|P a g e W W W . E D U T A P . C O . I N QUERY? HELLO@EDUTAP.CO.IN / 8146207241
(c) Cristiano Ronaldo (Portugal)
(d) Mario Gomez (Germany) Q.61 Match List-I with List-II and select the
correct answer using the code given below:
Q.58 Match List-1 with List-II and select the List-I (Head)
correct answer using the code given below the A. Jim Yong Kim
lists: B. Kamlesh Sharma
List-I (Person) C. Christine Lagarde
A. Steve Jobs D. Catherine Day
B. Sheryl Sandberg List-II (Institution)
C. Larry Page 1. President Word Bank
D. Ross Levinsohn 2. Secretary-General Commonwealth
List-II (Company) 3. Managing Director, IMF
1. Google 4. Secretary-General of the European
2. Apple Commission
3. Yahoo Code:
4. Facebook A B C D
Code: (a) 3 4 2 1
A B C D (b) 1 4 2 3
(a) 2 1 4 3 (c) 3 2 4 1
(b) 3 1 4 2 (d) 1 2 4 3
(c) 2 4 1 3
(d) 3 4 1 2 Q62. It is said that, in order to control inflation,
foreign inflow needs to be sterilized. Sterilization
Q.59 Match List-I with List-II and select the here refers to
correct answer using the code given below PSC (a) Ensuring that counterfeit currency does not
the lists: enter circulation
List-I (Book) (b) Ensuring that black money is accounted for
A. Rahul Dravid : Timeless Steel (c) Withdrawing equivalent local currency to
B. An Ashes Summer maintain a desirable rate of exchange
C. Sachin : A Hundred Hundreds Now (d) Compliance with import-export regulations
D. Opening Up : My Autography
List-II (Author) Q63. Which of the following are the main causes
1. Steve Waugh and Nasser Hussain of the slow rate of growth of per capita income in
2. Greg Chappell India?
3. Mike Atherton 1. High rate of capital formation
4. V. Krishnaswamy 2. High level of fiscal deficits
Code: 3. High rate of growth of population
A B C D
(a) 3 4 1 2 a) 1, 2 and 3
(b) 2 4 1 3 b) 1 and 2 only
(c) 3 1 4 2 c) 1 and 3 only
(d) 2 1 4 3 d) 2 and 3 only

Q.60 Why was Justice Dalveer Bhandari in the Q64. If the RBI adopts an expansionist open
news recently? market operations policy, this means it will:
(a) He became member of UN Human Rights a) Sell securities in the open market
Council. b) Buy securities from non-government holders
(b) He was elected to the International Court c) Openly announce to the market that it intends
of Justice. to expand credit
(c) He became Director General of World d) Offer commercial banks more credit in the
Trade Organization. open market
(d) He became Secretary-General of SAARC.
7|P a g e W W W . E D U T A P . C O . I N QUERY? HELLO@EDUTAP.CO.IN / 8146207241
Q65. Structural Planning refers to:
(a) Centralized planning Q71. In which of the following Acts, housing
(b) Laying down broad goals and strategies facility is a statutory provision?
(c) Changing existing institutions or creating new (a) The Plantations Labour Act, 1951
ones (b) The Factories Act, 1948
(d) Fixing flexible targets (c) The Mines Act, 1952
(d) None of the above
Q66. Which of the following is NOT one of the
features of the Special Economic Zones (SEZ) Q72. Statement (I): Industrial relation is currently
being set up for promoting exports? more influenced by the external market forces
(a) Foreign workers will be allowed free entry than the power play between employers and
without Visa restrictions employees.
(b) The SEZ area will be treated as foreign territory Statement (II): The forces of globalization have
for trade operations, duties and tariff made competition so imperative that unions and
(c) There will be no routine examination by their tactics like stopping productivity no more
customs authorities of import/export cargo hold good
(d) No license is required for import into the zone Codes:
(a) Both Statement (I) and Statement (II) are
Q67. Which one of the following expresses the individually true, and Statement (II) is the
relation between normal price and market price? correct explanation of Statement (1)
(a) Market price is greater than normal price (b) Both Statement (I) and Statement (II) are
(b) Market price is equal to normal price individually true but Statement (II) is NOT the
(c) Market price tends to be equal to normal price correct explanation of Statement (1)
(d) Market price is lesser than normal price (c) Statement (I) is true but Statement (II) is false
(d) Statement (I) is false, but Statement (II) is True
Q68. Inflation can be controlled by:
(a) Increase in wages Q73. The main objective of the Minimum Wage
(b) Decrease in taxation Act, 1948 is to safeguard the interests the
(c) Reduction in public expenditure workers engaged in:
(d) Making the rupee dearer (a) Unorganized sector
(b) Organized sector
Q69. Which of the following can be termed an (c) Industrial sector
infrastructural bottleneck in the development of (d) Agricultural sector
India's economy?
(a) The federal nature of Indian Polity. Q74. Match List-I with List-II and select the
(b) Existence of a large variety of financial correct answer using the code given below the
institutions lists:
(c) Delay in the administration of justice relating
to land acquisition and displacement List-1 (Contribution)
(d) The volatility of the Indian rupee A. Industrial Welfare Movement
B. Human Relations Thought
Q70. Which of the following statements is true C. Concept of Third Force
about Industrial Policy since 1991? D. Ahmedabad Experiment
(a) Only 5 industries related to security, strategic
and environmental concerns require industrial List-II (Contributor)
License 1. Charles A Myer
(b) An investor need not file an industrial 2. A.K. Rice
entrepreneur Memorandum 3. Robert Owen
(c) There is no reservation of products for 4. Elton Mayo
production in small scale sectors Code:
(d) The number of industries reserved for public A B C D
sector has been enhanced (a) 2 1 4 3
8|P a g e W W W . E D U T A P . C O . I N QUERY? HELLO@EDUTAP.CO.IN / 8146207241
(b) 3 1 4 2 enhancing livelihood security in rural areas of
(c) 2 4 1 3 India by providing at least one hundred:
(d) 3 4 1 2 a) Days of guaranteed employment in a financial
year to able adults in the Information
Q75. Match List-I with List-II and select the Technology sector.
correct answer using the code given below the b) Days of guaranteed wage employment in a
lists: financial year to adult members of a rural
List-I (Board/Committee) household.
A. First National Commission on Labour, 1969 c) Meals to children of rural households in one
B. Wage Board for Working Journalists, 2009 financial year.
C. Second National Commission on Labour, d) Employees for rural developmental schemes in
2002 a financial year.
D. Index Review Committee, 2009
Q79. One of the following Government of India
List-II (Chairperson) programmes aims to help, build or upgrade
1. G.K. Chadha dwelling units of below the poverty line rural
2. Ravindra Verma families:
3. P.B. Gajendragadkar a) National Programme Social Assistance
4. G.R. Majithia b) Jawahar Rozgar Yojana
Code: c) Indira Awas Yojana
A B C D d) Jawaharlal Nehru National Urban Renewal
(a) 3 4 2 1 Mission TAL
(b) 1 4 2 3
(c) 3 2 4 1 Q80. The Mid-Day Meal Scheme was launched in
(d) 1 2 4 3 1995 with the aim to :
1. Enhance enrolment, retention and
Q76. In which part of the Indian Constitution, attendance of primary school children
Worker's participation in Management has been 2. Improve the nutritional status of primary
incorporated? school children
(a) The Preamble 3. Improve the habit of reading among rural
(b) The Fundamental Rights households
(c) The Directive Principles of State Policy 4. Encourage the use of Tiffin boxes among
(d) None of the above primary school children
a) 1 and 2 only
Q77. How does National Rural Livelihood Mission b) 1 and 4 only
(NRLM) seek to improve livelihood options of c) 2 and 3 only
rural poor? d) 1, 2 and 4
1. By setting up a large number of new
manufacturing industries and agribusiness Q81. Which of the following is NOT covered by
centers in rural areas. the Employees Provident Fund and
2. By strengthening Self Help Groups (SHG) and Miscellaneous Provisions Act, 1952?
providing skill development. a) Pension
3. By supplying seeds, fertilizers, diesel pump- b) Provident Fund
sets and micro irrigation equipment free of c) Deposit Linked Insurance
cost to farmers. d) Injury Compensation
a) 1, 2 and 3
b) 2 only Q82. Which of the following statements about
c) 3 only Workmen's Compensation Act, 1923 is true?
d) 1 only a) It is not social security legislation.
b) Its name has been changed to the Employee's
Q78. The Mahatma Gandhi National Rural Compensation Act in 2009.
Employment Guarantee Act (MGNREGA) aims at
9|P a g e W W W . E D U T A P . C O . I N QUERY? HELLO@EDUTAP.CO.IN / 8146207241
c) It provides maximum compensation in the youth, they have foreseen nothing but ruination
event of death. staring the world in its face. And yet the world
d) It does not provide compensation for goes on. Every generation passes from the
occupational diseases. spontaneity and exuberance of youth to the
caution and prudence of old age, and then yield
Q83. Which of the following legislations is place to the next.
comprehensive social security legislation?
a) The Maternity Benefit Act Q86. What, according to you, is the theme and
b) The Employees State Insurance Act the idea of the passage:
c) The Employees Compensation Act (a) Generation gap
d) The Employees Provident Funds and (b) Problems of the youth
Miscellaneous Provisions Act (c) Optimism of the older generation
(d) None of the above
Q84. What is the maximum limit of gratuity
payable under the Payment of Gratuity Act, Q87. The older generation is suspicious about the
1972? younger generation as the youngsters lack:
a) 3 Lakhs rupees (a) knowledge
b) 7-5 Lakhs rupees (b) experience
c) 10 Lakhs rupees (c) patience
d) 10.5 Lakhs rupees (d) time

Q85. The Endeavour of 'Janani Suraksha Yojana' Q88. The author seems to be supportive of the
Programme is to: idea that
1. Promote institutional deliveries (a) the people of the older generation are men of
2. Provide monetary assistance to the mother to words rather than action
meet the cost of delivery (b) the young generation is impatient
3. Provide for wage loss due to pregnancy and (c) the younger generation today is much
confinement misunderstood and more maligned than it
a) 1 and 2 only deserves
b) 1 and 3 only (d) none of the above
c) 2 and 3 only
d) 1, 2 and 3 Q89. "And yet the world goes on"-what is the
tone of the author in this statement?
Next Four (04) items are based on the passage (a) Optimistic
given below: (b) Pessimistic
It has been rightly said that we spend the first half (c) Cynical
of our lives trying to understand the older (d) Critical
generation, and the second half trying to
understand the younger generation. Youth has Next Four (04) items are based on the passage
always felt somewhat exasperated with age, and given below:
age has always been suspicious of youth. With Who deserves more severe punishment? One who
their natural ebullience and impatience, a majority gives bribes or the one who takes them? The
of young people are keen to act and learn on their corrupt practice of bribery is possible because
own rather than be guided by the experience of there is someone who is ready to pay money for
their elders. The older people being more at home illegal action or decision in his favor. Otherwise,
with words rather than with action, often make how can one demand a bribe? The bribe-giver
noises about the problems of youth. In every tempts others to be corrupt and thus demoralizes
generation, old men are found shaking their hoary our national character. Even Jesus Christ, fearing
heads and waxing nostalgic about the good old the power of temptation, had said, "Lead me not
days when young people knew better and showed into temptation". A bribe-giver is generally
due reverence to age and tradition. In all ages, moneyed and influential, while, on the other hand,
whenever they have pondered over the ways of one who demands bribe does so because of his
10 | P a g e W W W . E D U T A P . C O . I N QUERY? HELLO@EDUTAP.CO.IN / 8146207241
poor circumstances and compulsions. Hence, one (b) Headstrong: controlled
who gives bribe should be awarded more severe (c) Elastic: stretch
punishment because he exploits the weakness of (d) Persuasive: convince
the poor. Giving and taking bribe happens in a
more vicious circle which can continue only Q96. Choose the word which is the nearest
because of money. Naturally, therefore, it is the opposite to the meaning of the underlined word:
affluent that grease this wheel of corruption and I wish I could pursue my studies.
should accordingly be dealt with firmly and (a) Discontinue
suitably punished to put a stop to this nefarious (b) Abandon
practice. (c) Restrain
(d) Deter
Q90. The author feels that the practice of bribery
is there because there are willing bribe givers. Q97. Which of the following options is the closest
(a) True in meaning to the word CIRCUITOUS?
(b) False (a) Indirect
(c) Partially true (b) Confusing
(d) None of the above (c) Crooked
(d) Cyclic
Q91. Who, according to the author, is more
responsible among the following? Q98. Archaeological studies suggest the theory
(a) The bribe-taker (a) All the continents were settled with human
(b) The bribe-giver societies at about the same time
(c) The witness (b) Farming societies developed before hunting
(d) The general public and gathering
(c) The earliest human evolved in the Rift valley in
Q92. Who, according to the author, should be East Africa
punished more severely? (d) The wheel was in use in all ancient societies
(a) The Police who allow this practice
(b) The bribe-giver Q99. Which of the following is the earliest to be
(c) The person who demands bribe constituted?
(d) None of the above (a) Press Council of India
(b) United News of India
Q93. Who, according to the author, is more (c) NAM News Network
corrupted? (d) Press Trust of India
(a) Our system
(b) The person who demands bribe Q100. Which of the following sets of countries
(c) The affluent who exploits the weakness of the has only federations?
poor (a) New Zealand, India, Zimbabwe and Argentina
(d) The supporter of bribe who is left with little (b) Malaysia, Australia, Nigeria and Brazil
choice (c) India, Nepal, Sri Lanka and South Africa
(d) France, Germany, Sweden and Switzerland
Q94. A judgment made before all the facts are
known must be called:
(a) Deliberate
(b) Sensible
(c) Premature
(d) Harsh

Q95. Which of the following pairs of words


expresses the same relationship as in ELUSIVE:
CAPTURE?
(a) Sensible: decide
11 | P a g e W W W . E D U T A P . C O . I N QUERY? HELLO@EDUTAP.CO.IN / 8146207241
Visit: www.civilstap.com
For Any Query Mail us: hello@civilstap.com or call us at - (+91)-8146207241
0
inside the earth. People use geothermal heat
SOLUTIONS – PREVIOUS YEAR for bathing, to heat buildings, and to generate
QUESTIONS electricity.
• Geothermal power plants require high-
temperature (300°F (150oC) to 700°F (370oC))
UPSC EPFO APFC – 2012 hydrothermal resources that come from either
dry steam wells or from hot water wells. Hence
Q1. For calculating Body Mass Index (BMI), Statement (I) is correct.
weight of the person (in kg) is divided by the: • To drive steam turbines high pressure, high
(a) Square of the weight (in kg) temperature steam is used. The high pressure
(b) Square of the height (in meters) creates the necessary thrust while the high
(c) Square root of the height (in meters) temperature prevents the steam from
(d) Vitamins intake condensing into water which can be
Answer: B detrimental for the turbine. Hence Statement
Explanation: (II) is correct but not the correct explanation
Body Mass Index is a simple calculation using a of Assertion as high-pressure steam drives
person's height and weight. steam turbines.
• The formula is BMI = W/h2 where W is a
person's weight in kilograms and h2 is their Q3. Gun metal is an alloy of
height in metres squared. (a) Copper, Tin and Zinc
• A BMI of 25.0 or more is overweight, while the (b) Aluminium, Tin and Steel
healthy range is 18.5 to 24.9. (c) Copper, Steel and Zinc
(d) Aluminium, Tin and Zinc
Directions: Each of the next One (2) item consists Answer: A
of two statements, one labeled as the 'Statement Explanation:
(I) and the other as 'Statement (II)'. You are to Gunmetal, also called G Metal is a variety of
examine these two statements carefully and select bronze, formerly used for ordnance.
the answers to these items using the codes given • Modern admiralty gunmetal is composed of 88
below: percent copper, 10 percent tin, and 2 percent
zinc and is used for gears and bearings that are
Codes: to be subjected to heavy loads and low speeds.
(a) Both Statement (I) and Statement (II) are • Hence option A is correct.
individually true, and Statement (II) is the
correct explanation of Statement (I) Q4. If the electrical resistance of a typical
(b) Both Statement (I) and Statement (II) are substance suddenly drops to zero, then the
individually true but Statement (II) is NOT the substance is called :
correct explanation of Statement (I) (a) Semiconductor
(c) Statement (I) is true but Statement (II) is false (b) Conductor
(d) Statement (I) is false, but Statement (II) is True (c) Superconductor
(d) Super semiconductor
Q2. Statement (I): In order to produce electric Answer: C
power from a geothermal reservoir, temperature Explanation:
above 1800C is required. In superconductors, resistance drops suddenly to
Statement (II): To drive steam turbines, high zero at a sufficiently low temperature.
temperature steam is used. • A superconductor is a material that achieves
Answer: B superconductivity, which is a state of matter
Explanation: that has no electrical resistance and does not
Geothermal energy is heat within the earth. The allow magnetic fields to penetrate.
word geothermal comes from the Greek words • An electric current in a superconductor can
geo (earth) and therme (heat). persist indefinitely.
• Geothermal energy is a renewable energy • Superconductivity can only typically be
source because heat is continuously produced achieved at very cold temperatures.
1|P a g e W W W . E D U T A P . C O . I N QUERY? HELLO@EDUTAP.CO.IN / 8146207241
• Prominent examples of superconductors Answer: D
include aluminium, niobium, magnesium Explanation:
diboride, cuprates such as yttrium barium File Transfer Protocol (FTP)
copper oxide and iron pnictides. • The File Transfer Protocol is a standard
• These materials only become superconducting network protocol used for the transfer of
at temperatures below a certain value, known computer files between a client and server on
as the critical temperature. a computer network.
• FTP is built on a client-server model
Q5. The material used for electric fuse is an alloy architecture using separate control and data
of tin and lead. This alloy should have: connections between the client and the server
(a) High specific resistance and low melting point.
(b) Low specific resistance and high melting point. Q8. RAM stands for:
(c) Low specific resistance and low melting point. (a) Random Access Memory
(d) High specific resistance and high melting point. (b) Read Access Memory
Answer: A (c) Random Attribute Memory
Explanation: (d) Random Applicable Memory
The material used for electric fuse is an alloy of tin Answer: A
and lead. Explanation:
• This alloy has high specific resistance and also Random Access Memory (RAM) –
low melting point so that it melts as soon as • A semiconductor storage structure that
large current flows through the wire. Hence accesses temporary data with a random or
option A is correct. direct accessing method.
• The specific resistance is constant for a wire • It is accurately referred to as 'erasable
irrespective of the length or cross-sectional read/write' memory.
area of the wire. • It is also referred to as volatile memory.

Q6. Bancassurance is: Q9. Which of the following is also known as brain
(a) An insurance scheme to insure bank deposits of computer?
(b) An insurance scheme exclusively for bank (a) Monitor
employees (b) Arithmetic Logic Unit (ALU)
(c) A composite financial service offering both (c) Control Unit
bank and insurance products (d) Central Processing Unit (CPU)
(d) A bank deposit scheme exclusively for Answer: D
employees of insurance companies Explanation:
Answer: C Central Processing Unit (CPU)
Explanation: • The Central Processing Unit (CPU) performs
• Bancassurance is an arrangement between a the actual processing of data.
bank and an insurance company allowing the • It is the part of a computer system that
insurance company to sell its products to the interprets and carries out the instructions
bank's client base. contained in the software.
• This partnership arrangement can be • The CPU is generally called by its generic
profitable for both companies. name 'Processor'. It is also known as the brain
• Banks earn additional revenue by selling of computer.
insurance products, and insurance companies
expand their customer bases without Q10. A technique in which data is written to two
increasing their sales force. duplicate disks simultaneously, is called as _____.
(a) Mirroring
Q7. The word FTP stands for: (b) Multiplexing
(a) File Transit Provision (c) Duplicating
(b) File Translate Protocol (d) Copying
(c) File Transfer Provision Answer: A
(d) File Transfer Protocol Explanation:
2|P a g e W W W . E D U T A P . C O . I N QUERY? HELLO@EDUTAP.CO.IN / 8146207241
• Disk mirroring or mirroring is a technique Q13. Three sets of data on comparable situations
used to protect a computer system from loss are available as under:
of data and other potential losses due to disk
failures. Set No. of Mean Standard
• In this technique, the data is duplicated by No. data Value Deviation
being written to two or more identical hard
drives, all of which are connected to one disk 1 9 8 1.6
controller card. 2 12 7 12
• If one hard drive fails, the data can be
3 15 9 1.4
retrieved from the other mirrored hard drives.

Q11. The term 'e-Waste' refers to : Considering all the data sets together, the overall
(a) The files that are deleted and enter the mean value would be:
'Waste-bin' folder in a computer (a) 7.24
(b) The temporary files, folders, links etc. that are (b) 7.66
rarely used in a computer (c) 8.08
(c) The electronic products such as mobiles, PCs (d) 8.50
etc. that are disposed off after their useful life Answer: C
(d) A portal that offers services for collecting Explanation:
household waste If there are multiple datasets, containing some
Answer: C observations, their weighted mean can be
Explanation: calculated as -
• Electronic waste, or e-waste, refers to all
items of electrical and electronic equipment
(EEE) and its parts that have been discarded by
its owner as waste without the intent of re-
use.
• E-waste is also referred to as WEEE (Waste
Electrical and Electronic Equipment),
electronic waste or e-scrap in different regions Where,
and under different circumstances in the x1, x2, …, and x3 are the mean of their respective
world. datasets.
• It includes a wide range of products – almost n1, n2, …, and n3 are the total number of
any household or business item with circuitry observations in each data set.
or electrical components with power or So, in the given question,
battery supply. • x1 = 8, x2 = 7, and x3 = 9
• n1 = 9, n2 = 12, and n3 = 15
Q12. CAD stands for: • Thereby overall mean = (8*9 + 12*7 + 15*9)/
(a) Computer Aided Design (9+12+15)
(b) Computer Application in Design • = (72+84+135)/(36)
(c) Coded Algorithm in Design • =291/36 = 8.08
(d) Compute Advance Design • Hence option C is correct.
Answer: A
Q14. If 20% of P= 30% of Q = 1/6 of R, then P: Q:
Explanation:
R is:
• Computer-Aided Design (CAD) is a computer
technology that designs a product and (a) 2 : 3 : 16
documents the design's process. (b) 3 : 2 : 16
• CAD may facilitate the manufacturing process (c) 10 : 15 : 18
by transferring detailed diagrams of a (d) 15 : 10 : 18
product's materials, processes, tolerances Answer: D
and dimensions with specific conventions for Explanation:
the product in question 20% of P = 30% of Q = 1/6 of R
3|P a g e W W W . E D U T A P . C O . I N QUERY? HELLO@EDUTAP.CO.IN / 8146207241
1/5 of P = 3/10 of Q = 1/6 of R = k Q17. A train travels at a certain average speed
So, P= 5k, Q = (10/3)k, R= 6k for a distance of 63 km. Thereafter it travels a
distance of 72 km with an average speed of 6
So, the ratio of P, Q, and R = 5k: 10/3k: 6k
km/hour more than the original speed. Total
Multiply by 3
P: Q: R = 15: 10: 18 time taken to complete the journey is 3 hours.
What is the original average speed of the train?
(a) 36 km/hour
Q15. A tree increases annually by 1/8th of its
(b) 42 km/hour
height. What will be its height after 2 years, if it
(c) 48 km/hour
stands today 64 cm high?
(d) 54 km/hour
(a) 72 cm
(b) 74 cm Answer: B
(c) 81 cm Explanation:
(d) 85 cm We have to find the original average speed of the
Answer: C train.
Explanation: Total journey completed in 3 hours.
Height of tree today = 64cm We know, distance = speed/time
After 1 year it will increase by 1/8th of its height =
(1/8) *64 = 8. So, total height after 1 year = 64 + 8
Given, that a train travels a distance of 63km at
= 72cm.
an average speed of x km/hr,
After 2nd year again increase of 1/8th of its height
= (1/8) *72 = 9. So, total height after 2 years = 72 Time = 63/x
+ 9 = 81cm Given, that the same train travels a distance of
72km at an average speed of (x+6)km/hr,
Q16. The sides of a triangle GHL are GH = 65 m,
Time = 72/(x+6)
HL75 m and LG = 80 m. What is the area of this
triangle?
(a) 2100 m² So, 3 = (63/x) + 72/(x+6)
(b) 2160 m² Dividing by 3 to both sides,
(c) 2200 m² 1 = 21/x + 24/(x+6)
(d) 2280 m² x (x + 6) = 21(x + 6) + 24(x)
Answer: D x² + 6x = 21x + 126 + 24x
Explanation: By grouping,
The sides of a triangle are 65, 75, and 80m. This is x² + 6x - 21x - 24x = 126
a scalene triangle. x² - 39x - 126 = 0
So, are can be find out by heron’s formula = √{s(s- x² - 42x + 3x - 126 = 0
a)(s-b)(s-c)} x (x - 42) + 3(x - 42) = 0
Where s= semi perimeter, and a, b, c = sides of (x - 42)*(x + 3) = 0
the triangle. Now, x - 42 = 0
So, a= 65m, b= 75m, c= 80m x = 42
Perimieter of triangle = a + b + c = 65 + 75 + 80 = Also, x + 3 = 0
220m x = -3
Semi perimeter = Perimeter/2 = 220/2 = 110m Since the average speed x cannot be negative, So,
x = 42km/hr
Now put values in the formula
= √ {110(110 – 65)(110 – 75)(110 – 80)} Q18. An iron rod of 1 cm diameter and 8 cm
= √ {110*45*35*30} length is drawn into a wire of 18 m length with
= 150*√231
= 2279.80 ~ 2280m2
4|P a g e W W W . E D U T A P . C O . I N QUERY? HELLO@EDUTAP.CO.IN / 8146207241
uniform thickness. The thickness of the wire long and travels at a uniform speed of 36
would be: km/hour in the direction opposite to that of
(a) 1/21 cm Train A. If these trains are crossing at a double-
(b) 1/18 cm track stretch, what is the time taken for the two
(c) 1/15 cm trains to fully clear each other?
(d) 1/12 cm (a) 10 seconds
Answer: C (b) 8 seconds
Explanation: (c) 7.2 seconds
Diameter of the iron rod = 1 cm So, radius of the (d) 6.6 seconds
iron rod = 1/2 cm = 0.5 cm Answer: B
Length of the iron rod = 8 cm Explanation:
We know that, Volume of the cylinder = πr2h = π Length of train A = 75m, speed of train A =
× 0.52 × 8 ……. (i) 54km/hr = 54*1000/60*60 = 15m/s
Length of the wire = 18 m = 1800 cm Length of train B = 125m, Speed of train B =
Volume of the wire = πr2h = π r2 × 1800 ….. (ii) 36km/hr = 36*1000/60*60 = 10m/s
Now they are moving in opposite direction
On equating both the equations So, relative speed will be sum of both
we have π × 0.52 × 8 = π r2 × 1800 So, S = D/T
r2 = 2 /1800 15 + 10 = (75 + 125)/ T
r2 = 1/900 25 = 200/T
r = 1/30 cm T = 8 seconds

Q21. Consider the following statements in


Therefore, the diameter of the wire is 1/15 cm
respect of the Comptroller and Auditor General
(CAG):
Q19. A 60 m long train travels at a uniform 1. Reports on the accounts of the States are
speed of 72 km/hour. It passes non-stop along submitted to the President who shall cause
the 600 m platform of a wayside station. What is these to be placed before the Parliament.
the elapsed time for the train to entirely clear 2. CAG is appointed by the President of India and
the platform? can be removed only on grounds and
procedure similar to those of a Supreme Court
(a) 30 seconds
Judge.
(b) 31 seconds 3. The form in which accounts of the Centre and
(c) 32 seconds States are to be kept is prescribed by CAG.
(d) 33 seconds Which of these statements are correct?
Answer: D (a) 1, 2 and 3
Explanation: (b) 1 and 2 only
Speed = Distance/ Time (c) 2 and 3 only
Speed = 72km/hr = 72*1000/(60*60) = 20m/s (d) 1 and 3 only
Length of train= 60m and length of platform= Answer: C
Explanation:
600m. So, total distance = 660m
• The CAG submits three audit reports to the
Now, President - audit report on appropriation
S = D/T accounts, audit report on finance accounts,
20 = 660/T and audit report on public undertakings.
So, T = 33seconds. - The President lays these reports before
both the Houses of Parliament.
Q20. Train A is 75 m long and travels at a
uniform speed of 54 km/hour. Train B is 125 m
5|P a g e W W W . E D U T A P . C O . I N QUERY? HELLO@EDUTAP.CO.IN / 8146207241
- After this, the Public Accounts Committee
examines them and reports its findings to Q23. Statement (I): The Directive Principles of
the Parliament. State Policy enshrined in the constitution, aim at
- He submits his audit reports relating to the providing the social and economic base of a
accounts of a state to governor, who shall, genuine democracy.
in turn, place them before the state Statement (II): The Directive Principles are merely
legislature. Hence Statement 1 is not directives which the government has to keep in
correct. mind while framing policy and is not enforceable
• The CAG is appointed by the President of India through courts.
by a warrant under his hand and seal. Codes:
- He holds office for a period of six years or (a) Both Statement (I) and Statement (II) are
up to the age of 65 years, whichever is individually true, and Statement (II) is the
earlier. He can resign any time from his correct explanation of Statement (1)
office by addressing the resignation letter (b) Both Statement (I) and Statement (II) are
to the president. individually true but Statement (II) is NOT the
- He can also be removed by the president correct explanation of Statement (1)
on same grounds and in the same manner (c) Statement (I) is true but Statement (II) is false
as a judge of the Supreme Court. Hence (d) Statement (I) is false, but Statement (II) is True
statement 2 is correct. Answer: B
• CAG advises the President with regard to Explanation:
prescription of the form in which the accounts • The Directive Principles constitute a very
of the Centre and the states shall be kept. comprehensive economic, social and political
Hence statement 3 is correct. programme for a modern democratic State.
- They aim at realising the high ideals of
Q22. Statement (I): Every State in India has a High justice, liberty, equality and fraternity as
Court in its (State’s) territory. outlined in the Preamble to the
Statement (II): As per Constitution of India, each Constitution. Thus, statement 1 is correct.
State is provided to have a high Court. • The phrase ‘Directive Principles of State Policy’
Codes: denotes the ideals that the State should keep
(a) Both Statement (I) and Statement (II) are in mind while formulating policies and
individually true, and Statement (II) is the enacting laws. These are the constitutional
correct explanation of Statement (1) instructions or recommendations to the State
(b) Both Statement (I) and Statement (II) are in legislative, executive and administrative
individually true but Statement (II) is NOT the matters.
correct explanation of Statement (1) - The Directive Principles are non-justiciable
(c) Statement (I) is true but Statement (II) is false in nature, that is, they are not legally
(d) Statement (I) is false, but Statement (II) is True enforceable by the courts for their
Answer: D violation. Thus, statement 2 is correct.
Explanation:
• At present, there are 24 high courts in the Q24. Which one of the following Institutions was
country. NOT set up by the Constitution of India?
- Out of them, four are common high (a) Union Public Service Commission
courts. Delhi is the only union territory that (b) Election Commission
has a high court of its own (since 1966). (c) Planning Commission
Hence statement (I) is false. (d) Union Judiciary
• The Constitution of India provides for a high Answer: C
court for each state, but the Seventh Explanation:
Amendment Act of 1956 authorised the Planning Commission was non – constitutional
Parliament to establish a common high court and non – statutory body established by an
for two or more states or for two or more executive resolution
states and a union territory. Hence statement
(II) is true.
6|P a g e W W W . E D U T A P . C O . I N QUERY? HELLO@EDUTAP.CO.IN / 8146207241
Q25. The following provision, "Protection of 3. The elected members of the legislative
monuments and places and objects of national assemblies of the Union Territories of Delhi
importance" is a: and Puducherry.
(a) Fundamental Right guaranteed by the • Elected members of Union Council are elected
Constitution of India MPs and hence eligible to participate in
(b) Directive Principle of State Policy of the Presidential Election.
Constitution of India
(c) Fundamental Duty laid down by the Q28. The responsibility of preparation of
Constitution of India electoral rolls in India rests with:
(d) Presidential order in accordance with the (a) The Parliament
Constitution of India (b) The Local Administration
Answer: B (c) The Election Commission
Explanation: (d) The Returning Officer
• Article 49 of the Indian Constitution in the Answer: C
Directive Principle of State Policy directs the Explanation:
state to protect monuments, places and One of the main responsibility of Election
objects of artistic or historic interest which Commission of India is to prepare and periodically
are declared to be of national importance. revise electoral rolls and to register all eligible
voters.
Q26. The Seventh schedule of the Constitution of
India contains provisions regarding: Q29. At the time of becoming the Prime Minister
(a) Scheduled Languages of our country one:
(b) Oaths and affirmations (a) Must be a member of one of the Houses of the
(c) Administration of Tribal areas Parliament. If not, must become a member of
(d) Union, State, Concurrent Lists one of the Houses within Six months.
Answer: D (b) Need not necessarily be a member of one of
Explanation: the Houses of Parliament but must get elected
• Seventh Schedule of the Indian Constitution as a member of Lok Sabha within Six months.
deals with the division of powers between the (c) Must be a member of one of the Houses of
Union and the States in terms of List I (Union Parliament prior to appointment.
List), List II (State List) and List III (Concurrent (d) Must be a member of Rajya Sabha.
List). Answer: B
Explanation:
Q27. The President of India is elected by the • The Supreme Court held that a person who is
members of an electoral college consisting of not a member of either House of Parliament
elected members of: can be appointed as Prime Minister for six
1. Both Houses of Parliament months, within which, he should become a
2. The Legislative Assemblies of the States member of either House of Parliament;
3. Union Council of Ministers otherwise, he ceases to be the Prime Minister.
Codes:
(a) 1 only Q30. Match List-I with List-II and select the
(b) 2 only correct answer using the code given below the
(c) 1 and 2 lists:
(d) 2 and 3 List-I (Commission)
Answer: All of the above A. Second Administrative Reforms Commission
Explanation: B. Second Commission on Centre State relations
The President is elected not directly by the people C. Thirteenth Finance Commission
but by members of electoral college consisting of: D. National Commission for the Review of the
1. The elected members of both the Houses of working of the Constitution
Parliament; List-II (Chairperson)
2. The elected members of the legislative 1. Vijay Kelkar
assemblies of the states; 2. Justice M.N. Venkatachaliah
7|P a g e W W W . E D U T A P . C O . I N QUERY? HELLO@EDUTAP.CO.IN / 8146207241
3. Justice Madan Mohan Punchi Answer: A
4. M. Veerappa Moily Explanation:
Codes: Adjournment Motion is introduced in the
A B C D Parliament to draw attention of the House to a
(a) 4 3 1 2 definite matter of urgent public importance and
(b) 2 3 1 4 needs the support of 50 members to be admitted.
(c) 4 1 3 2
(d) 2 1 3 4 Q33. Consider the following statements
Answer: A regarding limitations on the authority of the
Explanation: Indian Parliament:
• The second ARC constituted in 2005 was 1. Most of the important laws can be introduced
chaired by M. Veerappa Moily. in the Parliament with the prior consent of the
• The Punchhi Commission was constituted by President.
the Government of India in 2007 as a 2. Parliament has to operate within the
Commission on Centre-State relations. It was jurisdiction earmarked by the Constitution.
chaired by Justice Madan Mohan Punchhi who Which of these statements is/are correct?
was formerly the Chief Justice of India. (a) Both 1 and 2
• The Thirteenth Finance Commission of India (b) Neither 1 nor 2
was constituted by the under the (c) 1 only
chairmanship of Vijay L. Kelkar on 13 (d) 2 only
November 2007. Answer: D
• The National Commission to review the Explanation:
working of the Constitution (NCRWC) also • Only certain bills like Money Bills, Financial Bill
known as Justice Manepalli Narayana Rao under Article 117 (1) can be introduced in the
Venkatachaliah Commission was set up by a Parliament with the prior consent of the
resolution of the NDA Government of India led President. Not most of the important laws like
by Atal Bihari Vajpayee on 22 February 2000 President assent before their introduction.
for suggesting possible amendments to the Hence statement 1 in not correct.
Constitution of India. • The Constitution is the fundamental law of the
land in our country. It has defined the
Q31. What is common to Sardar Hukum Singh, authority and jurisdiction of all the three
Jagjivan Ram, Inderjit Gupta, Somnath Chatterjee organs of the Union government and the
and Seth Govind Das? nature of interrelationship between them.
(a) They were Speakers of the Lok Sabha Hence, the Parliament has to operate within
(b) They were Deputy Speakers of the Lok Sabha the limits prescribed by the Constitution.
(c) They were Pro-tem Speakers of the Lok Sabha Hence statement 2 is correct.
(d) None of the Above
Answer: D Q34. Consider the following statements:
Explanation: 1. By the early nineteenth century British carried
• There is nothing common to above names. out detailed surveys.
2. The effort was to know the topography, the
Q32. The purpose of Adjournment motion in our soil quality, the flora, the fauna, the logical
Parliament is: histories and the cropping pattern.
(a) To allow a discussion on a definite matter of Which of these statements is/are correct?
urgent public importance. (a) 1 only
(b) To let opposition members, collect (b) 2 only
information from concerned Ministers. (c) Both 1 and 2
(c) To allow a reduction of specific amount in the (d) Neither 1 nor 2
demand of grant. Answer:
(d) To postpone the proceedings to check the Explanation:
inappropriate or the violent behaviour on the The British gave much importance to the practice
part of some members. of surveying because they believed that a country
8|P a g e W W W . E D U T A P . C O . I N QUERY? HELLO@EDUTAP.CO.IN / 8146207241
had to be properly known before it could be The major sources for the agrarian history of the
effectively administered. Therefore, they carried 16th and early 17th centuries are chronicles and
out detailed surveys by the early 19th century in documents from the Mughal court.
order to map the entire country:
• They conducted revenue surveys in villages. Q37. Which of the following is a correct
• They made efforts to know the topography, statement about both the Mughal and Ottoman
the soil quality, the flora, the fauna, the local empires in the sixteenth century?
histories and the cropping pattern. (a) Both of these empires had powerful navies
• They also introduced census operations, held that engaged European navies.
at the interval of every ten years from the end (b) Both of these empires expanded through the
of the 19th century. use of gunpowder weapons and extensive
• They prepared detailed records of the number bureaucracies.
of people in all the provinces of India, noting (c) Both of these empires gave little monetary
information on castes, religions and support to artistic and cultural endeavours.
occupation separately. (d) In both of these empires the majority of the
• The British also carried on several other people were Muslims.
surveys such as botanical surveys, zoological Answer: B
surveys, archaeological surveys, forest Explanation:
surveys, etc. In this way, they gathered all the The Ottoman, Safavid, and Mughal Empires are
facts that were essential for administering a called the Gunpowder Empires because they had
country. strong military powers that utilized gunpowder
Hence option C is the correct answer. and innovative artillery. That successfully helped
them to expand and protect their territory.
Q35. Which is the correct meaning of the term • The Gunpowder Empires were significant due
'rule of primogeniture? to their impact on the trade and military. Due
(a) Favourite son inheriting his father's estate to their power, these empires controlled the
(b) Eldest son inheriting his father's estate important region of Eurasia that oversaw trade
(c) Division of the inheritance amongst all the between Europe and Asia.
sons
(d) Division of the inheritance amongst all the Q38. Consider the following:
sons and daughters 1. Rowlatt Act movement
Answer: B 2. Kheda movement
Explanation: 3. Champaran
Primogeniture is a system of inheritance in which 4. Ahmedabad mill strike
a person's property passes to their firstborn What is the correct chronological order of the
legitimate child upon their death. above-mentioned Gandhian movements?
• The term comes from the Latin "primo” which (a) 3, 4, 2 and 1
means first, and “genitura” which relates to a (b) 2, 3, 1 and 4
person's birth. (c) 3, 2, 4 and 1
(d) 2, 1, 3 and 4
Q36. Which of the following would be the most Answer: C
useful source of evidence for research about the Explanation:
agrarian history of the sixteenth and early • On 6 April 1919, Mahatma Gandhi started a
seventeenth centuries? non-violent Satyagraha against the unjust
(a) Accounts of contemporary travellers Rowlatt Act passed by the British government.
(b) Individual revenue records • The Champaran Satyagraha of 1917 was the
(c) Chronicles and documents from the Mughal first satyagraha movement led by Mahatma
court Gandhi in British India.
(d) Records of the East India company • Kheda Satyagraha of March, 1918 is known to
Answer: C be the first non-cooperation movement led
Explanation: by Mahatma Gandhi. This satyagraha was
centred on the peasant-Patidar community of
9|P a g e W W W . E D U T A P . C O . I N QUERY? HELLO@EDUTAP.CO.IN / 8146207241
Kheda, who refused to agree to a 23 percent • In cultural anthropology and cultural
tax hike imposed on them despite a disastrous geography, cultural diffusion is the spread of
crop failure and outbreak of plague and cultural items—such as ideas, styles, religions,
cholera. technologies, languages — between
• Ahmedabad Mill Strike of 1918 is considered individuals, whether within a single culture or
the first hunger strike led by Gandhi. In 1918, from one culture to another.
Mahatma Gandhi intervened in a dispute
between Ahmedabad workers and Q41. Consider the following statements about
millowners. heritage :
- In March 1918, Gandhi intervened in a 1. Heritage is that which has been or may be
dispute between Ahmedabad cotton mill inherited.
owners and workers over the cessation of 2. Heritage is anything given or received to be a
the plague bonus. proper possession.
So the correct sequence is: Champaran Which of these statements is/are correct?
movement, Kheda Satyagraha, Ahmedabad mill (a) 1 only
strike, Rowlatt Act. (b) 2 only
(c) Both 1 and 2
Q39. Which one of the following sets of industries (d) Neither 1 nor 2
were among first modern industries in India? Answer: A
(a) Tea, Cotton and Sugar
(b) Jute, Silk and Cotton Q42. Consider the following statements
(c) Cotton, Jute and Coal Mines regarding human development:
(d) Sugar, Silk and Steels 1. Skills may be harnessed in the society through
Answer: C participation of NGOs.
Explanation: 2. To fill the internal gaps in the system, reviews
• Cotton, Jute and Coalmines are the sets of and international experiences can prove to be
industries were among first modern industries helpful.
in India. Which of these statements is/are correct?
• The emergence of the cotton textile (a) 1 only
industry started with the introduction of the (b) 2 only
first machinery in Kolkata in the year 1818 at (c) Neither 1 nor 2
fort Gloster. Followed by this, another major (d) Both 1 and 2
event is the installation of machinery in Answer: C
Mumbai in the year 1854 by Nanabhai Davar. • NGOs, through capacity building, develop
• The first jute mill was established at Rishra, on community capacities such as ability, skill and
the River Hooghly near Calcutta in 1855 when knowledge of mobilizing resources, planning
Mr. George Acland brought jute spinning and evaluating community initiation and
machinery from Dundee. solving problems to gain the mastery over
• Coal mining started in 1774 by M/s Sumner their lives. Hence, statement 1 is correct.
and Heatly of East India Company in the • Reviews of ongoing schemes and initiatives
Raniganj Coalfield along the Western bank of and learnings from international experiences
river Damodar. can help in pinpointing the lacunae in the
Hence Option C is the correct answer. internal systems. Hence, statement 2 is
correct.
Q40. Which term is used to describe the spread
of Buddhism from India through Southeast Asia? Q43. Biome, largest recognizable assemblage of
(a) Social mobility animals and plants on the Earth, is controlled
(b) Cultural diffusion mainly by:
(c) Ethnocentrism (a) Biological activity
(d) Interdependence (b) Landforms
Answer: B (c) Climate
(d) Soil
10 | P a g e W W W . E D U T A P . C O . I N QUERY? HELLO@EDUTAP.CO.IN / 8146207241
Answer: C (c) 2 and 4 only
The largest recognizable assemblage of plants and (d) 1 and 3 only
animals on the Earth. The distribution of the Answer: D
biomes is controlled mainly by climate. • Jowar and Arhar can be grown without
irrigation facilities.
Q44. Despite having huge coal reserve in our • In India, jowar plant is grown in areas with less
country, why do we import millions of tons of than 100 cm rainfall and temperature ranging
coal? from 26° to 33° C. After wheat, the sorghum
1. It is policy of Govt. of India to save its own coal or jowar is the grain with the highest
reserve for future use and import now from cultivable land in the country.
other countries for present use. • Arhar crop requires average rainfall of 600-
2. Most of the thermal plants in our country are 650 mm with moist conditions for the first
coal based and they are unable to get eight weeks and drier conditions during
sufficient coal supply from our coal mines. flowering and pod development stage ,this
3. Steel plants require large quantities of coking will result in highly successful crop. Rains
coal which we do not have, coking coal is to be during flowering results in poor pollination.
imported from other countries. • Arhar crop grown successfully in summer,
Select the correct answer using the codes given rainy and winter season, April-Summer, June-
below: Kharif or rainy, September-Rabi or winter
(a) 1, 2 and 3 season.
(b) 2 and 3 only
(c) 1 and 2 only Q46. Match List-1 with List-II and select the
(d) 1 and 3 only correct answer using the code given below the
Answer: B lists:
• There is no such government policy in India List-I List-II
which aims to save local coal for future. A. Buxa Tiger Reserve 1. Rajasthan
Hence, statement 1 is incorrect. B. Periyar Tiger 2. Gujarat
• Most of the thermal plants in our country are Reserve
coal based and they are unable to get C. Sariska National 3. West Bengal
sufficient coal supply from our coal mines. Park
Thus, to meet the shortfall between coal D. Wild Ass Sanctuary 4. Kerala
demand and domestic coal supply, coal has to Code:
be imported from other countries. Hence, A B C D
statement 2 is correct. (a) 2 1 4 3
• Coking coal is essential raw material in steel (b) 3 4 1 2
manufacture. But India does not have (c) 2 4 1 3
significant local reserves of coking grade coal. (d) 3 1 4 2
Thus we have to import coking coal from Answer: B
other countries. Hence, statement 3 is • Buxa Tiger Reserve is a Tiger reserve and
correct. National Park in northern West Bengal.
• Periyar National Park and Wildlife Sanctuary
Q45. A cultivator has about two hectares rain fed (PNP) is a protected area located in the
land without irrigation facilities. Which type of districts of Idukki and Pathanamthitta in
crops would he like grow? Kerala.
1. Jowar • Sariska Tiger Reserve is a tiger reserve in
2. Cotton Alwar district, Rajasthan.
3. Arhar
• Indian Wild Ass Sanctuary also known as the
4. Potatoes
Wild Ass Wildlife Sanctuary is located in the
Select the correct answer using the codes given
Little Rann of Kutch in the Gujarat
below:
(a) 1, 2, 3 and 4
(b) 1, 2 and 3 only
11 | P a g e W W W . E D U T A P . C O . I N QUERY? HELLO@EDUTAP.CO.IN / 8146207241
Q47. Which one of the following statements is (a) Destruction of their nesting sites by new
true about red soil? invasive species
(a) It is rich in humus. (b) A drug used by cattle owners for treating their
(b) It is rich in potash. diseased cattle
(c) It is rich in iron compounds. (c) Scarcity of food available to them
(d) It is derived from volcanic origin. (d) A widespread, persistent and fatal disease
Answer: C amongst them
• This type of soil is formed as a result of Answer: B
weathering of metamorphic and igneous • Diclofenac, a drug used by cattle owners for
rocks. The red colour of the soil comes from treating their cattle is very harmful for the
the high percentage of iron content. The soil’s vultures.
texture varies from being sandy to clayey, but • After the vultures consume the dead carcass
it is mainly loamy. It lacks phosphate, humus of cattle with diclofenac in their bodies, the
and nitrogen content. The red soil is found in organs of vultures fail, resulting in their death.
regions such as Tamil Nadu, Madhya Pradesh,
Jharkhand, Odisha, some parts of Karnataka Q51. NABARD means:
and southeast Maharashtra. (a) National Bank for Agriculture and Rural
Development
Q48. "Kyoto Protocol', an agreement signed by (b) National Agricultural Bank and Rural
various countries, is associated with Development
(a) International trade (c) National Agricultural Board and Rural
(b) Deep sea oil and mineral exploration Development
(c) Clean environment and climate change (d) National Board for Agricultural and Rural
(d) Building common food stock to save human Area Development
being from any natural disaster Answer: A
Answer: C Explanation
Kyoto Protocol is associated with reductions in • NABARD stands for National Agriculture
emission of Greenhouse Gases. Hence, option C is and Rural Development.
the correct answer. • It came into existence on 12 July 1982 by
transferring the agricultural credit
Q49. Which of the following can be threats to the functions of RBI and refinance functions of
biodiversity of a geographical area? the then Agricultural Refinance and
1. Global warming Development Corporation (ARDC).
2. Fragmentation of habitats
3. Invasion of alien species Q.52 Which of the following is NOT a
4. Promotion of vegetarianism recommendation of the National Cooperative
(a) 1, 2 and 3 only Farming Advisory Board for putting cooperative
(b) 1, 2 and 4 only farming on a sounder footing?
(c) 3 and 4 only (a) Every Society should have a definite
(d) 1, 2, 3 and 4 programme for total pooling of lands.
Answer: A (b) Financial assistance should be given only
• Global warming, Fragmentation of habitats, for purchase of new land.
invasion of alien species etc. are well known (c) State Governments should give priority to
causes responsible for decline in biodiversity. revitalization of the existing societies.
• Promotion of vegetarianism is not responsible (d) Joint farming must be practiced with
for decline in biodiversity. respect to all lands of the society.
Answer: B
Q50. Vultures, which were very common in our Explanation
countryside a few years ago, are rarely seen
nowadays. This is attributed to one of the
following:

12 | P a g e W W W . E D U T A P . C O . I N QUERY? HELLO@EDUTAP.CO.IN / 8146207241


Q.53 Three States which have Human • The symbol is a perfect blend of Devnagri
Development Index (HDI) higher than China are: Ra and Roman R, the letters derived from
(a) Kerala, Maharashtra and Madhya the word 'Rupiah' in 'Hindi' and 'Rupees'
Pradesh in 'English'.
(b) Kerala, Punjab and Maharashtra
(c) Maharashtra, Madhya Pradesh and Q.56 The regional project by name 'CASA-1000*
(d) Kerala, Madhya Pradesh and Gujarat (Central Asia-South Asia-1000), funded by World
Answer: D Bank, pertains to the field of:
(a) Eradication of HIV AIDS
Q.54 Match List-I with List-II and select the (b) Roads and Buildings
correct answer using the code given below the (c) Electricity
lists: (d) Eradication of illiteracy
List-I (Chairperson) Answer: C
A. D.M. Dharmadhikari Explanation
B. Anil Kakodkar • About CASA 1000
C. B.N. Srikrishna • Initiative of World Bank.
D. Adi Godrej • The CASA-1000 project is the first step
List-II (Issue) towards creating the Central Asia-South
1. National Corporate Governance Policy Asia Regional Electricity Market
2. Air India Indian Airlines merger (CASAREM), leveraging Central Asia's
3. Indian Railways High Level Safety Review significant energy resources to help
Committee alleviate South Asia's energy shortages on
4. Financial Sector Legislative Reforms a mutually beneficial basis.
Commission
Code: Q57. Who won the Golden Boot Award in the
A B C D recently concluded UEFA Euro 2012?
(a) 1 3 4 2 (a) Fernando Torres (Spain)
(b) 2 3 4 1 (b) Mario Balotelli (Italy)
(c) 1 4 3 2 (c) Cristiano Ronaldo (Portugal)
(d) 2 4 3 1 (d) Mario Gomez (Germany)
Answer: B Answer: A
Explanation Explanation
Chairperson Issue • Spain striker Fernando Torres has won
D.M. Dharmadhikari Air India Indian Airlines the Euro 2012 Golden Boot award.
merger
Anil Kakodkar Indian Railways High Q.58 Match List-1 with List-II and select the
Level Safety Review correct answer using the code given below the
Committee lists:
B.N. Srikrishna Financial Sector List-I (Person)
Legislative Reforms A. Steve Jobs
Commission B. Sheryl Sandberg
Adi Godrej National Corporate C. Larry Page
Governance Policy D. Ross Levinsohn
List-II (Company)
Q.55 The newly introduced rupee symbol is based 1. Google
on: 2. Apple
(a) Roman and Greek script 3. Yahoo
(b) Persian and Dravidian script 4. Facebook
(c) Roman and Devanagari script Code:
(d) Greek and Devanagari script A B C D
Answer: C (a) 2 1 4 3
Explanation (b) 3 1 4 2
13 | P a g e W W W . E D U T A P . C O . I N QUERY? HELLO@EDUTAP.CO.IN / 8146207241
(c) 2 4 1 3 • Justice Dalveer Bhandari was elected to
(d) 3 4 1 2 International Court of Justice.
Answer: D • He was the former judge of the Supreme
Explanation Court of India.
Person Company
Steve Jobs Apple Q.61 Match List-I with List-II and select the
Sheryl Sandberg Facebook correct answer using the code given below:
Larry Page Google List-I (Head)
A. Jim Yong Kim
Ross Levinsohn Yahoo
B. Kamlesh Sharma
C. Christine Lagarde
Q.59 Match List-I with List-II and select the D. Catherine Day
correct answer using the code given below PSC List-II (Institution)
the lists: 1. President Word Bank
List-I (Book) 2. Secretary-General Commonwealth
A. Rahul Dravid : Timeless Steel 3. Managing Director, IMF
B. An Ashes Summer 4. Secretary-General of the European
C. Sachin : A Hundred Hundreds Now Commission
D. Opening Up : My Autography Code:
List-II (Author) A B C D
1. Steve Waugh and Nasser Hussain (a) 3 4 2 1
2. Greg Chappell (b) 1 4 2 3
3. Mike Atherton (c) 3 2 4 1
4. V. Krishnaswamy (d) 1 2 4 3
Code: Answer: D
A B C D Explanation
(a) 3 4 1 2
Person Institution
(b) 2 4 1 3
(c) 3 1 4 2
Jim Yong Kim President World Bank
(d) 2 1 4 3 Present World Bank
Answer: D President: David
Explanation Malpass
Women Area Kamlesh Sharma Secretary
Rahul Dravid : Rahul Dravid/Sambit Commonwealth Games
Timeless Steel Bal Present
An Ashes Summer Steve Waugh Commonwealth
Sachin : A Hundred V. Krishnaswamy Secretary: General-
Hundreds Now Baroness Patricia
Opening Up : My Michael Atherton Scotland
Autography Christine Lagarde Secretary General of
European Union
Q.60 Why was Justice Dalveer Bhandari in the Present Secretary
news recently? General of European
(a) He became member of UN Human Rights
Union: Marija
Council.
Pejčinović Burić
(b) He was elected to the International Court
of Justice.
Catherine Day Managing Director IMF
(c) He became Director General of World Present Managing
Trade Organization. Director of IMF:
(d) He became Secretary-General of SAARC. Kristalina Georgieva
Answer: B
Explanation
14 | P a g e W W W . E D U T A P . C O . I N QUERY? HELLO@EDUTAP.CO.IN / 8146207241
Q62. It is said that, in order to control inflation, d) Offer commercial banks more credit in the
foreign inflow needs to be sterilized. Sterilization open market
here refers to Answer: B
(a) Ensuring that counterfeit currency does not Explanation:
enter circulation • If the RBI adopts an expansionist open market
(b) Ensuring that black money is accounted for operations policy, this means it will Buy
(c) Withdrawing equivalent local currency to securities from non-government holders. This
maintain a desirable rate of exchange will lead to increase in money supply in the
(d) Compliance with import-export regulations economy as for the purchase of government
Answer: C securities, RBI needs to pay INR in
Explanation: consideration. Hence, B is the right answer.
• Sterilisation in the context of monetary policy
refers to the activity of the RBI of taking away Q65. Structural Planning refers to:
the excess money supply created due to its (a) Centralized planning
foreign exchange market intervention. Hence, (b) Laying down broad goals and strategies
C is the right answer. (c) Changing existing institutions or creating new
ones
Q63. Which of the following are the main causes (d) Fixing flexible targets
of the slow rate of growth of per capita income in Answer: C
India? Explanation:
1. High rate of capital formation • In structural planning, the present social and
2. High level of fiscal deficits economic structure is changed and a new
3. High rate of growth of population structure emerges. In the developing
a) 1, 2 and 3 countries, there is a structure planning. Big
b) 1 and 2 only economic and social changes are brought
c) 1 and 3 only about to usher into a new system. For
d) 2 and 3 only instance, shift from capitalist to socialist
Answer: D economy can be called a structural change.
Explanation: Structural planning can help in accelerating
• Slow rate of capital formation in the economy the pace of economic development. The
leads to slow growth rate of national income Communist countries like Russia and China
which in turn leads to slow rate of growth of followed structural planning. Hence, C is the
per capita income in India. Hence, statement right answer.
1 is incorrect.
• High level of fiscal deficits leads to slow Q66. Which of the following is NOT one of the
growth rate of GDP which in turn leads to slow features of the Special Economic Zones (SEZ)
rate of growth of per capita income in India. being set up for promoting exports?
Hence, statement 2 is correct. (a) Foreign workers will be allowed free entry
without Visa restrictions
• Per capita income is national income divided
by the total population of country. Therefore, (b) The SEZ area will be treated as foreign territory
when there is high rate of growth of for trade operations, duties and tariff
population, this will automatically lead to (c) There will be no routine examination by
slow rate of growth of per capita income in customs authorities of import/export cargo
India. Hence, statement 3 is correct. (d) No license is required for import into the zone
Answer: A
• Therefore, D is the right answer.
Explanation:
• Allowing foreign workers free entry without
Q64. If the RBI adopts an expansionist open
Visa restrictions is not the feature the Special
market operations policy, this means it will:
Economic Zones (SEZ) being set up for
a) Sell securities in the open market
b) Buy securities from non-government holders promoting exports. This may create a serious
c) Openly announce to the market that it intends challenge with regard to internal security.
to expand credit Hence, A is the right answer.
15 | P a g e W W W . E D U T A P . C O . I N QUERY? HELLO@EDUTAP.CO.IN / 8146207241
Q69. Which of the following can be termed an
Q67. Which one of the following expresses the infrastructural bottleneck in the development of
relation between normal price and market price? India's economy?
(a) Market price is greater than normal price (a) The federal nature of Indian Polity.
(b) Market price is equal to normal price (b) Existence of a large variety of financial
(c) Market price tends to be equal to normal price institutions
(d) Market price is lesser than normal price (c) Delay in the administration of justice relating
Answer: A to land acquisition and displacement
Explanation: (d) The volatility of the Indian rupee
• Market price is that price which prevails in a Answer: C
market on a single day or on very few days. It Explanation:
is a very short-period price which prevails at a • Delay in the administration of justice relating
particular time. On the other hand, normal to land acquisition and displacement leads to
price is that price which tends to prevail in the delay in starting of development projects in
long-run. It is a price which has a tendency to the country. Therefore, it becomes
prevail over a period of time. Since market infrastructural bottleneck in the development
price caters the inflation, therefore, it is of India's economy. Hence, C is the right
greater than normal price. Hence, A is the answer.
right answer.
Q70. Which of the following statements is true
Q68. Inflation can be controlled by: about Industrial Policy since 1991?
(a) Increase in wages (a) Only 5 industries related to security, strategic
(b) Decrease in taxation and environmental concerns require industrial
(c) Reduction in public expenditure License
(d) Making the rupee dearer (b) An investor need not file an industrial
Answer: C entrepreneur Memorandum
Explanation: (c) There is no reservation of products for
• Increase in wages leads to increase in production in small scale sectors
disposable income of people which will lead (d) The number of industries reserved for public
to increase in demand and consequent sector has been enhanced
increase in inflation. Hence, A is incorrect. Answer: B
• Decrease in taxes leads to increase in Explanation:
disposable income of people as now they • There are only 4 industries at present
need to pay less taxes which will lead to related to security, strategic and
increase in demand and consequent increase environmental concerns, where an
in inflation. Hence, B is incorrect. industrial license is currently required -
• Reduction in public expenditure leads to Electronic aerospace and defence
decrease in income of people as now they will equipment, Specified hazardous chemicals,
have less source of income which they were Industrial explosives, Cigars and cigarettes
getting earlier, e.g., MGNREGA wages. This of tobacco and manufactured tobacco
will lead to decrease in demand and substitutes. Hence, A is incorrect.
consequent decrease in inflation. Hence, C is • Industrial Entrepreneur Memorandum (IEM)
correct. is required by the government for statistical
• Making INR dearer means there is a high purposes only, similar to Foreign Assets and
demand of INR. When demand is high then Liabilities statement filed by the enterprise
people will demand more to spend more and to the RBI. Through IEM, the government
that will lead to increase in inflation. Hence, D conduct a limited post facto check to see
is incorrect. whether the proposed manufacturing
• Therefore, C is the right answer. activities require an industrial license or not.
Hence, B is correct.
• The new industrial policy announced on
24th July, 1991 as part of the process of
16 | P a g e W W W . E D U T A P . C O . I N QUERY? HELLO@EDUTAP.CO.IN / 8146207241
economic reforms has stated that the policy • Unions and their tactics are still playing a
of reservation of products in the small-scale crucial role irrespective of the forces of
industries sector will continue. Hence, C is globalisation. Hence statement (II) is
incorrect. incorrect.
• Government stakes in Public Sector • Therefore, C is the right answer.
Enterprises were reduced to enhance their
efficiency and competitiveness. Hence, D is Q73. The main objective of the Minimum Wage
incorrect. Act, 1948 is to safeguard the interests the
• Therefore, B is the right answer. workers engaged in:
(a) Unorganized sector
Q71. In which of the following Acts, housing (b) Organized sector
facility is a statutory provision? (c) Industrial sector
(a) The Plantations Labour Act, 1951 (d) Agricultural sector
(b) The Factories Act, 1948 Answer: A
(c) The Mines Act, 1952 Explanation:
(d) None of the above • The Minimum Wages Act, 1948 is an Act to
Answer: A provide for fixing minimum rates of wages in
Explanation: certain employments related to the
• Housing facility is a statutory provision under unorganised sector. Hence, A is the right
the Plantations Labour Act, 1951. In the Act, it answer.
is mentioned that it shall be the duty of every
employer to provide and maintain for every Q74. Match List-I with List-II and select the
worker and his family residing in the correct answer using the code given below the
plantation necessary housing lists:
accommodation. Hence, A is the right List-1 (Contribution)
answer. A. Industrial Welfare Movement
B. Human Relations Thought
Q72. Statement (I): Industrial relation is currently C. Concept of Third Force
more influenced by the external market forces D. Ahmedabad Experiment
than the power play between employers and
employees. List-II (Contributor)
Statement (II): The forces of globalization have 1. Charles A Myer
made competition so imperative that unions and 2. A.K. Rice
their tactics like stopping productivity no more 3. Robert Owen
hold good 4. Elton Mayo

Codes:
(a) Both Statement (I) and Statement (II) are Code:
individually true, and Statement (II) is the A B C D
correct explanation of Statement (1) (a) 2 1 4 3
(b) Both Statement (I) and Statement (II) are (b) 3 1 4 2
individually true but Statement (II) is NOT the (c) 2 4 1 3
correct explanation of Statement (1) (d) 3 4 1 2
(c) Statement (I) is true but Statement (II) is false Answer: D
(d) Statement (I) is false, but Statement (II) is True Explanation:
Answer: C • Robert Owen was associated with Industrial
Explanation: welfare Movement
• Due to globalisation, Industrial relation is • Elton Mayo was associated with Human
currently more influenced by the external Relations Thought
market forces than the power play between • Charles A Myer was associated with Concept
employers and employees. Hence, statement of Third Force
(I) is correct.
17 | P a g e W W W . E D U T A P . C O . I N QUERY? HELLO@EDUTAP.CO.IN / 8146207241
• A.K Rice was associated with Ahmedabad • Since the release of current series of CPI-IW
Experiment. with base year 2001=100, various Central
• Hence, D is the right answer. Trade Union Organisations had been pressing
for a review of these Index Numbers by a high
Q75. Match List-I with List-II and select the powered tripartite Committee. Accordingly,
correct answer using the code given below the the Ministry of Labour & Employment,
lists: Government of India constituted an Index
List-I (Board/Committee) Review Committee under the Chairmanship of
A. First National Commission on Labour, 1969 Prof. G.K. Chadha.
B. Wage Board for Working Journalists, 2009 • Therefore, A is the right answer.
C. Second National Commission on Labour,
2002 Q76. In which part of the Indian Constitution,
D. Index Review Committee, 2009 Worker's participation in Management has been
incorporated?
List-II (Chairperson) (a) The Preamble
1. G.K. Chadha (b) The Fundamental Rights
2. Ravindra Verma (c) The Directive Principles of State Policy
3. P.B. Gajendragadkar (d) None of the above
4. G.R. Majithia Answer: C
Code: Explanation:
A B C D • Article 43 A of the Constitution of India has
(a) 3 4 2 1 provided for worker’s participation in
(b) 1 4 2 3 management in these words: “The State shall
(c) 3 2 4 1 take steps, by suitable legislation, or in any
(d) 1 2 4 3 other way, to secure the participation of
Answer: A workers in management of undertakings,
Explanation: establishments or other organisations
• The first National Commission on Labour engaged in an industry”. Hence, C is the right
was set up on 24 December 1966 under the answer.
Chairmanship of Justice P.B. Gajendragadkar.
The Commission submitted its report in Q77. How does National Rural Livelihood Mission
August , 1969 after detailed examination of all (NRLM) seek to improve livelihood options of
aspects of labour problems, both in the rural poor?
organised and unorganised sectors. 1. By setting up a large number of new
• The Government of India constituted two manufacturing industries and agribusiness
wage boards (Majithia Wage Boards), one for centres in rural areas.
working journalists and other for non- 2. By strengthening Self Help Groups (SHG) and
journalists newspaper employees in 2007 as providing skill development.
sixth Wage Board under the Chairmanship of 3. By supplying seeds, fertilizers, diesel pump-
Justice Majithia as per the provisions of The sets and micro irrigation equipment free of
Working Journalists and Other Newspaper cost to farmers.
Employees (Conditions of Service) and a) 1, 2 and 3
Miscellaneous Provisions Act, 1955. The b) 2 only
Majithia Wage Boards submitted their final c) 3 only
report to the Government of India on 31st d) 1 only
Dec., 2010. Answer: B
• The second National Commission on Labour Explanation:
(NCL) was set up on 15 October 1999 under • It is a centrally sponsored
the chairmanship of Ravindra Varma which programme, launched by the Ministry of
submitted its report to the then Prime Rural Development in June 2011. It
Minister Atal Bihari Vajpayee on 29 June 2002. involves working with community
institutions through community
18 | P a g e W W W . E D U T A P . C O . I N QUERY? HELLO@EDUTAP.CO.IN / 8146207241
professionals in the spirit of self-help which c) Indira Awas Yojana
is a unique proposition of DAY-NRLM. It d) Jawaharlal Nehru National Urban Renewal
impacts the livelihoods through universal Mission TAL
social mobilization by inter alia organising Answer: C
one-woman member from each rural poor Explanation:
household into Self Help Groups (SHGs), • Indira Awaas Yojana (IAY) is a sub-scheme of
their training and capacity building, Rural Landless Employment Guarantee
facilitating their micro-livelihoods plans, and Programme (RLEGP) which was launched by
enabling them to implement their the Ministry of Rural Development. The main
livelihoods plans through accessing financial objective of the Indira Awaas Yojana is to
resources from their own institutions and provide a grant for the construction of houses
the banks. Hence, statement 2 is correct. to members of Scheduled Caste
• NRLM does not include setting up a large (SC)/Scheduled Tribes (ST), freed
number of new manufacturing industries bonded labours and to non-SC/ST category
and agribusiness centres in rural below the poverty line. Hence, C is the right
areas. Hence, statement 1 is incorrect. answer.
• NRLM does not include supplying seeds,
fertilizers, diesel pump-sets and micro
irrigation equipment free of cost to Q80. The Mid-Day Meal Scheme was launched in
farmers. Hence, statement 3 is incorrect. 1995 with the aim to:
• Therefore, B is the right answer. 1. Enhance enrolment, retention and
attendance of primary school children
Q78. The Mahatma Gandhi National Rural 2. Improve the nutritional status of primary
Employment Guarantee Act (MGNREGA) aims at school children
enhancing livelihood security in rural areas of 3. Improve the habit of reading among rural
India by providing at least one hundred: households
a) Days of guaranteed employment in a financial 4. Encourage the use of Tiffin boxes among
year to able adults in the Information primary school children
Technology sector. a) 1 and 2 only
b) Days of guaranteed wage employment in a b) 1 and 4 only
financial year to adult members of a rural c) 2 and 3 only
household. d) 1, 2 and 4
c) Meals to children of rural households in one Answer: A
financial year. Explanation:
d) Employees for rural developmental schemes • The Mid-Day Meal Scheme was launched in
in a financial year. 1995 with the aim to enhance enrolment,
Answer: B retention and attendance of primary school
Explanation: children. Hence, statement 1 is correct.
• The MGNREGA provides a legal guarantee for • It also aimed at improving the nutritional
one hundred days of employment in every status of primary school children. Hence,
financial year to adult members of any rural statement 2 is correct.
household willing to do public work-related • It is not aimed at improving the habit of
unskilled manual work at the statutory reading among rural households. Hence,
minimum wage. Hence, B is the right answer. statement 3 is incorrect.
• It had no provisions with respect to
Q79. One of the following Government of India encouraging the use of Tiffin boxes among
programmes aims to help, build or upgrade primary school children. Hence, statement 4
dwelling units of below the poverty line rural is incorrect.
families: • Therefore, A is the right answer.
a) National Programme Social Assistance
b) Jawahar Rozgar Yojana

19 | P a g e W W W . E D U T A P . C O . I N QUERY? HELLO@EDUTAP.CO.IN / 8146207241


Q81. Which of the following is NOT covered by comprehensive social security legislation.
the Employees Provident Fund and Rest of the given Acts unidimensional in terms
Miscellaneous Provisions Act, 1952? of providing social security. Hence, D is the
a) Pension right answer.
b) Provident Fund
c) Deposit Linked Insurance Q84. What is the maximum limit of gratuity
d) Injury Compensation payable under the Payment of Gratuity Act,
Answer: D 1972?
Explanation: a) 3 Lakhs rupees
• Employees Provident Fund and Miscellaneous b) 7-5 Lakhs rupees
Provisions Act, 1952 deals with pension, c) 10 Lakhs rupees
provident fund and deposit linked insurance d) 10.5 Lakhs rupees
but it has no provisions with respect of injury Answer: None of the above
compensation. Hence, D is the right answer. Explanation:
• The ceiling of Gratuity amount under the
Q82. Which of the following statements about Payment of Gratuity Act, 1972 has been raised
Workmen's Compensation Act, 1923 is true? from time to time keeping in view over-all
a) It is not social security legislation. economic condition and employers’ capacity
b) Its name has been changed to the Employee's to pay and the salaries of the employees,
Compensation Act in 2009. which have been increased in private sector
c) It provides maximum compensation in the and in PSUs. The latest such enhancement of
event of death. ceiling of gratuity was made vide Government
d) It does not provide compensation for of India Notification dated 29.03.2018 under
occupational diseases. which the gratuity amount ceiling has been
Answer: B increased from Rs.10 Lakhs to 20 Lakhs w.e.f.
Explanation: 29.3.2018.
• Workmen’s Compensation Act is a social • Therefore, as per previous limits C is the right
security legislation. Hence, statement 1 is answer but since that limit does not exist at
incorrect. present, hence, none of the above is right.
• Its name has been changed to the Employee's
Compensation Act in 2009. Hence, statement Q85. The Endeavour of 'Janani Suraksha Yojana'
2 is correct. Programme is to:
• It provides maximum compensation in the 1. Promote institutional deliveries
event of permanent total disablement results 2. Provide monetary assistance to the mother to
from injury. Hence, statement 3 is incorrect. meet the cost of delivery
• It provides compensation for occupational 3. Provide for wage loss due to pregnancy and
diseases. Hence, statement 4 is incorrect. confinement
• Therefore, B is the right answer. a) 1 and 2 only
b) 1 and 3 only
Q83. Which of the following legislations is c) 2 and 3 only
comprehensive social security legislation? d) 1, 2 and 3
a) The Maternity Benefit Act Answer: A
b) The Employees State Insurance Act Explanation:
c) The Employees Compensation Act • Janani Suraksha Yojana (JSY) is a safe
d) The Employees Provident Funds and motherhood intervention under the National
Miscellaneous Provisions Act Rural Health Mission (NRHM) being
Answer: D implemented with the objective of reducing
Explanation: maternal and neo-natal mortality by
• The Employees Provident Funds and promoting institutional delivery among the
Miscellaneous Provisions Act, 1952 deals with poor pregnant women. Hence, statement 1 is
provident fund, pension fund and deposit correct.
linked insurance scheme. Therefore, it is a
20 | P a g e W W W . E D U T A P . C O . I N QUERY? HELLO@EDUTAP.CO.IN / 8146207241
• Disbursement of Cash Assistance: As the cash with words rather than with action, often make
assistance to the mother is mainly to meet the noises about the problems of youth’ and ‘In every
cost of delivery, it should be disbursed generation, old men are found shaking their hoary
effectively at the institution itself. Hence, heads and waxing nostalgic about the good old
statement 2 is correct. days when young people knew better and showed
• There is no feature in the scheme with respect due reverence to age and tradition.’ are the
to the provision of wage loss due to pregnancy examples of the difference of the generation
and confinement. Hence, statement 3 is mentioned in the passage. Hence, option (a) is the
incorrect. correct answer.
• Therefore, A is the right answer.
Q87. The older generation is suspicious about the
Next Four (04) items are based on the passage younger generation as the youngsters lack:
given below: (a) knowledge
It has been rightly said that we spend the first half (b) experience
of our lives trying to understand the older (c) patience
generation, and the second half trying to (d) time
understand the younger generation. Youth has Answer: (b)
always felt somewhat exasperated with age, and Explanation: According to the passage, ‘with their
age has always been suspicious of youth. With natural ebullience and impatience, a majority of
their natural ebullience and impatience, a majority young people are keen to act and learn on their
of young people are keen to act and learn on their own rather than be guided by the experience of
own rather than be guided by the experience of their elders’. This suggests that the older
their elders. The older people being more at home generation is suspicious about the younger
with words rather than with action, often make generation as the youngsters lack ‘experience’.
noises about the problems of youth. In every Therefore, option (b) is the correct answer.
generation, old men are found shaking their hoary
heads and waxing nostalgic about the good old Q88. The author seems to be supportive of the
days when young people knew better and showed idea that
due reverence to age and tradition. In all ages, (a) the people of the older generation are men of
whenever they have pondered over the ways of words rather than action
youth, they have foreseen nothing but ruination (b) the young generation is impatient
staring the world in its face. And yet the world (c) the younger generation today is much
goes on. Every generation passes from the misunderstood and more maligned than it
spontaneity and exuberance of youth to the deserves
caution and prudence of old age, and then yield (d) none of the above
place to the next. Answer: (c)
Explanation: According to the passage, there is a
Q86. What, according to you, is the theme and lack of connection and understanding between
the idea of the passage: the young generation and the older generation as
(a) Generation gap the younger generation is often misunderstood by
(b) Problems of the youth the older generation. It is given in the passage that
(c) Optimism of the older generation ‘In every generation, old men are found shaking
(d) None of the above their hoary heads and waxing nostalgic about the
Answer: (a) good old days when young people knew better
Explanation: Accordingly, the theme of the and showed due reverence to age and tradition.’,
passage should be ‘generation gap’ as the passage which shows that the author is supportive of the
talks about the differences between younger idea that ‘the younger generation today is much
generation and older generation. ‘With their misunderstood and more maligned than it
natural ebullience and impatience, a majority of deserves’. Therefore, option (c) is the correct
young people are keen to act and learn on their answer.
own rather than be guided by the experience of
their elders. The older people being more at home
21 | P a g e W W W . E D U T A P . C O . I N QUERY? HELLO@EDUTAP.CO.IN / 8146207241
Q89. "And yet the world goes on"-what is the Explanation: As given in the passage, ‘The corrupt
tone of the author in this statement? practice of bribery is possible because there is
(a) Optimistic someone who is ready to pay money for illegal
(b) Pessimistic action or decision in his favor.’, which describes
(c) Cynical the author’s view on the practice of bribery.
(d) Critical Therefore, it is true that the author feels that the
Answer: (a) practice of bribery is there because there are
Explanation: As given in the passage ‘and yet the willing bribe givers. Hence, option (a) is the correct
world goes on’, this suggests that the author is answer.
hopeful and optimistic as despite of all the
misunderstanding and differences between the Q91. Who, according to the author, is more
younger generation and older generation the responsible among the following?
world still goes on. ‘Every generation passes from (a) The bribe-taker
the spontaneity and exuberance of youth to the (b) The bribe-giver
caution and prudence of old age, and then yield (c) The witness
place to the next.’, this statement describes the (d) The general public
optimism of the author. Hence, option (a) is the Answer: (b)
correct answer. Explanation: According to the author, ‘The corrupt
practice of bribery is possible because there is
Next Four (04) items are based on the passage someone who is ready to pay money for illegal
given below: action or decision in his favor. Otherwise, how can
Who deserves more severe punishment? One who one demand a bribe? The bribe-giver tempts
gives bribes or the one who takes them? The others to be corrupt and thus demoralizes our
corrupt practice of bribery is possible because national character.’. Therefore, it is clearly
there is someone who is ready to pay money for mentioned that the bribe-giver is more
illegal action or decision in his favor. Otherwise, responsible for the corruption as per the author.
how can one demand a bribe? The bribe-giver Hence, option (b) is the correct answer.
tempts others to be corrupt and thus demoralizes
our national character. Even Jesus Christ, fearing Q92. Who, according to the author, should be
the power of temptation, had said, "Lead me not punished more severely?
into temptation". A bribe-giver is generally (a) The Police who allow this practice
moneyed and influential, while, on the other hand, (b) The bribe-giver
one who demands bribe does so because of his (c) The person who demands bribe
poor circumstances and compulsions. Hence, one (d) None of the above
who gives bribe should be awarded more severe Answer: (b)
punishment because he exploits the weakness of Explanation: As per the author, ‘A bribe-giver is
the poor. Giving and taking bribe happens in a generally moneyed and influential, while, on the
more vicious circle which can continue only other hand, one who demands bribe does so
because of money. Naturally, therefore, it is the because of his poor circumstances and
affluent that grease this wheel of corruption and compulsions. Hence, one who gives bribe should
should accordingly be dealt with firmly and be awarded more severe punishment because he
suitably punished to put a stop to this nefarious exploits the weakness of the poor’, this describes
practice. the authors view that the person who gives bribe
should be punished more severely in case of
Q90. The author feels that the practice of bribery bribery and corruption. Therefore, option (b) is the
is there because there are willing bribe givers. correct answer.
(a) True
(b) False Q93. Who, according to the author, is more
(c) Partially true corrupted?
(d) None of the above (a) Our system
Answer: (a) (b) The person who demands bribe

22 | P a g e W W W . E D U T A P . C O . I N QUERY? HELLO@EDUTAP.CO.IN / 8146207241


(c) The affluent who exploits the weakness of the ‘controlled’. ‘Headstrong’ means ‘very determined
poor to do what you want without listening to others’,
(d) The supporter of bribe who is left with little while ‘controlled’ means ‘not showing emotion;
choice having one's feelings under control.’,Therefore,
Answer: (c) option (b) is the correct answer.
Explanation: According to the author, ‘The corrupt ‘Sensible’ means done or chosen in accordance
practice of bribery is possible because there is with wisdom or prudence; likely to be of benefit.
someone who is ready to pay money for illegal ‘Decide’ means make a choice from a number of
action or decision in his favor. Otherwise, how can alternatives.
one demand a bribe? The bribe-giver tempts ‘Elastic’ means (of an object or material) able to
others to be corrupt and thus demoralizes our resume its normal shape spontaneously after
national character’ and also ‘one who gives bribe being stretched or compressed.
should be awarded more severe punishment ‘Stretch’ means (of something soft or elastic) be
because he exploits the weakness of the poor’, this made or be capable of being made longer or wider
shows that the affluent who exploits the weakness without tearing or breaking.
of the poor is more corrupted. Hence, Option (c) ‘Persuasive’ means good at persuading someone
is the correct answer. to do or believe something through reasoning or
the use of temptation.
Q94. A judgment made before all the facts are ‘Convince’ means cause (someone) to believe
known must be called: firmly in the truth of something.
(a) Deliberate
(b) Sensible Q96. Choose the word which is the nearest
(c) Premature opposite to the meaning of the underlined word:
(d) Harsh I wish I could pursue my studies.
Answer: (c) (a) Discontinue
Explanation: A judgement made before all the (b) Abandon
facts are known is called as ‘premature’ which (c) Restrain
means that decision was made too hastily, and (d) Deter
probably would have been different if more time Answer: (b)
had been taken. Therefore, option (c) is the Explanation: ‘Pursue’ means ‘follow or chase
correct answer. (someone or something)’, the opposite of pursue
‘Deliberate’ means done consciously and should be ‘abandon’ which means ‘give up
intentionally. completely (a practice or a course of action).
‘Sensible’ means done or chosen in accordance Therefore, option (b) is the correct answer.
with wisdom or prudence; likely to be of benefit. ‘Discontinue’ means cease from doing or providing
‘Harsh’ means unpleasantly rough or jarring to the (something), especially something that has been
senses. provided on a regular basis.
‘Restrain’ means to control the actions or
Q95. Which of the following pairs of words behaviour of someone by force, especially in order
expresses the same relationship as in ELUSIVE: to stop them from doing something.
CAPTURE? ‘Deter’ means discourage (someone) from doing
(a) Sensible: decide something by instilling doubt or fear of the
(b) Headstrong: controlled consequences.
(c) Elastic: stretch
(d) Persuasive: convince Q97. Which of the following options is the closest
Answer: (b) in meaning to the word CIRCUITOUS?
Explanation: ‘Elusive’ means ‘difficult to find, (a) Indirect
catch, or achieve.’ while ‘capture’ means ‘take into (b) Confusing
one's possession or control by force.’. Hence, (c) Crooked
elusive and capture are the opposite of each other. (d) Cyclic
The pair which expresses the same relationship as Answer: (c)
that of elusive and capture is ‘headstrong’ and
23 | P a g e W W W . E D U T A P . C O . I N QUERY? HELLO@EDUTAP.CO.IN / 8146207241
Explanation: ‘Circuitous’ means ‘(of a route or Answer: (D)
journey) longer than the most direct way’, the Explanation:
word which is closest to the meaning of circuitous • On 7th September, 1978, an Act to
is ‘crooked’ which means ‘bent or twisted out of establish a Press Council of India for the
shape or out of place’. Therefore, option (c) is the purpose of preserving the freedom of the
correct answer. Press and of maintaining and improving
‘Indirect’ means not directly caused by or resulting the standards of newspapers and news
from something. agencies in India was enacted.
‘Confusing’ means bewildering or perplexing. • The United News of India (UNI), founded
‘Cyclic’ means occurring in cycles; recurrent. in 1961, has emerged as one of the largest
news agencies in India.
Q98. Archaeological studies suggest the theory • The NAM News Network (NNN) is the
(a) All the continents were settled with human transformation of the now-defunct NANAP
societies at about the same time or Non-Aligned News Agencies Pool. The
(b) Farming societies developed before hunting resurrection was with the concurrence and
and gathering endorsement of the Sixth Conference of
(c) The earliest human evolved in the Rift valley in Ministers of Information of Non-Aligned
East Africa Countries (COMINAC VI) hosted by
(d) The wheel was in use in all ancient societies Malaysia in November 2005.
Answer: (C) • Press Trust of India (PTI), news agency
Explanation: cooperatively owned by Indian
• Homo sapiens, the first modern humans, newspapers, which joined together to take
evolved from their early hominid over the management of the Associated
predecessors between 200,000 and Press of India and the Indian outlets of the
300,000 years ago. They developed a Reuters news agency of Great Britain. It
capacity for language about 50,000 years began operating in February 1949 and is
ago. The first modern humans began headquartered in Mumbai. Hence PTI is
moving outside of Africa starting about the earliest and D is the correct answer.
70,000-100,000 years ago. Thus it cannot
be said that human societies emerged at Q100. Which of the following sets of countries
the same time in all the continents. Hence has only federations?
statement (a) is incorrect. (a) New Zealand, India, Zimbabwe and Argentina
• As per archaeological studies, hunting and (b) Malaysia, Australia, Nigeria and Brazil
gathering societies emerged before the (c) India, Nepal, Sri Lanka and South Africa
farming societies. Hence statement (b) is (d) France, Germany, Sweden and Switzerland
incorrect. Answer: (B)
• The earliest human evolved in the Rift Explanation:
valley in East Africa. Hence statement (c) is • France is a unitary State organised on a
correct. decentralised basis under the 1958
• The wheel was invented in the 4th Constitution. Hence D is not the correct
millennium BC in Lower Mesopotamia answer.
(modern-day Iraq), where the Sumerian • New Zealand is a unitary state rather than
people inserted rotating axles into solid a federation—local government has only
discs of wood. Hence statement (d) is the powers conferred upon it by the
incorrect. national Parliament. Hence A is not the
correct answer.
Q99. Which of the following is the earliest to be • With the promulgation of its constitution in
constituted? 2015, Nepal replaced a unitary
(a) Press Council of India government with a federal system of
(b) United News of India government. This process has made Nepal
(c) NAM News Network a federal democratic republic governed
(d) Press Trust of India with three levels of government: a federal
24 | P a g e W W W . E D U T A P . C O . I N QUERY? HELLO@EDUTAP.CO.IN / 8146207241
level, seven provinces and 753 local
government. Hence C is not the correct
answer as per year 2012.
• Hence B is the correct answer as Malaysia,
Australia, Nigeria and Brazil are
federations.

25 | P a g e W W W . E D U T A P . C O . I N QUERY? HELLO@EDUTAP.CO.IN / 8146207241


Visit: www.civilstap.com
For Any Query Mail us: hello@civilstap.com or call us at - (+91)-8146207241
0
(c) S-Q-R-P
UPSC EPFO APFC – 2004 (d) R-P-S-Q

PART- A
Q2. S1: In the roll call of human history, the sleepy
ORDERING OF SENTENCES capital town of Madhya Pradesh is indelibly linked
with one of the world's worst man-made
Directions (for the next Five items): catastrophes.
In the following items each passage consists of six S6: Industries across the globe became less
sentences. The first sentence (S₁) and the sixth complacent and began tightening their safety
sentence (S6) are given in the beginning. The protocols.
middle four sentences in each have been removed P: More than 2,000 died instantly.
and jumbled up. These are labelled P, Q, R and S. Q: On the night of December 3, 1984, a poisonous
You are required to find out the proper sequence grey cloud spread out from the Union Carbide
of the four sentences and mark accordingly on the Plant in Bhopal, suffocating more than three lakhs
Answer Sheet. of its unsuspecting residents.
R: The disaster shook the world, especially when
Example X' has been solved for you. hindsight revealed that almost all safety
X S1: There wsa a boy named Jack. regulations had been flouted by the plant.
S6: At last she turned him out of the house. S: The rest were doomed to wheeze, hobble and
P: So the mother asked him to find work. gasp their way through life, their organs "eaten up
Q: They were very poor. by the deadly combination of gases that leaked
R: He lived with his mother. out of the plant.
S: But Jack refused to work. The correct sequence should be
The correct sequence should be: (a) Q-R-S-P
(a) R-Q-P-S (b) S-P-Q-R
(b) P-Q-R-S (c) Q-P-S-R
(c) Q-P-R-S (d) S-R-Q-P
(d) R-P-S-Q
Answer: (a) Q3. S₁: From an importer of milk powder to the
Explanation: The correct sequence in this example world's largest milk producer.
is R-Q-P-S which is marked by (a). Therefore, (a) is S6: And to think that it began as a simple
the correct answer. experiment in a village in Gujarat only in the
1960s.
Q1. S₁: We Indians by nature are extremely P: Operation Flood is arguably India's biggest and
emotional. most successful revolution since the Quit India
S6: What must be appreciated is that none of the Movement.
members of the Indian team has reacted to this Q: India's Operation Flood has come a long way
mind game utterance of Hayden. from the days when even baby food was imported
P: Had the political parties in India not been busy to producing. 84 million tonnes a year.
in fighting among themselves, they would have R: From abject desperation to becoming a global
asked the Central Government to refuse visa to player in the dairy sector.
Hayden. S: From a mere few villagers and a 275-litre
Q: Statisticians are working overtime to prove processing plant to an operation spread across 23
how our players have not been selfish. States, 170 Districts and 90,000 Village
R: Suddenly Hayden has become a villain. Cooperatives.
S: Without realizing Hayden's intent in calling The correct sequence should be:
subcontinent players selfish, we have reacted (a) P-S-Q-R
angrily. (b) R-Q-S-P
The correct sequence should be: (c) P-Q-S-R
(a) S-P-R-Q (d) R-S-Q-P
(b) R-Q-S-P

1|P a g e W W W . E D U T A P . C O . I N QUERY? HELLO@EDUTAP.CO.IN / 8146207241


Q4. S1: One of the ironies of the Indian economy's
consistent under-performance is that it has been Example Z' has been solved for you.
planned. Z. It is well-known that the effect/(P) is very
S6: As planning prospered, regional imbalances bad/(Q) On children/(R) of cinema/(S)
widened and competitiveness and free enterprise The correct sequence should be:
were crushed-defeating the fundamental purpose (a) P-S-R-Q
of planned development. (b) S-P-Q-R
P: It is another matter that in each of the nine Five- (c) S-R-P-Q
Year Plans, barring the first, more targets were (d) Q-S-R-P
missed than met.
Q: Within two months of India becoming a Q6. War minus shooting but there was nothing to
Republic, the Planning Commission was set up to suggest any/(P) was how cricket between India
prepare a blueprint for India's future, roughly once and Pakistan/(Q) increase of animosity between
in five years. the players or the spectators/(R) was visualised
R: Yet the process of planning didn't change and before the first ball was bowled/(S).
excuses were found wars and droughts in the The correct sequence should be:
1960s, war and oil shocks in the 70s. (a) P-R-Q-S
S: Elaborate models were developed, backed by (b) Q-S-P-R
immaculate calculations, on the investments and (c) P-S-Q-R
policies required to achieve the targets. (d) Q-R-P-S
The correct sequence should be:
(a) Q-S-P-R Q7. In 1997, NASA fact-finding mission on
(b) P-R-Q-S microgravity/(P) by becoming the first Indian
(c) Q-R-P-S woman to rocket into space on a/(Q) to reach out
(d) P-S-Q-R for the stars, created history/(R) Kalpana Chawla,
a Karnal girl who wanted /(S).
Q5. S1: In neighbouring Tamil Nadu, Kodaikanal The correct sequence should be:
has fallen on bad times. (a) S-R-Q-P
S6: It is still a source water-albeit unpotable for (b) P-Q-R-S
the thirsty Kodai. (c) S-Q-R-P
P: Trouble started when the focus was shifted (d) P-R-Q-S
from Ooty to Kodai in the mid 80s.
Q: The once pristine surroundings have been Q8. That so suggests that there are certain cultural
destroyed by construction that paid scant respect resources/(P) many Indians excel as
to the ecological sensitivity of the area. expatriates/(Q) that enable them to succeed while
R: Commercial forces took over the hill station living away from home/(R) perhaps unique to
large-scale activities and building commenced Indians/(S).
around the artificial lake created in the 1800s. The correct sequence should be:
S: There is large-scale deforestation as well. (a) S-P-Q-R
The correct sequence should be: (b) Q-R-S-P
(a) P-S-Q-R (c) S-R-Q-P
(b) Q-R-P-S (d) Q-P-S-R
(c) Q-S-P-R
(d) P-R-Q-S Q9. The advent we entertained and informed
ourselves/(P) of satellite television was a silent/(Q)
Directions (For the following 5 items): In the and completely changed the way/(R) revolution
following items some parts of the sentence have which created a whole new industry/(S).
been jumbled up. You are required to re-arrange The correct sequence should be:
these parts which are labelled P, Q. R and S to (a) Q-R-S-P
produce the correct sentence. Choose the proper (b) P-S-R-Q
sequence and mark in your Answer Sheet (c) Q-S-R-P
accordingly. (d) P-R-S-Q
2|P a g e W W W . E D U T A P . C O . I N QUERY? HELLO@EDUTAP.CO.IN / 8146207241
Q10. The President the aspirations of the store vast quantities of heat, thereby often
Scheduled Castes and the Scheduled Tribes/(P) preventing climatic extremes. In addition, water
with political parties, industry and other dissolves more substances than any other liquid. It
bodies/(Q) revealed that the Government would is this characteristic which helps make oceans a
initiate a dialogue/(R) on how best the private great storehouse for minerals which have been
sector could fulfill/(S). washed down. from the continents. In several
The correct sequence should be: areas of the world these minerals are being
(a) S-P-R-Q commercially exploited. Solar evaporation of salt
(b) R-Q-S-P is widely practised, potash is extracted from the
(c) S-Q-R-P Dead Sea, and magnesium is produced from sea
(d) R-P-S-Q water along the American Gulf Coast.
Q11. Which one of the following is correct?
COMPREHENSION According to the passage, the hydrosphere is not
(a) responsible for all forms of life
Directions (For the following 5 items): (b) able to modify weather
The passage given below is followed by questions (c) a source of natural resources:
based on its content. After reading the passage, (d) in danger of freezing over
choose the best answer to each question. Answer
all the five questions following the passage on the Q12. What is the reason that fish can survive in the
basis of what is stated or implied in the passage. oceans?
(a) They do not need oxygen
PASSAGE (b) Ice floats on the surface of the oceans
(c) Evaporation and condensation create a water
Ocean water plays an indispensable role in cycle
supporting life. The great ocean basins hold. about (d) There are oceans currents in the
300 million cubic miles of water. From this vast
amount, about 80,000 cubic miles of water are Q13. What is the author's main purpose in the
sucked into the atmosphere each year by passage given above?
evaporation and returned by precipitation and (a) To describe the properties and uses of water
drainage to the ocean. More than 24,000 cubic (b) To illustrate the importance of conserving
miles of rain descend annually upon the water
continents. This vast amount is required to (c) To explain how water is used in commerce and
replenish the lakes and streams, springs and water industry
tables on which all flora and fauna are dependent. (d) To reveal the extent of the earth's ocean
Thus, the hydrosphere permits organic existence. masses
The hydrosphere has strange characteristics
because water has properties unlike those of any Q14. What is the reason that climatic extremes are
other liquid. One anomaly is that water upon prevented in nature?
freezing expands by about 9 percent, whereas (a) A vast quantity of water is held in the great
most other liquids contract on cooling. For this ocean basins
reason, ice floats on water bodies instead of (b) The heat capacity of water is very high
sinking to the bottom. If the ice sank, the (c) Solar evaporation from the oceans uses high
hydrosphere would soon be frozen solidly, except technology
for a thin layer of surface melt water during the (d) Water dissolves many substances
summer season. Thus, all aquatic life would be
destroyed and the interchange of warm and cold Q15. Which of the following characteristics of
currents, which moderates climates, would be water does the author mention in the passage?
notably absent. 1. Water expands when it is frozen
Another outstanding characteristic of water is that 2. Water is used as a condensing medium for
water has a heat capacity which is highest of all thermal power plants
liquids and solids except ammonia. This 3. Water is a good solvent
characteristic enables the oceans to absorb and
3|P a g e W W W . E D U T A P . C O . I N QUERY? HELLO@EDUTAP.CO.IN / 8146207241
Select the correct answer using the codes given is most similar in meaning to the word in capital
below: letters.
Codes: Q21. COMMISERATION
(a) 1 only (a) Commission
(b) 2 and 3 (b) Vastness
(c) 1 and 3 (c) Sympathy
(d) 1, 2 and 3 (d) Commendation.

ANTONYMS Q22. SLIPSHOD


(a) Vulgar
Directions (For the following 5 items): (b) Careless
Each of the following five items consists of a word (c) Commonplace
in capital letters, followed by four words or groups (d) Retaliatory
of words. Select the word or group of words that
is most nearly opposite in meaning to the word in Q23. COGENT
capital letters. (a) Forceful
Q16. OUTLANDISH (b) Objectionable
(a) Stubborn (c) Exceptional
(b) Conventional (d) False
(c) Agitated
(d) Fearful Q24. PONDEROUS
(a) Facile
Q17. SCANTY (b) Exorbitant
(a) Arrogant (c) Cautious
(b) Plentiful (d) Unwieldy
(c) Painful
(d) Handsome Q25. MASQUERADE
(a) To provide support
Q18. PROPENSITY (b) To go in disguise
(a) Disinclination (c) To mesmerize
(b) Forecast (d) To run in a marathon race.
(c) Stagnation
(d) Restlessness Part - B

Q19. RECTITUDE Q26. Which one of the following is a super-cooled


(a) Self-condemnation liquid?
(b) Deceitfulness (a) Ice-cream
(c) Reprisal (b) Teflon
(d) Punctuality (c) Glass
(d) Mercury
Q20. PONTIFICATE
(a) To pilfer Q27. Why does bleeding occur through nose in
(b) To leave one's country high mountainous regions?
(c) To speak in a humble manner. (a) The pressure of blood in capillaries is greater
(d) To spend recklessly than the outside pressure
(b) The pressure at high altitudes is greater than
SYNONYMS that on the plains
(c) The blood pressure of a person increases at
Directions (For the following 5 items): high altitudes
Each of the following five items consists of a word (d) The blood pressure fluctuates and finally
in capital letters, followed by four words or groups decreases Drastically
of words. Select the word or group of words that
4|P a g e W W W . E D U T A P . C O . I N QUERY? HELLO@EDUTAP.CO.IN / 8146207241
Q28. Which one of the following causes small air is a correct explanation of the Assertion (A). Select
bubbles in water, which is stored in a glass vessel, your answers to these items using the Codes given.
to appear to shine with a silvery luster? below and mark your answer-sheet accordingly
(a) Reflection of light rays
(b) Refraction of light rays Codes:
(c) Polarization of light rays (a) Both A and R are true and R is the correct
(d) Total internal reflection explanation of A
(b) Both A and R are true but R is NOT the correct
Q29. Weight of a body on the surface of the earth explanation of A
is W₁. The weight of the same body is W2, at a (c) A is true but R is false
height of 500 metres above the surface, and W3 (d) A is false but R
at a depth of 500 metres below the surface of the
earth. Which one of the following is the correct Q33. Assertion (A): Glycerol is a constituent of
relation? shaving cream.
(a) W3 > W1 > W2 Reason (R): Glycerol is an antiseptic.
(b) W₁ > W₂; W₁ > W3
(c) W₁ < W₂ <W3 Q34. Assertion (A): Phenol is commonly used as a
(d) W₂ > W3 > W₁ disinfectant.
Reason (R): Phenol is not harmful to living human
Q30. Which one of the following is not a Vitamin? tissues even at a very high concentration.
(a) Folic acid
(b) Oleic acid
(c) Pantothenic acid Q35. What is the number of triangles that can be
(d) Ascorbic acid formed whose vertices are at the vertices of an
octagon but have only one side common with
Q31. Which one of the following statements is that of the octagon?
not correct? (a) 16
(a) The deficiency of Vitamin A causes night (b) 24
blindness (c) 32
(b) Pain in the muscles and joints along with (d) 48
fatigue, loss of weight and poor wound healing
Q36. A letter is taken out at random from the
are the symptoms of the deficiency of Vitamin
word 'ASSAM' and a letter is taken out at random
C
from the word 'NAGALAND'. What is the
(c) Bleeding gums and loosening of teeth are
probability that the two letters are the same?
symptoms of the deficiency of Vitamin B,
(d) The deficiency of Vitamin B can cause (a) 1/20
inflammation of skin, memory disorder and (b) 3/20
Diarrhoea (c) 1/40
(d) 31/40
Q32. Which one of the following elements is not Q37. A set of 5 parallel lines is intersected by
naturally found in human beings? another set of 4 parallel lines so as to form a
(a) Copper number of parallelograms. How many
(b) Zinc parallelograms can be formed?
(c) Iodine
(a) 12
(d) Lead
(b) 20
(c) 40
Directions: The following Two (8 & 9) items consist
(d) 60
of two statements, one labelled the 'Assertion (A)
and the other labelled the 'Reason (R)'. You are to Q38. Three dice are rolled. What is the
examine these two statements carefully and probability that at least one die will show 6?
decide if the Assertion (A) and the Reason (R) are (a) 1/6
individually true and if so, whether the Reason (R) (b) 1/36
5|P a g e W W W . E D U T A P . C O . I N QUERY? HELLO@EDUTAP.CO.IN / 8146207241
(c) 91/216 envelope without seeing the address written on
(d) 125/216 any envelope. What is the probability that each
Q39. How many numbers are there between 999 of the three letters goes into the envelope with
and 10000 such that at least one of their digits is correct address written on it?
5? (a) 1/27
(a) 2879 (b) 1/6
(b) 3168 (c) 1/9
(c) 3200 (d) 1/3
(d) 3224
Q40. Four different toys are to be distributed Q45. A person and his wife appear in the
among two children such that each child gets at interview for the two vacancies of a post. The
least one toy. In how many different ways can probability of the selection of the husband and
this be done? that of the wife are 1/7 and 1/5 respectively.
(a) 6 What is the probability that only one of them will
(b) 8 be selected?
(c) 12 (a) 1/7
(d) 14 (b) 1/5
Q41. A stadium has 10 gates. In how many (c) 2/7
different ways can 3 persons enter the stadium? (d) 11/35
(a) 10
(b) 30 Q46. What is the nth term of the series √3, √6,
(c) 310 2√3, ...?
(d) 1000 (a) √3(√2)n-1
(b) √2 (√3)n-1
Q42. At the conclusion of a party, a total number (c) √3 (√2)n
of 28 handshakes was exchanged. Every person in (d) √2 (√3)n
the party shook hands with every other person
who attended the party. What was the total Q47. The length of a rectangle is reduced by 20%
number of persons who attended the party? and breadth is kept constant, and the new figure
(a) 8 that is formed is a square.
(b) 14 Consider the following statements:
(c) 28 1. The area of square is 25% less than the area of
(d) 56 rectangle.
2. The perimeter of square is approximately 11%
Q43. Amit has 7 friends whom he wishes to invite less than 1 the perimeter of rectangle.
to a dinner. Out of his 7 friends, 1 or more may 3. The diagonal of square is approximately 12%
less than the diagonal of rectangle.
accept the invitation. In how many different ways
Which of the statements given. above is/are
can Amit's 7 friends attend the party? correct?
(a) 96 (a) 1 only
(b) 112 (b) 1 and 2
(c) 127 (c) 1, 2 and 3
(d) 128 (d) 2 and 3

Q44. There are three envelopes with three Directions (For the next four items):
different addresses written on them. Three Refer to the graphs given below and answer the
items that follow:
letters are to be enclosed in these three
envelopes such that one letter goes inside one

6|P a g e W W W . E D U T A P . C O . I N QUERY? HELLO@EDUTAP.CO.IN / 8146207241


Q51. In which year is the ratio of foreign
investment approvals to foreign investment
inflows highest?
(a) 1992
(b) 1993
(c) 1994
(d) 1995

Q52. There are four persons A, B, C and D. The


total amount of money with A and B together is
equal to the total amount of money with C and D
together. But the total amount of money with B
and D together is more than the amount of
money with A and C together. The amount of
money with A is more than that with B. Who has
the least amount of money?
(a) B
(b) C
(c) D
Q48. What is the contribution of EU in the foreign
(d) Cannot be determined
investment inflows for the year 1996 in US $ Mn.?
(a) 1840
Q53. A printer uses a total number of 4893 digits
(b) 2000
in order to number all the pages of his book. He
(c) 460
starts with the digit 1 for the first page of the
(d) Cannot be determined with the given data
book. How many pages does the book have?
(a) 4892
Q49. What are the foreign investment approvals
(b) 2322
from SAARC, ASEAN and EU combined together in
(c) 2100
the year 1996 (in US $ Mn.)?
(d) 1500
(a) 2240
(b) 5000
Directions (For the next two items):
(c) 3500
Based on the information given below, answer the
(d) 2600
two items which follow it:
Five ladies A, B, C, D, E go to a cinema hall along
Q50. If the net foreign investment inflows for the
with their husbands P, Q, R, S, T (not necessarily in
year 1997 were to grow by 20% and if the
that order). Seats numbered from 1 to 10 are
proportion of inflows was to remain the same as
reserved for these 5 married couples. Each of the
in case of the approvals for the year 1996, what
five ladies sits on the seat immediately to the left
would be the foreign investment inflows from
of her husband.
NAFTA (in US $ Mn.) If NAFTA got 800 inflow in
(i) The couple E and S occupy the seats at extreme
1996??
right.
(a) 800
(ii) C, who is not sitting at extreme left and Q, who
(b) 500
is not sitting on the seat number 2, are not
(c) 840
husband and wife.
(d) 920

7|P a g e W W W . E D U T A P . C O . I N QUERY? HELLO@EDUTAP.CO.IN / 8146207241


(iii) B and R are celebrating their wedding
anniversary and hence are sitting at the centre. Q58. Consider the following statements:
(iv) A and T are not husband and wife. 1. The first report of the Administrative Reforms
Commission recommended the creation of Lok
(v) D is seating on the seat number 3.
Pal and Lok Ayukta in India.
2. Different articles under the Constitution of
Q54. Who is sitting on the seat number 7? India provide for setting up the Union Public
(a) A Service Commission, the Planning Commission
(b) B and the Vigilance Commission.
(c) C Which of the statements given above is/are
(d) E correct?
(a) 1 only
(b) 2 only
Q55. If in the last condition (v), it is stated that D (c) Both 1 and 2
sits on the seat number 7, then who will sit on the (d) Neither 1 nor 2
seat number 4?
(a) P Q59. Consider the following statements:
(b) T 1. Money Bill can be introduced in any House of
(c) P or T Parliament.
2. If any question arises whether a bill is a Money
(d) Cannot be determined
Bill or not, the decision of the Chairman of the
Public Accounts Committee is final.
Q56. A 2-member committee is to be constituted 3. Vote of Credit enables a grant to be approved
out of 4 men and 2 women. What is the by the Parliament in advance of the detailed
probability that a committee thus formed will examination of various demands presented to
have exactly one woman? it.
Which of the statements given above is/are
(a) 1/3
correct?
(b) 1/4 (a) 1 and 2
(c) 7/15 (b) 2 and 3
(d) 8/15 (c) 1 and 3
(d) None
Q57. Match List-I (Commission/Committee) with
List-II (Chairman) and select the correct answer Q60. Consider the following statements:
using the code given below the lists: 1. If forty or more members support an
List-I (Commission/Committee) Adjournment Motion, only then the related
A. Economic Reforms Commission matter can be taken up for discussion.
B. Commission on Centre-State Relations 2. Under an Adjournment Motion, even a sub
C. Administrative Reforms Commission judice matter can be taken up for discussion.
D. Committee on the Prevention of Corruption 3. Under Article 112 of the Constitution of India,
List-II (Chairman) the Union Finance Minister causes to be laid
1. R.S. Sarkaria before both the Houses of Parliament and
2. K. Hanumanthaiya 'Annual Financial Statement' for each financial
3. L.K. Jha year.
4. K. Santhanam Which of the statements given above is/are
5. D.S. Kothari correct?
Code: (a) 1 only
A B C D (b) 3 only
(a) 3 4 2 1 (c) 2 and 3
(b) 2 1 5 4 (d) None
(c) 3 1 2 4
(d) 2 4 5 1
8|P a g e W W W . E D U T A P . C O . I N QUERY? HELLO@EDUTAP.CO.IN / 8146207241
Q61. Assertion (A): Recently, the four public Which of the statements given above is/are
sector general insurance companies have correct?
launched a community-based ‘Universal Health (a) 1 only
Insurance Scheme’. (b) 2 only
Reason (R): According to the Constitution of India, (c) Both 1 and 2
the task of promoting public health primarily rests (d) Neither 1 nor 2
with the Union Government.
Codes: Q65. Consider the following statements:
(a) Both A and R are true and R is the correct 1. Uttar Pradesh and Bihar are two States with
explanation of A maximum population of the Scheduled Castes.
(b) Both A and R are true but R is NOT the correct 2. Punjab has the highest ratio of the Scheduled
explanation of A Caste population to its total population.
(c) A is true but R is false 3. There was a provision for setting up a National
(d) A is false but R Commission for the Scheduled Castes and the
Scheduled Tribes under the Article 338 of the
Q62. Consider the following statements: Constitution of India. However, by a
1. Dadra and Nagar Haveli which merged with the Constitutional Amendment Act, separate
Indian Union in 1961 were ruled by the National Commission for the Scheduled Castes
Portuguese. and National Commission for the Scheduled
2. Both Pondicherry and Chandernagore were Tribes have been set up.
ruled by the French. Which of the statements given above is/are
3. Goa got its freedom in 1961. correct?
Which of the statements given above is/are (a) 2 only
correct? (b) 1 and 2
(a) 1, 2 and 3 (c) 2 and 3
(b) 2 and 3 (d) 1 and 3
(c) 1 and 3
(d) 2 only Q66. Consider the following statements:
1. An amendment of the Constitution of India can
Q63. Consider the following statements: be initiated by the introduction of a Bill for the
1. The Chief Minister of Delhi is appointed by the purpose in Lok Sabha only.
President of India. 2. A Bill providing for the formation of new States
2. The Delhi Police and Municipal Corporation of and for the alteration of boundaries or names
Delhi do not come under the administrative of existing States in India can be introduced in
purview of the Delhi Government. the Parliament only on the recommendation
Which of the statements given above is/are of the President of India.
correct? 3. In India, a Bill seeking to amend the
(a) 1 only representation of States in Parliament has to
(b) 2 only be passed by a simple majority of both Houses
(c) Both 1 and 2 of Parliament.
(d) Neither 1 nor 2 Which of the statements given above is/are
correct?
Q64. Consider the following statements: (a) 1 and 2
1. The Legislative Assembly of a State cannot (b) 2 only
have more than 450 and not less than 50 (c) 1 and 3
members chosen by direct election from (d) 1, 2 and 3
territorial constituencies.
2. The number of members of Legislative Council Q67. Which one of the following is the correct
of a State cannot exceed one-third of the total statement?
membership of the Legislative Assembly of
that State subject to a minimum of 40. The term of Lok Sabha can be extended by the
Parliament for a period
9|P a g e W W W . E D U T A P . C O . I N QUERY? HELLO@EDUTAP.CO.IN / 8146207241
(a) not exceeding six months at a time during (d) 3 and 4
proclamation of emergency but not exceeding
in any case a period of three months after the Q70. Assertion (A): During the British rule, the
proclamation of emergency has ceased to Congress Ministers resigned in 1939 in all the
operate provinces where they were in office.
(b) not exceeding one year at a time during Reason (R): Congress wanted fresh elections as it
proclamation of emergency but not exceeding was frustrated with Jinnah’s campaign of two-
in any case a period of six months after the nation theory.
proclamation of emergency has ceased to Codes:
operate (a) Both A and R are true and R is the correct
(c) not exceeding one year at a time during explanation of A
proclamation of emergency but not exceeding (b) Both A and R are true but R is NOT the correct
in any case a period of three months after the explanation of A
proclamation of emergency has ceased to (c) A is true but R is false
operate (d) A is false but R
(d) not exceeding six months at a time during
proclamation of emergency but not exceeding Q71. Assertion (A): Aurangzeb released Shahu
in any case a period of one month after the from the prison shortly before Aurangzeb died in
proclamation of emergency has ceased to 1707.
operate Reason (R): Zulfiqar pointed that Shahu’s return to
his Kingdom would cause a division among the
Q68. Consider the following statements: Marathas who would thus be disabled from
1. The Indian Parliament is not sovereign, and the plundering the imperial territories.
legislation passed by it is subject to judicial Codes:
review. (a) Both A and R are true and R is the correct
2. In India, the Fundamental Rights and Directive explanation of A
Principles of State Policy are enforceable by (b) Both A and R are true but R is NOT the correct
the courts. explanation of A
Which of the statements given above is/are (c) A is true but R is false
correct? (d) A is false but R is true
(a) 1 only
(b) 2 only Q72. Match List-I with List-II and select the
(c) Both 1 and 2 correct answer using the code given below the
(d) Neither 1 nor 2 lists:
List-I (Indian Kings)
Q69. Consider the following statements related A. Karikala
to the World War II: B. Rudradaman
1. Germany attacked the US naval base at Pearl C. Milinda
Harbour. D. Kanishka
2. Winston Churchill was the British Prime
Minister during the World War II. List-II (Dynasty)
3. The Versailles Treaty which is generally cited as 1. Bacteria Greek
one of the main causes of the World War II was 2. Chola
signed just before the World War II in 1939. 3. Chalukya
4. Franklin Roosevelt was the President of the 4. Kushanas
United States were dropped over the Japanese 5. Shakas
cities of Hiroshima and Nagasaki. Codes:
Which of the statement given above is/are A B C D
correct? (a) 2 4 1 5
(a) 1, 2 and 3 (b) 1 5 3 4
(b) 1, 2 and 4 (c) 2 5 1 4
(c) 2 only (d) 1 4 3 5
10 | P a g e W W W . E D U T A P . C O . I N QUERY? HELLO@EDUTAP.CO.IN / 8146207241
(a) Sayyids – Khaljis – Lodis – Tughlaqs
Q73. During the World War II, Indian soldiers (b) Khaljis – Tughlaqs – Sayyids – Lodis
fought heroically in the Battle at Monte Cassino. (c) Khaljis – Sayyids – Lodis – Tughlaqs
Where is Monte Cassino located? (d) Tughlaqs – Khaljis – Sayyids – Lodis
(a) Poland
(b) Italy Q78. Consider the following statements:
(c) Germany According to the Mountbatten Plan
(d) Greece 1. The Union of India and Pakistan were to be
granted freedom not later than June 1948.
Q74. Match List-I with List-II and select the 2. The Legislative Assembly of Sindh was to
correct answer using the code given below the decide whether it wanted to join the
lists: Constituent Assembly of India or not.
List-I (Battles) 3. Baluchistan was to decide whether it wanted
A. Battle of Chausa to stay with the Indian Union or become
B. Battle of Ghagra separate.
C. Battle of Kanwah Which of the statements given above is/are
D. Battle of Talikota correct?
(a) 1 and 2
List-II (Fought between) (b) 2 and 3
1. Babar and Rajputs (c) 2 only
2. Humayun and Sher Khan (d) 1 and 3
3. Babar and Afghans of Bihar & Bengal
4. Vijayanagar ruler and the Decca Sultanates Q79. In the year 1919, what was the reason for
Codes: Mahatma Gandhi to warn the Viceroy that a
A B C D countrywide Satyagraha would be launched?
(a) 2 1 3 4 (a) He wanted the Government to withdraw the
(b) 4 3 1 2 Rowlatt Act immediately
(c) 2 3 1 4 (b) He was forcing the British rulers to be sensitive
(d) 4 1 3 2 to the Khilafat Movement
(c) He wanted the Government to abolish the
Q75. When Akbar besieged the Fort of Chittoor, Zamindari System to alleviate the miseries of
who among the following defended it for four peasants
months? (d) He was forcing the British rulers to give at least
(a) Uday Singh the dominion status to India to make Home
(b) Rana Pratap Rule possible
(c) Bahman Shah
(d) Jaimal Q80. Who among the following introduced the
celebration of Ganesh Chaturthi and Shivaji
Q76. Which one of the following statements is Festivals to bring the Indian society together and
not correct? inspire patriotic feelings among the people?
(a) Lord Wellesley set up the first three Indian (a) Bal Gangadhar Tilak
Universities (b) Gopal Krishna Gokhale
(b) Lord Dalhousie introduced telegraph in India (c) Jyotirao Govind Rao Phule
(c) Lord Ripon introduced a system of local self- (d) Mahadev Govind Ranade
government both for towns and country-side
areas Q81. Which one of the following is the correct
(d) Lord Curzon presided over the Delhi Durbar of chronological sequence of the given Bhakti
1903 Saints?
(a) Guru Nanak-Tulsidas Ramdas Tukaram
Q77. Which one of the following is the correct (b) Tulsidas-Guru Nanak Tukaram-Ramdas
chronological sequence of the given dynasties of (c) Guru Nanak-Tulsidas Tukaram-Ramdas
Delhi Sultanate? (d) Tulsidas-Guru Nanak Ramdas-Tukaram
11 | P a g e W W W . E D U T A P . C O . I N QUERY? HELLO@EDUTAP.CO.IN / 8146207241
5. Uttar Pradesh
Q82. Match List-I. (Persons) with List-II (Writings) Code:
and select the correct answer using the codes A B C D
given below the lists: (a) 4 2 3 1
LIST I (Persons) LIST II (Writings) (b) 1 3 5 2
A. Asvaghosha 1. Buddha Charita (c) 4 3 1 2
B. Nagarjuna 2. Madhyamika (d) 1 2 5 3
Sutra
C. Patanjali 3. Vedanta Sangraha Q86. Match List-I (Produce) with List-II (Major
D. Ramanuja 4. Yogasutra Producer State) and select the correct answer
Code: using the code given below the lists:
A B C D List-I (Produce) List-II (Major
(a) 4 3 1 2 Producer State)
(b) 1 2 4 3 A. Rubber 1. Andhra Pradesh
(c) 4 2 1 3 B. Soyabean 2. Tamil Nadu
(d) 1 3 4 2 C. Groundnut 3. Madhya Pradesh
D. Wheat 4. Kerala
Q83. Match List-I. (Historical Sites) with List-II 5. Uttar Pradesh
(State) and select the correct answer using the Code:
codes given below the lists: A B C D
LIST I (Historical Site) LIST II (State) (a) 4 1 2 5
A. Shore Temple 1. Karnataka (b) 5 3 1 4
B. Bhimbetka 2. Tamil Nadu (c) 4 3 1 5
C. Kesava Temple 3. Kerala (d) 5 1 2 4
(Hoysala
Monument) Q87. Consider the following statements:
D. Hampi 4. Madhya Pradesh 1. In India, intensity of cropping is high in the
5. Rajasthan States of the peninsular plateau.
Code: 2. In India, the States of Punjab and Haryana
A B C D have the highest proportion of the net sown
(a) 3 5 2 1 area to total geographical area.
(b) 2 4 1 1 3. South-west Monsoon season is termed as the
(c) 3 4 2 2 Kharif season of crops.
(d) 2 5 1 4 Which of the statements given above are correct?
(a) 2 and 3
Q84. Which country among the following is the (b) 1 and 2
biggest producer of cotton? (c) 1 and 3
(a) China (d) 1, 2 and 3
(b) India
(c) Indonesia Q88. Match List-I (Country) with List-II (Location)
(d) United States of America and select the correct answer using the codes
given below the Lists:
Q85. Match List-I (Wildlife Sanctuary) with List-II List-I (Country) List-II (Location)
(State) and select the correct answer using the A. Bahamas 1. Europe
code given below the lists: B. Belize 2. North America
List-I (Wildlife List-II (Host Country) C. Macedonia 3.North Atlantic Ocean
Sanctuary) D. Gabon 4. Africa
A. Bhitar Kanika 1. Andhra Pradesh 5. South America
B. Pachmarhi 2. Karnataka Code:
C. Pocharam 3. Madhya Pradesh A B C D
D. Sharavathi 4. Orissa (a) 3 4 1 2

12 | P a g e W W W . E D U T A P . C O . I N QUERY? HELLO@EDUTAP.CO.IN / 8146207241


(b) 1 2 5 4
(c) 3 2 1 4 Q94. Which one of the following is the correct
(d) 1 4 5 2 statement?
(a) Spring tides occur on the full moon day
Q89. Going from the West of the United States of (b) Neap tides occur on the new moon day only
America to its East, which one of the following is (c) The West Coast of India experiences tides four
the correct sequence of the given major times a day
American cities? (d) Tides do not occur in the gulfs
(a) Houston-Los Angeles San Francisco New York
(b) San Francisco-New York Houston-Los Angeles Q95. Which one of the following pairs is not
(c) Houston-New York-San Francisco-Los Angeles correctly matched.
(d) San Francisco-Los Angeles Houston-New York City River
(a) Ahmedabad : Sabarmati
Q90. Match List-1 (Beach Resort) with List-II (b) Hyderabad : Musi
(State) and select the correct answer using the (c) Lucknow : Gomti
codes given below the Lists: (d) Surat : Narmada
List-I (Beach Resort) List-II (State)
A. Digha 1. Kerala Q96. Match List-I (Famous Place) with List-II
B. Covelong 2. West Bengal (Country) and select the correct answer using the
C. Cherai 3. Maharashtra codes given below the lists:
D. Murud-Janjira 4. Tamil Nadu List-I (Famous Place) List-II (Country)
Code: A. Alexandria 1. Turkey
A B C D B. Blackpool Pleasure 2. Great Britain
(a) 2 4 1 3 Beach
(b) 3 1 4 2 C. Constantinople 3. Italy
(c) 2 1 4 3 D. Florence 4. Greece
(d) 3 4 1 2 5. Egypt
Code:
Q91. Where are the maximum numbers of major A B C D
ports located in India? (a) 1 3 4 2
(a) Maharashtra (b) 5 2 1 3
(b) Kerala (c) 1 2 4 3
(c) Goa (d) 5 3 1 2
(d) Tamil Nadu
Q97. Consider the following statements:
Q92. Which one of the following is not a tributary 1. Kaziranga National Park is a World Heritage
of the river Godavari? Site recognized by the UNESCO.
(a) Koyna 2. Kaziranga National Park is a home to sloth
(b) Manjra bear and hoolock gibbon.
(c) Pranhita Which of the statements given above is/are
(d) Wardha correct?
(a) 1 only
Q93. Which one of the following is not a correct (b) 2 only
statement? (c) Both 1 and 2
(a) The height of the Western half of the (d) Neither 1 nor 2
Himalayas is greater than that of the Eastern
half. Q98. Match List-I (Institute) with List-II (Location)
(b) The Himalayas are young fold mountains and select the correct answer using the code
(c) The Shivalik ranges are made of given below the lists:
unconsolidated river deposits List-I (Institute)
(d) The Himalayas are wide in Kashmir and A. Indian Institute of Public Administration
become narrow towards the East B. V.V. Giri National Labour Institute
13 | P a g e W W W . E D U T A P . C O . I N QUERY? HELLO@EDUTAP.CO.IN / 8146207241
C. National Institute of Financial Management Which of the statements given above is/are
D. National Law School of India University correct?
List-II (Location) (a) 1 only
1. Fardiabad (b) 2 only
2. Bangalore (c) Both 1 and 2
3. NOIDA (d) Neither 1 nor 2
4. Mumbai
5. Delhi Q103. Consider the following statements with
Code: reference to the Census 2001 of India:
A B C D 1. For the first time, the country has
(a) 1 2 4 3 witnessed a faster growth in female
(b) 5 3 1 2 literary compared to that of males during
(c) 1 3 4 2 the decade 1991-2001.
(d) 5 2 1 3 2. During the decade 1991-2001, the rural-
urban literacy gap continued to diverge
Q99. Where is the Holy Shrine of Imam Ali in Which of the statement given above is/are
Najaf located? correct?
(a) Saudi Arabia (a) 1 only (b) 2 only
(b) Iraq (c) Both 1 and 2 (d) Neither 1 nor 2
(c) Iran
(d) Kuwait Q104. Match List-I (Railway Zone) with List-II
(Headquarters) and select the correct answer
Q100. Why was the region of Darfur of Sudan in using the code given below the lists:
news recently? List-I (Railway Zone)
(a) Americans bombed the terrorist training A. East-Central Railway
camps set up over there by the Janaweed B. North-Western Railway
militiamen of Sudan C. North-Central Railway
(b) Thousands of black African civilians were D. South-Western Railway
killed or displaced by the Janaweeds who List-II (Headquarters)
are supported by the Arab-dominated 1. Hubli
Sudan's Government 2. Allahabad
(c) Bird flu started from there 3. Hajipur
(d) A dam over the river Blue Nile was swept 4. Jabalpur
away killing thousands of people 5. Jaipur
Code:
Q101. What was the total expenditure on A B C D
education both by the Central and State (a) 3 5 2 1
Governments during the year 2002-03 as an (b) 2 1 4 5
approximate percentage of the Gross Domestic (c) 3 1 2 5
Product? (d) 2 5 4 1
(a) 2%
(b) 3% Q105. Consider the following statements:
(c) 4% 1. The average population density of India is
(d) 5% between 600-700 persons per square
kilometer.
Q102. Consider the following statements with 2. 2002-2007 is the duration of the Tenth
reference to the United Nations Organisation: Five-Year Plan.
1. In the General Assembly of the UNO, no 3. Installed electricity generation capacity in
member-nation has veto power. India is in excess of 2 lakh Megawatt.
2 In the Security Council, all permanent Which of the statements given above is/are
members must vote in the affirmative if a correct?
resolution is to pass. (a) 1 and 2
14 | P a g e W W W . E D U T A P . C O . I N QUERY? HELLO@EDUTAP.CO.IN / 8146207241
(b) 2 only List-II (Country)
(c) 1 and 3 1. Belarus
(d) 1, 2 and 3 2. United States of America
3. Australia
Q106. Match List-I (Institute) with List-II (City) 4. Chile
and select the correct answer using the codes 5. Sweden
given below the Lists: Codes:
List-I (Institute) A B C D
A. Rashtriya Sanskrit Vidyapeeth (a) 2 4 1 3
B. Maharishi Sandipani Rashtriya Veda (b) 1 3 5 4
Vidya Prathishthan (c) 2 3 1 4
C. Central Institute of Indian Languages (d) 1 4 5 3
D. Central Institute of English and Foreign
Language Q109. Who won the 100-metre race for men in
List-II (City) the Athens Olympics in August, 2004?
1. Hyderabad (a) Maurice Greene
2. Varanasi (b) Justine Gatlin
3. Mysore (c) Asafa Powell
4. Tirupati (d) Francis Obekwelu
5. Ujjain
Code: Q110. Who among the following Indian film
A B C D directors/actors was honoured with the Order of
(a) 2 3 1 5 the British Empire (OBE) in year 2004?
(b) 4 5 3 1 (a) Naseeruddin Shah
(c) 2 5 3 1 (b) Amitabh Bachchan
(d) 4 3 1 5 (c) Shekhar Kapoor
(d) Om Puri

Q107. Who among the following are the Bharat Q111. Match List-I (Distinguished Women) with
Ratna recipients? List-II (Known As) and select the correct answer
1. Amartya Sen using the codes given below the Lists:
2. Gulzari Lal Nanda List-I (Distinguished Women)
3. M.S. Swaminathan A. Maria Sharapova
4. Satyajit Ray B. Aung San Suu Kyi
Select the correct answer using the codes C. Asma Jahangir
given below: D. Norah Jones
Codes: List-II (Known As)
(a) 1 and 2 1. Human rights activist
(b) 3 and 4 2. Political leader
(c) 1, 2 and 4 3. Singer
(d) 1, 2, 3 and 4 4. Sportsperson
Codes:
Q108. Match List-I (Outstanding Sports persons A B C D
at the Athens Olympics) with List-II (Country) and (a) 1 2 3 4
select the correct answer using the codes given (b) 4 3 2 1
below the Lists : (c) 1 3 2 4
List-I (Outstanding Sports persons at the (d) 4 2 1 3
Athens Olympics)
A. Michael Phelps Q112. Match List-I (Author) with List-II (Book) and
B. Ian Thorpe select the correct answer using the codes given
C. Yulia Nesterenko below the Lists:
D. Nicolas Massu List-I (Author)
15 | P a g e W W W . E D U T A P . C O . I N QUERY? HELLO@EDUTAP.CO.IN / 8146207241
A. Bill Clinton India had plan holiday during 1966-69 due to
B. Henry Kissinger 1. Indo-Pakistan conflict in 1965.
C. Nelson Mandela 2. Severe drought for two successive years.
D. Hillary Clinton Which of the statements given above is/are
List-II (Book) correct?
1. Between Hope and History (a) 1 only
2. Living History (b) 2 only
3. Diplomacy (c) Both 1 and 2
4. Long Walk to Freedom (d) Neither 1 nor 2
Codes:
A B C D Q116. Consider the following statements:
(a) 2 4 3 1 1. The members of the Employees' Provident
(b) 1 3 4 2 Fund and exempted Provident Funds are
(c) 2 3 4 1 eligible for the Employees' Deposit-Linked
(d) 1 4 3 2 Insurance Scheme.
2. Employees' Pension Scheme, 1995 was
Q113. Match List-1 (Distinguished Women) with introduced for the industrial workers.
List-II (Known As/Area) and select the correct 3. Coverage under the Employees' Provident
answer using the codes given below the Lists: Fund and Miscellaneous Provisions Act is
List-I (Distinguished Women) restricted to establishments employing 20 or
A. Shanta Sinha more persons.
B. Kalpana Lajmi Which of the statements given above is/are
C. Romila Thapar correct?
D. Radha Reddy a) 1 and 2
List-II (Known As/Area) b) 1, 2 and 3
1. Film Direction c) 1 and 3
2. Community Leadership d) 3 only
3. Dancer
4. Historian
Q117. Which one of the following statements is
5. Business-woman
not correct?
Codes:
A B C D a) The Special Marriage Act, 1954 extends to all
(a) 2 1 4 3 the States and Union Territories of India
(b) 4 3 5 1 b) The Hindu Marriage Act, 1955 applies to
(c) 2 3 4 1 Buddhists, Jains and Sikhs in addition to Hindus
(d) 4 1 5 3 c) The Hindu Succession Act, 1956 recognizes the
right of women to inherit property of an
intestate equally with men
Q114. Assertion (A): In its Global Development
d) Muslims, Christians and Parsis in India have no
Finance Report 2004, the World Bank has
adoption laws and can take a child under the
categorised India as a less indebted country for the
Guardians and Wards Act, 1890 only under
year 2002.
foster care
Reason (R): During the last decade, India’s
outstanding external debt has considerable
reduced. Q118. Why did Motilal Nehru and Chittaranjan
(a) Both A and R are individually true and R is the Das form a separate group known as Swaraj Party
correct explanation of A. within the Congress?
(b) Both A and R are individually true and R is not a) They were not satisfied with the progress
the correct explanation of A. made by the Congress in achieving Swaraj
(c) A is true but R is false. b) It was their reaction to Mahatma Gandhi's
(d) A is false but R is true sudden decision to suspend the Non-
Cooperation Movement
Q115. Consider the following statements:
16 | P a g e W W W . E D U T A P . C O . I N QUERY? HELLO@EDUTAP.CO.IN / 8146207241
c) They were interested in forming more active
party with revolutionary ideas
d) They felt that their opinions and views were
not given due importance in the Congress
Sessions
Q119. Which of the following is detected and
estimated by the Pollution Check' carried out on
motor cars at service stations?
a) Lead and carbon particles
b) Oxides of nitrogen and sulphur
c) Carbon monoxide
d) Carbon dioxide

Q120. Directions: The following items consist of


two statements, one labelled the 'Assertion (A)
and the other labelled the 'Reason (R)'. You are to
examine these two statements carefully and
decide if the Assertion (A) and the Reason (R) are
individually true and if so, whether the Reason (R)
is a correct explanation of the Assertion (A).
Select your answers to these items using the
Codes given. below and mark your answer-sheet
accordingly
Assertion (A): The salinity of the open seas is very
high compared to that of inland seas.
Reason (R): The water of rivers flowing into open
seas contains dissolved salts.
Codes:
(a) Both A and R are true and R is the correct
explanation of A
(b) Both A and R are true but R is NOT the correct
explanation of A
(c) A is true but R is false
(d) A is false but R

17 | P a g e W W W . E D U T A P . C O . I N QUERY? HELLO@EDUTAP.CO.IN / 8146207241


Visit: www.civilstap.com
For Any Query Mail us: hello@civilstap.com or call us at - (+91)-8146207241
0
S: Without realizing Hayden's intent in calling
SOLUTIONS – PREVIOUS YEAR subcontinent players selfish, we have reacted
QUESTIONS angrily.
The correct sequence should be:
UPSC EPFO APFC – 2004 (a) S-P-R-Q
(b) R-Q-S-P
(c) S-Q-R-P
PART- A (d) R-P-S-Q
Answer: (c)
ORDERING OF SENTENCES Explanation: The correct sequence is S-Q-R-P
which is given in option (c). Therefore, option (c) is
Directions (for the next Five items): the correct answer. S1 talks about ‘we Indians’ so
In the following items each passage consists of six the next sentence which goes here is S as in S it is
sentences. The first sentence (S₁) and the sixth said that ‘we have reacted angrily’. After S the next
sentence (S6) are given in the beginning. The sentence should be Q because S talks about
middle four sentences in each have been removed ‘calling subcontinent players selfish’ while Q states
and jumbled up. These are labelled P, Q, R and S. that ‘our players have not been selfish’. The
You are required to find out the proper sequence following statement should be R as it is the only
of the four sentences and mark accordingly on the one that fits here. The fourth statement is P.
Answer Sheet.
Q2. S1: In the roll call of human history, the sleepy
Example X' has been solved for you. capital town of Madhya Pradesh is indelibly linked
X S1: There wsa a boy named Jack. with one of the world's worst man-made
S6: At last she turned him out of the house. catastrophes.
P: So the mother asked him to find work. S6: Industries across the globe became less
Q: They were very poor. complacent and began tightening their safety
R: He lived with his mother. protocols.
S: But Jack refused to work. P: More than 2,000 died instantly.
The correct sequence should be: Q: On the night of December 3, 1984, a poisonous
(a) R-Q-P-S grey cloud spread out from the Union Carbide
(b) P-Q-R-S Plant in Bhopal, suffocating more than three lakhs
(c) Q-P-R-S of its unsuspecting residents.
(d) R-P-S-Q R: The disaster shook the world, especially when
Answer: (a) hindsight revealed that almost all safety
Explanation: The correct sequence in this example regulations had been flouted by the plant.
is R-Q-P-S which is marked by (a). Therefore, (a) is S: The rest were doomed to wheeze, hobble and
the correct answer. gasp their way through life, their organs "eaten up
by the deadly combination of gases that leaked
out of the plant.
Q1. S₁: We Indians by nature are extremely The correct sequence should be
emotional. (a) Q-R-S-P
S6: What must be appreciated is that none of the (b) S-P-Q-R
members of the Indian team has reacted to this (c) Q-P-S-R
mind game utterance of Hayden. (d) S-R-Q-P
P: Had the political parties in India not been busy Answer: (c)
in fighting among themselves, they would have Explanation: The correct sequence is Q-P-S-R,
asked the Central Government to refuse visa to which is given in option (c). Therefore, option (c) is
Hayden. the correct answer. The second statement after S1
Q: Statisticians are working overtime to prove should be Q as S talks about the worst man-made
how our players have not been selfish. catastrophes and Q talks about what happened
R: Suddenly Hayden has become a villain. there exactly. P should be the following statement
after Q as it tells us that what happened after the
1|P a g e W W W . E D U T A P . C O . I N QUERY? HELLO@EDUTAP.CO.IN / 8146207241
catastrophe. S should come after P because it tells prepare a blueprint for India's future, roughly once
about the rest of the population who survived. R in five years.
should follow S as it gives the reason that why the R: Yet the process of planning didn't change and
disaster happened. excuses were found wars and droughts in the
Q3. S₁: From an importer of milk powder to the 1960s, war and oil shocks in the 70s.
world's largest milk producer. S: Elaborate models were developed, backed by
S6: And to think that it began as a simple immaculate calculations, on the investments and
experiment in a village in Gujarat only in the policies required to achieve the targets.
1960s. The correct sequence should be:
P: Operation Flood is arguably India's biggest and (a) Q-S-P-R
most successful revolution since the Quit India (b) P-R-Q-S
Movement. (c) Q-R-P-S
Q: India's Operation Flood has come a long way (d) P-S-Q-R
from the days when even baby food was imported Answer: (a)
to producing. 84 million tonnes a year. Explanation: The correct sequence is Q-S-P-R,
R: From abject desperation to becoming a global which is given in option (a). Therefore, option (a)
player in the dairy sector. is the correct answer. Statement S1 should be
S: From a mere few villagers and a 275-litre followed by statement Q as it states the action of
processing plant to an operation spread across 23 which statement S1 is mentioning about. After Q,
States, 170 Districts and 90,000 Village statement S should come as it explains what
Cooperatives. happens in a five year plan. According to the
The correct sequence should be: passage, statement P should come after
(a) P-S-Q-R statement S. Statement R should be the last
(b) R-Q-S-P statement after arrangement.
(c) P-Q-S-R
(d) R-S-Q-P Q5. S1: In neighbouring Tamil Nadu, Kodaikanal
Answer: (a) has fallen on bad times.
Explanation: The correct sequence is P-S-Q-R, S6: It is still a source water-albeit unpotable for
which is given in option (a). Therefore, option (a) the thirsty Kodai.
is the correct answer. Here, statement P should P: Trouble started when the focus was shifted
follow statement S1, it is the most suitable of the from Ooty to Kodai in the mid 80s.
following statements as it introduces operation Q: The once pristine surroundings have been
flood. S should come after P because it talks about destroyed by construction that paid scant respect
the expansion of the operation flood. Statement Q to the ecological sensitivity of the area.
should follow statement S here as it states about R: Commercial forces took over the hill station
the ‘long way’ that is covered by ‘operation flood’. large-scale activities and building commenced
R should be the last statement after around the artificial lake created in the 1800s.
rearrangement according to the passage. S: There is large-scale deforestation as well.
The correct sequence should be:
Q4. S1: One of the ironies of the Indian economy's (a) P-S-Q-R
consistent under-performance is that it has been (b) Q-R-P-S
planned. (c) Q-S-P-R
S6: As planning prospered, regional imbalances (d) P-R-Q-S
widened and competitiveness and free enterprise Answer: (d)
were crushed-defeating the fundamental purpose Explanation: The correct sequence is P-R-Q-S,
of planned development. which is given in option (d). Therefore, option (d)
P: It is another matter that in each of the nine Five- is the correct answer. Statement P should be the
Year Plans, barring the first, more targets were first statement after statement S1 as it tells about
missed than met. the starting of the bad times which is mentioned
Q: Within two months of India becoming a in statement S1. Statement R should follow
Republic, the Planning Commission was set up to statement P as it proceeds to tell the further

2|P a g e W W W . E D U T A P . C O . I N QUERY? HELLO@EDUTAP.CO.IN / 8146207241


reason. Statement Q-S should come last in the The correct sequence should be:
arrangement. (a) S-R-Q-P
(b) P-Q-R-S
Directions (For the following 5 items): In the (c) S-Q-R-P
following items some parts of the sentence have (d) P-R-Q-S
been jumbled up. You are required to re-arrange Answer: (a)
these parts which are labelled P, Q. R and S to Explanation: The correct sequence of the
produce the correct sentence. Choose the proper sentence is S-R-Q-P. Therefore, option (a) is the
sequence and mark in your Answer Sheet correct answer. The sentence should start with S
accordingly. as it talks about the karnal girl Kalpana Chawla and
her dreams and her final achievement which is
Example Z' has been solved for you. given by the sequence SRQP. Therefore, after
Z. It is well-known that the effect/(P) is very rearrangement option (a) will be the right answer.
bad/(Q) On children/(R) of cinema/(S)
The correct sequence should be: Q8. That so suggests that there are certain cultural
(a) P-S-R-Q resources/(P) many Indians excel as
(b) S-P-Q-R expatriates/(Q) that enable them to succeed while
(c) S-R-P-Q living away from home/(R) perhaps unique to
(d) Q-S-R-P Indians/(S).
Answer: (a) The correct sequence should be:
Explanation: (a) S-P-Q-R
The proper way of writing the sentence is "It is (b) Q-R-S-P
well-known that the effect of cinema on children (c) S-R-Q-P
is very bad." This is indicated by the sequence P-S- (d) Q-P-S-R
R-Q and so (a) is the correct answer. Answer: (b)
Explanation: The correct sequence after
Q6. War minus shooting but there was nothing to rearrangement is Q-R-S-P. Therefore, option (b) is
suggest any/(P) was how cricket between India the correct answer. The sentence can not start
and Pakistan/(Q) increase of animosity between with S as it does not make any sense. Hence,
the players or the spectators/(R) was visualised options (a) and (c) are eliminated. Q should be
before the first ball was bowled/(S). followed by R to make the sentence grammatically
The correct sequence should be: and meaningfully correct, so option (d) will be
(a) P-R-Q-S eliminated. Hence, after rearrangement option (b)
(b) Q-S-P-R will be the right answer which will make the
(c) P-S-Q-R sentence correct.
(d) Q-R-P-S
Answer: (b)
Explanation: The correct sequence of the Q9. The advent we entertained and informed
sentence is Q-S-P-R. Therefore, option (b) is the ourselves/(P) of satellite television was a silent/(Q)
correct answer. The sentence can not end with P and completely changed the way/(R) revolution
as it ends with ‘any’ which is never the end of any which created a whole new industry/(S).
sentence. Neither the sentence can start with P as The correct sequence should be:
is does not make any sense. Hence, options (a) and (a) Q-R-S-P
(c) should be eliminated. Option (b) makes the (b) P-S-R-Q
correct sense of the sentence after (c) Q-S-R-P
rearrangement. Therefore, it is the right answer. (d) P-R-S-Q
Answer: (c)
Q7. In 1997, NASA fact-finding mission on Explanation: The correct sequence of the
microgravity/(P) by becoming the first Indian sentence is Q-S-R-P. Therefore, option (c) is the
woman to rocket into space on a/(Q) to reach out correct answer. The sentence can not start with P
for the stars, created history/(R) Kalpana Chawla, it does not make any sense as ‘advent’ means the
a Karnal girl who wanted /(S). arrival of a notable person or thing. So, the
3|P a g e W W W . E D U T A P . C O . I N QUERY? HELLO@EDUTAP.CO.IN / 8146207241
sentence should start with Q as it tells about the tables on which all flora and fauna are dependent.
advent of satellite television. Hence, according to Thus, the hydrosphere permits organic existence.
the sentence the correct sequence after The hydrosphere has strange characteristics
rearrangement is SQRP which is given in option (c). because water has properties unlike those of any
other liquid. One anomaly is that water upon
freezing expands by about 9 percent, whereas
Q10. The President the aspirations of the most other liquids contract on cooling. For this
Scheduled Castes and the Scheduled Tribes/(P) reason, ice floats on water bodies instead of
with political parties, industry and other sinking to the bottom. If the ice sank, the
bodies/(Q) revealed that the Government would hydrosphere would soon be frozen solidly, except
initiate a dialogue/(R) on how best the private for a thin layer of surface melt water during the
sector could fulfill/(S). summer season. Thus, all aquatic life would be
The correct sequence should be: destroyed and the interchange of warm and cold
(a) S-P-R-Q currents, which moderates climates, would be
(b) R-Q-S-P notably absent.
(c) S-Q-R-P Another outstanding characteristic of water is that
(d) R-P-S-Q water has a heat capacity which is highest of all
Answer: (b) liquids and solids except ammonia. This
Explanation: The correct sequence of the characteristic enables the oceans to absorb and
sentence is R-Q-S-P. Therefore, option (b) is the store vast quantities of heat, thereby often
correct answer. The sentence should start with preventing climatic extremes. In addition, water
The President ‘revealed’ which is given in R, so R dissolves more substances than any other liquid. It
should come first after rearrangement, followed is this characteristic which helps make oceans a
by Q as it mentions the dialogue ‘with political great storehouse for minerals which have been
parties’. Q should be followed by S-P as it makes washed down. from the continents. In several
the sentence correct in meaning. The correct areas of the world these minerals are being
sequence after rearrangement is RQSP which is commercially exploited. Solar evaporation of salt
given in option (b). Hence, option (b) is the right is widely practised, potash is extracted from the
answer. Dead Sea, and magnesium is produced from sea
water along the American Gulf Coast.
COMPREHENSION Q11. Which one of the following is correct?
According to the passage, the hydrosphere is not
Directions (For the following 5 items): (a) responsible for all forms of life
The passage given below is followed by questions (b) able to modify weather
based on its content. After reading the passage, (c) a source of natural resources:
choose the best answer to each question. Answer (d) in danger of freezing over
all the five questions following the passage on the Answer: (d)
basis of what is stated or implied in the passage. Explanation: Option (d) is the correct answer here.
As given in the second paragraph of the passage
PASSAGE about the hydrosphere, we can say that
hydrosphere is not in danger of freezing over
Ocean water plays an indispensable role in because if that happens then all the aquatic life
supporting life. The great ocean basins hold. about would be destroyed and the interchange of warm
300 million cubic miles of water. From this vast and cold currents, which moderates climates,
amount, about 80,000 cubic miles of water are would be notably absent. Therefore, option (d) is
sucked into the atmosphere each year by correct.
evaporation and returned by precipitation and
drainage to the ocean. More than 24,000 cubic Q12. What is the reason that fish can survive in the
miles of rain descend annually upon the oceans?
continents. This vast amount is required to (a) They do not need oxygen
replenish the lakes and streams, springs and water (b) Ice floats on the surface of the oceans

4|P a g e W W W . E D U T A P . C O . I N QUERY? HELLO@EDUTAP.CO.IN / 8146207241


(c) Evaporation and condensation create a water (b) i.e. ‘the heat capacity of water is very high’ is
cycle the right choice as it is reason that climate
(d) There are oceans currents in the extremes are prevented in nature.
Answer: (b)
Explanation: Option (b) is the correct answer. Q15. Which of the following characteristics of
According to the second paragraph of the passage water does the author mention in the passage?
it is given that ice floats on water bodies instead of 1. Water expands when it is frozen
sinking to the bottom and if the ice sank, the 2. Water is used as a condensing medium for
hydrosphere would soon be frozen solidly, except thermal power plants
for a thin layer of surface melt water during the 3. Water is a good solvent
summer season. Thus, all aquatic life would be Select the correct answer using the codes given
destroyed and the interchange of warm and cold below:
currents, which moderates climates, would be Codes:
notably absent. This explains that fishes can (a) 1 only
survive in the ocean because of the ice that floats (b) 2 and 3
on the surface of the ocean. Hence, (b) is the (c) 1 and 3
correct option. (d) 1, 2 and 3
Answer: (c)
Q13. What is the author's main purpose in the Explanation: Option (c) is the correct answer. It
passage given above? given in the passage that, “One anomaly is that
(a) To describe the properties and uses of water water upon freezing expands by about 9 percent,
(b) To illustrate the importance of conserving whereas most other liquids contract on cooling”
water and also “water dissolves more substances than
(c) To explain how water is used in commerce and any other liquid”. Therefore, only option 1 and 3
industry are mentioned in the passage whereas option 2 is
(d) To reveal the extent of the earth's ocean not mentioned anywhere in the given passage.
masses Hence, option (c) is the right answer.
Answer: (b)
Explanation: Option (b) is the correct answer. ANTONYMS
According to the author, the main purpose of the
given passage is to illustrate the importance of Directions (For the following 5 items):
conserving water and not to commercially exploit Each of the following five items consists of a word
the water resources and minerals as the in capital letters, followed by four words or groups
hydrosphere permits organic existence. of words. Select the word or group of words that
is most nearly opposite in meaning to the word in
Q14. What is the reason that climatic extremes are capital letters.
prevented in nature? Q16. OUTLANDISH
(a) A vast quantity of water is held in the great (a) Stubborn
ocean basins (b) Conventional
(b) The heat capacity of water is very high (c) Agitated
(c) Solar evaporation from the oceans uses high (d) Fearful
technology Answer: (b)
(d) Water dissolves many substances Explanation: Option (b) is the correct answer.
Answer: (b) ‘Outlandish’ means ‘looking or sounding bizarre or
Explanation: Option (b) is the correct answer. It is unfamiliar.’, the opposite for this is ‘conventional’
given in the third paragraph of the passage that which means ‘based on or in accordance with what
another outstanding characteristic of water is that is generally done or believed.’
water has a heat capacity which is highest of all ‘Stubborn’ means having or showing dogged
liquids and solids except ammonia. This determination not to change one's attitude or
characteristic enables the oceans to absorb and position on something, especially in spite of good
store vast quantities of heat, thereby often arguments or reasons to do so.
preventing climatic extremes. Therefore, Option
5|P a g e W W W . E D U T A P . C O . I N QUERY? HELLO@EDUTAP.CO.IN / 8146207241
‘Agitated’ means feeling or appearing troubled or ‘deceitfulness’ which means ‘having a tendency or
nervous. disposition to deceive or give false impressions’.
‘Fearful’ means feeling or appearing troubled or ‘Self-condemnation’ means the blaming of oneself
nervous. for something.
‘Reprisal’ means an act of retaliation.
Q17. SCANTY ‘Punctuality’ means the fact or quality of being on
(a) Arrogant time.
(b) Plentiful
(c) Painful Q20. PONTIFICATE
(d) Handsome (a) To pilfer
Answer: (b) (b) To leave one's country
Explanation: Option (b) is the correct answer. (c) To speak in a humble manner.
‘Scanty’ means ‘small or insufficient in quantity or (d) To spend recklessly
amount.’, the opposite for this is ‘plentiful’ which Answer: (c)
means ‘existing in or yielding great quantities; Explanation: Option (c) is the correct answer.
abundant.’ ‘Pontificate’ means ‘to speak or write and give
‘Arrogant’ means having or revealing an your opinion about something as if you knew
exaggerated sense of one's own importance or everything about it and as if only your opinion was
abilities. correct’, the opposite of this is option (c) i.e. to
‘Painful’ means (of a part of the body) affected speak in a humble manner.
with pain. ‘To pilfer’ means to steal (things of little value).
‘Handsome’ means (of a number, sum of money, ‘Recklessly’ means without regard to the danger or
or margin) substantial or (of a man) good-looking. the consequences of one's actions; rashly.

Q18. PROPENSITY SYNONYMS


(a) Disinclination
(b) Forecast Directions (For the following 5 items):
(c) Stagnation Each of the following five items consists of a word
(d) Restlessness in capital letters, followed by four words or groups
Answer: (a) of words. Select the word or group of words that
Explanation: Option (a) is the correct answer. is most similar in meaning to the word in capital
‘Propensity’ means ‘an inclination or natural letters.
tendency to behave in a particular way.’, the Q21. COMMISERATION
opposite for this is ‘disinclination’ which means ‘a (a) Commission
reluctance or lack of enthusiasm.’ (b) Vastness
‘Forecast’ means a calculation or estimate of (c) Sympathy
future events, especially coming weather or a (d) Commendation.
financial trend. Answer: (c)
‘Stagnation’ means the state of not flowing or Explanation: Option (c) is the correct answer.
moving. ‘Commiseration’ means ‘sympathy and sorrow for
‘Restlessness’ means the inability to rest or relax the misfortunes of others; compassion.’, the word
as a result of anxiety or boredom. which has the same meaning as commiseration is
‘sympathy’.
Q19. RECTITUDE ‘Commission’ means an instruction, command, or
(a) Self-condemnation role given to a person or group.
(b) Deceitfulness ‘Vastness’ means very great extent or size;
(c) Reprisal immensity.
(d) Punctuality ‘Commendation’ means a formal or official praise.
Answer: (b)
Explanation: Option (b) is the correct answer. Q22. SLIPSHOD
‘Rectitude’ means ‘morally correct behaviour or (a) Vulgar
thinking; righteousness.’, the opposite of this is (b) Careless
6|P a g e W W W . E D U T A P . C O . I N QUERY? HELLO@EDUTAP.CO.IN / 8146207241
(c) Commonplace (b) To go in disguise
(d) Retaliatory (c) To mesmerize
Answer: (b) (d) To run in a marathon race.
Explanation: Option (b) is the correct answer. Answer: (b)
‘Slipshod’ means ‘characterized by a lack of care, Explanation: Option (b) is the correct answer.
thought, or organization.’, the word which has the ‘Masquerade’ means ‘a false show or pretence.’,
same meaning as slipshod is ‘careless’. the group of words that is similar to the meaning
‘Vulgar’ means lacking sophistication or good of masquerade is ‘to go in disguise’, which means
taste. ‘to give a new appearance to a person or thing,
‘Retaliatory’ means (of an action) characterized by especially in order to hide its true form’.
a desire for revenge. ‘To provide support’ means to give
encouragement and approval to someone or
Q23. COGENT something because you want the person or thing
(a) Forceful to succeed.
(b) Objectionable ‘To mesmerize’ means to hold completely the
(c) Exceptional attention or interest of someone.
(d) False ‘To run in a marathon race’ means to run in a long-
Answer: (a) distance race.
Explanation: Option (a) is the correct answer.
‘Cogent’ means ‘(of an argument or case) clear, Part - B
logical, and convincing.’, hence, a cogent Q26. Which one of the following is a super-cooled
argument, reason, etc. is clearly expressed and liquid?
persuades people to believe it. Here the synonym (a) Ice-cream
of cogent is ‘forceful’ which ‘means strong and (b) Teflon
assertive; vigorous and powerful.’ (c) Glass
‘Objectionable’ means arousing distaste or (d) Mercury
opposition; unpleasant or offensive. Answer: C
‘Exceptional’ means not like most others of the Explanation:
same type; unusual Supercooling is the process of chilling a liquid
‘False’ means not according with truth or fact; below its freezing point, without it becoming solid.
incorrect. • A super-cooled liquid is a liquid below its
freezing point that has not crystalized to
Q24. PONDEROUS freeze.
(a) Facile • Glass is an example of supercooled liquid.
(b) Exorbitant Hence option C is correct.
(c) Cautious • Teflon is not a product on its own, but a brand
(d) Unwieldy name of a product. It refers to a chemical
Answer: (d) coating known as polytetrafluoroethylene
Explanation: Option (d) is the correct answer. (PTFE).
‘Ponderous’ means ‘slow and clumsy because of • Mercury is the only liquid metal at room
great weight.’, the synonym of ponderous is temperature.
‘unwieldy’ which means ‘(of an object) difficult to
move because of its size, shape, or weight.’ Q27. Why does bleeding occur through nose in
‘Facile’ means ignoring the true complexities of an high mountainous regions?
issue; superficial. (a) The pressure of blood in capillaries is greater
‘Exorbitant’ means (of a price or amount charged) than the outside pressure
unreasonably high. (b) The pressure at high altitudes is greater than
‘Cautious’ means (of a person) careful to avoid that on the plains
potential problems or dangers. (c) The blood pressure of a person increases at
high altitudes
Q25. MASQUERADE (d) The blood pressure fluctuates and finally
(a) To provide support decreases Drastically
7|P a g e W W W . E D U T A P . C O . I N QUERY? HELLO@EDUTAP.CO.IN / 8146207241
Answer: A Q30. Which one of the following is not a Vitamin?
Explanation: (a) Folic acid
Atmospheric pressure decreases with increase in (b) Oleic acid
altitude. (c) Pantothenic acid
• At higher altitudes the blood pressure inside (d) Ascorbic acid
our body is more than the atmospheric Answer: B
pressure which forces the blood to ooze out Explanation:
from openings like the nose. • Folate is the natural form of vitamin B9, water-
• Hence the nose bleeding and Hence option A soluble and naturally found in many foods like
is correct. citrus fruits. It is also added to foods and sold
as a supplement in the form of folic acid
Q28. Which one of the following causes small air • Vitamin B5, also called pantothenic acid, is
bubbles in water, which is stored in a glass vessel, one of 8 B vitamins. All B vitamins help the
to appear to shine with a silvery luster? body convert food (carbohydrates) into fuel
(a) Reflection of light rays (glucose), which the body uses to produce
(b) Refraction of light rays energy. These B vitamins, often referred to as
(c) Polarization of light rays B complex vitamins, also help the body use fats
(d) Total internal reflection and protein.
Answer: D • Ascorbic acid (vitamin C) is used as a dietary
Explanation: supplement when the amount of ascorbic acid
The reason why the air bubble shines in water is in the diet is not enough.
because of the total internal reflection. • Oleic acid is an omega-9 fatty acid. It can be
• Total internal reflection is a phenomenon in made by the body. It is also found in foods.
which light travels back in the same medium Highest levels are found in olive oil and other
or reflects back. It occurs when light travels edible oils. It is not related to any Vitamin.
from an optically denser to a rarer medium. Hence option B is correct answer.
• This happens in an air bubble in water when
light travels from water to air and back out. Q31. Which one of the following statements is
This creates the shine or the silvery lustre. not correct?
Hence option D is correct. (a) The deficiency of Vitamin A causes night
blindness
Q29. Weight of a body on the surface of the earth (b) Pain in the muscles and joints along with
is W₁. The weight of the same body is W2, at a fatigue, loss of weight and poor wound healing
height of 500 metres above the surface, and W3 are the symptoms of the deficiency of Vitamin
at a depth of 500 metres below the surface of the C
earth. Which one of the following is the correct (c) Bleeding gums and loosening of teeth are
relation? symptoms of the deficiency of Vitamin B
(a) W3 > W1 > W2 (d) The deficiency of Vitamin B can cause
(b) W₁ > W₂; W₁ > W3 inflammation of skin, memory disorder and
(c) W₁ < W₂ <W3 Diarrhoea
(d) W₂ > W3 > W₁ Answer: C
Answer: A Explanation:
Explanation: Vitamins and deficiency diseases
The Weight of an object is given by multiplying its The deficiency of different Vitamins causes
mass (m) and acceleration due to gravity (g) as different deficiency diseases described as follows:
W=mxg. • In its more severe forms, vitamin A deficiency
• The value of g varies with altitude in an inverse contributes to blindness by making the cornea
manner and as the altitude increases the value very dry, thus damaging the retina and cornea.
of g decreases and as altitude (height) Hence statement A is correct.
decreases, the value of g increases. • Vitamin C is found in citrus fruits and
• Thus, W3>W2>W1 is the correct sequence. vegetables. Scurvy results from a deficiency of
vitamin C in the diet.
8|P a g e W W W . E D U T A P . C O . I N QUERY? HELLO@EDUTAP.CO.IN / 8146207241
o Symptoms may not occur for a few is a correct explanation of the Assertion (A). Select
months after a person's dietary intake your answers to these items using the Codes given.
of vitamin C drops too low. below and mark your answer-sheet accordingly
o Bruising, bleeding gums, weakness,
fatigue, weight loss, poor wound Codes:
healing, and rash are among scurvy (a) Both A and R are true and R is the correct
symptoms. Hence statement B is explanation of A
correct and statement C is incorrect. (b) Both A and R are true but R is NOT the correct
• Vitamin B12 deficiency may lead to a reduction explanation of A
in healthy red blood cells (anaemia). The (c) A is true but R is false
nervous system may also be affected. Diet or (d) A is false but R is true
certain medical conditions may be the cause.
Symptoms are rare but can include fatigue, Q33. Assertion (A): Glycerol is a constituent of
breathlessness, numbness, poor balance and shaving cream.
memory trouble. Hence statement D is Reason (R): Glycerol is an antiseptic.
correct. Answer: C
Explanation:
Q32. Which one of the following elements is not Glycerol is added to shaving soaps to prevent
naturally found in human beings? rapid drying and softening the skin. Hence
(a) Copper Assertion is correct but Reason is false.
(b) Zinc
(c) Iodine Q34. Assertion (A): Phenol is commonly used as a
(d) Lead disinfectant.
Answer: C Reason (R): Phenol is not harmful to living human
Explanation: tissues even at a very high concentration.
• Copper is a mineral that is found throughout Answer: C
the body. It helps your body make red blood Explanation:
cells and keeps nerve cells and your immune Phenol is an antiseptic and disinfectant used in a
system healthy. It also helps form collagen, a variety of settings.
key part of bones and connective tissue. • Phenol is an antiseptic and disinfectant. It is
• Zinc is found in cells throughout the body. It is active against a wide range of micro-organisms
needed for the body's defensive (immune) including some fungi and viruses but is only
system to properly work. It plays a role in cell slowly effective against spores. Phenol has
division, cell growth, wound healing, and the been used to disinfect skin and to relieve
breakdown of carbohydrates. Zinc is also itching. Hence Assertion is true.
needed for the senses of smell and taste. • Exposure to phenol may cause irritation to the
• Iodine is an essential trace mineral not made skin, eyes, nose, throat, and nervous system.
by the body so must be obtained by food or • Some symptoms of exposure to phenol are
supplements. It is found naturally in some weight loss, weakness, exhaustion, muscle
foods and is added to supplements and some aches, and pain. Severe exposure can cause
salt seasonings. Hence option C is correct. liver and/or kidney damage, skin burns,
• Although Lead is not made by the human body tremor, convulsions, and twitching. Hence
and is non-biodegradable and harmful for the Reason is false.
human body, it is found in the human bones
Q35. What is the number of triangles that can be
and teeth.
formed whose vertices are at the vertices of an
octagon but have only one side common with
Directions: The following Two (7 & 8) items consist
that of the octagon?
of two statements, one labelled the 'Assertion (A)
and the other labelled the 'Reason (R)'. You are to (a) 16
examine these two statements carefully and (b) 24
decide if the Assertion (A) and the Reason (R) are (c) 32
individually true and if so, whether the Reason (R) (d) 48
9|P a g e W W W . E D U T A P . C O . I N QUERY? HELLO@EDUTAP.CO.IN / 8146207241
Answer: C Q38. Three dice are rolled. What is the
Explanation: probability that at least one die will show 6?
There are 2 vertices that will be used for base (a) 1/6
side. (b) 1/36
Another 2 vertices which are adjacent to each (c) 91/216
vertices taken will make common side, so we (d) 125/216
have to minus these ae well. Answer: C
Therefore, Explanation:
The number of triangles formed with only one Dice have values from 1 to 6
side common with octagon is 8× (8-2 -2) = 32 So, the probability of Dice 1/Dice2/Dice3 will
show 6 = 1/6
Q36. A letter is taken out at random from the
Probability of not showing 6 = 5/6
word 'ASSAM' and a letter is taken out at random
from the word 'NAGALAND'. What is the
So, require probability = 6 on only 1 Dice or 6 on
probability that the two letters are the same?
any 2 Dice or 6 on all 3 dice
(a) 1/20 = 3C1*(1/6)*(5/6)*(5/6) + 3C2*(1/6)*(1/6*(5/6) +
(b) 3/20 (1/6)*(1/6)*(1/6)
(c) 1/40 = 75/216 + 15/216 + 1/216
(d) 31/40 = 91/216
Answer: B Alternate method-
Explanation: Probability of no 6 by any dice = (5/6)*(5/6)*(5/6)
In ASSAM, we have 2As, 2Ss, 1 M = 125/216
In NAGALAND, we have 2Ns, 3As, 1G,1L, 1D So, required probability = 1 – 125/216 = 91/216
1 letter is taken from each word-That needs to be
the same. So, these can be As only.
So, probability of A taken from ASSAM is 2C1/5C1 = Q39. How many numbers are there between 999
2/5 and 10000 such that at least one of their digits is
and A from NAGALAND is 3C1/8C1 = 3/8 5?
(a) 2879
So, required probability = (2/5)*(3/8) = 3/20 (b) 3168
(c) 3200
(d) 3224
Q37. A set of 5 parallel lines is intersected by
another set of 4 parallel lines so as to form a Answer: B
number of parallelograms. How many Explanation:
parallelograms can be formed? There is no digit to be 5 on any place in 4 digits
(a) 12 number from 1000 to 9999 = 8*9*9*9 (because
(b) 20 of at 1000s place 0 and 5 both cannot come so, 8
(c) 40 chances likewise for the unit, 10s and 100s place
(d) 60 5 cannot come so chance will be 9)
Answer: D = 5832
Explanation:
Total number between 999 and 10000 = 9000
For a parallelogram, we need 2 pairs of parallel
So, required numbers are = 9000 – 5832 = 3168
lines. So, we can "choose" any two lines from the
first set of 4 parallel lines 'and' choose any two
lines from the second set of 5 parallel lines. Q40. Four different toys are to be distributed
among two children such that each child gets at
Hence the value we need is least one toy. In how many different ways can
4C × 5C = 6 × 10 = 60
2 2 this be done?
(a) 6
Thus 60 parallelograms are formed.
(b) 8
(c) 12
10 | P a g e W W W . E D U T A P . C O . I N QUERY? HELLO@EDUTAP.CO.IN / 8146207241
(d) 14 (c) 127
Answer: (d) 128
Explanation: Answer: C
There are 4 toys to be distributed among 2 Explanation:
children such that each child gets at least 1 toy. There are 7 people he wishes to invite.
Required ways = 4C1*3C3 + 4C2*2C2 + 4C3*1C1 Every friend has 2 choices either accept or decline
=4+6+4 the invitation.
= 14 ways. So, total ways = 2*2*2*2*2*2*2 =27
Now, we know 1 or more may accept the
Q41. A stadium has 10 gates. In how many invitation(given). So, no person is coming to part =
different ways can 3 persons enter the stadium? 1 way.
(a) 10 So, required ways = 27 – 1
(b) 30 = 128 – 1
(c) 310 = 127
(d) 1000
Answer: Q44. There are three envelopes with three
Explanation: D
different addresses written on them. Three
There are 10 gates
letters are to be enclosed in these three
Total persons = 3
envelopes such that one letter goes inside one
Each person can choose any gate out of 10.
envelope without seeing the address written on
Total ways are 10*10*10 = 1000
any envelope. What is the probability that each
of the three letters goes into the envelope with
Q42. At the conclusion of a party, a total number
correct address written on it?
of 28 handshakes were exchanged. Every person
(a) 1/27
in the party shook hands with every other person
(b) 1/6
who attended the party. What was the total (c) 1/9
number of persons who attended the party? (d) 1/3
(a) 8 Answer: B
(b) 14 Explanation:
(c) 28 Total no. of ways =3!=6 and favourable no.of ways
(d) 56 =1.
Answer: A So, required probability = 1/6
Explanation:
Let n be the number of people present.
Q45. A person and his wife appear in the
For every handshake, 2 persons are required.
interview for the two vacancies of a post. The
Since each handshake is between two persons, the
probability of the selection of the husband and
total number of handshakes = nC2 =28
that of the wife are 1/7 and 1/5 respectively.
n! /{(n-2)! * 2!} = 28
What is the probability that only one of them will
n(n-1) = 56
be selected?
So, n =8
(a) 1/7
(b) 1/5
Q43. Amit has 7 friends whom he wishes to invite (c) 2/7
to a dinner. Out of his 7 friends, 1 or more may (d) 11/35
accept the invitation. In how many different ways Answer:
can Amit's 7 friends attend the party? Explanation: C
(a) 96 Probability of selection of husband = 1/7, it means
(b) 112 of not selecting = 6/7
11 | P a g e W W W . E D U T A P . C O . I N QUERY? HELLO@EDUTAP.CO.IN / 8146207241
Probability of selection of wife = 1/5, it means of Now, length reduced by 20%. So, now length –
not selecting = 4/5 (4/5)L
Breadth remains same
So, new figure is square. So, (4/5)L = B
So, only 1 will be selected = “Wife selected, and
So, New area = (4/5)L*B
husband is not selected” or “husband selected and
wife doesn’t” 1. Difference in area = [{L/B – (4/5) L*B}/ L*B]*100
= (1/5)* (6/7) + (1/7)* (4/5) = 20% (So, wrong as given in the statement 1 to be
= 6/35 + 4/35 25%)
= 10/35 2. perimeter of square = 2(4/5) L + 2B = (8/5) L +
= 2/7 2B
Perimeter of rectangle = 2(L + B)
Difference = [{2L + 2B – (8/5)L – 2B}/ (2(L + B)]*100
Q46. What is the nth term of the series √3, √6, = [(2/5)L/ 2(L + (4/5)L]*100 = 11.11% (So, right it is
2√3, ...? approx. 11%)
(a) √3(√2)n-1
(b) √2 (√3)n-1 3. Diagonal of rectangle = √(L2 + B2) = √(L2 + (4/5 L)2
(c) √3 (√2)n = (L/5)*√41
(d) √2 (√3)n Diagonal of square =side*√2 = (4/5)L*√2
Answer: A Require %age = [{(√41 -
Explanation: 4√2)*(L/5)}/(L/5)*√41]*100 = 11.65% (So, right it
√6/√3 = 2√3/ √6 is approx. 12%)
√2 = √2 Hence option D.
So, this means it is a G.P.
Directions (For the next four items):
Tn = arn-1 (where Tn = nth term, a = first term, r =
Refer to the graphs given below and answer the
difference) items that follow:
Tn = √3(√2)n-1

Q47. The length of a rectangle is reduced by 20%


and breadth is kept constant, and the new figure
that is formed is a square.
Consider the following statements:
1. The area of the square is 25% less than the area
of rectangle.
2. The perimeter of square is approximately 11%
less than the perimeter of rectangle.
3. The diagonal of square is approximately 12%
less than the diagonal of rectangle.
Which of the statements given. above is/are
correct?
(a) 1 only
(b) 1 and 2
(c) 1, 2 and 3
(d) 2 and 3
Answer: D
Explanation:
Let length of the rectangle = L, breadth of the Q48. What is the contribution of EU in the foreign
rectangle = B investment inflows for the year 1996 in US $ Mn.?
Area= L*B (a) 1840
(b) 2000

12 | P a g e W W W . E D U T A P . C O . I N QUERY? HELLO@EDUTAP.CO.IN / 8146207241


(c) 460 So, foreign investment approvals in 1997 = 4*2400
(d) Cannot be determined with the given data = 9600
Answer: D
Explanation: Now, NAFTA got $ 700mn in 1996
As per the information 23% approval for EU in That is 800/2000)*100 = 40%
1996.
But we don’t know how much inflows out of it. So, in 1997 it got 40% of 2400 = 960
So, cannot be determined with the given data.
Q51. In which year is the ratio of foreign
Q49. What are the foreign investment approvals investment approvals to foreign investment
from SAARC, ASEAN and EU combined together in inflows highest?
the year 1996 (in US $ Mn.)? (a) 1992
(a) 2240 (b) 1993
(b) 5000 (c) 1994
(c) 3500 (d) 1995
(d) 2600 Answer: D
Answer: A Explanation:
Explanation: We can clearly see in 1995 ratio of foreign
Foreign investment approval for 1996 = US $ 8000 investment approval to inflows in highest =
mn. 9900/2000
%age of approval for SAARC, ASEAN and EU = 2 +
3 + 23 =28% Q52. There are four persons A, B, C and D. The
So, 28% of 8000 = 2240 total amount of money with A and B together is
equal to the total amount of money with C and D
Q50. If the net foreign investment inflows for the together. But the total amount of money with B
year 1997 were to grow by 20% and if the and D together is more than the amount of
proportion of inflows was to remain the same as money with A and C together. The amount of
in case of the approvals for the year 1996, what money with A is more than that with B. Who has
would be the foreign investment inflows from the least amount of money?
NAFTA (in US $ Mn.) If NAFTA got 800 inflow in (a) B
1996? (b) C
(a) 800 (c) D
(b) 500 (d) Cannot be determined
(c) 840 Answer: B
(d) 960 Explanation:
Answer: D A+B=C+D
Explanation: B+D>A+C
Net foreign investment inflows for 1996 = 2000 A>B
Net foreign investment inflows for 1997 increased
Now,
by 20% = 1.2*2000 = 2400
B + D > A + C and A > B
⇒B+D>A+C>B+C
Proportion of inflows with respect to approvals in
B+D>B+C
1996 = (2000/8000) *100 = 25% or 1/4
So, D > C
As per given proportion is same in the year 1997
Now, B + D > A + C ⇒ B + D = A + C + K
with respect to 1996.
Also, A + B = C + D ⇒ B - D = C - A
⇒ 2B = 2C + K
13 | P a g e W W W . E D U T A P . C O . I N QUERY? HELLO@EDUTAP.CO.IN / 8146207241
B>C (v) D is seating on the seat number 3.
A>B>C&D>C Q54. Who is sitting on the seat number 7?
Clearly, each one of A, D, and B has more amount (a) A
than C. Hence, C has the least amount. (b) B
(c) C
Q53. A printer uses a total number of 4893 digits (d) E
in order to number all the pages of his book. He Answer: C
starts with the digit 1 for the first page of the Explanation:
book. How many pages does the book have? The couple E and S occupy the seats at extreme
(a) 4892 right.
(b) 2322 D is seating on the seat number 3
(c) 2100 B and R are celebrating their wedding anniversary
(d) 1500 and hence are sitting at the centre.
Answer: D
Explanation:
1 digit pages = 1 to 9 = 9
2 digits pages = 10 to 99 = 90
3 digits pages = 100 to 999 = 900
4 digit pages = let’s say x
Now, C, who is not sitting at extreme left and Q, who is
Number of digits used till 999 pages = 9*1 + 90*2 not sitting on the seat number 2, are not husband
+ 900*3 = 9 + 180 + 2700 = 2889 and wife.
Therefore, A and T are not husband and wife.
4x + 2889 = 4893
4x = 2004
X = 501
So, total pages are 9 + 90 + 900 + 501 =1500

Directions (For the next two items):


Based on the information given below, answer the So, C is sitting at seat number 7.
two items which follow it:
Five ladies A, B, C, D, E go to a cinema hall along Q55. If in the last condition (v), it is stated that D
with their husbands P, Q, R, S, T (not necessarily in sits on the seat number 7, then who will sit on the
that order). Seats numbered from 1 to 10 are seat number 4?
reserved for these 5 married couples. Each of the (a) P
five ladies sits on the seat immediately to the left (b) T
of her husband. (c) P or T
(i) The couple E and S occupy the seats at extreme (d) Cannot be determined
right. Answer:
(ii) C, who is not sitting at extreme left and Q, who Explanation:
is not sitting on the seat number 2, are not Now, D sits on seat number is 7.
husband and wife.
(iii) B and R are celebrating their wedding
anniversary and hence are sitting at the centre.
(iv) A and T are not husband and wife.

14 | P a g e W W W . E D U T A P . C O . I N QUERY? HELLO@EDUTAP.CO.IN / 8146207241


- Its chairman was L.K. Jha, a member of the
Indian Civil Service who had specialised in
the economic affairs of the country.
• The agitation for State autonomy led to the
creation of the R. S. Sarkaria Commission by
the Central Government to recommend
changes in the Centre-State relationship in
Q56. A 2-member committee is to be constituted
1983.
out of 4 men and 2 women. What is the
• The first Administrative Reform Commission
probability that a committee thus formed will was established on 5 January 1966. The
have exactly one woman? Administrative Reforms Commission was
(a) 1/3 initially chaired by Morarji Desai, and later on
(b) 1/4 K. Hanumanthaiya became its chairman when
(c) 7/15 Desai became the Deputy Prime Minister of
India.
(d) 8/15
• In 1962, Lal Bahadur Shastri appointed K.
Answer: D
Santhanam to preside over the committee on
Explanation:
anti-corruption. Because of its thorough
Total member = 4 + 2 = 6
investigative work and recommendations, the
Committee should have exactly 1 woman. Committee earned a reputation as
So, required probability = (1 Men and 1 women) Santhanam's Committee on Anti-Corruption.
out of toal Hence C is the correct answer.
= 4C1 * 2C1/ 6C2
= 8/15 Q58. Consider the following statements:
1. The first report of the Administrative Reforms
Q57. Match List-I (Commission/Committee) with Commission recommended the creation of Lok
List-II (Chairman) and select the correct answer Pal and Lok Ayukta in India.
using the code given below the lists: 2. Different articles under the Constitution of
List-I (Commission/Committee) India provide for setting up the Union Public
A. Economic Reforms Commission Service Commission, the Planning Commission
B. Commission on Centre-State Relations and the Vigilance Commission.
C. Administrative Reforms Commission Which of the statements given above is/are
D. Committee on the Prevention of Corruption correct?
List-II (Chairman) (a) 1 only
1. R.S. Sarkaria (b) 2 only
2. K. Hanumanthaiya (c) Both 1 and 2
3. L.K. Jha (d) Neither 1 nor 2
4. K. Santhanam Answer: A
5. D.S. Kothari Explanation:
Code: • The Administrative Reforms Commission (ARC)
A B C D of India (1966–1970) recommended the
(a) 3 4 2 1 setting up of two special authorities
(b) 2 1 5 4 designated as ‘Lokpal’ and ‘lokayukta’ for the
(c) 3 1 2 4 redressal of citizens’ grievances. Hence
(d) 2 4 5 1 statement 1 is correct.
Answer: C • Article 315 to 323 contains provisions related
Explanation: to the Union Public Service Commission but
• The Economic Administration Reforms planning commission and vigilance
Commission (EARC) was set up in the early commission are not formed under the Articles
eighties to suggest changes in the economic of Indian Constitution.
administration of the country. • Planning Commission is non - Constitutional
and non-Statutory body.
15 | P a g e W W W . E D U T A P . C O . I N QUERY? HELLO@EDUTAP.CO.IN / 8146207241
• Vigilance Commission is statutory body before both the Houses of Parliament and
established by the CVC Act of 2003. 'Annual Financial Statement' for each financial
year.
Q59. Consider the following statements: Which of the statements given above is/are
1. Money Bill can be introduced in any House of correct?
Parliament. (a) 1 only
2. If any question arises whether a bill is a Money (b) 3 only
Bill or not, the decision of the Chairman of the (c) 2 and 3
Public Accounts Committee is final. (d) None
3. Vote of Credit enables a grant to be approved Answer: D
by the Parliament in advance of the detailed Explanation:
examination of various demands presented to • Adjournment Motion is introduced in the
it. Parliament to draw attention of the House to a
Which of the statements given above is/are definite matter of urgent public importance
correct? and needs the support of 50 members to be
(a) 1 and 2 admitted.
(b) 2 and 3 • An Adjournment Motion, should not deal with
(c) 1 and 3 any matter that is under adjudication by court.
(d) None • Under Article 112 of the Constitution of India,
Answer: D the President of India (not Union Finance
Explanation: Minister) causes to be laid before both the
• The Constitution lays down a special Houses of Parliament and 'Annual Financial
procedure for the passing of money bills in the Statement' for each financial year.
Parliament.
- A money bill can only be introduced in the Q61. Assertion (A): Recently, the four public
Lok Sabha and that too on the sector general insurance companies have
recommendation of the President. Thus, launched a community-based ‘Universal Health
statement 1 is not correct. Insurance Scheme’.
• If any question arises whether a bill is a money Reason (R): According to the Constitution of India,
bill or not, the decision of the Speaker of the the task of promoting public health primarily rests
Lok Sabha is final. with the Union Government.
- His decision in this regard cannot be Codes:
questioned in any court of law or in the (a) Both A and R are true and R is the correct
either House of Parliament or even by the explanation of A
President. Thus, statement 2 is not (b) Both A and R are true but R is NOT the correct
correct. explanation of A
• Vote of Credit is granted for meeting an (c) A is true but R is false
unexpected demand upon the resources of (d) A is false but R
India, when on account of the magnitude or Answer: B
the indefinite character of the service, the Explanation:
demand cannot be stated with the details • There are 4 public sector general insurance
ordinarily given in a budget. Thus statement 3 providers who have implemented Universal
is not correct. Health Insurance Scheme with the intention to
enhance people's access of health care
Q60. Consider the following statements: especially for families at or below the poverty
1. If forty or more members support an line.
Adjournment Motion, only then the related - This Scheme was launched on a
matter can be taken up for discussion. countrywide basis in July 2003. Hence
2. Under an Adjournment Motion, even a sub Assertion is correct.
judice matter can be taken up for discussion. • According to Article 47 in Part IV of Indian
3. Under Article 112 of the Constitution of India, Constitution, it is the duty of the State to raise
the Union Finance Minister causes to be laid
16 | P a g e W W W . E D U T A P . C O . I N QUERY? HELLO@EDUTAP.CO.IN / 8146207241
the level of nutrition and the standard of Which of the statements given above is/are
living of people and to improve public health. correct?
(a) 1 only
Q62. Consider the following statements: (b) 2 only
1. Dadra and Nagar Haveli which merged with the (c) Both 1 and 2
Indian Union in 1961 were ruled by the (d) Neither 1 nor 2
Portuguese. Answer: C
2. Both Pondicherry and Chandernagore were Explanation:
ruled by the French. • The Chief Minister of National Capital Territory
3. Goa got its freedom in 1961. of Delhi is appointed by the Indian President
Which of the statements given above is/are (not by the lt. governor).
correct? • Municipal Corporation of Delhi (MCD) is the
(a) 1, 2 and 3 municipal corporation that governs most of
(b) 2 and 3 Delhi, India.
(c) 1 and 3 - The Municipal Corporation of Delhi was
(d) 2 only replaced by three new bodies, the North
Answer: A Delhi Municipal Corporation, the South
Explanation: Delhi Municipal Corporation and the East
• The Portuguese ruled Dadar and Nagar Haveli Delhi Municipal Corporation in 2012.
until its liberation in 1954. Subsequently, the - They are controlled by the Government of
administration was carried on till 1961 by an India.
administrator chosen by the people • Delhi Police comes under the jurisdiction of
themselves. It was converted into a union the Ministry of Home Affairs (MHA),
territory of India by the 10th Constitutional Government of India.
Amendment Act, 1961.
• The territory of Puducherry comprises the Q64. Consider the following statements:
former French establishments in India known 1. The Legislative Assembly of a State cannot
as Puducherry, Karaikal, Mahe and Yanam. have more than 450 and not less than 50
The French handed over this territory to India members chosen by direct election from
in 1954. territorial constituencies.
- On 8th June 1948, the French government 2. The number of members of Legislative Council
had an agreement with the Indian of a State cannot exceed one-third of the total
Government under which it declared that membership of the Legislative Assembly of
the people of the French colonies in India that State subject to a minimum of 40.
would have the right to choose their own Which of the statements given above is/are
future status. correct?
- The French government conducted a (a) 1 only
plebiscite on 19th June 1949 in which a (b) 2 only
whopping 97% of the people voted in (c) Both 1 and 2
favour of a merger with India. 7463 votes (d) Neither 1 nor 2
were received in favour of the merger and Answer: B
114 against it. Explanation:
• India acquired Goa from the Portuguese by • The Legislative assembly of State consists of
means of a police action in 1961. representatives directly elected by the people
Hence A is the correct answer. on the basis of universal adult franchise. Its
maximum strength is fixed at 500 and
Q63. Consider the following statements: minimum strength at 60.
1. The Chief Minister of Delhi is appointed by the - It means that its strength varies from 60 to
President of India. 500 depending on the population size of
2. The Delhi Police and Municipal Corporation of the state. Thus statement 1 is not correct.
Delhi do not come under the administrative • The maximum strength of the Legislative
purview of the Delhi Government. Council in a state with bicameral legislature is
17 | P a g e W W W . E D U T A P . C O . I N QUERY? HELLO@EDUTAP.CO.IN / 8146207241
fixed at one-third of the total strength of the
assembly and the minimum strength is fixed at Q66. Consider the following statements:
40. Thus statement 2 is correct. 1. An amendment of the Constitution of India can
be initiated by the introduction of a Bill for the
Q65. Consider the following statements: purpose in Lok Sabha only.
1. Uttar Pradesh and Bihar are two States with 2. A Bill providing for the formation of new States
maximum population of the Scheduled Castes. and for the alteration of boundaries or names
2. Punjab has the highest ratio of the Scheduled of existing States in India can be introduced in
Caste population to its total population. the Parliament only on the recommendation
3. There was a provision for setting up a National of the President of India.
Commission for the Scheduled Castes and the 3. In India, a Bill seeking to amend the
Scheduled Tribes under the Article 338 of the representation of States in Parliament has to
Constitution of India. However, by a be passed by a simple majority of both Houses
Constitutional Amendment Act, separate of Parliament.
National Commission for the Scheduled Castes Which of the statements given above is/are
and National Commission for the Scheduled correct?
Tribes have been set up. (a) 1 and 2
Which of the statements given above is/are (b) 2 only
correct? (c) 1 and 3
(a) 2 only (d) 1, 2 and 3
(b) 1 and 2 Answer: B
(c) 2 and 3 Explanation:
(d) 1 and 3 • An amendment of the Constitution can be
Answer: C initiated only by the introduction of a bill for
Note: This answer is per the current information as the purpose in either House of Parliament and
in 2022. not in the state legislatures. Thus, statement 1
Explanation: is wrong.
• Uttar Pradesh stands first with 20.5% of the • Article 3 of Indian Constitution provides for the
total scheduled caste (SC) population, formation of new States and for the alteration
followed by West Bengal with 10.7%. of boundaries or names of existing States in
- Bihar with 8.2% and Tamil Nadu with 7.2 % India. However there are 2 conditions in this
come third and fourth. Thus, statement 1 regard:
is not correct. - a bill contemplating the above changes can
• As per Census 2011, the State of Punjab, has be introduced in the Parliament only with
the highest percentage of Scheduled Caste the prior recommendation of the
population amongst all the States of the President. Thus, statement 2 is correct.
Country. - before recommending the bill, the
- The Scheduled Caste population in Punjab President has to refer the same to the state
is 88.60 lakh which is 31.94% of the total legislature concerned for expressing its
population i.e. 277.43 lakh of the State. views within a specified period.
• The 65th Constitutional Amendment Act of • Representation of states in Parliament is
1990 provided for the establishment of a high related to the federal structure of the polity
level multi-member National Commission for can be amended by a special majority of the
SCs and STs under Article 338 in the place of a Parliament and also with the consent of half of
single Special Officer for SCs and STs. the state legislatures by a simple majority.
- Later the 89th Constitutional Amendment Thus, statement 3 is not correct.
Act of 2003 bifurcated the combined
National Commission for SCs and STs into Q67. Which one of the following is the correct
two separate bodies, namely, National statement?
Commission for Scheduled Castes (under
Article 338) and National Commission for The term of Lok Sabha can be extended by the
Scheduled Tribes (under Article 338-A). Parliament for a period
18 | P a g e W W W . E D U T A P . C O . I N QUERY? HELLO@EDUTAP.CO.IN / 8146207241
(a) not exceeding six months at a time during - Both the Supreme Court and high courts
proclamation of emergency but not exceeding can declare the laws enacted by the
in any case a period of three months after the Parliament as void and ultra vires
proclamation of emergency has ceased to (unconstitutional), if they contravene any
operate provision of the Constitution.
(b) not exceeding one year at a time during • Only Fundamental Rights are justiciable and
proclamation of emergency but not exceeding enforceable, allowing persons to move the
in any case a period of six months after the courts for their enforcement, if and when they
proclamation of emergency has ceased to are violated.
operate - The Directive Principles are non-justiciable
(c) not exceeding one year at a time during in nature, that is, they are not legally
proclamation of emergency but not exceeding enforceable by the courts for their
in any case a period of three months after the violation.
proclamation of emergency has ceased to
operate Q69. Consider the following statements related
(d) not exceeding six months at a time during to the World War II:
proclamation of emergency but not exceeding 1. Germany attacked the US naval base at Pearl
in any case a period of one month after the Harbour.
proclamation of emergency has ceased to 2. Winston Churchill was the British Prime
operate Minister during the World War II.
Answer: B 3. The Versailles Treaty which is generally cited as
Explanation: one of the main causes of the World War II was
• The Lok Sabha is not a continuing chamber. Its signed just before the World War II in 1939.
normal term is five years from the date of its 4. Franklin Roosevelt was the President of the
first meeting after the general elections, after United States were dropped over the Japanese
which it automatically dissolves. cities of Hiroshima and Nagasaki.
- The term of the Lok Sabha can be extended Which of the statement given above is/are
during the period of national emergency correct?
be a law of Parliament for one year at a (a) 1, 2 and 3
time for any length of time. (b) 1, 2 and 4
- However, this extension cannot continue (c) 2 only
beyond a period of six months after the (d) 3 and 4
emergency has ceased to operate. Answer: C
Explanation:
Q68. Consider the following statements: • The attack on Pearl Harbour was a surprise
1. The Indian Parliament is not sovereign, and the military strike by the Imperial Japanese Navy
legislation passed by it is subject to judicial Air Service upon the United States.
review. • Winston Churchill was an inspirational
2. In India, the Fundamental Rights and Directive statesman, writer, orator and leader who led
Principles of State Policy are enforceable by Britain to victory in the Second World War.
the courts. • The Treaty of Versailles was the most
Which of the statements given above is/are important of the peace treaties of World War
correct? I. It ended the state of war between Germany
(a) 1 only and the Allied Powers.
(b) 2 only • In August of 1945, the United States was still
(c) Both 1 and 2 fighting in World War II against the nation of
(d) Neither 1 nor 2 Japan. Having been told about the successful
Answer: A Trinity Test of an atomic bomb, President
Explanation: Truman decided to drop an atomic bomb on
• The adoption of an independent Judiciary with Japan on August 6, 1945.
the power of judicial review restricts the Hence option C is the correct answer.
sovereignty of Indian Parliament.
19 | P a g e W W W . E D U T A P . C O . I N QUERY? HELLO@EDUTAP.CO.IN / 8146207241
Q70. Assertion (A): During the British rule, the desire of starting an interesting conflict
Congress Ministers resigned in 1939 in all the between the Marathas and expecting Sahu
provinces where they were in office. to be on the side of Mughals for the
Reason (R): Congress wanted fresh elections as it succession fight of the Mughal Rule.
was frustrated with Jinnah’s campaign of two- Hence D is the correct answer.
nation theory.
Codes: Q72. Match List-I with List-II and select the
(a) Both A and R are true and R is the correct correct answer using the code given below the
explanation of A lists:
(b) Both A and R are true but R is NOT the correct List-I (Indian Kings)
explanation of A A. Karikala
(c) A is true but R is false B. Rudradaman
(d) A is false but R C. Milinda
Answer: D. Kanishka
Explanation:
• When World War II started in 1939, the British List-II (Dynasty)
were fighting against the Axis Powers. 1. Bacteria Greek
- The Viceroy of India announced India’s 2. Chola
involvement without consulting its 3. Chalukya
representative political leaders. Congress 4. Kushanas
asked for transfer of power in repayment 5. Shakas
of their cooperation in war, which the Codes:
British government denied. As a result, A B C D
Congress ministries resigned after 28 (a) 2 4 1 5
Months of power. (b) 1 5 3 4
Hence option C is correct answer. (c) 2 5 1 4
(d) 1 4 3 5
Q71. Assertion (A): Aurangzeb released Shahu Answer: C
from the prison shortly before Aurangzeb died in Explanation:
1707. • Karikala was a Chola dynasty king who ruled
Reason (R): Zulfiqar pointed that Shahu’s return to southern India.
his Kingdom would cause a division among the - He is credited with the construction of the
Marathas who would thus be disabled from flood banks of the river Kaveri. He is
plundering the imperial territories. recognised as the greatest of the Early
Codes: Cholas.
(a) Both A and R are true and R is the correct • Rudradaman I (130–150AD), a Saka monarch
explanation of A from the Western Kshatrapas dynasty, was a
(b) Both A and R are true but R is NOT the correct Saka ruler from the Western Kshatrapas
explanation of A dynasty. He was the king of Castana's
(c) A is true but R is false grandson.
(d) A is false but R is true • The Milindapañha is set up as a compilation of
Answer: D questions posed by King Milinda to a revered
Explanation: senior monk named Nagasena.
• Shahu Maharaj I was the Chhatrapati of the - This Milinda has been identified with
Maratha domain and the grandson of Shivaji. considerable confidence by scholars as the
- In his early life stages, when he was just a Greek king Menander of Bactria, in the
7-year-old boy, he was taken as a prisoner dominion founded by Alexander the Great,
alongside his mother in the year 1689 by which corresponds with much of present
the Mughal forces after the conflict of day Afghanistan.
Raigad. • Kanishka, was an emperor of the Kushan
- After the decease of Aurangzeb in the year dynasty.
1707 Bahadur Shah released Shahu in the Hence option C is the correct answer.
20 | P a g e W W W . E D U T A P . C O . I N QUERY? HELLO@EDUTAP.CO.IN / 8146207241
- In the battle that ensued along the banks
Q73. During the World War II, Indian soldiers of Ghagra, a tributary of Ganges, Babur
fought heroically in the Battle at Monte Cassino. defeated the Afghans.
Where is Monte Cassino located? • Babur decided to take on Rana Sanga of
(a) Poland Chittoor, who as ruler of Mewar, had a strong
(b) Italy influence over Rajasthan and Malwa. Babur
(c) Germany selected Khanwa, near Agra, as a favourable
(d) Greece site for this inevitable encounter.
Answer: B • The Deccan states, joined hands to wage the
Explanation: great battle against their common enemy
• The Battle of Monte Cassino was a series of Vijayanagar.
four assaults by the Allies against the Winter - The battle was fought at Talikota or
Line in Italy held by Axis forces during the Rakshasi-Tangadi in January 1565 in which
Italian Campaign of World War II. The intention Ramaraya, in spite of his old age,
was a breakthrough to Rome. personally commanded the forces along
with his cousins and brothers.
Q74. Match List-I with List-II and select the Hence C is the correct answer.
correct answer using the code given below the
lists: Q75. When Akbar besieged the Fort of Chittoor,
List-I (Battles) who among the following defended it for four
A. Battle of Chausa months?
B. Battle of Ghagra (a) Uday Singh
C. Battle of Kanwah (b) Rana Pratap
D. Battle of Talikota (c) Bahman Shah
(d) Jaimal
List-II (Fought between) Answer: A
1. Babar and Rajputs Explanation:
2. Humayun and Sher Khan • The ruler of Mewar, Rana Uday Singh, put up
3. Babar and Afghans of Bihar & Bengal a great fight before losing Chittoor, which was
4. Vijayanagar ruler and the Decca Sultanates conquered by Akbar after a siege of six
Codes: months.
A B C D
(a) 2 1 3 4 Q76. Which one of the following statements is
(b) 4 3 1 2 not correct?
(c) 2 3 1 4 (a) Lord Wellesley set up the first three Indian
(d) 4 1 3 2 Universities
Answer: C (b) Lord Dalhousie introduced telegraph in India
Explanation: (c) Lord Ripon introduced a system of local self-
• The Battle of Chausa was a famous battle government both for towns and country-side
between the Mughal Emperor, Humayun, and areas
the Afghan warlord, Sher Shah Suri. (d) Lord Curzon presided over the Delhi Durbar of
- It was fought on 26 June 1539 at Chausa, 1903
10 miles southwest of Buxar in modern-day Answer: A
Bihar, India. Explanation:
• The Battle of Ghagra was the last battle Babar • The three universities of Calcutta, Madras and
fought against the Afghans. Bombay came into existence in 1857 during
- Sultan Ibrahim Lodi’s brother Mahmud Lord Canning. Hence statement 1 is not
Lodi and Sultan Nusrat Shah, son-in-law of correct.
Ibrahim Lodi, conspired against Babur. • Lord Dalhousie was credited with the
Realising the danger Babar marched introduction of telegraph in India. Calcutta and
against them. Agra were connected by telegraph.

21 | P a g e W W W . E D U T A P . C O . I N QUERY? HELLO@EDUTAP.CO.IN / 8146207241


• Lord Ripon is known for introducing the Local the ‘Dickie Bird Plan’. The important provisions of
Self Government in 1882. this plan were:
- His scheme of local self-government • British India was to be partitioned into two
developed the Municipal institutions dominions – India and Pakistan.
which had been growing up in the country • The date for the transfer of power was to be
ever since India was occupied by the British August 15, 1947. Hence statement 1 is not
Crown. correct.
• The Delhi Durbar was held twice more in 1903 • The princely states were given the choice to
and 1911 to proclaim first King Edward VII and either remain independent or accede to India
then King George V as Emperor of India. or Pakistan. The British suzerainty over these
However, it was the 1903 Delhi Durbar kingdoms was terminated.
presided over by Viceroy of India Lord Curzon. • The legislative assembly of Sind would decide
whether to join the Indian constituent
Q77. Which one of the following is the correct assembly or not. It decided to go to Pakistan.
chronological sequence of the given dynasties of Hence statement 2 is correct.
Delhi Sultanate? • A referendum was to be held on NWFP (North-
(a) Sayyids – Khaljis – Lodis – Tughlaqs Western Frontier Province) to decide which
(b) Khaljis – Tughlaqs – Sayyids – Lodis dominion to join. NWFP decided to join
(c) Khaljis – Sayyids – Lodis – Tughlaqs Pakistan while Khan Abdul Gaffar Khan
(d) Tughlaqs – Khaljis – Sayyids – Lodis boycotted and rejected the referendum. There
Answer: B is no such choice for Baluchistan. Hence
Explanation: statement 3 is not correct.
• Five dynasties ruled over the Delhi Sultanate
sequentially: the Mamluk or Slave dynasty Q79. In the year 1919, what was the reason for
(1206–1290), the Khalji dynasty (1290–1320), Mahatma Gandhi to warn the Viceroy that a
the Tughlaq dynasty (1320–1414), the Sayyid countrywide Satyagraha would be launched?
dynasty (1414–1451), and the Lodi dynasty (a) He wanted the Government to withdraw the
(1451–1526). Rowlatt Act immediately
(b) He was forcing the British rulers to be sensitive
Q78. Consider the following statements: to the Khilafat Movement
According to the Mountbatten Plan (c) He wanted the Government to abolish the
1. The Union of India and Pakistan were to be Zamindari System to alleviate the miseries of
granted freedom not later than June 1948. peasants
2. The Legislative Assembly of Sindh was to (d) He was forcing the British rulers to give at least
decide whether it wanted to join the the dominion status to India to make Home
Constituent Assembly of India or not. Rule possible
3. Baluchistan was to decide whether it wanted Answer: A
to stay with the Indian Union or become Explanation:
separate. Gandhiji wanted the Government to withdraw the
Which of the statements given above is/are Rowlatt Act immediately and for this he warned
correct? the British Government.
(a) 1 and 2 • Gandhi started Satyagraha in 1919 against the
(b) 2 and 3 Rowlatt Act because the act laid down
(c) 2 only measures which were insulting and
(d) 1 and 3 challenging for the Indians.
Answer: B
Explanation: Q80. Who among the following introduced the
In May 1947, Mountbatten came up with a plan celebration of Ganesh Chaturthi and Shivaji
under which he proposed that the provinces be Festivals to bring the Indian society together and
declared independent successor states and then inspire patriotic feelings among the people?
be allowed to choose whether to join the (a) Bal Gangadhar Tilak
constituent assembly or not. This plan was called (b) Gopal Krishna Gokhale
22 | P a g e W W W . E D U T A P . C O . I N QUERY? HELLO@EDUTAP.CO.IN / 8146207241
(c) Jyotirao Govind Rao Phule B. Nagarjuna 2. Madhyamika
(d) Mahadev Govind Ranade Sutra
Answer: A C. Patanjali 3. Vedanta Sangraha
Explanation: D. Ramanuja 4. Yogasutra
Bal Gangadhar transformed the simple Ganesh Code:
Puja performed at home into a social and public A B C D
Ganesh festival. (a) 4 3 1 2
• He used the Ganesh Chaturthi and Shiv (b) 1 2 4 3
Jayanti (birth anniversary of Shivaji) festivals (c) 4 2 1 3
to create unity and a national spirit among the (d) 1 3 4 2
people. Answer: B
• Buddhacharita is an epic poem in the Sanskrit
Q81. Which one of the following is the correct mahakavya style on the life of Gautama
chronological sequence of the given Bhakti Buddha by Aśvaghoṣa.
Saints? • Madhyamika sutra, also known as
(a) Guru Nanak-Tulsidas-Ramdas-Tukaram Mūlamadhyamakakārikā, is the foundational
(b) Tulsidas-Guru Nanak-Tukaram-Ramdas text of the Madhyamaka school of Mahāyāna
(c) Guru Nanak-Tulsidas-Tukaram-Ramdas Buddhist philosophy. It was composed by the
(d) Tulsidas-Guru Nanak-Ramdas-Tukaram Indian philosopher Nāgārjuna.
Answer: C • The Yoga Sutras was compiled in the early
• Gurū Nānak (15 April 1469 – 22 September centuries CE, by the sage Patanjali in India
1539), was the founder of Sikhism and is the who synthesized and organized knowledge
first of the ten Sikh Gurus in the Sikhism about yoga from much older traditions.
religion. • Of the nine works that Ramanujacharya
• Tulsidas (1511–1623) was a Ramanandi wrote, Vedartha Sangraha was the first. It was
Vaishnava Hindu saint and poet, renowned for in fact a discourse that he gave in Tirumala.
his devotion to the deity Rama. The title can be translated as 'Essence of the
• He wrote several popular works in Sanskrit and meaning of the Vedas.
Awadhi but is best known as the author of the • Hence, option B is the correct answer.
Hanuman Chalisa and of the epic
Ramcharitmanas, a retelling of the Sanskrit Q83. Match List-I. (Historical Sites) with List-II
Ramayana based on Rama's life in the (State) and select the correct answer using the
vernacular Awadhi. codes given below the lists:
• Sant Tukaram Maharaj (1600 - 1650) was a LIST I (Historical Site) LIST II (State)
17th-century Marathi poet, Hindu sant (saint), A. Shore Temple 1. Karnataka
popularly known as Tuka, Tukobaraya, Tukoba B. Bhimbetka 2. Tamil Nadu
in Maharashtra.[4] He was a Sant of Varkari C. Kesava Temple 3. Kerala
sampradaya (Marathi-Vaishnav tradition) - (Hoysala
that venerates the god Vithoba - in Monument)
Maharashtra D. Hampi 4. Madhya Pradesh
• Swami Ramdas (born Vittal Rao on 10 April 5. Rajasthan
1884) was an Indian saint, philosopher, Code:
philanthropist and pilgrim. Swami Ramdas A B C D
became a wandering ascetic in his late 30s and (a) 3 5 2 1
later established Anandashram in Kanhangad, (b) 2 4 1 1
Kerala. (c) 3 4 2 2
(d) 2 5 1 4
Q82. Match List-I. (Persons) with List-II (Writings) Answer: B
and select the correct answer using the codes • The Shore Temple (c. 725 AD) is a complex of
given below the lists: temples and shrines that overlooks the shore
LIST I (Persons) LIST II (Writings) of the Bay of Bengal. It is located in
A. Asvaghosha 1. Buddha Charita
23 | P a g e W W W . E D U T A P . C O . I N QUERY? HELLO@EDUTAP.CO.IN / 8146207241
Mahabalipuram, about 60 kilometres (37 mi) the Satpura Range of Madhya Pradesh.
south of Chennai in Tamil Nadu. Hence, B is matched with 3.
• The Bhimbetka rock shelters are an • Pocharam Wildlife Sanctuary is a forest and
archaeological site located in the Raisen wildlife sanctuary located 15 km from Medak
District in the state of Madhya Pradesh. and 115 km from Hyderabad, Telangana. At
• Chennakeshava Temple, also referred to as the time of exam, the Sanctuary was located
Keshava, Kesava or Vijayanarayana Temple of in Andhra Pradesh. Hence, C is matched with
Belur, is a 12th-century Hindu temple in the 1.
Hassan district of Karnataka. • Sharavathi Wildlife Sanctuary is a protected
• Hampi or Hampe, also referred to as the wildlife sanctuary in the Western Ghats of
Group of Monuments at Hampi, is a UNESCO Karnataka. Hence, D is matched with 2.
World Heritage Site located in Hampi town,
Vijayanagara district, east-central Karnataka. Q86. Match List-I (Produce) with List-II (Major
• Hence, correct answer is B. Producer State) and select the correct answer
using the code given below the lists:
Q84. Which country among the following is the List-I (Produce) List-II (Major
biggest producer of cotton? Producer State)
(a) China A. Rubber 1. Andhra Pradesh
(b) India B. Soyabean 2. Tamil Nadu
(c) Indonesia C. Groundnut 3. Madhya Pradesh
(d) United States of America D. Wheat 4. Kerala
Answer: A/B 5. Uttar Pradesh
• Different sources cite India or China as the Code:
biggest cotton producers in the world. A B C D
• According to FAO, India and China both are (a) 4 1 2 5
the biggest cotton producers in the world. (b) 5 3 1 4
Their relative position is subject to change (c) 4 3 1 5
every year. (d) 5 1 2 4
Answer: C
Q85. Match List-I (Wildlife Sanctuary) with List-II The statistics of state-wise production of different
(State) and select the correct answer using the crops in 2004 was different from what it is today.
code given below the lists: Thus, data presented here is the latest data
List-I (Wildlife List-II (Host Country) available.
Sanctuary) • Natural rubber is cultivated in 16 states in
A. Bhitar Kanika 1. Andhra Pradesh India. With over 600,000 hectares, Kerala tops
B. Pachmarhi 2. Karnataka rubber cultivation, followed by Tripura with
C. Pocharam 3. Madhya Pradesh over 85,038 hectares under plantation.
D. Sharavathi 4. Orissa Hence, A is matched with 4.
5. Uttar Pradesh • The major soyabean growing states are
Code: Madhya Pradesh, Maharashtra, Rajasthan,
A B C D Karnataka, and Telangana. Hence, B is
(a) 4 2 3 1 matched with 3.
(b) 1 3 5 2 • Groundnut producing States are led by
(c) 4 3 1 2 Gujarat, Andhra Pradesh, Tamil Nadu and
(d) 1 2 5 3 Maharashtra. Hence, C is matched with 1.
Answer: C • Uttar Pradesh is the largest wheat producer
• Bhitarkanika National Park is a 145 km2 large in India, followed very closely by Madhya
national park in northeast Kendrapara district Pradesh. Hence, D is matched with 5.
in Odisha. Hence, A is matched with 4.
• Pachmarhi Biosphere Reserve is a non-use Q87. Consider the following statements:
conservation area and biosphere reserve in

24 | P a g e W W W . E D U T A P . C O . I N QUERY? HELLO@EDUTAP.CO.IN / 8146207241


1. In India, intensity of cropping is high in the • Macedonia is a geographical and historical
States of the peninsular plateau. region of the Balkan Peninsula in Southeast
2. In India, the States of Punjab and Haryana Europe. Hence, C is matched with 1.
have the highest proportion of the net sown • Gabon, officially the Gabonese Republic, is a
area to total geographical area. country on the west coast of Central Africa.
3. South-west Monsoon season is termed as the Hence, D is matched with 4.
Kharif season of crops.
Which of the statements given above are correct? Q89. Going from the West of the United States of
(a) 2 and 3 America to its East, which one of the following is
(b) 1 and 2 the correct sequence of the given major
(c) 1 and 3 American cities?
(d) 1, 2 and 3 (a) Houston-Los Angeles-San Francisco-New York
Answer: A (b) San Francisco-New York-Houston-Los Angeles
• In India, cropping intensity is higher in the (c) Houston-New York-San Francisco-Los Angeles
States of Northern Plains as compared to (d) San Francisco-Los Angeles-Houston-New York
States in peninsular plateau. Various reasons Answer: D
like fertile soil, plain topography, perennial
rivers etc. are the reasons for this. Hence,
statement 1 is incorrect.
• In Punjab & Haryana, net sown area is around
80% of the total area of the State, which is
much higher than the other States. Hence,
statement 2 is correct.
• South west monsoon is associated with the
Kharif cropping season. Hence, statement 3 is
correct.
Q88. Match List-I (Country) with List-II (Location)
and select the correct answer using the codes
given below the Lists:
List-I (Country) List-II (Location) Q90. Match List-1 (Beach Resort) with List-II
(State) and select the correct answer using the
A. Bahamas 1. Europe
codes given below the Lists:
B. Belize 2. North America
List-I (Beach Resort) List-II (State)
C. Macedonia 3.North Atlantic Ocean
A. Digha 1. Kerala
D. Gabon 4. Africa
B. Covelong 2. West Bengal
5. South America
C. Cherai 3. Maharashtra
Code:
D. Murud-Janjira 4. Tamil Nadu
A B C D
(a) 3 4 1 2 Code:
(b) 1 2 5 4 A B C D
(c) 3 2 1 4 (a) 2 4 1 3
(d) 1 4 5 2 (b) 3 1 4 2
Answer: C (c) 2 1 4 3
(d) 3 4 1 2
• The Bahamas, officially the Commonwealth of
Answer: A
The Bahamas, is a country within the Lucayan
Archipelago of the West Indies in the North • Digha is a seaside resort town in the state of
Atlantic Ocean. Hence, A is matched with 3. West Bengal. Hence, A is matched with 2.
• Belize is a Caribbean country on the north- • Covelong Beach is actually Kovalam beach
eastern coast of Central America. Hence, B is that is located on the coast of the Bay of
most appropriately matched with 2. Bengal near a village named Covelong,
Chennai. Hence, B is matched with 4.

25 | P a g e W W W . E D U T A P . C O . I N QUERY? HELLO@EDUTAP.CO.IN / 8146207241


• Cherai beach is located in Kochi Taluk, a (c) The West Coast of India experiences tides four
suburb of the city of Kochi, in the state of times a day
Kerala. Hence, C is matched with 1. (d) Tides do not occur in the gulfs
• Murud-Janjira is the local name of a famous Answer: A
fort and tourist spot situated on an island just • Spring tides occur on full moon and new moon
off the coastal town/city of Murud, in the days. Hence, option A is the correct answer.
Raigad district of Maharashtra. Hence, D is • Neap tides occur near the time when the
matched with 3. Moon and the Sun are in quadrature. Hence,
option B is incorrect.
Q91. Where are the maximum numbers of major • West coast of India experiences tides twice a
ports located in India? day.
(a) Maharashtra • Tides do occur in Gulfs as well.
(b) Kerala
(c) Goa
(d) Tamil Nadu Q95. Which one of the following pairs is not
Answer: D correctly matched.
• Tamil Nadu has three major ports located in City River
its territory: Chennai port, Kamarajar Port (a) Ahmedabad : Sabarmati
(Ennore) and Chidambaranar Port (Tuticorin). (b) Hyderabad : Musi
(c) Lucknow : Gomti
Q92. Which one of the following is not a tributary (d) Surat : Narmada
of the river Godavari? Answer: D
(a) Koyna Surat is located on the banks of Tapti river. Hence,
(b) Manjra option D is the correct answer.
(c) Pranhita
(d) Wardha Q96. Match List-I (Famous Place) with List-II
Answer: A (Country) and select the correct answer using the
The Koyna River is a tributary of the Krishna River. codes given below the lists:
List-I (Famous Place) List-II (Country)
Q93. Which one of the following is not a correct A. Alexandria 1. Turkey
statement? B. Blackpool Pleasure 2. Great Britain
(a) The height of the Western half of the Beach
Himalayas is greater than that of the Eastern C. Constantinople 3. Italy
half. D. Florence 4. Greece
(b) The Himalayas are young fold mountains 5. Egypt
(c) The Shivalik ranges are made of Code:
unconsolidated river deposits A B C D
(d) The Himalayas are wide in Kashmir and (a) 1 3 4 2
become narrow towards the East (b) 5 2 1 3
Answer: A (c) 1 2 4 3
• The height of eastern Himalayas is not lower (d) 5 3 1 2
than of the western half. In fact, highest peaks Answer: B
of Mt. Everest & Kanchenjunga etc. are • Alexandria is a historical city located on the
located in the eastern half. Hence, option A is Mediterranean coast of Egypt. Hence, A is
the correct answer. matched with 5.
• Rest three statements are correct. • Blackpool Pleasure Beach is an amusement
park situated on Blackpool's South Shore, in
Q94. Which one of the following is the correct the county of Lancashire, North West
statement? England. Hence, B is matched with 2.
(a) Spring tides occur on the full moon day • Constantinople was the capital of the Roman
(b) Neap tides occur on the new moon day only Empire, and later, the Eastern Roman Empire

26 | P a g e W W W . E D U T A P . C O . I N QUERY? HELLO@EDUTAP.CO.IN / 8146207241


(also known as the Byzantine Empire), the (d) 5 2 1 3
Latin Empire (1204–1261), and the Ottoman Answer: B
Empire (1453–1922). Officially renamed Explanation:
Istanbul in 1930, the city is today the largest Institutes Headquarters
city and financial centre of the Republic of Indian Institute of Public Delhi
Turkey (1923–present). It remains the largest Administration
city in Europe. Hence, C is matched with 1.
V. V. Giri National Labour Noida
• Florence is a city in Central Italy. Hence, D is
Institute
matched with 3.
National Institute of Financial Faridabad
Management
Q97. Consider the following statements:
1. Kaziranga National Park is a World Heritage
National Law School of India Mumbai
University
Site recognized by the UNESCO.
2. Kaziranga National Park is a home to sloth
bear and hoolock gibbon. Q99. Where is the Holy Shrine of Imam Ali in
Which of the statements given above is/are Najaf located?
correct? (a) Saudi Arabia
(a) 1 only (b) Iraq
(b) 2 only (c) Iran
(c) Both 1 and 2 (d) Kuwait
(d) Neither 1 nor 2 Answer: B
Answer: C Explanation:
• Kaziranga National Park is located in the • Imam Ali Shrine, the holiest site of Shia
Golaghat and Nagaon districts of the state of Muslims and one of the most important
Assam. sites of Najaf. Najaf is located in Iraq.
• The park, which hosts two-thirds of the
world's great one-horned rhinoceroses, is a Q100. Why was the region of Darfur of Sudan in
UNESCO World Heritage Site. Hence, news recently?
statement 1 is correct. (a) Americans bombed the terrorist training
• Kaziranga National Park is a home to sloth camps set up over there by the Janaweed
bear and hoolock gibbon. Hence, statement 2 militiamen of Sudan
is also correct. (b) Thousands of black African civilians were
killed or displaced by the Janaweeds who
Q98. Match List-I (Institute) with List-II (Location) are supported by the Arab-dominated
and select the correct answer using the code Sudan's Government
given below the lists: (c) Bird flu started from there
List-I (Institute) (d) A dam over the river Blue Nile was swept
A. Indian Institute of Public Administration away killing thousands of people
B. V.V. Giri National Labour Institute Answer: B
C. National Institute of Financial Management Explanation:
D. National Law School of India University • In 2003, Sudanese government forces
List-II (Location) launched the first of 2 major offensives
1. Fardiabad against rebels in Darfur.
2. Bangalore • In 2004, the US State Department
3. NOIDA accused Sudan of joining its
4. Mumbai
government military forces with the
5. Delhi
Arab Janjaweed (“men with guns on
Code:
A B C D horses or camels”) militias to target and
(a) 1 2 4 3 carry out genocidal violence against
(b) 5 3 1 2 African tribal villages populated by
(c) 1 3 4 2 Black Africans.
27 | P a g e W W W . E D U T A P . C O . I N QUERY? HELLO@EDUTAP.CO.IN / 8146207241
the concurring votes of the permanent
members
Q101. What was the total expenditure on
education both by the Central and State Q103. Consider the following statements with
Governments during the year 2002-03 as an reference to the Census 2001 of India:
approximate percentage of the Gross Domestic 1. For the first time, the country has
Product? witnessed a faster growth in female
(a) 2% literary compared to that of males during
(b) 3% the decade 1991-2001.
(c) 4% 2. During the decade 1991-2001, the rural-
(d) 5% urban literacy gap continued to diverge
Answer: B Which of the statement given above is/are
Explanation: correct?
(a) 1 only (b) 2 only
(c) Both 1 and 2 (d) Neither 1 nor 2
Answer: C
Explanation:
• As per the Census 2001, The literacy rate
recorded an increase of 13.17 percentage
points to from 1991 to 2001, the highest
increase in any one decade.
• During the decade the 1991-2001, rural-
urban literacy gap continued to diverge:
o The rate of growth of literacy in the
decade ending 2001 has been
higher in the rural areas, at 14.75
per cent as compared to the 7.2 per
Q102. Consider the following statements with cent increase in urban areas.
reference to the United Nations Organisation: Despite these improvements
1. In the General Assembly of the UNO, no literacy in urban areas was 80.3 per
member-nation has veto power. cent and that in rural areas 59.4 per
2 In the Security Council, all permanent cent.
members must vote in the affirmative if a
resolution is to pass.
Which of the statements given above is/are Q104. Match List-I (Railway Zone) with List-II
correct? (Headquarters) and select the correct answer
(a) 1 only using the code given below the lists:
(b) 2 only List-I (Railway Zone)
(c) Both 1 and 2 A. East-Central Railway
(d) Neither 1 nor 2 B. North-Western Railway
Answer: D C. North-Central Railway
Explanation: D. South-Western Railway
• All five permanent members have List-II (Headquarters)
exercised the right of veto at one time or 1. Hubli
another. 2. Allahabad
• Decisions of the Security Council on 3. Hajipur
procedural matters shall be made by an 4. Jabalpur
affirmative vote of nine members. 5. Jaipur
• Decisions of the Security Council on all Code:
other matters shall be made by an A B C D
affirmative vote of nine members including (a) 3 5 2 1
(b) 2 1 4 5
28 | P a g e W W W . E D U T A P . C O . I N QUERY? HELLO@EDUTAP.CO.IN / 8146207241
(c) 3 1 2 5 sectoral approach to bring down
(d) 2 5 4 1 regional inequalities.
Answer: A o Expenditure of ₹43,825 crore (US$5.5
Explanation billion) for tenth five years.
Railway Zone Headquarter
East-Central Railway Hajipur Q106. Match List-I (Institute) with List-II (City)
North-Western Railway Jaipur and select the correct answer using the codes
given below the Lists:
North-Central Railway Allahabad
List-I (Institute)
(Prayagraj)
A. Rashtriya Sanskrit Vidyapeeth
South-Western Railway Hubli B. Maharishi Sandipani Rashtriya Veda
Vidya Prathishthan
C. Central Institute of Indian Languages
Q105. Consider the following statements: D. Central Institute of English and Foreign
1. The average population density of India is Language
between 600-700 persons per square List-II (City)
kilometer. 1. Hyderabad
2. 2002-2007 is the duration of the Tenth 2. Varanasi
Five-Year Plan. 3. Mysore
3. Installed electricity generation capacity in 4. Tirupati
India is in excess of 2 lakh Megawatt. 5. Ujjain
Which of the statements given above is/are Code:
correct? A B C D
(a) 1 and 2 (a) 2 3 1 5
(b) 2 only (b) 4 5 3 1
(c) 1 and 3 (c) 2 5 3 1
(d) 1, 2 and 3 (d) 4 3 1 5
Answer: As the question is based on latest trends, Answer: B
students are advised to prepare the latest trends Explanation:
related to important parameters (Like GPD, Institutes City
Inflation, Electricity generation etc.) for the Rashtriya Sanskrit Vidyapeeth Tirupati
examination.
Maharishi Sandipani Rashtriya Ujjain
Explanation:
Veda Vidya Prathishthan
• As per the latest Data of the government for
Central Institute of Indian Mysore
July 2022, Total Installed electricity generation
Languages
of India is 4,03,760 MW.
Central Institute of English and Hyderabad
• Tenth Five Year Plan was implemented during
Foreign Language
the duration of 2002-2007 with following
targets:
o Attain 8% GDP growth per year.
Q107. Who among the following are the Bharat
o Reduction of poverty rate by 5% by
Ratna recipients?
2007.
1. Amartya Sen
o Providing gainful and high-quality
2. Gulzari Lal Nanda
employment at least to the addition to
3. M.S. Swaminathan
the labour force.
4. Satyajit Ray
o Reduction in gender gaps in literacy
Select the correct answer using the codes
and wage rates by at least 50% by 2007.
given below:
o 20-point program was introduced.
Codes:
o Target growth: 8.1% – growth
(a) 1 and 2
achieved: 7.7%.
(b) 3 and 4
o The Tenth Plan was expected to follow
(c) 1, 2 and 4
a regional approach rather than
(d) 1, 2, 3 and 4
29 | P a g e W W W . E D U T A P . C O . I N QUERY? HELLO@EDUTAP.CO.IN / 8146207241
Answer: C
Explanation Q110. Who among the following Indian film
Recipient of Bharat Ratan Year directors/actors was honoured with the Order of
Award the British Empire (OBE) in year 2004?
Satyajit Ray 1992 (a) Naseeruddin Shah
(b) Amitabh Bachchan
Gulzarilal Nanda 1997
(c) Shekhar Kapoor
Amratya Sen 1999
(d) Om Puri
Answer: D
Q108. Match List-I (Outstanding Sports persons Explanation
at the Athens Olympics) with List-II (Country) and • Om Puri has honorary Officer of the
select the correct answer using the codes given Order of the British Empire (OBE) in
below the Lists : 2004 for his contributions to the British film
List-I (Outstanding Sports persons at the industry.
Athens Olympics)
A. Michael Phelps Q111. Match List-I (Distinguished Women) with
B. Ian Thorpe List-II (Known As) and select the correct answer
C. Yulia Nesterenko using the codes given below the Lists:
D. Nicolas Massu List-I (Distinguished Women)
List-II (Country) A. Maria Sharapova
1. Belarus B. Aung San Suu Kyi
2. United States of America C. Asma Jahangir
3. Australia D. Norah Jones
4. Chile List-II (Known As)
5. Sweden 1. Human rights activist
Codes: 2. Political leader
A B C D 3. Singer
(a) 2 4 1 3 4. Sportsperson
(b) 1 3 5 4 Codes:
(c) 2 3 1 4 A B C D
(d) 1 4 5 3 (a) 1 2 3 4
Answer: C (b) 4 3 2 1
Explanation (c) 1 3 2 4
Athlete Country Event (d) 4 2 1 3
Michael Phelp United Swimming Answer: D
States of Explanation
America Person Profession
Ian Thorpe Australia Swimming Maria Sharapova Tennis
Yulia Nesterenko Belarus Athlete Aung San Suu Kyi Myanmar
Nicolas Massu Chile Tennis Politician
Asma Jahangir Pakistan
Q109. Who won the 100-metre race for men in Human Right
the Athens Olympics in August, 2004? Activist
(a) Maurice Greene Norah Jones American
(b) Justine Gatlin Singer
(c) Asafa Powell
(d) Francis Obekwelu Q112. Match List-I (Author) with List-II (Book) and
Answer: B select the correct answer using the codes given
Explanation below the Lists:
• Justine Gatlin has won the Gold Medal for List-I (Author)
Men’s 100 metre race for men in Athens A. Bill Clinton
Olympics 2004.
30 | P a g e W W W . E D U T A P . C O . I N QUERY? HELLO@EDUTAP.CO.IN / 8146207241
B. Henry Kissinger She is the founder of
C. Nelson Mandela Mamidipudi
D. Hillary Clinton Venkatarangaiya
List-II (Book) Foundation
1. Between Hope and History
Kalpana Lajmi Film Director
2. Living History
3. Diplomacy She was an Indian film
4. Long Walk to Freedom director, producer and
Codes: screenwriter.
A B C D Romila Thapar Historians
(a) 2 4 3 1 She is an Indian historian.
(b) 1 3 4 2 Her principal area of
(c) 2 3 4 1 study is ancient India, a
(d) 1 4 3 2 field in which she is pre-
Answer: D eminent.
Explanation Radha Reddy Indian Kuchipudi Dancer
Book Author
Between Hope and History Bill Clinton Q114. Assertion (A): In its Global Development
Living History Hillary Finance Report 2004, the World Bank has
Clinton categorised India as a less indebted country for the
Diplomacy Nelson year 2002.
Mandela Reason (R): During the last decade, India’s
Long Walk to Freedom Henry outstanding external debt has considerable
Kissinger reduced.
(a) Both A and R are individually true and R is the
Q113. Match List-1 (Distinguished Women) with correct explanation of A.
List-II (Known As/Area) and select the correct (b) Both A and R are individually true and R is not
answer using the codes given below the Lists: the correct explanation of A.
List-I (Distinguished Women) (c) A is true but R is false.
A. Shanta Sinha (d) A is false but R is true
B. Kalpana Lajmi Answer: Latest Edition of the Reports released by
C. Romila Thapar International Organizations are important for the
D. Radha Reddy examination, in these reports India Specific
List-II (Known As/Area) Findings are the most important for the
1. Film Direction examination.
2. Community Leadership
3. Dancer Q115. Consider the following statements:
4. Historian India had plan holiday during 1966-69 due to
5. Business-woman 1. Indo-Pakistan conflict in 1965.
Codes: 2. Severe drought for two successive years.
A B C D Which of the statements given above is/are
(a) 2 1 4 3 correct?
(b) 4 3 5 1 (a) 1 only
(c) 2 3 4 1 (b) 2 only
(d) 4 1 5 3 (c) Both 1 and 2
Answer: D (d) Neither 1 nor 2
Explanation Answer: A
Explanation:
Women Area
• The government had to declare "Plan
Shanta Sinha Community Leadership
Holidays" (1966-67, 1967-68, and 1968-69).
She is an Indian anti-child The Sino-Indian War of 1962 and the Indo-Pak
labour activist.
31 | P a g e W W W . E D U T A P . C O . I N QUERY? HELLO@EDUTAP.CO.IN / 8146207241
War of 1965, which caused the Third Five Year b) The Hindu Marriage Act, 1955 applies to
Plan to fail, were the primary causes of the Buddhists, Jains and Sikhs in addition to Hindus
plan holidays. Hence, statement 1 is correct. c) The Hindu Succession Act, 1956 recognizes the
• During 1966-67, there was the problem of right of women to inherit property of an
drought but it was not the reason behind the intestate equally with men
plan holiday. Hence, statement 2 is incorrect. d) Muslims, Christians and Parsis in India have no
• Therefore, A is the right answer. adoption laws and can take a child under the
Guardians and Wards Act, 1890 only under
foster care
Q116. Consider the following statements: Answer: A
1. The members of the Employees' Provident Explanation:
Fund and exempted Provident Funds are • The Special Marriage Act, 1954 extended to
eligible for the Employees' Deposit-Linked all the States and Union Territories of India
Insurance Scheme. except Jammu & Kashmir. Hence statement
2. Employees' Pension Scheme, 1995 was (a) was incorrect in 2004 but now Jammu &
introduced for the industrial workers. Kashmir has become a UT and so it applies
3. Coverage under the Employees' Provident there also.
Fund and Miscellaneous Provisions Act is • The Hindu Marriage Act, 1955 applies to
restricted to establishments employing 20 or many religions such as the Hindus, Jains,
more persons. Sikhs and the Buddhists. It is also applicable
Which of the statements given above is/are to the persons if they have converted to any
correct? of these religions from any other religion.
a) 1 and 2 Hence statement (b) is correct.
b) 1, 2 and 3 • As per the Hindu Succession Act 1956, in a
c) 1 and 3 case where a woman dies intestate leaving
d) 3 only property, her property will firstly devolve
Answer: C upon her sons and daughters so also the
Explanation: husband. The children of any pre-deceased
• The members of the Employees' Provident son or daughter are also included in the first
Fund and exempted Provident Funds are category of heirs of a female Hindu. Hence
eligible for the Employees' Deposit-Linked statement (c) is correct.
Insurance Scheme. Hence, statement 1 is • Adoption is a subject matter of personal law
correct. since it is a legal affiliation of a child. Similar
• The Employee's Pension Scheme (EPS) was to Muslims and Parsis, Christians also can
introduced in the year 1995 with the main adopt a child from an orphanage with the
aim of helping employees in the organised permission of the concerned court under the
sector. All employees who are eligible for the Guardian and Ward Act, 1980. Hence
Employees Provident Fund (EPF) scheme will statement (d) is correct.
also be eligible for EPS. Hence, statement 2 • Therefore, A is the right answer.
is incorrect.
• Coverage under the Employees' Provident
Fund and Miscellaneous Provisions Act is Q118. Why did Motilal Nehru and Chittaranjan
restricted to establishments employing 20 or Das form a separate group known as Swaraj Party
more persons. Hence, statement 3 is within the Congress?
correct. a) They were not satisfied with the progress
• Therefore, C is the right answer. made by the Congress in achieving Swaraj
b) It was their reaction to Mahatma Gandhi's
sudden decision to suspend the Non-
Q117. Which one of the following statements is Cooperation Movement
not correct? c) They were interested in forming more active
a) The Special Marriage Act, 1954 extends to all party with revolutionary ideas
the States and Union Territories of India
32 | P a g e W W W . E D U T A P . C O . I N QUERY? HELLO@EDUTAP.CO.IN / 8146207241
d) They felt that their opinions and views were Select your answers to these items using the
not given due importance in the Congress Codes given. below and mark your answer-sheet
Sessions accordingly
Answer: B
Explanation: Assertion (A): The salinity of the open seas is very
high compared to that of inland seas.
• The Withdrawal of Non-Cooperation
movement, and arrest of Gandhiji in March Reason (R): The water of rivers flowing into open
1922 through the freedom struggle into a seas contains dissolved salts.
passive phase, there was disintegration, Codes:
disorganisation and demoralisation among (a) Both A and R are true and R is the correct
nationalist ranks. explanation of A
• There were different proposals regarding (b) Both A and R are true but R is NOT the correct
the activities that need to be undertaken in explanation of A
the passive phase of the freedom (c) A is true but R is false
movement. (d) A is false but R is true
• Consequently, there emerged two major Answer: D
schools of thought - Swarajist: They wanted Explanation:
a re-entry and utilization of Legislative • Mostly inland seas have greater salinity
Councils and No-Changers: They were than open seas. This may be because of
against the council re-entry and rather evaporation. Hence Assertion (A) is false.
emphasized on constructive work during • Throughout the world, rivers carry an
the passive phase of the movement. estimated four billion tons of dissolved
• Therefore, B is the right answer. salts to the ocean annually. Hence Reason
Q119. Which of the following is detected and (R) is true. Hence option D is correct.
estimated by the Pollution Check' carried out on
motor cars at service stations?
a) Lead and carbon particles
b) Oxides of nitrogen and sulphur
c) Carbon monoxide
d) Carbon dioxide
Answer: C
Explanation:
In India, a valid driving license, insurance coverage
and a Pollution Under Control Certificate are legal
mandatory requirements for a car.
• The smoke emitted from vehicles, if left
unchecked, could pollute the environment
to a great extent. PUC is a certification mark
that is provided to vehicles that undergo the
PUC Test successfully.
• Carbon monoxide and hydrocarbons are
checked.
• Therefore, C is the right answer.

Q120. Directions: The following items consist of


two statements, one labelled the 'Assertion (A)
and the other labelled the 'Reason (R)'. You are to
examine these two statements carefully and
decide if the Assertion (A) and the Reason (R) are
individually true and if so, whether the Reason (R)
is a correct explanation of the Assertion (A).
33 | P a g e W W W . E D U T A P . C O . I N QUERY? HELLO@EDUTAP.CO.IN / 8146207241
Visit: www.civilstap.com
For Any Query Mail us: hello@civilstap.com or call us at - (+91)-8146207241
0
Q4. (a) A little younger
UPSC EPFO APFC - 2002 (b) to myself we, both were
PART- A (c) devoted to cricket and boating
(d) No error
SPOTTING ERRORS
Q5. (a) He had no sooner
(b) arrived than
Directions (for the next Five items):
(c) he was asked to leave again
(i) In this section a number of sentences are
(d) No error
given. Each sentence has three underlined
parts. indicated by (a), (b) and (c). Read each Directions (for the next three items):
sentence to find out whether there is an error.
If you find an error in any one of the Select the most suitable word from (a), (b), (c) and
underlined parts (a, b, c), indicate your (d) given below each sentence to fill in the blanks
response by blackening the letter related to Q6. The Rajputs always fought _____________
that part in the Answer Sheet provided. If a the last man.
sentence has no error. indicate this by
(a) upto (b) till
blackening (d) which stands for "No error".
(c) to (d) for
(ii) Errors may belong to grammar, usage or
idiom. Examples P and Q have been solved for Q7. I have been surfing internet._________ the
you last two hours.
(a) since (b) for
P. (a) My friend and myself (c) from (d) right from
(b) Study together
(c) During holidays Q8. You should be considerate __________ your
(d) No error employees and I assure you of your success in
Answer: (d) every walk of life.
Explanation: The correct answer for P is letter 'd' (a) for (b) to
because the sentence has no mistake in it. (c) upon (d) on
Q. (a) The rice from Dehradun is
(b) More superior Directions (for the Five items which follow):
(c) To that of Saharanpur Match words in Last I with the words/group of
(d) No error words in the List-II such that they convey the
Answer: (b) same meaning or sense. Select the correct
Explanation: The correct answer for Q is 'b' answer using the codes given below the lists
because the mistake in the sentence is in the part Q9. List-I
carrying the letter 'b'.
A. Obsequious
Q1. (a) Choose only B. Obstreperous
(b) Such friends C. Inveterate
(c) That you can trust D. Impecunious
(d) No error List-II
1. Dishonest
Q2. (a) While he was digging sand 2. Having little or no money
(b) a large mass fell 3. Too willing to obey
(c) and buried him completely 4. Noisy and unruly
(d) No error 5. Deep-rooted
Q3. (a) The book is well printed and attractively 6. Fearless
bound Code:
(b) making together A B C D
(c) an attractive volume (a) 3 4 5 2
(d) No error (b) 2 6 5 3
(c) 3 6 1 2

1|P a g e W W W . E D U T A P . C O . I N QUERY? HELLO@EDUTAP.CO.IN / 8146207241


(d) 2 4 1 3 6. Person who imposes strict discipline
Code:
Q10. List-I A B C D
A. Cadaverous (a) 6 3 4 1
B. Prodigal (b) 2 5 6 3
C. Decadence (c) 6 5 4 3
D. Gratuitous (d) 2 3 6 1
List-II
1. Falling to lower moral level Q13. List-I
2. Jubilation A. Adjunct
3. Something done or given unnecessarily B. Contiguous
4. Looking very pale C. Consanguine
5. Stout or well-built D. Recluse
6. Extravagant List-II
Code: 1. Same family relationship
A B C D 2. Diametrically opposite places
(a) 4 5 1 6 3. A person who has withdrawn from society
(b) 1 6 2 3 4. Enthusiastic person
(c) 4 6 1 3 5. Thing that is less important and not essential
(d) 1 5 2 6 6. Neighbouring
Code:
Q11. List-I A B C D
A. Dilettante (a) 5 1 2 3
B. Profligate (b) 4 6 1 5
C. Prescient (c) 5 6 1 3
D. Parochial (d) 4 1 2 5
List-II
1. A person with futuristic vision Directions (for the Five items which follow): Each
2. Miserly of the following five items carries a phrase/idiom.
3. A person who studies or does something Select the correct answer out of the four given
without serious interest or understanding choices (a), (b), (c) or (d) which conveys the
4. To show narrow interest closest meaning of the given phrase/idiom.
5. Wasteful Q14. The world is one's oyster
6. Pungent remarks
Code: (a) One feels that nobody in the world would help
A B C D him/her.
(a) 6 4 2 3 (b) The feeling one gets when he lands in an
(b) 3 5 1 4 unfamiliar country.
(c) 6 5 2 4 (c) One is able to enjoy all the pleasures and
(d) 3 4 1 5 opportunities that life has to offer.
(d) When one has an uphill task to perform
Q12. List-I
Q15. To go for the jugular?
A. Iconoclast
B. Humdinger (a) To work very hard for the success.
C. Martinet (b) To make a fierce destructive attack on the
D. Callow weakest point in an opponent’s argument.
List-II (c) To go in for the costliest thing.
1. Immature and inexperienced (d) To go in for dishonest means.
2. Person who attacks established customs Q16. To follow the primrose path
3. Excellent person or thing
(a) To follow one's objectives very steadfastly.
4. Popular leader
(b) To go on wandering.
5. Mysterious plot
(c) To trek to the mountains.
2|P a g e W W W . E D U T A P . C O . I N QUERY? HELLO@EDUTAP.CO.IN / 8146207241
(d) To go in for pursuit of pleasure or an easy life. J. Which one of the following assumptions or steps
is essential in developing the author's
Q17. A bee in one's bonnet position?
(a) To be very enterprising (a) All forms of life have a single overriding goal
(b) To be very timid and shy (b) The will to survive of a creature is identified
(c) To have an impractical idea with a desire for peace
(d) To have a particular idea which occupies one's (c) All beings are divided into higher and lower
thoughts continually groups
Q18. To take a rain-check (d) A parallel is drawn between happiness and life,
and pain and death
(a) To decline an offer but promise to accept it
later. Explanation:
(b) To go through the weather report.
(c) To proceed on sightseeing. I. The idea which represents the author's main
(d) To take to farming as a source of livelihood. point is 'peace and security are the chief goals
of all living beings', which is response (c). So
COMPREHENSION 'c' is the correct answer.
J. The best assumption underlying the passage
Directions: is 'The will to survive of a creature is identified
In this section you have one short passage. After with a desire for peace', which is response (b).
the passage you will find several questions based So 'b' is the correct answer.
on the passage. First, read Passage and answer the
questions based on it. Examples I and J are solved PASSAGE
for you.
When you buy a car, examine carefully the
PASSAGE important features of the model you are
considering. Many a buyer of the latest models has
In our approach to life, be it pragmatic or got into trouble by buying a car too broad or too
otherwise, a basic fact that confronts us squarely long for his garage. Furthermore a long car is much
and unmistakably is the desire for peace, security harder to manoeuvre in traffic and much more
and happiness. Different forms of life at different difficult to park. Another feature is the comfort
levels of existence make up the teeming denizens afforded. Are the seats nice and durably
of this earth of ours. And, no matter whether they upholstered? Is there enough glass area to give the
belong to the higher groups such as human beings driver a good view in all directions, particularly to
or to the lower groups such as animals, all beings the rear? It should be remembered that the
primarily seek peace, comfort and security. Life is heavier and more powerful a car is, the more
as dear to a mute creature as it is to a man. Even expensive it will be to operate. High powered
the lowliest insect strives for protection against motors require expensive high-octane petrol. The
dangers that threaten its life. Just as each one of greater weight means greater tyre wear and
us wants to live and not to die, so do all other enlarged brakes. The old cliché is still true; it is not
creatures. the initial cost but the upkeep which matters!
I. The author's main point is that Q19. According to this passage, the two factors
(a) different forms of life are found on earth that the buyer of a new car should keep in mind
(b) different levels of existence are possible in are
nature (a) its price and size
(c) peace and security are the chief goals of all (b) its model and upholstery
living beings (c) its size and the comfort it offers
(d) even the weakest creature struggles to (d) its upholstery and glass area
preserve its life
Q20. The size of the car should be in accordance
with
(a) the money one can afford to pay
3|P a g e W W W . E D U T A P . C O . I N QUERY? HELLO@EDUTAP.CO.IN / 8146207241
(b) the size of the garage the buyer has (c) non-serious (d) calculated
(c) the width of the road where the buyer lives
(d) the volume of traffic on the road Q26. Sanguine as he is in his attitude to life and
its problems, he cannot but be noticed wherever
Q21. The writer favours a car with a wide glass he goes.
area so that the driver can (a) optimistic (b) enthusiastic
(a) enjoy the scene outside while driving (c) realistic (d) dispassionate
(b) have a good view of the rear
(c) roll down the glasspanes to get enough fresh Q27. One's remonstration against social ills has
air to be consistent to be fruitful.
(d) display his beautiful upholstery with pride (a) outrage (b) demonstration
(c) protest (d) criticism
Q22. It is very expensive to maintain a large car
because it Directions (for the next Three items):
(a) is heavy
(b) requires more space for parking ANTONYMS
(c) needs changing its tyres more frequently In this section each item consists of a word or a
(d) needs more durable upholstery phrase which is underlined in the sentence given.
It is followed by four words or phrases. Select the
Q23. While buying a new car, one should word or phrase which is closest to the opposite in
specially consider its meaning of the underlined word or phrase.
(a) price Example F has been solved for you.
(b) maintenance cost F. Lucy is a smart girl.
(c) upholstery (a) lazy
(d) brakes (b) active
(c) indecent
Directions (for the next Four items): (d) casual
In this section you find a number of sentences, Answer: (a)
parts of which are underlined. You may also find Explanation: The word which is nearest to the
only a group of words which is underlined. For opposite in meaning of the underlined word.
each underlined part, four words/phrases are "smart" is "lazy". So 'a' is the correct answer.
listed below. Choose the word/phrase nearest in
meaning to the underlined part. Q28. The advice rendered to him sounded to be
Example E is solved for you. quite sagacious.
E. His style is quite transparent. (a) stupid (b) shallow
(a) verbose (c) impracticable (d) insincere
(b) involved
(c) lucid Q29. The Indian cultural fair was held at Rome in
(d) witty a grandiose manner.
Answer: (c) (a) unobtrusive (b) unimposing
Explanation: In item E the word "lucid" is nearest (c) unimpressive (d) simple
in meaning to the word "transparent". So 'c' is the
Q30. His voice sounded haughty when I spoke to
correct answer.
him over the telephone.
Q24. I rather like the quaint little house at the (a) pleasant (b) humble
end of the street. (c) soft (d) cheerful
(a) old (b) quiet
(c) unusual (d) haunted Part - B

Q25. If she continues to spend her time in such a Q31. The device to convert alternating current
frivolous manner, she will fail in her into direct current is
examinations. (a) Transformer
(a) frightful (b) leisurely (b) Rectifier
4|P a g e W W W . E D U T A P . C O . I N QUERY? HELLO@EDUTAP.CO.IN / 8146207241
(c) Alternator Which of these statements is/are correct?
(d) Condenser (a) 2 only
(b) 1 and 2
Q32. Combustion of a candle is a (c) 1 and 3
(a) Photochemical reaction (d) 1, 2 and 3
(b) Exothermic reaction
(c) Physical change Q36. Carbohydrates are stored in the body as
(d) Endothermic reaction (a) Sugars
(b) Starch
Q33. Which of the following is a chemical (c) Glucose
compound? (d) Glycogen
(a) Air
(b) Oxygen Q37. Bacteria and viruses cause
(c) Ammonia (a) Allergies
(d) Mercury (b) Non-communicable diseases
(c) Degenerative diseases
Q34. A and B ran over a distance starting from the (d) Infectious diseases
same place. The distance-time graph is shown:
Q38. Substances which are obtained from micro
organisms but used to destroy micro organisms
are
(a) Antigens
(b) Antibiotics
(c) Antibodies
(d) Antiseptics

Consider the following statements: Directions: The following Two (8 & 9) items consist
1. A ran throughout the given period of time with of two statements, one labelled the 'Assertion (A)
uniform velocity. and the other labelled the 'Reason (R)'. You are to
2. A ran throughout the given period of time with examine these two statements carefully and
uniform acceleration. decide if the Assertion (A) and the Reason (R) are
3. B stopped running after 2 hours. individually true and if so, whether the Reason (R)
4. A ran faster than B near the 2-hr stage. is a correct explanation of the Assertion (A). Select
Which of these statements is/are correct? your answers to these items using the Codes given.
(a) 2 and 3 below and mark your answer-sheet accordingly
(b) 1, 3 and 4
(c) 4 only Codes:
(d) 1, 2, 3 and 4 (a) Both A and R are true and R is the correct
explanation of A
Q35. Consider the following statements (b) Both A and R are true but R is NOT the correct
regarding the Nuclear Power Programme of explanation of A
India: (c) A is true but R is false
1. Pressurised heavy water reactors have been set (d) A is false but R
up at Kaiga (Karnataka) and Rawatbhata
(Rajasthan). Q39. Assertion (A): A bus and a car (with lower
2. Variable Energy Cyclotron Centre, Kolkata is one weight than that of the bus) moving at the same
of the five Research Centres under the velocity, are brought to rest on a level road by
Department of Science and Technology. applying brakes which exert same retarding force.
3. India has a vast resource of Thorium and the The car will travel lesser distance before coming to
third stage of Indian Nuclear Power Programme rest.
envisages the use of Thorium for power Reason (R): The kinetic energy to be destroyed is
generation. equal to the work done by the retarding force.
5|P a g e W W W . E D U T A P . C O . I N QUERY? HELLO@EDUTAP.CO.IN / 8146207241
goods, individually by the conveyors 'A', 'B' and
Q40. Assertion (A): A train has come out of the 'C' are
platform and is accelerating. If a person sitting in (a) 17, 12, 8 hrs.
the train and facing engine, throws up a coin, it will (b) 15, 10, 6 hrs.
fall ahead of him. (c) 16, 11, 7 hrs.
Reason (R): A force acts on the coin whose (d) 14, 9, 5 hrs.
magnitude is equal to the product of the mass of
the coin and acceleration of the train. Q45. According to a certain code
* means 'greater than'
Q41. Consider the following statements:
1. Intel's Pentium IV is an operating system used + means 'less than'
in PCs.
2. During booting period of a PC, the operating ÷ means 'equal to'
system is first loaded. % means 'plus'
3. Analogue computers are useful only in
scientific field while digital computers have If A * X, B ÷ Z, Y + C; then the relation. between
application in scientific as well as in business, ABC and XYZ is
administrative functions.
(a) ABC + XYZ
4. Micro-computer is another name for micro-
processor. (b) (ABC % XYZ) * (2XYZ)
Which of these statements are correct?
(a) 1, 2, 3 and 4 (c) (2XYZ) * (ABC % XYZ)
(b) 2 and 3 (d) (ABC/3) + (XYZ/4)
(c) 1 and 4
(d) 1, 2 and 4 Q46. What is the maximum number of pieces 1
cm in diameter and 20 cm long that can be
Q42. Which positional value divides the series cut out of a cylindrical wooden block 20 cm
into equal parts? in length and 3 cm in diameter?
(a) Median
(a) 4
(b) Second quartile
(c) 5th Décile (b) 5
(d) All the above three
(c) 6

Q43. A town 'P' is located in a particular district. (d) 7


The town 'A' is west of 'P'. Town 'T' is east of 'P'.
Q47. A group of boys decided to buy a few
Town 'K' is east of 'B' but west of 'T' and 'A'. They
cassettes whose price was between Rs.200/-
are all in the same district. Which town is the
and Rs.250/-. But at the time of purchase,
farthest west?
two of the boys declined to contribute as a
(a) P result of which the remaining boys had to
(b) K pay Rs. 1/- more than they had originally
(c) B planned. What was the price of the cassettes
(d) A if the boys contributed equally and in whole
Q44. A factory has three belt conveyors fitted in number of rupees?
its loading station. Conveyors 'A' and 'B' (a) Rs. 220/-
operating simultaneously lift all the goods in the
same time during which the goods can be lifted (b) Rs. 210/-
by the conveyor 'C' operating alone. Conveyor 'B'
(c) Rs. 230/-
lifts all the goods five hours faster than conveyor
'A' but four hours slower than the conveyor 'C' (d) Rs. 240/-
The durations of time required to lift all the

6|P a g e W W W . E D U T A P . C O . I N QUERY? HELLO@EDUTAP.CO.IN / 8146207241


Directions (for next Five items): Sumeet, Philips, (b) Bishan
Wasim, Bishan and Chetan are five players of
the College Cricket team and their home (c) Chetan
towns are Surat, Pune, Warangal, Bangalore (d) Philips
and Chandigarh but not in that order. The
five specialist slots of spinner, pace bowler, Q52. Chetan's hometown is
wicket-keeper, batsman and the captain are
(a) Bangalore
held by them, again not in the order of their
names stated above. (b) Warangal
(i) Their names, home towns and the specialities (c) Surat
do not start with the same letter.
(d) Pune
(ii) Neither Philips nor Wasim is the captain and
they do not belong to either Surat or Q53. A and B stand in a circular ring with 10
Bangalore. other persons. If the arrangement of 12
persons is randomly done, the chance, that
(iii) Sumeet is neither a wicket-keeper nor a there are exactly 3 persons between A and B
batsman. is
(iv) Pune is not Bishan's home town. (a) 3/12
(v) The player who hails from Bangalore is a (b) 2/11
wicket-keeper
(c) 2/13
(vi) The captain's hometown is Pune while the
batsman does not hail from Warangal. (d) 3/11

Q48. The spinner's hometown is Q54. 15 men can complete a work in 280 days.
They started the work and after every 10
(a) Pune days, 15 additional men were employed. In
how many days the work was completed?
(b) Warangal
(a) 70 days
(c) Bangalore
(b) 60 days
(d) Chandigarh
(c) 55 days
Q49. Chandigarh is hometown of
(d) 50 days
(a) Philips
Q55. Bag 'A' contains 5 white and 2 black balls.
(b) Wasim
Bag 'B' contains 2 white and 3 black balls. If
(c) Bishan any one bag is chosen and a ball is taken out
of it at random, what is the probability that
(d) Sumeet the ball is black?
Q50. Who is the pace bowler? (a) 31/70
(a) Bishan (b) 1/2
(b) Sumeet (c) 5/12
(c) Wasim (d) 3/5
(d) Chetan Q56. Three containers have their volumes in the
Q51. Who is the spinner? ratio 3: 4: 5. They are full of mixtures of milk
and water. The mixtures contain milk and
(a) Wasim water in the ratio of 4:1, 3:1 and 5:2
7|P a g e W W W . E D U T A P . C O . I N QUERY? HELLO@EDUTAP.CO.IN / 8146207241
respectively. The contents of all these three (c) 25
containers are poured into a fourth
container. The ratio of milk and water in the (d) 11
fourth container is Q61. How many lines other than those shown in
(a) 5:2 the figure are required to join each comer
with another?
(b) 157: 53
(c) 151 48
(d) 4:1
Q57. Two dice are thrown simultaneously. The
probability that the product of the two
numbers on the two dice is an even number
is
(a) 56
(a) 1/2
(b) 48
(b) 3/4
(c) 20
(c) 5/16
(d) 32
(d) 3/8
Q62. Two pipes 'A' and 'B' can fill a tank in 15
Q58. The flowers in a basket become double
minutes and 20 minutes respectively. Both
every minute and the basket gets full in one
the pipes are opened together but after 4
hour. In how much time, the basket was 1/32
minutes, pipe 'A' is turned off. What is the
full?
total time required to fill the tank?
(a) 12 minutes
(a) 12 mt 30 sec
(b) 32 minutes
(b) 14 mt 40 sec
(c) 45 minutes
(c) 10 mt 20 sec
(d) 55 minutes
(d) 11 mt 45 sec
Q59. If X is mortal, Y is wise. If Z is not good, A
Directions (For next Two items): A cube has six
will suffer. Y is not wise, if A suffers. If B is a
faces, each of a different colour. The red face
slave, X is immortal. If A suffers, what
is opposite to black. The green face is
follows?
between red and black. The blue face is
(a) X is mortal adjacent to white. The brown face is
adjacent to blue. The red face is the bottom
(b) B is a slave most face.
(c) Z is good Q63. The four colours adjacent to green are
(d) Y is wise (a) red, black, blue and white
Q60. In a group of 36 persons, a total of 16 take (b) red, black, brown and blue
tea while 9 take tea but not coffee. How
many persons in this group take coffee but (c) red, black, brown and white
not tea?
(d) red, brown, blue and white
(a) 27
Q64. The face opposite to the brown face is
(b) 20
(a) red
8|P a g e W W W . E D U T A P . C O . I N QUERY? HELLO@EDUTAP.CO.IN / 8146207241
(b) white (b) President
(c) Finance Minister
(c) green (d) Finance Minister
(d) blue
Q69. Assertion (A): Point of order is raised to
criticize a policy of the government.
Q65. Which of the following statements is not
Reason (R): It is an important tool of legislative
correct?
control.
(a) The 42nd Amendment extended the term of
Codes:
Lok Sabha and State Legislative Assemblies
(a) Both A and R are true and R is the correct
from 5 years to 6 years
explanation of A
(b) The President can dissolve the Lok Sabha on
(b) Both A and R are true but R is NOT the correct
the advice of the Prime Minister
explanation of A
(c) Money Bills cannot be introduced in the Rajya
(c) A is true but R is false
Sabha
(d) A is false but R
(d) One third members of the Rajya Sabha retire
on the expiry of every third year
Q70. Who of the following is the Chairman of
Planning Commission in India?
Q66. Consider the following statements:
(a) Finance Minister of India
1. For the three All-India Services, there are 24
(b) Prime Minister of India
State cadres including three joint cadres.
(c) President of India
2. Union Public Service Commission is the cadre
(d) Home Minister of India
controlling authority for Indian Administrative
Service.
Q71. Who among the following became the
3. Staff Selection Commission is entrusted with
President of India without having been the Vice –
the task of making recruitment to all the Group
President?
'B' and Group 'C' posts under the Government.
(a) Shri V. V. Giri
Which of these statements is/are correct?
(b) Shri Venkataraman
(a) 1 only
(c) Shri N. Sanjeeva Reddy
(b) 1 and 2
(d) Shri Zakir Hussain
(c) 2 and 3
(d) 1, 2 and 3
Q72. Nalanda University flourished during the
reign of which of the following rulers?
Q67. Consider the following statements:
(a) Chandragupta Maurya
1. Part 'B' of the Budget speech of the finance
(b) Ashoka
minister includes description of the initiatives
(c) Kanishka
on economic front to be taken up by the
(d) Harsha
Government for the coming year in addition to
direct and indirect tax proposals.
Q73. The foreign traveller who visited India
2. The receipts on account of the partial
during Vijayanagar period was?
disinvestment of Central Government's
(a) Megasthenes
holdings in the equity capital of public sector
(b) Yuan Chwang
enterprises are revenue receipts.
(c) Fa Hien
Which of these statements is/are correct?
(d) Nicolo Conti
(a) 1 only
(b) 2 only
Q74. Who among the following was a great
(c) Both 1 and 2
protagonist of the 'Doctrine of Lapse'?
(d) Neither 1 nor 2
(a) Lord Dalhousie
(b) Lord Minto
Q68. The Comptroller and Auditor General of
(c) Lord Curzon
India is appointed by the
(d) Lord Wellesley
(a) Prime Minister

9|P a g e W W W . E D U T A P . C O . I N QUERY? HELLO@EDUTAP.CO.IN / 8146207241


Q75. Chauri Chaura is known in the history of (a) 4 3 1
India's struggle for freedom because (b) 2 5 4
(a) Gandhiji launched his first Satyagraha in India (c) 4 5 1
from this place (d) 2 3 4
(b) Gandhiji withdrew his Satyagraha movement
when an irate mob set fire to a police station Q81. Consider the following statements :
and burnt some policemen 1. Bermuda situated in Western Atlantic Ocean
(c) Hundreds of freedom fighters were shot dead is a United Kingdom Overseas Territory.
at this place by the police 2. 38th Parallel divided boundary between North
(d) Gandhiji sat on a hunger strike and South Vietnam before unification.
3. Bahamas situated near Florida is one of the
Q76. The 'Grand Trunk Road' connects states of USA.
(a) Kolkata and Mumbai Which of these statements is/are correct?
(b) Delhi and Chennai (a) 1 only
(c) Kolkata and Amritsar (b) 2 and 3
(d) Tirupati and Ludhiana (c) 1 and 3
(d) 1 and 2
Q77. The Civil Disobedience Movement of 1930
culminated in the signing of a pact in 1931 by Q82. Consider the following statements :
Gandhiji and 1. Falkland Islands are situated in Pacific Ocean.
(a) Lord Irwin 2. Red Sea separates Sudan from Egypt.
(b) Lord Linlithgow 3. England, Norway, Denmark surround North Sea.
(c) Lord Wavell Which of these statements is/are correct?
(d) Lord Curzon (a) 2 only
(b) 1 and 3
Q78. Dhanvantari was (c) 2 and 3
(a) a famous General of Chandragupta Maurya (d) 3 only
(b) a noted physician in the Court of Chandragupta
Vikramaditya Q83. Which is the correct sequence of the given
(c) a famous dramatist who lived during the time American cities from West to East?
of Harsha (a) Salt Lake City-Pittsburgh-San Francisco-Boston
(d) a musician in the Court of Ashoka (b) San Francisco-Salt Lake City-Pittsburgh-Boston
(c) Salt Lake City-San Francisco-Boston-Pittsburgh
Q79. Which leader is called the ‘father' of India's (d) San Francisco-Pittsburgh-Salt Lake City-Boston
social reform movement?
(a) Mahatma Gandhi Q84. Latitude of a place is indicative of its
(b) Raja Rammohun Roy (a) time
(c) G.K. Gokhale (b) altitude
(d) Lokmanya Tilak (c) amount of rainfall
(d) temperature
Q80. Match List-I. (Classical Dance Form) with
List-II (State) and select the correct answer using Q85. Consider the following statements :
the codes given below the lists: 1. Alluvial soil is rich in chemical properties and
LIST I (Classical Dance LIST II (State) is capable of yielding Rabi and Kharif crops.
Form) 2. Black soil is suitable for cotton, groundnut.
A. Bharat Natyam 1. Odisha 3. Rabi crops are reaped in autumn after sowing
B. Kathakali 2. Tamil Nadu in June.
C. Kuchipudi 3. Manipur Which of these statements are correct?
4. Andhra Pradesh (a) 1, 2 and 3
5. Kerala (b) 1 and 2
Code: (c) 2 and 3
A B C (d) 1 and 3
10 | P a g e W W W . E D U T A P . C O . I N QUERY? HELLO@EDUTAP.CO.IN / 8146207241
C. Kaziranga Sanctuary 3. Uttar Pradesh
Q86. Consider the following statements : While it D. Periyar Sanctuary 4. Rajasthan
is 6:30 AM (Sunday) in London, it is around 5. Uttaranchal
1. 2.30 AM (Sunday) in Honolulu (Hawaii) Code:
2. 8.30 PM (Saturday) in Honolulu (Hawaii) A B C D
3. 3.30 PM (Sunday) in Tokyo (a) 2 5 4 1
4. 1:30 AM (Sunday) in Tokyo (b) 4 3 1 2
Which of these statements is/are correct? (c) 2 3 4 1
(a) 4 only (d) 4 5 1 2
(b) 2 and 3
(c) 1 and 4 Q90. Match List-I (Multi-purpose river valley
(d) 2 and 4 projects) with List-II (State) and select the correct
answer using the codes given below the lists:
Q87. Consider the following statements: List-I (Multi-purpose List-II (State)
1. Prime Meridian passes through Greenwich river valley projects)
and is at 0° Longitude. A. Purna Project 1. Gujarat
2. International Date Line, on the map appears B. Bhima Project 2. Maharashtra
as a straight line along 180° Longitude. C. Kakrapara Project 3. Uttaranchal
3. While crossing International Date Line from D. Tehri Dam Project 4. Uttar Pradesh
east to west, one gains a day. 5. Karnataka
4. Tropic of Cancer lies at 23½° Latitude S. Code:
Which of these statements are correct? A B C D
(a) 1, 2, 3 and 4 (a) 1 2 5 3
(b) 1 and 2 (b) 2 5 1 3
(c) 1 and 3 (c) 1 3 5 4
(d) 2 and 3 (d) 2 4 1 3
Q88. Consider the following statements : Q91. Match List-I (City) with List-II (River Passing
According to Census 2001 through the city) and select the correct answer
1. The literacy rate increase for males is more using the codes given below the lists:
than that of the females during last one List-I (City) List-II (River passing
decade. through the city)
2. Amongst the States and Union Territories,
A. Nasik 1. Krishna
Delhi has the highest population density while
B. Surat 2. Cauvery
Sikkim has the lowest.
C. Ujjain 3. Godawari
3. The correct sequence of population in
D. Vijayawada 4. Shipra
descending order of Rajasthan, Madhya
5. Tapti
Pradesh, Tamil Nadu and Karnataka is Tamil
Nadu-Madhya Pradesh-Rajasthan-Karnataka. Code:
Which of these statements is/are correct? A B C D
(a) 1 and 2 (a) 3 5 4 1
(b) 2 and 3 (b) 4 1 3 2
(c) 1, 2 and 3 (c) 3 1 4 2
(d) 3 only (d) 4 5 3 1

Q89. Match List-I (Park/Sanctuary) with List-II Q92. Recently, Cannes Film Festival was held in
(State) and select the correct answer using the (a) Switzerland
codes given below the lists: (b) France
(c) Italy
List-I (Park/Sanctuary) List-II (State)
(d) England
A. Keoladeo National 1. Assam
Park
Q93. Shri Kiran Karnik is the
B. Corbett National Park 2. Kerala
11 | P a g e W W W . E D U T A P . C O . I N QUERY? HELLO@EDUTAP.CO.IN / 8146207241
(a) President of NASSCOM (d) 2 3 4 1
(b) Chairman of the Board of Control of Cricket for
India Q97. Match List-I (Person) with List-II
(c) Chairman of Maharashtra State Public Service (Organization) and select the correct answer
Commission using the code given below the lists:
(d) Managing Director of Engineers India Ltd. List-I (Person)
A. Rajendra S. Pawar
Q94. Knesset is the name of the Parliament of B. Ashok Soota
(a) Norway C. R.S. Lodha
(b) Sweden D. R.K. Pachauri
(c) Israel List-II (Organization)
(d) Spain 1. CII
2. TERI
Q95. Match List-I with List-II and select the 3. NIIT
correct answer using the code given below the 4. FICCI
lists: Code:
List-I A B C D
A. Dennis Tito (a) 4 2 3 1
B. Colin Powell (b) 3 1 4 3
C. Jacques Chirac (c) 4 1 3 2
D. Donald H. Rumsfeld (d) 3 2 4 1
List-II
1. President of France Q98. Raksha Kavach' is
2. Defence Secretary, USA (a) a new safety equipment designed for the
3. Secretary of State, USA soldiers of Indian Army
4. First space tourist (b) an anti-collision device to prevent
Code: collision between trains designed by the
A B C D Konkan Railway Corporation
(a) 1 3 4 2 (c) a new design for earthquake resistant
(b) 4 2 1 3 buildings
(c) 1 2 4 3 (d) a new device for the Indian airports for
(d) 4 3 1 2 the safe landing of aircrafts

Q96. Match List-I (Football World Cup/Year) with Q99. Consider the following statements in
List-II (Host Country) and select the correct respect of Soccer World Cup :
answer using the code given below the lists: 1. Prior to FIFA-2002 Cup, Brazil had won
List-I (Football World Cup/Year) the World Cup four times.
A. 1986 2. Indomitable Lions' is the nickname for
B. 1990 Cameroon team.
C. 1994 3. 'Azzurri' is the nickname for the Italian
D. 1998 team.
List-II (Host Country) 4. Senegal, Slovenia and Ecuador
1. Italy participating in FIFA-2002 are the only
2. Mexico teams participating in the World Cup for
3. France the first time.
4. USA Which of these statements are correct?
5. Spain (a) 1, 2 and 3
Code: (b) 2 and 3
A B C D (c) 1, 3 and 4
(a) 5 3 2 1 (d) 1, 2 and 4
(b) 2 1 4 3
(c) 5 1 2 3
12 | P a g e W W W . E D U T A P . C O . I N QUERY? HELLO@EDUTAP.CO.IN / 8146207241
Q100. Recently, Government disinvested stake in B. Allahabad
VSNL, IBP and IPCL to C. Kamptee
(a) Bharti Telesonic, Reliance and GAIL, D. Mhow
respectively List-II (Establishment)
(b) Tata Group of Industries, Indian Oil 1. Central Air Command HQ
Corporation and Reliance, respectively 2. Naval Air Base
(c) Bharti Telesonic, Royal Dutch Shell and 3. Institute of Military Law
Pfizer, respectively 4. Coast Guard District HQ
(d) Motorala, Kuwait Petroleum and 5. College of Combat
Ranbaxy, respectively Code:
A B C D
Q101. Stockholm Syndrome is a mental condition (a) 2 5 3 1
experienced by (b) 3 1 4 5
(a) the young persons who take up jobs in (c) 2 1 3 5
the industry after passing out from the (d) 3 5 4 1
Management Schools
(b) persons who migrate from rural areas to Q104. Match List-I (Library) with List-II (City) and
urban cities, during the earlier stages of select the correct answer
their stay List-I (Library)
(c) drug-addicts in the rehabilitation centres A. National Library
(d) hostages and kidnap victims in which B. State Central Library
positive feelings develop towards their C. Connemara Public Library
captors List-II (City)
1. New Delhi
Q102. Match List-I (Football World 2. Kolkata
Cup/Year) with List-II (Host Country) and select 3. Mumbai
the correct answer using the code given below 4. Chennai
the lists: Code:
List-I (Prominent Football Player) A B C
A. Michael Owen (a) 1 3 4
B. Luis Figo (b) 2 3 4
C. Gabriel Batistuta (c) 1 4 3
D. Raul Gonzalez (d) 2 4 3
List-II (Country)
1. Italy Q105. Crop Yield Formulation Unit which has
2. Spain developed statistical models to forecast crop
3. Portugal yield works under the control of
4. Argentina (a) Ministry of Water Resources
5. England (b) Department of Agriculture and Co
6. Brazil operation
Code: (c) India Meteorological Department
A B C D (d) Department of Food and Public
(a) 5 2 4 3 Distribution
(b) 6 3 1 2
(c) 5 3 4 2 Q106. East Timor, the latest independent nation
(d) 6 2 1 3 in the world was ruled by
(a) Indonesia (b) Australia
Q103. Match List-I (City) with List-II (c) Malaysia (d) Philippines
(Establishment) and select the correct answer
using the codes given below the lists: Q107. What is the chronological sequence of the
List-I (City) following developments?
A. Arkonnam
13 | P a g e W W W . E D U T A P . C O . I N QUERY? HELLO@EDUTAP.CO.IN / 8146207241
(a) National Aluminium Company Ltd. : (a) 1 only
Bhubaneswar (b) 2 and 3
(b) Indian Bureau of Mines : Dhanbad (c) 1 and 2
(c) National Film Archives of India : Pune (d) 1, 2 and 3
(d) National Archives of India : New Delhi
Q114. Consider the following statements:
Q108. Sagar Samrat' is the name of 1. Launching of Fourth five-year plan was
(a) an island near Port Blair postponed and three annual plans had to be
(b) a drilling ship in Bombay High resorted to between 1966 to 1969.
(c) an island near Maldives 2. Fifth five-year plan was terminated a year
(d) name of a 5-star hotel in Goa earlier.
3. Eighth five-year plan commenced in 1992
Q109. Which tourist destination from amongst instead of 1990.
the following will be developed as an Which of these statements is/are correct?
international destination for tourism according (a) 2 and 3
to the Budget 2002-03? (b) 1 and 2
(a) Port Blair (c) 1 and 3
(b) Hampi (d) 1, 2 and 3
(c) Pushkar
(d) Nalanda Q115. Assertion (A): Macro-economic deals with
the study of aggregates and averages.
Q110. Shri Rajender Singh won the Ramon Reason (R): Central problem of macro-economics
Magsaysay Award for is the determination of income and employment
(a) Emergent leadership
(b) Community leadership Codes:
(c) Illiteracy eradication (a) Both A and R are true and R is the correct
(d) Bio-technology explanation of A
(b) Both A and R are true but R is NOT the correct
Q111. Which among the following became the explanation of A
first woman Prime Minister in the world? (c) A is true but R is false
(a) Indira Gandhi (d) A is false but R
(b) Kim Campbell
(c) Margaret Thatcher Q116. Directions: The following items consist of
(d) Sirimavo Bandaranaike two statements, one labelled the 'Assertion (A)
and the other labelled the 'Reason (R)'. You are to
Q112. Who among the following wrote the book examine these two statements carefully and
Algebra of Infinite Justice? decide if the Assertion (A) and the Reason (R) are
(a) Anita Desai individually true and if so, whether the Reason (R)
(b) Vikram Seth is a correct explanation of the Assertion (A).
(c) Arundhati Roy Select your answers to these items using the
(d) Rohington Mistry Codes given. below and mark your answer-sheet
accordingly

Q113. Consider the following statements: Assertion (A): Amongst the three strategies
overall cost leadership, differentiation and market
1. The demand for commodities having a large
segmentation; a company with differentiation
number of close substitutes is less elastic than
strategy has less competition from both its direct
the commodities having a smaller number of
competitors and from potential substitutes.
substitutes
2. Luxury items have a highly elastic demand Reason (R): Its consumers have greater brand
3. The commodities whose consumption cannot loyalty, and, therefore, less price sensitivity.
easily be postponed have less elastic demand Codes:
Which of these statements is/are correct?
14 | P a g e W W W . E D U T A P . C O . I N QUERY? HELLO@EDUTAP.CO.IN / 8146207241
(a) Both A and R are true and R is the correct (c) A is true but R is false
explanation of A (d) A is false but R
(b) Both A and R are true but R is NOT the correct
explanation of A Q119. Directions: The following items consist of
(c) A is true but R is false two statements, one labelled the 'Assertion (A)
(d) A is false but R and the other labelled the 'Reason (R)'. You are to
examine these two statements carefully and
Q117. Directions: The following items consist of decide if the Assertion (A) and the Reason (R) are
two statements, one labelled the 'Assertion (A) individually true and if so, whether the Reason (R)
and the other labelled the 'Reason (R)'. You are to is a correct explanation of the Assertion (A).
examine these two statements carefully and Select your answers to these items using the
decide if the Assertion (A) and the Reason (R) are Codes given. below and mark your answer-sheet
individually true and if so, whether the Reason (R) accordingly
is a correct explanation of the Assertion (A).
Select your answers to these items using the Assertion (A): Enterprise Resource Planning (ERP)
Codes given. below and mark your answer-sheet is useful in a manufacturing company. It does not
accordingly find application in service industry.
Reason (R): Enterprise Resource Planning (ERP)
Assertion (A): In the organizational structure of enables a high level of interaction and
the modern companies, the trend is towards wider coordination along the supply chain.
span of control. Codes:
Reason (R): Narrow span of control discourages (a) Both A and R are true and R is the correct
autonomy. explanation of A
Codes: (b) Both A and R are true but R is NOT the correct
(a) Both A and R are true and R is the correct explanation of A
explanation of A (c) A is true but R is false
(b) Both A and R are true but R is NOT the correct (d) A is false but R
explanation of A
(c) A is true but R is false Q120. Consider the following statements
(d) A is false but R concerning United Arab Emirates:
1. Oman is one of the seven Emirates which form
Q118. Directions: The following items consist of the United Arab Emirates.
two statements, one labelled the 'Assertion (A) 2. Dubai is the capital of United Arab Emirates.
and the other labelled the 'Reason (R)'. You are to 3. Fishing and tourism are the key industries of
examine these two statements carefully and United Arab Emirates.
decide if the Assertion (A) and the Reason (R) are Which of these statements is/are correct?
individually true and if so, whether the Reason (R) (a) 1 and 2
is a correct explanation of the Assertion (A). (b) 3 only
Select your answers to these items using the (c) 2 and 3
Codes given. below and mark your answer-sheet (d) 1, 2 and 3
accordingly
Assertion (A): Unity of command cannot always
be strictly applied in practice.
Reason (R): Workers should report to different
supervisors for different aspects or tasks technical,
financial, administrative etc.
Codes:
(a) Both A and R are true and R is the correct
explanation of A
(b) Both A and R are true but R is NOT the correct
explanation of A

15 | P a g e W W W . E D U T A P . C O . I N QUERY? HELLO@EDUTAP.CO.IN / 8146207241


Visit: www.civilstap.com
For Any Query Mail us: hello@civilstap.com or call us at - (+91)-8146207241
0
(b) a large mass fell
SOLUTIONS – PREVIOUS YEAR (c) and buried him completely
QUESTIONS (d) No error
Answer: (a)
Explanation: Use ‘the’ before ‘sand’ as ‘the’ is
UPSC EPFO APFC - 2002 used to refer to specific or particular nouns.
PART- A Q3. (a) The book is well printed and attractively
bound
SPOTTING ERRORS (b) making together
(c) an attractive volume
Directions (for the next Five items): (d) No error
(i) In this section a number of sentences are Answer: (b)
given. Each sentence has three underlined Explanation: Use ‘it’ after ‘making’. “The book is
parts. indicated by (a), (b) and (c). Read each well-printed and attractively bound making
sentence to find out whether there is an error. altogether an attractive volume.” The sentence
If you find an error in any one of the has two clauses, “The book is well-printed and
underlined parts (a, b, c), indicate your attractively bound,” and “making altogether an
response by blackening the letter related to attractive volume.” The first clause has a noun,
that part in the Answer Sheet provided. If a book, and it talks about it, however, the second
sentence has no error. indicate this by clause does not have a noun and so it is not clear
blackening (d) which stands for "No error". that what subject or object it is referring to. The
(ii) Errors may belong to grammar, usage or second clause needs a noun or pronoun to
idiom. Examples P and Q have been solved for complete its meaning. As the second clause is also
you talking about the book mentioned in the first
clause, it requires a pronoun for the book. So, the
P. (a) My friend and myself second clause needs ‘it’ as an object for verb
(b) Study together ‘making.’ Hence, the second clause should be” and
(c) During holidays “making it altogether an attractive volume.”
(d) No error Hence, the word or phrase that must be changed
Answer: (d) for the sentence to be correct is indicated by
Explanation: The correct answer for P is letter 'd' Option (b). Option (b) is the correct answer.
because the sentence has no mistake in it.
Q. (a) The rice from Dehradun is Q4. (a) A little younger
(b) More superior (b) to myself we, both were
(c) To that of Saharanpur (c) devoted to cricket and boating
(d) No error (d) No error
Answer: (b) Answer: (b)
Explanation: The correct answer for Q is 'b' Explanation: Use ‘than’ in place of ‘to’. "Than'' in
because the mistake in the sentence is in the part the sentence is a conjunction, a word connecting
carrying the letter 'b'. two complete sentences. When you add the
implied verb at the end, it sounds more logical:
Q1. (a) Choose only She's younger than I am.
(b) Such friends
(c) That you can trust Q5. (a) He had no sooner
(d) No error (b) arrived than
Answer: (c) (c) he was asked to leave again
Explanation: ‘as’ will be used in place of ‘that’ as (d) No error
relative pronoun, because relative pronoun is used Answer: (d)
in the form of ‘as’ after ‘such’. Explanation: The sentence has no mistake in it.
Ex. He is such a good man as can help you.
Directions (for the next three items):
Q2. (a) While he was digging sand

1|P a g e W W W . E D U T A P . C O . I N QUERY? HELLO@EDUTAP.CO.IN / 8146207241


Select the most suitable word from (a), (b), (c) and 5. Deep-rooted
(d) given below each sentence to fill in the blanks 6. Fearless
Code:
Q6. The Rajputs always fought _____________ A B C D
the last man. (a) 3 4 5 2
(a) upto (b) till (b) 2 6 5 3
(c) to (d) for (c) 3 6 1 2
Answer: (c) (d) 2 4 1 3
Explanation: Definition of to the last man: until all Answer: (a)
the men in a group are killed, defeated, etc. They Explanation: Obsequious means obedient or
vowed to fight to the last man. attentive to an excessive or servile degree.
Obstreperous means noisy and difficult to control.
Q7. I have been surfing internet._________ the
Inveterate means having a particular habit,
last two hours.
activity, or interest that is long-established and
(a) since (b) for unlikely to change.
(c) from (d) right from Impecunious means having little or no money.
Answer: (b)
Explanation: The word 'for' is used to show a Q10. List-I
period of time, be it in the past, present or future. A. Cadaverous
The word 'since' is used to refer to a time that has B. Prodigal
begun in the past and is still continuing. Use ‘from’ C. Decadence
to mean the starting time and ‘to’ for the end or D. Gratuitous
finish time. Note: To can be a synonym of until and List-II
till so it’s perfectly correct to use for with until or 1. Falling to lower moral level
till. 2. Jubilation
3. Something done or given unnecessarily
Q8. You should be considerate __________ your
4. Looking very pale
employees and I assure you of your success
5. Stout or well-built
in every walk of life.
6. Extravagant
(a) for (b) to Code:
(c) upon (d) on A B C D
Answer: (b) (a) 4 5 1 6
Explanation: ‘to’ is used to show what somebody’s (b) 1 6 2 3
opinion or feeling about something is and it is also (c) 4 6 1 3
used to show somebody’s attitude or reaction to (d) 1 5 2 6
something. Answer: (c)
Directions (for the Five items which follow): Explanation: Cadaverous means very pale, thin, or
Match words in Last I with the words/group of bony.
words in the List-II such that they convey the Prodigal means spending money or using
same meaning or sense. Select the correct resources freely and recklessly; wastefully
answer using the codes given below the lists extravagant.
Decadence means moral or cultural decline as
Q9. List-I characterized by excessive indulgence in pleasure
A. Obsequious or luxury.
B. Obstreperous Gratuitous means done without good reason;
C. Inveterate uncalled for.
D. Impecunious
Q11. List-I
List-II
1. Dishonest A. Dilettante
2. Having little or no money B. Profligate
3. Too willing to obey C. Prescient
4. Noisy and unruly D. Parochial

2|P a g e W W W . E D U T A P . C O . I N QUERY? HELLO@EDUTAP.CO.IN / 8146207241


List-II Callow means (of a young person) inexperienced
1. A person with futuristic vision and immature.
2. Miserly
3. A person who studies or does something Q13. List-I
without serious interest or understanding A. Adjunct
4. To show narrow interest B. Contiguous
5. Wasteful C. Consanguine
6. Pungent remarks D. Recluse
Code: List-II
A B C D 1. Same family relationship
(a) 6 4 2 3 2. Diametrically opposite places
(b) 3 5 1 4 3. A person who has withdrawn from society
(c) 6 5 2 4 4. Enthusiastic person
(d) 3 4 1 5 5. Thing that is less important and not essential
Answer: (b) 6. Neighbouring
Explanation: Dilettante means a person who Code:
cultivates an area of interest, such as the arts, A B C D
without real commitment or knowledge. (a) 5 1 2 3
Profligate means recklessly extravagant or (b) 4 6 1 5
wasteful in the use of resources. (c) 5 6 1 3
Prescient means having or showing knowledge of (d) 4 1 2 5
events before they take place. Answer: (c)
Parochial means having a limited or narrow Explanation: Adjunct means a thing added to
outlook or scope. something else as a supplementary rather than an
essential part.
Q12. List-I Contiguous means sharing a common border;
A. Iconoclast touching.
B. Humdinger Consanguine means relating to or denoting people
C. Martinet descended from the same ancestor.
D. Callow Recluse means a person who lives a solitary life
List-II and tends to avoid other people.
1. Immature and inexperienced
2. Person who attacks established customs Directions (for the Five items which follow): Each
3. Excellent person or thing of the following five items carries a phrase/idiom.
4. Popular leader Select the correct answer out of the four given
5. Mysterious plot choices (a), (b), (c) or (d) which conveys the
6. Person who imposes strict discipline closest meaning of the given phrase/idiom.
Code: Q14. The world is one's oyster
A B C D
(a) One feels that nobody in the world would help
(a) 6 3 4 1
him/her.
(b) 2 5 6 3
(b) The feeling one gets when he lands in an
(c) 6 5 4 3
unfamiliar country.
(d) 2 3 6 1
(c) One is able to enjoy all the pleasures and
Answer:(d)
opportunities that life has to offer.
Explanation: Iconoclast means a person who
(d) When one has an uphill task to perform
attacks or criticizes cherished beliefs or
Answer: (c)
institutions.
Explanation: The world is one's oyster means that
Humdinger means a remarkable or outstanding
the person being spoken about has every
person or thing of its kind.
advantage necessary to achieve what he wishes to
Martinet means a person who demands complete
achieve, and to enjoy life.
obedience; a strict disciplinarian.

3|P a g e W W W . E D U T A P . C O . I N QUERY? HELLO@EDUTAP.CO.IN / 8146207241


Q15. To go for the jugular? questions based on it. Examples I and J are solved
(a) To work very hard for the success. for you.
(b) To make a fierce destructive attack on the
weakest point in an opponent’s argument. PASSAGE
(c) To go in for the costliest thing. In our approach to life, be it pragmatic or
(d) To go in for dishonest means. otherwise, a basic fact that confronts us squarely
Answer: (b) and unmistakably is the desire for peace, security
Explanation: To go for the jugular means to attack and happiness. Different forms of life at different
quickly and savagely in the most vicious and levels of existence make up the teeming denizens
effective way possible. of this earth of ours. And, no matter whether they
Q16. To follow the primrose path belong to the higher groups such as human beings
or to the lower groups such as animals, all beings
(a) To follow one's objectives very steadfastly.
primarily seek peace, comfort and security. Life is
(b) To go on wandering.
as dear to a mute creature as it is to a man. Even
(c) To trek to the mountains.
the lowliest insect strives for protection against
(d) To go in for pursuit of pleasure or an easy life.
dangers that threaten its life. Just as each one of
Answer: (d)
us wants to live and not to die, so do all other
Explanation: If you lead someone down the
creatures.
primrose path, you encourage that person to live
an easy life that is full of pleasure but bad for I. The author's main point is that
them. (a) different forms of life are found on earth
Q17. A bee in one's bonnet (b) different levels of existence are possible in
nature
(a) To be very enterprising
(c) peace and security are the chief goals of all
(b) To be very timid and shy
living beings
(c) To have an impractical idea
(d) even the weakest creature struggles to
(d) To have a particular idea which occupies one's
preserve its life
thoughts continually
Answer: (d) J. Which one of the following assumptions or steps
Explanation: To have a bee in one's bonnet means is essential in developing the author's
to be obsessed with a certain idea, to be position?
preoccupied with something. (a) All forms of life have a single overriding goal
Q18. To take a rain-check (b) The will to survive of a creature is identified
with a desire for peace
(a) To decline an offer but promise to accept it
(c) All beings are divided into higher and lower
later.
groups
(b) To go through the weather report.
(d) A parallel is drawn between happiness and life,
(c) To proceed on sightseeing.
and pain and death
(d) To take to farming as a source of livelihood.
Answer: (a) Explanation:
Explanation: To take a rain-check means an
offered or requested postponement of an I. The idea which represents the author's main
invitation until a more convenient, usually point is 'peace and security are the chief goals
unspecified time. of all living beings', which is response (c). So
'c' is the correct answer.
COMPREHENSION
J. The best assumption underlying the passage
is 'The will to survive of a creature is identified
Directions:
with a desire for peace', which is response (b).
In this section you have one short passage. After
So 'b' is the correct answer.
the passage you will find several questions based
on the passage. First, read Passage and answer the PASSAGE

4|P a g e W W W . E D U T A P . C O . I N QUERY? HELLO@EDUTAP.CO.IN / 8146207241


When you buy a car, examine carefully the Explanation: According to the passage the writer
important features of the model you are favours a car with a wide glass area so that the
considering. Many a buyer of the latest models has driver can have a good view of the rear. Therefore,
got into trouble by buying a car too broad or too option (b) is the correct answer.
long for his garage. Furthermore a long car is much
harder to manoeuvre in traffic and much more Q22. It is very expensive to maintain a large car
difficult to park. Another feature is the comfort because it
afforded. Are the seats nice and durably (a) is heavy
upholstered? Is there enough glass area to give the (b) requires more space for parking
driver a good view in all directions, particularly to (c) needs changing its tyres more frequently
the rear? It should be remembered that the (d) needs more durable upholstery
heavier and more powerful a car is, the more Answer: (c)
expensive it will be to operate. High powered Explanation: Option (c) is the correct answer as it
motors require expensive high-octane petrol. The is mentioned in the paragraph that the greater
greater weight means greater tyre wear and weight means greater tyre wear and enlarged
enlarged brakes. The old cliché is still true; it is not brakes and that’s why it is expensive to maintain a
the initial cost but the upkeep which matters! large car.

Q19. According to this passage, the two factors Q23. While buying a new car, one should
that the buyer of a new car should keep in mind specially consider its
are (a) price
(a) its price and size (b) maintenance cost
(b) its model and upholstery (c) upholstery
(c) its size and the comfort it offers (d) brakes
(d) its upholstery and glass area Answer: (b)
Answer: (b) Explanation: It is given in the passage that The old
Explanation: In the given passage the two factors cliché is still true; it is not the initial cost but the
that are mentioned to keep in mind while buying a upkeep which matters!. Which basically means
car are its model and upholstery. Therefore option that the maintenance cost should be considered
(b) is the correct answer here. by the buyer specifically. Therefore, option (b) is
the correct answer.
Q20. The size of the car should be in accordance
with Directions (for the next Four items):
(a) the money one can afford to pay In this section you find a number of sentences,
(b) the size of the garage the buyer has parts of which are underlined. You may also find
(c) the width of the road where the buyer lives only a group of words which is underlined. For
(d) the volume of traffic on the road each underlined part, four words/phrases are
Answer: (b) listed below. Choose the word/phrase nearest in
Explanation: As mentioned in the second line of meaning to the underlined part.
the passage that the buyer get into trouble while Example E is solved for you.
buying a car too long or too broad for his garage E. His style is quite transparent.
hence size of the car should be in accordance with (a) verbose
the size of the garage the buyer has. (b) involved
(c) lucid
Q21. The writer favours a car with a wide glass (d) witty
area so that the driver can Answer: (c)
(a) enjoy the scene outside while driving Explanation: In item E the word "lucid" is nearest
(b) have a good view of the rear in meaning to the word "transparent". So 'c' is the
(c) roll down the glasspanes to get enough fresh correct answer.
air
(d) display his beautiful upholstery with pride Q24. I rather like the quaint little house at the
Answer: (b) end of the street.

5|P a g e W W W . E D U T A P . C O . I N QUERY? HELLO@EDUTAP.CO.IN / 8146207241


(a) old (b) quiet Demonstration means an act of showing that
(c) unusual (d) haunted something exists or is true by giving proof or
Answer: (c) evidence.
Explanation: Quaint means attractively unusual or Criticism means the expression of disapproval of
old-fashioned. someone or something on the basis of perceived
Haunted means to have a disquieting or harmful faults or mistakes.
effect on.
Directions (for the next Three items):
Q25. If she continues to spend her time in such a
frivolous manner, she will fail in her ANTONYMS
examinations. In this section each item consists of a word or a
(a) frightful (b) leisurely phrase which is underlined in the sentence given.
(c) non-serious (d) calculated It is followed by four words or phrases. Select the
Answer: (c) word or phrase which is closest to the opposite in
Explanation: Frivolous means behaving in a silly meaning of the underlined word or phrase.
way and not taking anything seriously. Example F has been solved for you.
Frightful means very unpleasant, serious, or F. Lucy is a smart girl.
shocking. (a) lazy
Leisurely used to describe an action that is done in (b) active
a relaxed way, without hurrying. (c) indecent
Calculated means (of an action) done with full (d) casual
awareness of the likely consequences. Answer: (a)
Explanation: The word which is nearest to the
Q26. Sanguine as he is in his attitude to life and
opposite in meaning of the underlined word.
its problems, he cannot but be noticed wherever
"smart" is "lazy". So 'a' is the correct answer.
he goes.
(a) optimistic (b) enthusiastic Q28. The advice rendered to him sounded to be
(c) realistic (d) dispassionate quite sagacious.
Answer: (a) (a) stupid (b) shallow
Explanation: Sanguine means optimistic or (c) impracticable (d) insincere
positive, especially in an apparently bad or difficult Answer: (a)
situation. Explanation: Sagacious means having or showing
Optimistic means hopeful and confident about the keen mental discernment and good judgement;
future. wise or shrewd. ‘Stupid’ is opposite in meaning of
Enthusiastic means having or showing intense and ‘sagacious’.
eager enjoyment, interest, or approval. Shallow means of little depth.
Realistic means having or showing a sensible and Impracticable means (of a course of action)
practical idea of what can be achieved or impossible in practice to do or carry out.
expected. Insincere means not expressing genuine feelings.
Dispassionate means not influenced by strong
emotion, and so able to be rational and impartial. Q29. The Indian cultural fair was held at Rome in
a grandiose manner.
Q27. One's remonstration against social ills has (a) unobtrusive (b) unimposing
to be consistent to be fruitful. (c) unimpressive (d) simple
(a) outrage (b) demonstration Answer: (c)
(c) protest (d) criticism Explanation: Grandiose means impressive and
Answer: (c) imposing in appearance or style, especially
Explanation: Remonstration means the act of pretentiously so. Therefore, ‘unimpressive’ is the
expressing earnest opposition or protest. antonym of ‘grandiose’.
Outrage means an extremely strong reaction of Unobtrusive means not conspicuous or attracting
anger, shock, or indignation. attention.

6|P a g e W W W . E D U T A P . C O . I N QUERY? HELLO@EDUTAP.CO.IN / 8146207241


Unimposing means not imposing or impressive in (a) Photochemical reaction
appearance. (b) Exothermic reaction
Simple means easily understood or done; (c) Physical change
presenting no difficulty. (d) Endothermic reaction
Answer: B
Q30. His voice sounded haughty when I spoke to Explanation:
him over the telephone. The combustion of a candle is an exothermic
(a) pleasant (b) humble reaction. Hence option B is correct.
(c) soft (d) cheerful • An exothermic reaction is a chemical reaction
Answer: (b) that releases energy in the form of light or
Explanation: Haughty means arrogantly superior heat. The burning of the candle is an
and disdainful. ‘Humble’ is opposite in meaning of exothermic reaction.
‘haughty’. Therefore, option (b) is the correct
answer. Q33. Which of the following is a chemical
Pleasant means giving a sense of happy compound?
satisfaction or enjoyment. (a) Air
Cheerful means noticeably happy and optimistic. (b) Oxygen
Soft means having a pleasing quality involving a (c) Ammonia
subtle effect or contrast rather than sharp (d) Mercury
definition. Answer: C
Explanation:
Part - B Ammonia (NH3) is a chemical compound while
Oxygen and Mercury are elements and Air is a
Q31. The device to convert alternating current heterogeneous mixture. Hence option C is
into direct current is correct.
(a) Transformer • A chemical element is a species of atoms that
(b) Rectifier have a given number of protons in their nuclei,
(c) Alternator including the pure substance consisting only of
(d) Condenser that species. For example, Sodium, Lithium,
Answer: B Oxygen, Hydrogen etc. are elements.
Explanation: • A chemical compound is any substance
A rectifier is a device that converts alternating composed of identical molecules consisting of
electric current into direct current. It may be an atoms of two or more chemical elements
electron tube (either a vacuum or a gaseous type), which are chemically linked by chemical
vibrator, solid-state device, or mechanical device. bonds. For example, Water (H2O) is a chemical
Hence option B is correct. combination of Hydrogen and Oxygen
• A transformer is a device that transfers electric elements in a fixed ratio and is thus called a
energy from one alternating-current circuit to chemical compound.
one or more other circuits, either increasing • A mixture is a combination made up of two or
(stepping up) or reducing (stepping down) the more chemical components that are not
voltage. chemically linked but are physically present
• An alternator is an electrical generator that together. It can be of two kinds (viz.)
converts mechanical energy to electrical o Homogeneous mixture - A
energy in the form of alternating current. homogeneous mixture is a mixture in
• A Condenser is a device for reducing a gas or which the composition is uniform
vapour to a liquid. Condensers are employed throughout the mixture. All solutions
in power plants to condense exhaust steam would be considered homogeneous.
from turbines and in refrigeration plants to For example a sugar solution has
condense refrigerant vapours, such as uniform distribution of sugar particles
ammonia and fluorinated hydrocarbons. throughout the mixture making it a
homogeneous mixture.
Q32. Combustion of a candle is a
7|P a g e W W W . E D U T A P . C O . I N QUERY? HELLO@EDUTAP.CO.IN / 8146207241
o Heterogeneous mixture – A greater the gradient (and the steeper the line)
heterogeneous mixture is a mixture the faster the object is moving.
with a non-uniform composition. For • An increasing gradient on the distance-time
example, Air which may have different graph represents acceleration while a
gases present in a non-uniform reducing gradient indicates deceleration.
manner.
Q35. Consider the following statements
Q34. A and B ran over a distance starting from the regarding the Nuclear Power Programme of India
same place. The distance-time graph is shown: :
1. Pressurised heavy water reactors have been set
up at Kaiga (Karnataka) and Rawatbhata
(Rajasthan).
2. Variable Energy Cyclotron Centre, Kolkata is one
of the five Research Centres under the
Department of Science and Technology.
3. India has a vast resource of Thorium and the
third stage of Indian Nuclear Power Programme
Consider the following statements: envisages the use of Thorium for power
1. A ran throughout the given period of time with generation.
uniform velocity. Which of these statements is/are correct?
2. A ran throughout the given period of time with (a) 2 only
uniform acceleration. (b) 1 and 2
3. B stopped running after 2 hours. (c) 1 and 3
4. A ran faster than B near the 2-hr stage. (d) 1, 2 and 3
Which of these statements is/are correct? Answer: C
(a) 2 and 3 Explanation:
(b) 1, 3 and 4 The Government has accorded administrative
(c) 4 only approval and financial sanction for construction of
(d) 1, 2, 3 and 4 10 indigenous 700 MW Pressurized Heavy Water
Answer: D Reactors (PHWRs) in fleet mode.
Explanation: • The reactors are planned at Kaiga, Karnataka
Stetement-1 is correct: A ran throughout the (Kaiga-5&6), Gorakhpur, Haryana (GHAVP-
given period of time with uniform velocity as the 3&4), Chutka, Madhya Pradesh (Chutka-1&2)
slope of the graph does not change. and Mahi Banswara, Rajasthan (Units-1 to 4).
Statement-2 is correct: As the velocity of A • The Rajasthan Atomic Power Project (RAPP),
remains constant, it can be said that A ran with a located in Rawatbhata in the north Indian
zero acceleration. state of Rajasthan, currently has six
Statement-3 is correct: B stopped running after 2 pressurised heavy water reactor (PHWR)
hours. After, 2 hours the distance travelled by B is units, operating with a total installed capacity
not increasing though time is increasing which of 1,180MW. Hence Statement 1 is correct.
means B stopped after 2 hours. • Variable Energy Cyclotron Centre, Kolkata is
Statement-4 is correct: A ran faster than B near one of the five Research Centres under the
the 2-hr stage. A was running while B stopped Department of Atomic Energy. Hence
hence this can be said. Statement 2 is incorrect.
• India’s three-stage nuclear power programme
Distance-Time graph was formulated by Homi Bhabha in the 1950s.
If an object moves along a straight line, the o The first two stages, natural uranium-
distance travelled can be represented by a fueled heavy water reactors and
distance-time graph. plutonium-fueled fast breeder
• In a distance-time graph, the gradient of the reactors, are intended to generate
line is equal to the speed of the object. The sufficient fissile material from India’s
limited uranium resources, so that all
8|P a g e W W W . E D U T A P . C O . I N QUERY? HELLO@EDUTAP.CO.IN / 8146207241
its vast thorium reserves can be fully (a) Antigens
utilized in the third stage of thermal (b) Antibiotics
breeder reactors. Hence Statement 3 is (c) Antibodies
correct. (d) Antiseptics
Answer: B
Explanation:
Q36. Carbohydrates are stored in the body as An antibiotic is a chemical substance, produced by
(a) Sugars micro-organisms, which has the capacity to inhibit
(b) Starch the growth of and even to destroy bacteria and
(c) Glucose other micro-organisms.
(d) Glycogen • Alexander Fleming derived the first antibiotic,
Answer: D Penicillin, in 1928 from a fungi called
Explanation: Penicillium.
The carbohydrates act as the primary supply of • Antigens are any substance that causes the
energy in the body. It breaks down into smaller body to make an immune response against
units of sugar such as glucose and fructose. that substance.
• The Glucose is absorbed and used to generate o Antigens include toxins, chemicals,
energy in the body. The excess Glucose is bacteria, viruses, or other substances
stored in the muscles and liver as Glycogen. that come from outside the body.
• Thus, Carbohydrates are stored in the form of • Antibodies are proteins that protect when an
Glycogen in the human body. Hence option D unwanted substance enters the body.
is correct. Produced by one’s immune system, antibodies
bind to these unwanted substances in order to
Q37. Bacteria and viruses cause eliminate them from the system. Another
(a) Allergies word for antibody is immunoglobulin.
(b) Non-communicable diseases • An antiseptic is a substance that stops or
(c) Degenerative diseases slows down the growth of microorganisms.
(d) Infectious diseases They're frequently used in hospitals and other
Answer: D medical settings to reduce the risk of infection
Explanation: during surgery and other procedures.
Infectious diseases are disorders caused by
organisms — such as bacteria, viruses, fungi or Directions: The following Two (8 & 9) items consist
parasites. of two statements, one labelled the 'Assertion (A)
• Allergies occur when the immune system and the other labelled the 'Reason (R)'. You are to
reacts to a foreign substance — such as pollen, examine these two statements carefully and
bee venom or pet dander — or a food that decide if the Assertion (A) and the Reason (R) are
doesn't cause a reaction in most people. individually true and if so, whether the Reason (R)
• A disease in which the function or structure of is a correct explanation of the Assertion (A). Select
the affected tissues or organs changes for the your answers to these items using the Codes given.
worse over time. Osteoarthritis, osteoporosis, below and mark your answer-sheet accordingly
and Alzheimer disease are examples.
• Non-communicable diseases (NCDs) are Codes:
medical conditions or diseases that are not (a) Both A and R are true and R is the correct
caused by infectious agents. These are chronic explanation of A
diseases of long duration, and generally with (b) Both A and R are true but R is NOT the correct
slow progression and are the result of a explanation of A
combination of genetic, physiological, (c) A is true but R is false
environmental and behavioural factors. (d) A is false but R is true

Q38. Substances which are obtained from micro Q39. Assertion (A): A bus and a car (with lower
organisms but used to destroy micro organisms weight than that of the bus) moving at the same
are velocity, are brought to rest on a level road by
9|P a g e W W W . E D U T A P . C O . I N QUERY? HELLO@EDUTAP.CO.IN / 8146207241
applying brakes which exert same retarding force. moves up with the same speed and comes
The car will travel lesser distance before coming to down with the same speed.
rest. • As the train is accelerating, the speed of the
Reason (R): The kinetic energy to be destroyed is person will be more than that of the coin, thus
equal to the work done by the retarding force. the coin is slower and falls behind him.
Answer: A • A force acts on the coin whose magnitude is
Explanation: equal to the product of the mass of the coin
Both Assertion (A) and Reason (R) are correct. and acceleration of the train but it is not the
• When a car and a bus are moving with the reason for the coin falling ahead of him. Hence
same velocity, and weight of the car is less than A is false but R is true.
that of the bus, then the Kinetic Energy (K.E.)
of the bus will be higher than that of the car Q41. Consider the following statements:
as KE (Ek) is given as: 1. Intel's Pentium IV is an operating system used
in PCs.
2. During booting period of a PC, the operating
system is first loaded.
• The KE is to be destroyed to stop the car/bus 3. Analogue computers are useful only in
by the work done by the retarding force as per scientific field while digital computers have
Work-Energy theorem. application in scientific as well as in business,
• When one tries to stop the car/bus, it does not administrative functions.
stop immediately. "Stopping distance" refers 4. Micro-computer is another name for micro-
to the distance the vehicle travels while the processor.
brake is operating. Which of these statements are correct?
• By definition of work (work = force x distance), (a) 1, 2, 3 and 4
the car's kinetic energy is equal to the braking (b) 2 and 3
force multiplied by the stopping distance. The (c) 1 and 4
equation can be written as: (d) 1, 2 and 4
Answer: B
Explanation:
• Here, as the braking force is same, the • Pentium 4 was a series of single-core central
stopping distance depends on the KE of the processing units (CPU) for desktop PCs and
vehicle. As car has lower KE, its stopping laptops. The series was designed by Intel and
distance will be less. launched in November 2000. Therefore
• Hence both Assertion and Reason are correct, statement 1 is incorrect.
and Reason is the correct explanation of the • The operating system is loaded through a
Assertion. bootstrapping process, more succinctly
known as booting. A boot loader is a program
Q40. Assertion (A): A train has come out of the whose task is to load a bigger program, such
platform and is accelerating. If a person sitting in as the operating system. Therefore statement
the train and facing engine, throws up a coin, it will 2 is correct.
fall ahead of him. • An Analog computer stores data continuously
Reason (R): A force acts on the coin whose in the form of physical quantities and perform
magnitude is equal to the product of the mass of calculations with the help of measures. They
the coin and acceleration of the train. are used in the calculation of physical
Answer: D variables such as voltage, pressure,
Explanation: temperature, speed. Whereas, A Digital
When a person sitting in an accelerating train computer is the most commonly used type of
throws the coin in air, it falls behind him. Hence computer and they are extensively used for
Assertion is incorrect. solving complex problems in the field of
• As the coin is tossed, the coin has certain engineering & technology, design, research
speed acquired from the moving train. It thus and data processing. Therefore statement 3 is
correct.
10 | P a g e W W W . E D U T A P . C O . I N QUERY? HELLO@EDUTAP.CO.IN / 8146207241
• Microcomputers are also called as personal So, Town B is the farthest west.
computers and in general parlance, the
Q44. A factory has three belt conveyors fitted in
microprocessor, also known as the Central
its loading station. Conveyors 'A' and 'B'
Processing Unit (CPU), is the brain of all
operating simultaneously lift all the goods in the
computers and many household and
same time during which the goods can be lifted
electronic devices. Therefore statement 4 is
by the conveyor 'C' operating alone. Conveyor 'B'
incorrect.
lifts all the goods five hours faster than conveyor
Hence the correct answer will be option B.
'A' but four hours slower than the conveyor 'C'
The durations of time required to lift all the
Q42. Which positional value divides the series
goods, individually by the conveyors 'A', 'B' and
into equal parts?
'C' are
(a) Median
(b) Second quartile (a) 17, 12, 8 hrs.
(c) 5th Décile (b) 15, 10, 6 hrs.
(d) All the above three (c) 16, 11, 7 hrs.
Answer: D (d) 14, 9, 5 hrs.
Explanation: Answer: B
• The middle value in a sorted, ascending or Explanation:
descending list of numbers is known as the Let conveyor B lifts all the goods in X hours.
median. This is why the median is termed as a Conveyor A lifts all goods in X + 5
positional average of the data set because it Conveyor C lifts all goods in X – 4 hours.
divides the whole data set into two equal
parts, containing an equal number of So, conveyor B takes 1/x time to lift in 1 hour
elements. Conveyor A takes 1/(x+5) hours to lift in 1 hour
• Quartiles are values from the dataset which Conveyor C takes 1/(x-4) hours to lift in 1 hour.
divide the whole data sets into four equal
parts. The first quartile value or Q1 has 25% Now, according to the question, to lift all goods,
elements of the data set below it. Likewise, Q2 Conveyor A + Conveyor B = Conveyor C
has 50% elements below it and it divides the 1/(x+5) + 1/x = 1/(x-4)
data set into exactly two equal parts.
• Decile divides a data set into 10 equal parts, Take LCM on the left side
segregated by 9 positional elements, named
D1, D2, …, and D9. Out of these 9 values, D5 is (x+5+x)/x(x+5) = 1/(x-4)
the middlemost value which divides the 2x+5/x2+5x = 1/(x-4)
whole dataset into two equal parts. (2x+5) *(x-4) = x2 + 5x
• Hence the correct answer is option D. 2x2 - 8x + 5x – 20 = x2 + 5x
x2 - 8x - 20 = 0
x2 - 10x + 2x – 20 = 0
Q43. A town 'P' is located in a particular district. x(x-10) + 2(x-10) = 0
The town 'A' is west of 'P'. Town 'T' is east of 'P'. (x-10) *(x+2) = 0
Town 'K' is east of 'B' but west of 'T' and 'A'. They So, x= 10, -2
are all in the same district. Which town is the But x cannot be negative here.
farthest west? So, X = 10
(a) P
(b) K Hence Conveyor B takes X = 10 hours.
(c) B Conveyor A take X + 5 = 10 + 5 = 15 hours
(d) A Conveyor C takes X - 4 hours = 10 – 4 = 6 hours
Answer: C
Explanation: Q45. According to a certain code
As per the given information
* means 'greater than'

11 | P a g e W W W . E D U T A P . C O . I N QUERY? HELLO@EDUTAP.CO.IN / 8146207241


+ means 'less than' Now we need to find max number of pieces of 1
cm in diameter. So, radius = 1/2 cm, length =
÷ means 'equal to' 20cm
% means 'plus' So, volume of the required block = πr2h =
If A * X, B ÷ Z, Y + C; then the relation. between π*(1/2)2*(20) = 5 π
ABC and XYZ is
(a) ABC + XYZ So, max pieces = 45 π/5 π = 9 pieces.
(b) (ABC % XYZ) * (2XYZ)
(c) (2XYZ) * (ABC % XYZ) Q47. A group of boys decided to buy a few
(d) (ABC/3) + (XYZ/4) cassettes whose price was between Rs.200/-
and Rs.250/-. But at the time of purchase,
Answer: B two of the boys declined to contribute as a
result of which the remaining boys had to
Explanation:
pay Rs. 1/- more than they had originally
As per the given information planned. What was the price of the cassettes
if the boys contributed equally and in the
A > X, B = Z, Y < C whole number of rupees?
From this we got to know that ABC > XYZ (a) Rs. 220/-
So, option B: (ABC % XYZ) * (2XYZ) will be the (b) Rs. 210/-
answer because from this, ABC + XYZ > 2XYZ
(c) Rs. 230/-
That is ABC > XYZ
(d) Rs. 240/-
Hence it is the right answer.
Answer: A
Explanation:
Q46. What is the maximum number of pieces 1
cm in diameter and 20 cm long that can be Let the total number of boys= Y
cut out of a cylindrical wooden block 20 cm
Let the price of cassettes = Rs. C
in length and 3 cm in diameter?
According to question,
(a) 4
{C/(Y-2)} – {C/Y} = 1
(b) 5
(CY – CY + 2C)/ Y(Y-2) = 1
(c) 6
2C = Y2 – 2Y
(d) 9
So, if we put the value of Y = 22. (We also know C
Answer: D
to be between 200 and 250.)
Explanation:
Then 2C = (22)2 – 2(22)
Radius (r) of given cylindrical wooden block = 3/2
2C = 484 – 44
cm (because Diameter = 2*radius)
2C = 440
Length (h) = 20cm
C = 220.
The volume of the block = πr2h = π*(3/2)2*(20) =
45 π So, the cost price of the cassettes is Rs.220

12 | P a g e W W W . E D U T A P . C O . I N QUERY? HELLO@EDUTAP.CO.IN / 8146207241


Directions (for next Five items): Sumeet, Philips, Now, Philips cannot be from Bangalore as given
Wasim, Bishan, and Chetan are five players in (ii).
on the College Cricket team and their
hometowns are Surat, Pune, Warangal, The player who hails from Bangalore is a
Bangalore, and Chandigarh but not in that wicketkeeper. So, the only option left is
order. The five specialist slots of the spinner, Chetan.
pace bowler, wicketkeeper, batsman, and Further, Neither Philips nor Wasim is the captain
captain are held by them, again not in the and they do not belong to either Surat or
order of their names stated above. Bangalore. So, Bishan is the only option left to
(i) Their names, hometowns, and specialties do be from Surat.
not start with the same letter. Now, Philips is from Warangal and Wasim is from
(ii) Neither Philips nor Wasim is the captain and Chandigarh.
they do not belong to either Surat or the batsman does not hail from Warangal.
Bangalore.
(iii) Sumeet is neither a wicketkeeper nor a
batsman.
(iv) Pune is not Bishan's hometown.
(v) The player who hails from Bangalore is a
wicketkeeper
(vi) The captain's hometown is Pune while the This is the final arrangement.
batsman does not hail from Warangal.
So, spinner’s hometown is Warangal
Q48. The spinner's hometown is
(a) Pune
Q49. Chandigarh is the hometown of
(b) Warangal
(a) Philips
(c) Bangalore
(b) Wasim
(d) Chandigarh
(c) Bishan
Answer: B
(d) Sumeet
Explanation:
Answer: B
As per the given information
Explanation
Captain- Neither Wasim nor Philips. Also Chetan
As per the given information
cannot be Captain(because of same starting
letter), Also Bishan doesn’t hail from Pune but Captain- Neither Wasim nor Philips. Also Chetan
Captain does. So, only option left for Captain cannot be Captain(because of same starting
– Sumeet letter), Also Bishan doesn’t hail from Pune but
Captain does. So, only option left for Captain
– Sumeet

13 | P a g e W W W . E D U T A P . C O . I N QUERY? HELLO@EDUTAP.CO.IN / 8146207241


Captain does. So, only option left for Captain
– Sumeet

Now, Philips cannot be from Bangalore as given


in (ii).
The player who hails from Bangalore is a Now, Philips cannot be from Bangalore as given
wicketkeeper. So, the only option left is in (ii).
Chetan.
The player who hails from Bangalore is a
Further, Neither Philips nor Wasim is the captain wicketkeeper. So, the only option left is
and they do not belong to either Surat or Chetan.
Bangalore. So, Bishan is the only option left to
Further, Neither Philips nor Wasim is the captain
be from Surat.
and they do not belong to either Surat or
Now, Philips is from Warangal and Wasim is from Bangalore. So, Bishan is the only option left to
Chandigarh. be from Surat.

the batsman does not hail from Warangal. Now, Philips is from Warangal and Wasim is from
Chandigarh.
the batsman does not hail from Warangal.

This is the final arrangement.


So, Chandigarh is the hometown of Wasim.
This is the final arrangement.
So, Bishan is a pace bowler.
Q50. Who is the pace bowler?
(a) Bishan
Q51. Who is the spinner?
(b) Sumeet
(a) Wasim
(c) Wasim
(b) Bishan
(d) Chetan
(c) Chetan
Answer: A
(d) Philips
Explanation:
Answer: D
As per the given information
Explanation:
Captain- Neither Wasim nor Philips. Also Chetan
cannot be Captain(because of same starting As per the given information
letter), Also Bishan doesn’t hail from Pune but
Captain- Neither Wasim nor Philips. Also Chetan
cannot be Captain(because of same starting
14 | P a g e W W W . E D U T A P . C O . I N QUERY? HELLO@EDUTAP.CO.IN / 8146207241
letter), Also Bishan doesn’t hail from Pune but Captain- Neither Wasim nor Philips. Also Chetan
Captain does. So, only option left for Captain cannot be Captain(because of same starting
– Sumeet letter), Also Bishan doesn’t hail from Pune but
Captain does. So, only option left for Captain
– Sumeet

Now, Philips cannot be from Bangalore as given


in (ii).
Now, Philips cannot be from Bangalore as given
The player who hails from Bangalore is a in (ii).
wicketkeeper. So, the only option left is
Chetan. The player who hails from Bangalore is a
wicketkeeper. So, the only option left is
Further, Neither Philips nor Wasim is the captain Chetan.
and they do not belong to either Surat or
Bangalore. So, Bishan is the only option left to Further, Neither Philips nor Wasim is the captain
be from Surat. and they do not belong to either Surat or
Bangalore. So, Bishan is the only option left to
Now, Philips is from Warangal and Wasim is from be from Surat.
Chandigarh.
Now, Philips is from Warangal and Wasim is from
the batsman does not hail from Warangal. Chandigarh.
the batsman does not hail from Warangal.

This is the final arrangement.


So, Philips is a spinner. This is the final arrangement.
So, Chetan’s hometown is Bangalore.
Q52. Chetan's hometown is
(a) Bangalore Q53. A and B stand in a circular ring with 10
other persons. If the arrangement of 12
(b) Warangal
persons is randomly done, the chance, that
(c) Surat there are exactly 3 persons between A and B
is
(d) Pune
(a) 3/12
Answer: A
(b) 2/11
Explanation:
(c) 2/13
As per the given information
(d) 3/11
15 | P a g e W W W . E D U T A P . C O . I N QUERY? HELLO@EDUTAP.CO.IN / 8146207241
Answer: B 15*10 + 30*10 + ……………….+ nth term = 15*280
Explanation: 10(15 + 30 + ………….nth term) = 15*280
There are total 12 persons A + B + 10 others. So, this makes an AP.
So, total number of ways to be arranged around Sum= (n/2) *{2a + (n-1)d}
a circle = (12 -1)! = 11! Ways
Where n is the nth term, a= first element, d is the
Now, exactly 3 persons are to be arranged in difference
between A and B. So, 10P3
15 * 280 = 10[(n/2) *{2*15 +(n-1)15}]
Now, A and B can interchange their positions = 2!
Ways 15*28 = (n/2) *(15 + 15n)

Also, the remaining 7 can also interchange their 30*28 = 15n + 15n2
positions with each other. So, 7! Ways n2 +n = 56
So, total ways in arranging all with the condition of So, n = 7, -8 (but n cannot be negative)
exactly 3 in between A and B = 2! * 10P3 * 7! Ways
= 2*(10! /7!) * 7! = 2*10! Ways So, n=7
It means 7*10 = 70 days

So, the chance(Probability) = 2*10! / 11! So, work will be done in 70 days.

= 2/11
Q55. Bag 'A' contains 5 white and 2 black balls.
Bag 'B' contains 2 white and 3 black balls. If
Q54. 15 men can complete a work in 280 days. any one bag is chosen and a ball is taken out
They started the work and after every 10 of it at random, what is the probability that
days, 15 additional men were employed. In the ball is black?
how many days the work was completed?
(a) 31/70
(a) 70 days
(b) 1/2
(b) 60 days
(c) 5/12
(c) 55 days
(d) 3/5
(d) 50 days
Answer: B
Answer: A
Explanation:
Explanation:
Bag A = 5White, 2 Black balls
Initial Men = 15.
Bag B = 2 White, 3 Black balls.
These 15 men can finish the work in 280 days.
Probability of selecting a bag = 1/2
Total Work = 15 * 280 men-days = 4200 men-
days Now, the Probability of picking a black ball = from
bag A or from bag B = 2C1/5C1 + 3C1/5C1
They work for 10 days. So, work done = 15*10
men-days. = 2/5 + 3/5

Now 15 more men were added, and they worked =1


for 10 days= 30*10 men days
So, required probability = (1/2) *1
Similarly for the next days
= 1/2
16 | P a g e W W W . E D U T A P . C O . I N QUERY? HELLO@EDUTAP.CO.IN / 8146207241
The ratio of milk to water in the 4th container =
314x/35: 106x/35
Q56. Three containers have their volumes in the
ratio 3: 4: 5. They are full of mixtures of milk =157: 53
and water. The mixtures contain milk and
water in the ratio of 4:1, 3:1 and 5:2
respectively. The contents of all these three Q57. Two dice are thrown simultaneously. The
containers are poured into a fourth probability that the product of the two
container. The ratio of milk and water in the numbers on the two dice is an even number
fourth container is is
(a) 5:2 (a) 1/2
(b) 157: 53 (b) 3/4
(c) 151: 48 (c) 5/16
(d) 4:1 (d) 3/8
Answer: B Answer: B
Explanation: Explanation:
Let the volumes be 3x,4x, and 5x respectively. Total number of outcome is 6*6 = 36.
Container with volume 3x: Now, according to the question we need the
Milk = (4/ 4+1) *3x = 12x/5 product of numbers on two dice to be even.

Water = (1/ 4+1) *3x = 3x/5 We know “odd* even = even” as well as
“even*even = even”

Container with volume 4x:


But odd*odd = odd. So, the possibility of the odd
Milk = (3/3+1) *4x = 12x/4 number on the first dice is 3 (1, 3, 5) and
similarly for 2nd dice.
Water = (1/ 3+1) *4x = 4x/4
So, the number of outcomes when the product
will be odd = 3*3 = 9 ways.
Container with volume 5x:
Milk = (5/5+2) *5x = 25x/7 So, required probability = 36 – 9/36
Water = (2/5+2) *5x = 10x/7 = 27/36
= 3/4
Total milk = 12x/5 + 12x/4 + 25x/7 Q58. The flowers in a basket become double
= 314x/35 every minute and the basket gets full in one
hour. In how much time, the basket was 1/32
full?
Total water = 3x/5 + 4x/4 + 10x/7 (a) 12 minutes
= 106x/35 (b) 32 minutes
(c) 45 minutes
(d) 55 minutes

17 | P a g e W W W . E D U T A P . C O . I N QUERY? HELLO@EDUTAP.CO.IN / 8146207241


Answer: D (c) 25
Explanation: (d) 11
As per the given information, Answer: B
The flower gets double every minute. Explanation:
Basket in 60 minutes gets full There are total 36 persons.
So, 9 persons take Tea but not coffee (it means only
tea)
Basket in 59 minutes will be ½ full
Total persons who take tea (only Tea + both tea
Basket in 58 minutes will be ¼ full and coffee) = 16.
Basket in 57 minutes will be 1/8 full So, persons who take both coffee and tea = 16 –
Basket in 56 minutes will be 1/16 full 9 = 7.

Basket in 55 minutes will be 1/32 full. So, persons who take coffee but not tea (only
coffee) = total – only tea – both tea and coffee
So, this is the final answer 55 minutes. =36 - 9 – 7 = 20

Q59. If X is mortal, Y is wise. If Z is not good, A Q61. How many lines other than those shown in
will suffer. Y is not wise if A suffers. If B is a the figure are required to join each corner
slave, X is immortal. If A suffers, what with another?
follows?
(a) X is mortal
(b) B is a slave
(c) Z is good
(d) Y is wise
Answer: B
(a) 56
Explanation:
(b) 48
If A suffers, it means Y is not wise. (given). Also, If
A suffers, it means Z is not good (given) (c) 20
If Y is not wise, So, X is not mortal or immortal. (d) 32
If X is immortal, So B is a slave (given) Answer: C
So, B is slave follows. Explanation:
The number of diagonals in a polygon of n sides is
given by n(n-3)/2
Q60. In a group of 36 persons, a total of 16 take
tea while 9 take tea but not coffee. How Hence in an octagon (number of sides = 8) is given
many persons in this group take coffee but by 8(8–3)/2 = 20.
not tea?
(a) 27
Q62. Two pipes 'A' and 'B' can fill a tank in 15
(b) 20 minutes and 20 minutes respectively. Both

18 | P a g e W W W . E D U T A P . C O . I N QUERY? HELLO@EDUTAP.CO.IN / 8146207241


the pipes are opened together but after 4 (c) red, black, brown and white
minutes, pipe 'A' is turned off. What is the
total time required to fill the tank? (d) red, brown, blue and white

(a) 12 mt 30 sec Answer: C

(b) 14 mt 40 sec Explanation:

(c) 10 mt 20 sec
(d) 11 mt 45 sec
Answer: B
Explanation:
Amount of water filled by pipe A in one minute =
1/15
Amount of water filled by pipe B in one minute =
1/20
So, both pipes were opened for 4 minutes. ABEF = Red
So, the amount of water filled by both pipes in 4 GHDC = Black
minutes = 4* (1/15 + 1/20)
ABCD = Green
= 7/15
EFGH = Blue
So, remaining amount = 8/15
AFGD = White
Now, pipe A is turned off, So, this will be filled by
pipe B. let’s say in x minutes BCHE = Brown

So, x*(1/20) = 8/15 So, Red, black, white and brown are adjacent to
Green.
X = 32/3 minutes it means 10 minutes 40 seconds

Q64. The face opposite to the brown face is


So, total time taken = 4 minutes + 10 minutes 40
seconds (a) red

= 14 minutes 40 seconds (b) white


(c) green

Directions (For the next Two items): A cube has (d) blue
six faces, each of a different colour. The red Answer: B
face is opposite to black. The green face is
between red and black. The blue face is Explanation:
adjacent to the white. The brown face is
adjacent to the blue. The red face is the
bottommost face.
Q63. The four colours adjacent to green are
(a) red, black, blue and white
(b) red, black, brown and blue

19 | P a g e W W W . E D U T A P . C O . I N QUERY? HELLO@EDUTAP.CO.IN / 8146207241


- Prime Minister can recommend
dissolution of the Lok Sabha to President at
any time. Thus, statement 2 is correct.
• The Constitution lays down a special
procedure for the passing of money bills in the
Parliament.
- A money bill can only be introduced in the
Lok Sabha and that too on the
recommendation of the President. Thus,
statement 3 is correct.
• The Rajya Sabha (first constituted in 1952) is a
continuing chamber, that is, it is a permanent
ABEF = Red body and not subject to dissolution.
- However, one-third of its members retire
GHDC = Black every second year.
ABCD = Green - Their seats are filled up by fresh elections
and presidential nominations at the
EFGH = Blue beginning of every third year. Hence
Statement 4 is not correct.
AFGD = White
BCHE = Brown Q66. Consider the following statements:
1. For the three All-India Services, there are 24
So, White is opposite to brown face. State cadres including three joint cadres.
2. Union Public Service Commission is the cadre
Q65. Which of the following statements is not controlling authority for Indian Administrative
correct? Service.
(a) The 42nd Amendment extended the term of 3. Staff Selection Commission is entrusted with
Lok Sabha and State Legislative Assemblies the task of making recruitment to all the Group
from 5 years to 6 years 'B' and Group 'C' posts under the Government.
(b) The President can dissolve the Lok Sabha on Which of these statements is/are correct?
the advice of the Prime Minister (a) 1 only
(c) Money Bills cannot be introduced in the Rajya (b) 1 and 2
Sabha (c) 2 and 3
(d) One third members of the Rajya Sabha retire (d) 1, 2 and 3
on the expiry of every third year Answer: None of the above, as Options given in
Answer: D the Question above are as per 2002 information
Explanation: which has been modified over the period of time.
• 42nd Constitutional Amendment of 1976 Explanation:
raised the tenure of Lok Sabha and state • After being selected for the IAS, candidates are
legislative assemblies from 5 to 6 years. allocated to "Cadres." There is one cadre for
However, the 44th Amendment of 1978 each Indian State that is 21 Cadres in all,
restored the original term of the Lok Sabha and except for three joint cadres: Assam-
the state legislative assemblies to 5 years. Meghalaya, Manipur-Tripura, and Arunachal
Thus, Statement 1 is correct. Pradesh - Goa – Mizoram - Union Territories
• The normal term of Lok Sabha is five years (AGMUT). Thus, statement 1 is not correct.
from the date of its first meeting after the • The Constitution visualises the UPSC to be the
general elections, after which it automatically ‘watch-dog of merit system’ in India.
dissolves. - However UPSC is not concerned with the
- However, the President is authorised to classification of services, pay and service
dissolve the Lok Sabha at any time even conditions, cadre management, training,
before the completion of five years. and so on. Thus, statement 2 is not
correct.
20 | P a g e W W W . E D U T A P . C O . I N QUERY? HELLO@EDUTAP.CO.IN / 8146207241
• Staff Selection Commission is entrusted to sources (such as dividend income, profits,
make recruitment to interest receipts). Hence Statement 2 is
- All Group “B” posts in the various correct.
Ministries/Departments of the Govt. of
India and their Attached and Subordinate Q68. The Comptroller and Auditor General of
Offices which are in the pay scales the India is appointed by the
maximum of which is Rs.10,500 or below (a) Prime Minister
and (b) President
- All non-technical Group “C” posts in the (c) Finance Minister
various Ministries/Departments of the (d) Finance Minister
Govt. of India and their Attached and Answer:
Subordinate Offices, except those posts Explanation:
which are specifically exempt from the • The CAG is appointed by the President of India
purview of the Staff Selection Commission. by a warrant under his hand and seal. The CAG,
- Hence NOT ALL jobs in Group B and C. before taking over his office, makes and
Thus, statement 3 is not correct. subscribes before the president an oath or
affirmation.
Q67. Consider the following statements:
1. Part 'B' of the Budget speech of the finance Q69. Assertion (A): Point of order is raised to
minister includes description of the initiatives criticize a policy of the government.
on economic front to be taken up by the Reason (R): It is an important tool of legislative
Government for the coming year in addition to control.
direct and indirect tax proposals. Codes:
2. The receipts on account of the partial (a) Both A and R are true and R is the correct
disinvestment of Central Government's explanation of A
holdings in the equity capital of public sector (b) Both A and R are true but R is NOT the correct
enterprises are revenue receipts. explanation of A
Which of these statements is/are correct? (c) A is true but R is false
(a) 1 only (d) A is false but R is true
(b) 2 only Answer: D
(c) Both 1 and 2 Explanation:
(d) Neither 1 nor 2 • A point of order is an extraordinary process
Answer: A which, when raised, has the effect of
Explanation: suspending the proceedings before the House
• The budget is a statement of the estimated and the member who is on her/his legs gives
receipts and expenditure of the Government way.
of India in a financial year. - This is meant to assist the Speaker in
- In addition to the estimates of receipts and enforcing the Rules, Directions and
expenditure, the budget contains certain provisions of the Constitution for
other elements. regulating the business of the House.
- Part B of the Budget Speech contains Hence statement 1 is not correct.
details of economic and financial policy of • It should essentially refer to procedure and
the coming year, that is, taxation relate to the business before the House at the
proposals, prospects of revenue, spending moment and is considered to be an important
programme and introduction of new tool of legislative control. Hence statement 2
schemes/projects. Hence Statement 1 is is correct.
correct.
• Revenue Receipts are receipts which do not Q70. Who of the following is the Chairman of
have a direct impact on the assets and Planning Commission in India?
liabilities of the government. It consists of the (a) Finance Minister of India
money earned by the government through tax (b) Prime Minister of India
(such as excise duty, income tax) and non-tax (c) President of India
21 | P a g e W W W . E D U T A P . C O . I N QUERY? HELLO@EDUTAP.CO.IN / 8146207241
(d) Home Minister of India (d) Nicolo Conti
Answer: B Answer: D
Explanation: Explanation:
Prime Minister of India was the ex – officio Nicolo De Conti was Italian merchant, scholar, and
chairman of the Planning Commission. a traveller who visited the Vijayanagar Kingdom
during the time of Dev Raya II.
Q71. Who among the following became the • He left an account in the Travels of Nicolo
President of India without having been the Vice – Conti.
President? Hence Option D is the correct answer.
(a) Shri V. V. Giri
(b) Shri Venkataraman Q74. Who among the following was a great
(c) Shri N. Sanjeeva Reddy protagonist of the 'Doctrine of Lapse'?
(d) Shri Zakir Hussain (a) Lord Dalhousie
Answer: C (b) Lord Minto
Explanation: (c) Lord Curzon
Neelam Sanjeeva Reddy was an Indian politician (d) Lord Wellesley
who served as the sixth president of India, serving Answer: A
from 1977 to 1982. Explanation:
• Beginning a long political career with the The Doctrine of Lapse was an annexation policy
Indian National Congress Party in the followed widely by Lord Dalhousie when he was
independence movement, he went on to hold India’s Governor-General from 1848 to 1856.
several key offices in independent India—as • It was used as an administrative policy for the
the first Chief Minister of Andhra Pradesh, a extension of British Paramountcy.
two-time Speaker of the Lok Sabha and a Hence option A is the correct Answer.
Union Minister—before becoming the Indian
President. Q75. Chauri Chaura is known in the history of
India's struggle for freedom because
Q72. Nalanda University flourished during the (a) Gandhiji launched his first Satyagraha in India
reign of which of the following rulers? from this place
(a) Chandragupta Maurya (b) Gandhiji withdrew his Satyagraha movement
(b) Ashoka when an irate mob set fire to a police station
(c) Kanishka and burnt some policemen
(d) Harsha (c) Hundreds of freedom fighters were shot dead
Answer: D at this place by the police
Explanation: (d) Gandhiji sat on a hunger strike
• Nalanda was an acclaimed Mahavihara, a large Answer: B
Buddhist monastery in the ancient kingdom of Explanation:
Magadha in India. • Chauri Chaura is a town in Gorakhpur district
• The site is located about ninety five kilometres of Uttar Pradesh.
southeast of Patna near the town of Bihar • On 4th February, 1922, this town witnessed a
Sharif and was a centre of learning from the violent incident - a large crowd of peasants set
fifth century CE to c. 1200 CE. on fire a police station that killed 22
• Nalanda flourished under the patronage of the policemen.
Gupta Empire in the 5th and 6th centuries, and • Due to this incident Mahatma Gandhi called
later under Harsha, the emperor of Kannauj. off the Non-Cooperation Movement (1920-
Hence option D is the correct answer. 22). Hence option B is the correct answer.

Q73. The foreign traveller who visited India Q76. The 'Grand Trunk Road' connects
during Vijayanagar period was? (a) Kolkata and Mumbai
(a) Megasthenes (b) Delhi and Chennai
(b) Yuan Chwang (c) Kolkata and Amritsar
(c) Fa Hien (d) Tirupati and Ludhiana
22 | P a g e W W W . E D U T A P . C O . I N QUERY? HELLO@EDUTAP.CO.IN / 8146207241
Answer: C who inaugurated the age of enlightenment and
Explanation: liberal reformist modernisation in India.
This route started from Kabul in Afghanistan to
Chittagong in Bangladesh. It covered Khyber Q80. Match List-I. (Classical Dance Form) with
Bypass and connected cities like Rawalpindi, List-II (State) and select the correct answer using
Amritsar, Attari, Delhi, Mathura, Varanasi, Patna, the codes given below the lists:
Kolkata in India, Dhaka and Chittagong. LIST I (Classical Dance LIST II (State)
Form)
Q77. The Civil Disobedience Movement of 1930 A. Bharat Natyam 1. Odisha
culminated in the signing of a pact in 1931 by B. Kathakali 2. Tamil Nadu
Gandhiji and C. Kuchipudi 3. Manipur
(a) Lord Irwin 4. Andhra Pradesh
(b) Lord Linlithgow 5. Kerala
(c) Lord Wavell Code:
(d) Lord Curzon A B C
Answer: A (a) 4 3 1
Explanation: (b) 2 5 4
(c) 4 5 1
• Gandhi-Irwin Pact, was an agreement signed (d) 2 3 4
on March 5, 1931, between Mohandas K. Answer: B
Gandhi, leader of the Indian nationalist • Bharatanatyam is a major form of Indian
movement, and Lord Irwin, British viceroy classical dance that originated in Tamil Nadu.
(1926–31) of India. • Kathakali is native to the Malayalam-speaking
• It marked the end of a period of civil southwestern region of Kerala.
disobedience (satyagraha) in India against • Kuchipudi originates from a village named
British rule that Gandhi and his followers had Kuchipudi in the state of Andhra Pradesh.
initiated with the Salt March (March–April • Hence, option B is the correct answer.
1930).
Q81. Consider the following statements :
Q78. Dhanvantari was 1. Bermuda situated in Western Atlantic Ocean
(a) a famous General of Chandragupta Maurya is a United Kingdom Overseas Territory.
(b) a noted physician in the Court of Chandragupta 2. 38th Parallel divided boundary between North
Vikramaditya and South Vietnam before unification.
(c) a famous dramatist who lived during the time 3. Bahamas situated near Florida is one of the
of Harsha states of USA.
(d) a musician in the Court of Ashoka Which of these statements is/are correct?
Answer: B (a) 1 only
Explanation: (b) 2 and 3
Dhanvantari was a well-known physician during (c) 1 and 3
reign of Chandragupta Vikramaditya and was one (d) 1 and 2
of Chandragupta Vikramaditya's nine diamonds. Answer: A
• Bermuda is a British Overseas Territory in the
Q79. Which leader is called the ‘father' of India's Western Atlantic Ocean. Hence, statement 1
social reform movement? is correct.
(a) Mahatma Gandhi • 38th North Parallel divided the North and
(b) Raja Rammohun Roy South Korea from 1945 till 1950. Hence,
(c) G.K. Gokhale statement 2 is incorrect.
(d) Lokmanya Tilak
• The Bahamas, officially the Commonwealth of
Answer: B
The Bahamas, is a country within the Lucayan
Explanation:
Archipelago of the West Indies in the Atlantic.
Raja Ram Mohan Roy was the father of Modern
Hence, statement 3 is incorrect.
India's Renaissance and a tireless social reformer
23 | P a g e W W W . E D U T A P . C O . I N QUERY? HELLO@EDUTAP.CO.IN / 8146207241
• Longitude of a place is indicative of time of the
Q82. Consider the following statements : place.
1. Falkland Islands are situated in Pacific Ocean.
2. Red Sea separates Sudan from Egypt. Q85. Consider the following statements :
3. England, Norway, Denmark surround North Sea. 1. Alluvial soil is rich in chemical properties and
Which of these statements is/are correct? is capable of yielding Rabi and Kharif crops.
(a) 2 only 2. Black soil is suitable for cotton, groundnut.
(b) 1 and 3 3. Rabi crops are reaped in autumn after sowing
(c) 2 and 3 in June.
(d) 3 only Which of these statements are correct?
Answer: D (a) 1, 2 and 3
• The Falkland Islands, or the Malvinas Islands, (b) 1 and 2
is an archipelago in the South Atlantic Ocean. (c) 2 and 3
Hence, statement 1 is incorrect. (d) 1 and 3
• Red Sea separated the African continent with Answer: B
the Arabian Peninsula. Egypt and Sudan share • Alluvial soil is rich in chemical properties and
a land border. Hence, statement 2 is is capable of yielding Rabi and Kharif crops.
incorrect. Hence, statement 1 is correct.
• The North Sea is a sea of the Atlantic Ocean • Black soil is suitable for cotton & groundnut.
between Great Britain, Norway, Denmark, Hence, statement 2 is correct.
Germany, the Netherlands, Belgium and • Rabi crops are reaped in spring after sowing in
France. post monsoon season in month of
October/November. Hence, statement 3 is
Q83. Which is the correct sequence of the given incorrect.
American cities from West to East?
(a) Salt Lake City-Pittsburgh-San Francisco-Boston Q86. Consider the following statements : While it
(b) San Francisco-Salt Lake City-Pittsburgh-Boston is 6:30 AM (Sunday) in London, it is around
(c) Salt Lake City-San Francisco-Boston-Pittsburgh 1. 2.30 AM (Sunday) in Honolulu (Hawaii)
(d) San Francisco-Pittsburgh-Salt Lake City-Boston 2. 8.30 PM (Saturday) in Honolulu (Hawaii)
Answer: B 3. 3.30 PM (Sunday) in Tokyo
4. 1:30 AM (Sunday) in Tokyo
Which of these statements is/are correct?
(a) 4 only
(b) 2 and 3
(c) 1 and 4
(d) 2 and 4
Answer: B

Q84. Latitude of a place is indicative of its


(a) time
(b) altitude
(c) amount of rainfall
(d) temperature
Answer: D
• Latitude of a place is indicative of the
temperature of the place. Hence, option D is
the correct answer.
24 | P a g e W W W . E D U T A P . C O . I N QUERY? HELLO@EDUTAP.CO.IN / 8146207241
• Tokyo, Japan is 8 hours ahead of London, UK. 2. Amongst the States and Union Territories,
So, the time in Tokyo would be around 2.30 Delhi has the highest population density while
PM. Sikkim has the lowest.
• London, UK is 11 hours ahead of Honolulu, 3. The correct sequence of population in
Hawaii, USA. So, the time in Honolulu would descending order of Rajasthan, Madhya
be around 7:30 PM. Hence, option B is the Pradesh, Tamil Nadu and Karnataka is Tamil
correct answer. Nadu-Madhya Pradesh-Rajasthan-Karnataka.
Which of these statements is/are correct?
Q87. Consider the following statements: (a) 1 and 2
1. Prime Meridian passes through Greenwich (b) 2 and 3
and is at 0° Longitude. (c) 1, 2 and 3
2. International Date Line, on the map appears (d) 3 only
as a straight line along 180° Longitude. Answer: D
3. While crossing International Date Line from • During 1991 – 2001, literacy rate of males
east to west, one gains a day. increased by 11.72 percentage points while
4. Tropic of Cancer lies at 23½° Latitude S. female literacy rate increased by 14.87.
Which of these statements are correct? Hence, statement 1 is incorrect.
(a) 1, 2, 3 and 4 • In 2001 Census, Delhi had the highest
(b) 1 and 2 population density while Arunachal Pradesh
(c) 1 and 3 had the lowest. Hence, statement 2 is
(d) 2 and 3 incorrect.
Answer: C • Correct order of population is Tamil Nadu
• Prime Meridian is a meridian (a line of (62.1 million) – Madhya Pradesh (60.3 miliion)
longitude) in a geographic coordinate system – Rajasthan (56.4 million) – Karnataka (52.7
at which longitude is defined to be 0°. It is million). Hence, statement 3 is correct.
derived, but differs slightly, from the
Greenwich Meridian, the previous standard. Q89. Match List-I (Park/Sanctuary) with List-II
Hence, statement 1 is correct. (State) and select the correct answer using the
• International Date Line (IDL) is an codes given below the lists:
internationally accepted demarcation on the List-I (Park/Sanctuary) List-II (State)
surface of Earth, running between the South A. Keoladeo National 1. Assam
Pole and North Pole and serving as the Park
boundary between one calendar day and the B. Corbett National Park 2. Kerala
next. It passes through the Pacific Ocean, C. Kaziranga Sanctuary 3. Uttar Pradesh
roughly following the 180° line of longitude D. Periyar Sanctuary 4. Rajasthan
and deviating to pass around some territories 5. Uttaranchal
and island groups. It is not a straight line. Code:
Hence, statement 2 is incorrect. A B C D
• While crossing International Date Line from (a) 2 5 4 1
east to west, one gains a day. While crossing (b) 4 3 1 2
it towards the east one loses a day. Hence, (c) 2 3 4 1
statement 3 is correct. (d) 4 5 1 2
• Tropic of Cancer lies at 23½° Latitude N. Answer: D
Hence, statement 4 is correct. • Keoladeo National Park is a famous avifauna
sanctuary in Bharatpur, Rajasthan. Hence, A is
Q88. Consider the following statements : matched with 4.
According to Census 2001 • Corbett National Park is in Uttarakhand.
1. The literacy rate increase for males is more Hence, B is matched with 5.
than that of the females during last one • Kaziranga NP is located in Assam. Hence, C is
decade. matched with 1.

25 | P a g e W W W . E D U T A P . C O . I N QUERY? HELLO@EDUTAP.CO.IN / 8146207241


• Periyar Wildlife Sanctuary is located in Kerala. • Ujjain is located along the banks of Shipra.
Hence, D is matched with 2. Hence, C is matched with 4.
• Vijayawada is located along Krishna river.
Q90. Match List-I (Multi-purpose river valley Hence, D is matched with 1.
projects) with List-II (State) and select the correct
answer using the codes given below the lists: Q92. Recently, Cannes Film Festival was held in
List-I (Multi-purpose List-II (State) (a) Switzerland
river valley projects) (b) France
A. Purna Project 1. Gujarat (c) Italy
B. Bhima Project 2. Maharashtra (d) England
C. Kakrapara Project 3. Uttaranchal Answer: B
D. Tehri Dam Project 4. Uttar Pradesh Explanation
5. Karnataka • The Cannes Festival until 2003 called the
Code: International Film Festival and known in
A B C D English as the Cannes Film Festival, is an
(a) 1 2 5 3 annual film festival held in Cannes,
(b) 2 5 1 3 France.
(c) 1 3 5 4
(d) 2 4 1 3 Q93. Shri Kiran Karnik is the
Answer: B (a) President of NASSCOM
• Purna river project is located in the State of (b) Chairman of the Board of Control of
Maharashtra. Hence, A is matched with 2. Cricket for India
• Bhima project is located in Karnataka. Hence, (c) Chairman of Maharashtra State Public
B is matched with 5. Service Commission
• Kakrapara project is located in Gujarat. Hence, (d) Managing Director of Engineers India
C is matched with 1. Ltd.
• Tehri Dam project is located in Uttarakhand. Answer: A
Hence, D is matched with 3. Explanation
• Kiran Karnik was serving as the President
Q91. Match List-I (City) with List-II (River Passing of NASSCOM in 2001.
through the city) and select the correct answer
using the codes given below the lists: Q94. Knesset is the name of the Parliament of
List-I (City) List-II (River passing (a) Norway
through the city) (b) Sweden
A. Nasik 1. Krishna (c) Israel
B. Surat 2. Cauvery (d) Spain
Answer: C
C. Ujjain 3. Godavari
Explanation
D. Vijayawada 4. Shipra
• Knesset is the name of the Parliament
5. Tapti
of Israel.
Code:
A B C D
Q.95 Match List-I with List-II and select the
(a) 3 5 4 1
correct answer using the code given below the
(b) 4 1 3 2
lists:
(c) 3 1 4 2
List-I
(d) 4 5 3 1
A. Dennis Tito
Answer: A
B. Colin Powell
• Nasik is located along the banks of river
C. Jacques Chirac
Godavari. Hence, A is matched with 3.
D. Donald H. Rumsfeld
• Surat is located along banks of Tapti river.
List-II
Hence, B is matched with 5.
1. President of France
2. Defence Secretary, USA
26 | P a g e W W W . E D U T A P . C O . I N QUERY? HELLO@EDUTAP.CO.IN / 8146207241
3. Secretary of State, USA Q.97 Match List-I (Person) with List-II
4. First space tourist (Organization) and select the correct answer
Code: using the code given below the lists:
A B C D List-I (Person)
(a) 1 3 4 2 A. Rajendra S. Pawar
(b) 4 2 1 3 B. Ashok Soota
(c) 1 2 4 3 C. R.S. Lodha
(d) 4 3 1 2 D. R.K. Pachauri
Answer: D List-II (Organization)
Person Associated 1. CII
With 2. TERI
Dennis Tito First Space 3. NIIT
Tourist 4. FICCI
Colin Powell Secretary of Code:
State USA A B C D
Jacques Chirac President of (a) 4 2 3 1
France (b) 3 1 4 2
Donald H. Rumsfeld Defence (c) 4 1 3 2
Secretary, (d) 3 2 4 1
USA Answer: D
Explanation
Q.96 Match List-I (Football World Cup/Year) with Person Organization
List-II (Host Country) and select the correct Rajendra S Pawar NIIT
answer using the code given below the lists: Ashok Soota CII
List-I (Football World Cup/Year) R.S.Lodha FICCI
A. 1986 R.K.Pachauri TERI
B. 1990
C. 1994 Q.98 Raksha Kavach' is
D. 1998 (a) a new safety equipment designed for the
List-II (Host Country) soldiers of Indian Army
1. Italy (b) an anti-collision device to prevent
2. Mexico collision between trains designed by the
3. France Konkan Railway Corporation
4. USA (c) a new design for earthquake resistant
5. Spain buildings
Code: (d) a new device for the Indian airports for
A B C D the safe landing of aircrafts
(a) 5 3 2 1 Answer: B
(b) 2 1 4 3 Explanation
(c) 5 1 2 3 • Raksha Kavach is an anti-collision device to
(d) 2 3 4 1 prevent collision between trains designed
Answer: B by the Konkan Railway Corporation.
Explanation
Football World Cup Host Country Q.99 Consider the following statements in
1986 Mexico respect of Soccer World Cup :
1990 Italy 1. Prior to FIFA-2002 Cup, Brazil had won
1994 USA the World Cup four times.
1998 France 2. Indomitable Lions' is the nickname for
2022 Doha, Qatar Cameroon team.
3. 'Azzurri' is the nickname for the Italian
team.

27 | P a g e W W W . E D U T A P . C O . I N QUERY? HELLO@EDUTAP.CO.IN / 8146207241


4. Senegal, Slovenia and Ecuador • Stockholm syndrome is a coping
participating in FIFA-2002 are the only mechanism to a captive or abusive
teams participating in the World Cup for situation.
the first time. • This condition applies to situations
Which of these statements are correct? including child abuse, coach-athlete
(a) 1, 2 and 3 abuse, relationship abuse and sex
(b) 2 and 3 trafficking.
(c) 1, 3 and 4
(d) 1, 2 and 4 Q.102 Match List-I (Football World
Answer: A Cup/Year) with List-II (Host Country) and select
Explanation the correct answer using the code given below
• China, Ecuador, Senegal, and Slovenia the lists:
made their World Cup debuts in FIFA 2002. List-I (Prominent Football Player)
• FIFA World Cup 2022 will take place in A. Michael Owen
Qatar. B. Luis Figo
• La'ebb is the mascot of the Qatar 2022. C. Gabriel Batistuta
• Adidas 2022 World Cup ball is called "Al D. Raul Gonzalez
Rihla." List-II (Country)
1. Italy
2. Spain
Q.100 Recently, Government disinvested stake in 3. Portugal
VSNL, IBP and IPCL to 4. Argentina
(a) Bharti Telesonic, Reliance and GAIL, 5. England
respectively 6. Brazil
(b) Tata Group of Industries, Indian Oil Code:
Corporation and Reliance, respectively A B C D
(c) Bharti Telesonic, Royal Dutch Shell and (a) 5 2 4 3
Pfizer, respectively (b) 6 3 1 2
(d) Motorala, Kuwait Petroleum and (c) 5 3 4 2
Ranbaxy, respectively (d) 6 2 1 3
Answer: B Answer: C
Explanation Explanation
• Government has disinvested its Stake in Player Country
VSNL, IBP and IPCL to Tata Group of Michael Owen England
Industries, Indian Oil Corporation and Luis Figo Portugal
Reliance, respectively Gabriel Batistuta Argentina
Raul Gonzalez Spain
Q.101 Stockholm Syndrome is a mental condition
experienced by
(a) the young persons who take up jobs in Q.103 Match List-I (City) with List-II
the industry after passing out from the (Establishment) and select the correct answer
Management Schools using the codes given below the lists:
(b) persons who migrate from rural areas to List-I (City)
urban cities, during the earlier stages of A. Arkonnam
their stay B. Allahabad
(c) drug-addicts in the rehabilitation centres C. Kamptee
(d) hostages and kidnap victims in which D. Mhow
positive feelings develop towards their List-II (Establishment)
captors 1. Central Air Command HQ
Answer: D 2. Naval Air Base
Explanation 3. Institute of Military Law
4. Coast Guard District HQ
28 | P a g e W W W . E D U T A P . C O . I N QUERY? HELLO@EDUTAP.CO.IN / 8146207241
5. College of Combat Answer: B
Code: Explanation
A B C D • Crop Yield Formation unit works under
(a) 2 5 3 1 the Department of Agriculture and Co
(b) 3 1 4 5 operation.
(c) 2 1 3 5
(d) 3 5 4 1 Q.106 East Timor, the latest independent nation
Answer: C in the world was ruled by
Explanation (a) Indonesia
City Establishments (b) Australia
Arkonnam Naval Air Base (c) Malaysia
Allahabad Central Air (d) Philippines
Command Hq Answer: A
Kamptee Institute of Explanation
Military Law • East Timor declared itself independent
Mhow College of from Portugal in 1975, but was invaded by
Combat Indonesia.
• It became the first new sovereign state of
the twenty-first century on 20 May 2002.
Q.104 Match List-I (Library) with List-II (City) and
select the correct answer Q.107 Which of the following pair is not correctly
List-I (Library) matched ?
A. National Library (a) National Aluminium Company Ltd. :
B. State Central Library Bhubaneswar
C. Connemara Public Library (b) Indian Bureau of Mines : Dhanbad
List-II (City) (c) National Film Archives of India : Pune
1. New Delhi (d) National Archives of India : New Delhi
2. Kolkata Answer: B
3. Mumbai Explanation
4. Chennai Indian Bureau of Mines is headquartered at
Code: Nagpur.
A B C
(a) 1 3 4 Q.108 Sagar Samrat' is the name of
(b) 2 3 4 (a) an island near Port Blair
(c) 1 4 3 (b) a drilling ship in Bombay High
(d) 2 4 3 (c) an island near Maldives
Answer: B (d) name of a 5-star hotel in Goa
Explanation Answer: B
Library City Explanation
National Library Kolkata • Sagar Samrat is an oil drilling platform in
State Central Library Mumbai Mumbai high, which is 176 kilometres
Connemara Public Library Chennai away from the coastline of Mumbai.

Q.105 Crop Yield Formulation Unit which has Q.109 Which tourist destination from amongst
developed statistical models to forecast crop the following will be developed as an
yield works under the control of international destination for tourism according
(a) Ministry of Water Resources to the Budget 2002-03?
(b) Department of Agriculture and Co (a) Port Blair
operation (b) Hampi
(c) India Meteorological Department (c) Pushkar
(d) Department of Food and Public (d) Nalanda
Distribution Answer: B
29 | P a g e W W W . E D U T A P . C O . I N QUERY? HELLO@EDUTAP.CO.IN / 8146207241
Explanation Which of these statements is/are correct?
• According to union budget 2002-03, (a) 1 only
Hampi will be developed as an (b) 2 and 3
international destination for tourism. (c) 1 and 2
(d) 1, 2 and 3
Q.110 Shri Rajender Singh won the Ramon Answer: B
Magsaysay Award for Explanation:
(a) Emergent leadership • The extent of responsiveness of demand with
(b) Community leadership change in the price is not always the same.
(c) Illiteracy eradication The demand for a product can be elastic or
(d) Bio-technology inelastic, depending on the rate of change in
Answer: B the demand with respect to change in price of
Explanation a product. Elastic demand is the one when the
• Rajendra Singh is the winner of Stockholm response of demand is greater with a small
Water Prize' 2015. proportionate change in the price. On the
• He has been honoured by Asia's most other hand, inelastic demand is the one when
prestigious Ramon Magsaysay Award' there is relatively a less change in the demand
2001, for Community Leadership. with a greater change in the price.
• Therefore, the demand for commodities
Q.111 Which among the following became the having a large number of close substitutes is
first woman Prime Minister in the world? more elastic than the commodities having a
(a) Indira Gandhi smaller number of substitutes because
(b) Kim Campbell whenever there is a change in prices of
(c) Margaret Thatcher commodities having close substitutes there
(d) Sirimavo Bandaranaike will be drastic change in the demand of that
Answer: D commodity. Hence, statement 1 is incorrect.
Explanation • Luxury items Luxury items have a highly elastic
• Sirimavo Bandaranaike was the first demand. Since these items are not necessity
woman Prime Minister of the World. therefore, change in prices of these items lead
• She was the Prime Minister of Sri Lanka to drastic change in the demands of these
(then the Dominion of Ceylon) in 1960. items. Hence, statement 2 is correct.
• The commodities whose consumption cannot
Q.112 Who among the following wrote the book easily be postponed have less elastic demand.
Algebra of Infinite Justice? Since these commodities belong to the
(a) Anita Desai necessity class of goods e.g., flour, salt, sugar
(b) Vikram Seth etc. Therefore, irrespective of change in prices
(c) Arundhati Roy of these goods, demand of these commodities
(d) Rohington Mistry does not get affected too much. Hence,
Answer: C statement 3 is correct.
Explanation • Therefore, B is the right answer.
• Arundati Roy has written the book
“Algebra of Infinite Justice” Q114. Consider the following statements:
1. Launching of Fourth five-year plan was
postponed and three annual plans had to be
Q113. Consider the following statements: resorted to between 1966 to 1969.
1. The demand for commodities having a large 2. Fifth five-year plan was terminated a year
number of close substitutes is less elastic than earlier.
the commodities having a smaller number of 3. Eighth five-year plan commenced in 1992
substitutes instead of 1990.
2. Luxury items have a highly elastic demand Which of these statements is/are correct?
3. The commodities whose consumption cannot (a) 2 and 3
easily be postponed have less elastic demand (b) 1 and 2
30 | P a g e W W W . E D U T A P . C O . I N QUERY? HELLO@EDUTAP.CO.IN / 8146207241
(c) 1 and 3 or aggregate, economy. Hence, statement (A)
(d) 1, 2 and 3 is correct.
Answer: D • The central issues in Macroeconomics relate
Explanation: to the overall level of employment, growth
• The government had to declare "Plan rate of national output, general price level and
Holidays" (1966-67, 1967-68, and 1968-69). stability of the economy. Hence, statement
The Sino-Indian War and the Indo-Pak War, (R) is correct.
which caused the Third Five Year Plan to fail, • Though both the statements are correct but R
were the primary causes of the plan is not the correct explanation of A. Therefore,
holidays. Therefore, the Fourth five-year B is the right answer.
plan was postponed. Hence, statement 1 is
correct. Q116. Directions: The following items consist of
two statements, one labelled the 'Assertion (A)
• The new approach was “Rolling Plan”. It
and the other labelled the 'Reason (R)'. You are to
terminated the fifth five-year plan in
examine these two statements carefully and
1977-78 and launched its own sixth five-year
decide if the Assertion (A) and the Reason (R) are
plan for period 1978-83 and called
individually true and if so, whether the Reason (R)
it rolling plan. The Janata
is a correct explanation of the Assertion (A).
Party government rejected the Fifth Five-Year
Select your answers to these items using the
Plan and introduced a new Sixth Five-Year
Codes given. below and mark your answer-sheet
Plan (1978–1980). The plan was
accordingly
terminated in 1978, its fourth year, due to
change in the Government at the Centre. Assertion (A): Amongst the three strategies
Hence, statement 2 is correct. overall cost leadership, differentiation and market
• The Eight Five Year Plan was not introduced segmentation; a company with differentiation
in 1990 and the following years 1990-91 and strategy has less competition from both its direct
1991-92 were treated as Annual Plans. This competitors and from potential substitutes.
was largely because of the economic Reason (R): Its consumers have greater brand
instability. India faced a crisis of foreign loyalty, and, therefore, less price sensitivity.
exchange reserves during this time. Codes:
Therefore, Eighth five-year plan commenced (a) Both A and R are true and R is the correct
in 1992 instead of 1990. Hence, statement 3 explanation of A
is correct. (b) Both A and R are true but R is NOT the correct
• Therefore, D is the right answer. explanation of A
(c) A is true but R is false
Q115. Assertion (A): Macro-economic deals with (d) A is false but R
the study of aggregates and averages. Answer: A
Reason (R): Central problem of macro-economics Explanation:
is the determination of income and employment. • A market differentiation strategy is an
Codes: approach business develop by providing
(a) Both A and R are true and R is the correct customers with something unique, different
explanation of A and distinct from item their competitor. The
(b) Both A and R are true but R is NOT the correct main objective of implementing a
explanation of A differentiation strategy is to increase
(c) A is true but R is false competitive advantage.
(d) A is false but R • Product differentiation is a marketing strategy
Answer: B designed to distinguish a company's products
Explanation: or services from the competition. Successful
• Macroeconomics is the branch of economics product differentiation involves identifying
that deals with the structure, performance, and communicating the unique qualities of a
behavior, and decision-making of the whole, product or company while highlighting the

31 | P a g e W W W . E D U T A P . C O . I N QUERY? HELLO@EDUTAP.CO.IN / 8146207241


distinct differences between that product or • This concept affects organization design in
company and its competitors. a variety of ways, including speed of
• Product differentiation goes hand in hand communication flow, employee
with developing a strong value proposition so motivation, reporting relationships, and
that a product or service is attractive to a administrative overhead. Span of
target market or audience. management has been part of the
• If successful, product differentiation can historical discussion regarding the most
create a competitive advantage for the appropriate design and structure of
product's seller and ultimately build brand organizations.
awareness. Examples of differentiated • A small, or narrow, span of control results
products might include the fastest high-speed in each manager supervising a small
Internet service or the most gas-efficient number of employees, while a wide span of
electric vehicle on the market. Therefore, management occurs when more
assertion Is true. subordinates report directly to a given
• A differentiated product can increase brand manager.
loyalty and even survive a higher price point. If • A small span of management would make
a product is perceived to be better in some it necessary to have more managers and
way than its competitors, consumers will more layers of management to oversee the
consider it worth the higher price. Therefore, same number of operative employees than
the Reason is also correct. would be necessary for an organization
• Hence the correct option will be Option A. using a wider span of management. The
narrower span of management would
Q117. Directions: The following items consist of result in more layers of management and
two statements, one labelled the 'Assertion (A) slower communications between lower-
and the other labelled the 'Reason (R)'. You are to level employees and top level managers of
examine these two statements carefully and the firm. Recent moves to downsize
decide if the Assertion (A) and the Reason (R) are organizations and to eliminate
individually true and if so, whether the Reason (R) unnecessary positions has resulted in
is a correct explanation of the Assertion (A). many organizations moving to wider
Select your answers to these items using the spans of management and the elimination
Codes given. below and mark your answer-sheet of layers of middle-level managers.
accordingly • The trend in recent years has been to
move toward wider spans of control to
Assertion (A): In the organizational structure of reduce costs, speed decision making,
the modern companies, the trend is towards wider increase flexibility and empower
span of control. employees. Therefore, assertion is
Reason (R): Narrow span of control discourages correct.
autonomy. • A narrow span of control also discourages
Codes: autonomy among employees since every
(a) Both A and R are true and R is the correct employee is closely guided by a supervisor
explanation of A since every supervisor has very few
(b) Both A and R are true but R is NOT the correct subordinates to supervise. Therefore,
explanation of A reason is also correct.
(c) A is true but R is false • Hence the correct answer will be option A.
(d) A is false but R
Answer: A Q118. Directions: The following items consist of
Explanation: two statements, one labelled the 'Assertion (A)
• Span of control or span of management is and the other labelled the 'Reason (R)'. You are to
a dimension of organizational design examine these two statements carefully and
measured by the number of subordinates decide if the Assertion (A) and the Reason (R) are
that report directly to a given manager. individually true and if so, whether the Reason (R)
is a correct explanation of the Assertion (A).
32 | P a g e W W W . E D U T A P . C O . I N QUERY? HELLO@EDUTAP.CO.IN / 8146207241
Select your answers to these items using the employee is bound to visit the production
Codes given. below and mark your answer-sheet manager and that is a grave violation of
accordingly unity of command. Therefore, assertion is
correct.
Assertion (A): Unity of command cannot always
• Moreover, globalization has forced the
be strictly applied in practice.
Indian companies to create multiple
Reason (R): Workers should report to different business divisions, and therefore workers
supervisors for different aspects or tasks technical, should report to different supervisors for
financial, administrative etc. different aspects or tasks technical,
Codes: financial, administrative etc. and that will
(a) Both A and R are true and R is the correct lead to a higher specialization in the work
explanation of A profile of different employees. Therefore,
(b) Both A and R are true but R is NOT the correct reason is also correct.
explanation of A • Hence the correct answer will be option A.
(c) A is true but R is false
(d) A is false but R Q119. Directions: The following items consist of
Answer: A two statements, one labelled the 'Assertion (A)
Explanation: and the other labelled the 'Reason (R)'. You are to
• Unity of command: An employee should examine these two statements carefully and
receive orders from only one superior as decide if the Assertion (A) and the Reason (R) are
shown in the figure below. individually true and if so, whether the Reason (R)
is a correct explanation of the Assertion (A).
Select your answers to these items using the
Codes given. below and mark your answer-sheet
accordingly
Assertion (A): Enterprise Resource Planning (ERP)
is useful in a manufacturing company. It does not
find application in service industry.
Reason (R): Enterprise Resource Planning (ERP)
enables a high level of interaction and
coordination along the supply chain.
Codes:
(a) Both A and R are true and R is the correct
• This principle is against the Taylor’s
explanation of A
functional foremanship where multiple
(b) Both A and R are true but R is NOT the correct
superiors oversee the subordinate. As per
explanation of A
Fayol if there is no unity of command it will
(c) A is true but R is false
lead to conflict as subordinate will never
(d) A is false but R
know whom to obey. Adoption of this
Answer: D
principle leads to clarity about work
Explanation:
whereas violation will lead to confusion
• Enterprise resource planning (ERP) is a
about what to do and what not to do.
platform companies use to manage and
• However, Unity of command cannot
integrate the essential parts of their
always be strictly applied in practice,
businesses. Many ERP software
because no manager has all the required
applications are critical to companies
skills. For example, a marketing manager
because they help them implement
will not have the knowledge regarding the
resource planning by integrating all the
work done in the production department.
processes needed to run their companies
Therefore, if an employee of marketing
with a single system.
employee has a doubt regarding
• ERP applications also allow the different
production department, in that case, the
departments to communicate and share
33 | P a g e W W W . E D U T A P . C O . I N QUERY? HELLO@EDUTAP.CO.IN / 8146207241
information more easily with the rest of • Fishing and tourism are the key industries of
the company. It collects information about United Arab Emirates. Hence statement 3 is
the activity and state of different divisions, correct.
making this information available to other • Hence option B is the correct answer.
parts, where it can be used productively.
• An ERP software system can also integrate
planning, purchasing inventory, sales,
marketing, finance, human resources, and
more.
• ERP software can be used by variety of
companies, ERP software has expanded to
include nearly all types of businesses.
Typically, each company will have its own
specific reasons to implement an ERP
solution. Therefore, assertion is incorrect.
• he components of an ERP system depend
on the organization's needs. However,
there are key features that each ERP
should include. Generally, packages
include finance, human resource, logistics
and manufacturing, supply chain
management, and customer relationship
management and ERP also streamlines
various components of the Supply chain,
therefore the reason is correct.
• Hence the correct answer will be option D.

Q120. Consider the following statements


concerning United Arab Emirates:
1. Oman is one of the seven Emirates which form
the United Arab Emirates.
2. Dubai is the capital of United Arab Emirates.
3. Fishing and tourism are the key industries of
United Arab Emirates.
Which of these statements is/are correct?
(a) 1 and 2
(b) 3 only
(c) 2 and 3
(d) 1, 2 and 3
Answer: B
Explanation:
• The United Arab Emirates is an elective
monarchy formed from a federation of seven
emirates.
• UAE’s seven emirates are Abu Dhabi, Dubai,
Sharjah, Ajman, Umm Al Quwain, Ras Al
Khaimah and Fujairah. Hence statement 1 is
incorrect.
• Abu Dhabi is the capital city of Dubai. Hence
statement 2 is incorrect.

34 | P a g e W W W . E D U T A P . C O . I N QUERY? HELLO@EDUTAP.CO.IN / 8146207241


Visit: www.civilstap.com
For Any Query Mail us: hello@civilstap.com or call us at - (+91)-8146207241
0
the word or group of words you consider most
UPSC EPFO EO/AO - 2021 appropriate for the blank space and indicate
PART- A your response on the Answer Sheet accordingly.
Directions : Each item in this section consists of a
6. It’s time you went out and ______ your own
sentence with an underlined word followed by
living.
your words. Select the option that is opposite in
(a) afforded
meaning to the underlined word and mark your
(b) earned
response in your Answer Sheet accordingly.
(c) gained
(d) won
1. She always organises lavish parties on
birthdays.
(a) frugal
7. The elephant fell into a _______ the hunters
(b) decent
had set.
(c) cheap
(a) track
(d) strange
(b) trap
(c) trick
(d) trunk
2. The soldier was greatly praised for his valour.
(a) clumsiness
(b) cowardice
8. The dress isn’t really tight. It’ll ______when
(c) selfishness
you’ve worn it.
(d) tactlessness
(a) bend
(b) expand
(c) squeeze
3. These regions are very fertile.
(d) stretch
(a) sterile
(b) arid
(c) barren
9. With his back to the old man, he stood at the
(d) productive
window, fists clenched, and his shoulders ______
with his choked sobbing.
(a) shivered
4. Some sick immigrants were detained at the
(b) shook
airport.
(c) vibrated
(a) settlers
(d) moved
(b) natives
(c) vagabonds
(d) foreigners
10. The bereaved mother proudly received the
gallantry award ______ her brave son who had
laid down his life in the recent war.
5. The questions were so elementary that anyone
(a) in the name of
could pass the test.
(b) in lieu of
(a) unusual
(c) on behalf of
(b) primary
(d) for the sake of
(c) naive
(d) advanced
Spotting Errors
Directions : Each item in this section has a
Fill in the blanks
sentence with three underlined parts labelled
Directions : Each of the following sentences in
(a), (b) and (c). Read each sentence to find out
this section has a blank space and four words or
whether there is any error in any underlined
group of words given after the sentence. Select
part and indicate your response in the Answer
1|P a g e W W W . E D U T A P . C O . I N QUERY? HELLO@EDUTAP.CO.IN / 8146207241
Sheet against the corresponding letter i.e., (a) or Directions : Each item in this section consists of a
(b) or (c). If you find no error, your response sentence with an underlined word followed by
should be indicated as (d). four words. Select the option that is nearest in
meaning to the underlined word and mark your
11. Neither the servants/ nor the clerk/ has response in your Answer Sheet accordingly.
done this./ No error
16. Today the newspaper is full of carnage and
(a) Neither the servants violence.
(b) nor the clerk (a) bloodshed
(c) has done this. (b) lust
(d) No error (c) concern
(d) satire

12. The car which went past us/ when we were


driving on the highway/ must have been doing 17. This religious gift, that makes men human, is
at least a hundred miles an hour./ No error still alive in Indian souls.
(a) mystical
(a) The car which went past us (b) spiritual
(b) when we were driving on the highway (c) metaphysical
(c) must have been doing at least a hundred miles (d) philosophical
an hour.
(d) No error
18. It is difficult for us to imagine how much this
safety means to everyone.
13. Raju will be back home/ in an year/ after his (a) feel
stay abroad./ No error (b) understand
(c) think
(a) Raju will be back home (d) see
(b) in an year
(c) after his stay abroad.
(d) No error 19. Public service is not merely something that
occupies the hours you are doing it, but invades
all your life and experience and affects them in
14. I have/ much work/ to do./ No error. one way or the other.
(a) preserves
(a) I have (b) demolishes
(b) much work (c) covers
(c) to do. (d) spoils
(d) No error.

20. He was in the early part of the twentieth


15. Psychology did not develop into a science/ century, a staunch nationalist of the liberal fold.
based of careful observation/ and (a) profuse
experimentation until the late 1800s./ No error. (b) dogmatic
(c) plentiful
(a) Psychology did not develop into a science (b) tolerant
(b) based of careful observation
(c) and experimentation until the late 1800s. Part - B
(d) No error.
Q.21) How many times does the digit 3 appear
between 1 and 100 such that the number where
Synonyms 3 appears is not divisible by 3?
2|P a g e W W W . E D U T A P . C O . I N QUERY? HELLO@EDUTAP.CO.IN / 8146207241
(a) 11 (b) 2 only
(b) 12 (c) Both 1 and 2
(c) 13 (d) Neither 1 nor 2
(d) 17
Q27. Which one of the following is the remainder
Q22. Which one of the following is the Average of when 74100 is divided by 9?
all prime numbers between 21 & 55? (a) 2
(a) 35.85 (b) 5
(b) 36.71 (c) 3
(c) 38.00 (d) 7
(d) 39.00
Q28. Which one of the following is the arithmetic
Q23. If the ratio of Speed of ‘A’ & ‘B’ is 5:6 & ‘B’ mean of √3 - √2 and its reciprocal?
allows ‘A’ a starts of 70 m in a 1.2km race, who (a) √3
will win the race and by what distance? (b) √2
(a) ‘A’ wins by 30 m (c) 2
(b) ‘B’ wins by 200 m (d) 1
(c) ‘B’ wins by 130 m
(d) The race finishes in a dead heat Q29. Suppose x and y are two positive numbers
such that when x is reduced by 2 and y is
Q24. ‘M’ is 60 year old, ‘R’ is 5 years junior to ‘M’ increased by 2, the ratio becomes 2:1; and when
& 4 years senior to ‘V’. The youngest brother of x is increased by 2 and y is reduced by 2, the ratio
‘V’ is ‘B’ & he is 6 years junior to ‘V’. What is the becomes 3:1. Which one of the following is equal
age difference between ‘M’ & ‘B’ ? to x-y?
(a) 18 years (a) 20
(b) 15 years (b) 24
(c) 13 years (c) 18
(d) 11 years (d) 22

Q25. English alphabet is recorded in the following Q30. Let n(>1) be a composite natural number
manner: whose square root is not an integer. Consider the
following statements:
The first 6 letters are written in opposite order, 1. n has a factor that is greater than 1 but less
the next 6 letters are written in opposite order than the square root of n.
and so on. At the end, ‘Y’ is interchanged with ‘Z’. 2. n has a factor that is greater than the square
Which one of the following is the fourth letter to root of n but less than n.
the right of the 13th letter? Which of the statements given above is/are
correct?
(a) N (a) 1 only
(b) O (b) 2 only
(c) M (c) Both 1 and 2
(d) I (d) Neither 1 nor 2

Q26. If a varies as b then which of the following Q31. The average age of husband and wife was 23
statements is /are correct? years when they were married 5 years ago. The
1. nth root of a²b varies as (2n)th root of a4b². average age of a husband, his wife and child is 20
𝑎
2. 𝑏2 varies inversely as b years now. How old is the child now?
(a) 9 months
Select the correct answer using the code given (b) 1 years
below: (c) 3 years
(d) 4 years
(a) 1 only
3|P a g e W W W . E D U T A P . C O . I N QUERY? HELLO@EDUTAP.CO.IN / 8146207241
Q32. If W₁ and W₂ are weights of the two solid (b) ROM
iron ball of radii ½ metre and ⅓ metre (c) Flash Memory
respectively, then which one of the following is (d) Cache Memory
equal to W₁: W₂?
(a) 8: 27 Q38. Which one of the following is not a web
(b) 27: 8 browser?
(c) 4: 16 (a) Internet Explorer
(d) 16: 4 (b) Firefox
(c) Fedora
(d) Google Chrome
Q33. 40% of the students in a class are from India
and 50% are girls. If 25% of Indian students are Q39. Which one of the following represents 1 GB
girls, what percentage of non-Indian students are of information?
boys. (a) 1024 KB
(a) 33.33% (b) 1024 MB
(b) 40% (c) 1024 TB
(c) 25% (d) 1024 PB
(d) 20%
Q40. Which one of the following is not a
language
Q34. The average weight of a hundred students
translator?
in a class is 46 kg. The average weight of boys and
(a) Assembler
girls is 50 kg and 40 kg respectively. What is the
(b) Linker
difference between the number of boys and girls?
(c) Interpreter
(a) 30 (d) Compiler
(b) 25
(c) 20 Q41. Which one of the following statements is
(d) 10 correct?
A device driver of output devices
Q35. A hollow cube tube of side 10 cm each (a) Interprets input provided by users into
weighing 200 gm, is made. This cube is placed in computer usable form.
water with a horizontal face. How many cm of its (b) Interprets computer output into user
height sink in water if the density of water is 1 understandable form.
gm/cm³? (c) Translates user inputs into output device.
(d) Facilitates user to communicate with output
(a) 1 cm
device.
(b) 1.5 cm
(c) 2 cm
Q42. Which one of the following registers is used
(d) 2.5 cm
to keep track of the next instruction to be
executed?
Q36. Which one of the following basic operations
(a) Memory address register
raw input data into for converting useful
(b) Memory buffer register
information is not performed by all computers?
(c) Program counter
(a) Inputting
(d) Memory data register
(b) Storing
(c) Switching
Q43. Which one of the following is not an audio
(d) Outputting
file
format?
Q37. Which one of the following memories is
(a) MIDI
extremely fast and acts as a high-speed buffer
(b) WAV
between the CPU and the main memory?
(c) SWF
(a) RAM
(d) MPEG
4|P a g e W W W . E D U T A P . C O . I N QUERY? HELLO@EDUTAP.CO.IN / 8146207241
Q44. Which one of the following denotes a Q51. Which one of the following does `Nirguna
sequential electronic circuit that is used to store Bhakti' refer to?
1-bit of information? (a) Shaivite form of worship
(a) Register (b) Vaishnavite form of worship
(b) Transistor (c) Incarnation worship
(c) Flip-flop (d) Abstract form of God worship
(d) Capacitor
Q52. What did Ain-i-Akbari seek to promote
Q45. Where did Gandhiji put in his first major within the frontiers of the Mughal State?
public appearance after returning from South (a) Social disharmony influenced by the whims of
Africa in 1915? the nobility
(a) Bombay University (b) Cessation of the provincial units from central
(b) Kheda rule
(c) Champaran (c) Social harmony with the support of a strong
(d) Banaras Hindu University ruling class
(d) Establishment of a rule of rural republics by
Q46. Who among the following is the author of overthrowing the Mughal suzerainty
`Gandhi as Mahatma’?
(a) Mahadev Desai Q53. Gandhiji's Salt March to Dandi was started
(b) Shahid Amin from which one of the following places?
(c) Louis Fischer (a) Kochrab Ashram
(d) David Arnold (b) Sabarmati Ashram
(c) Ahmedabad Textile Mill
Q47. Who among the following was the President (d) Ahmedabad Jail
of the Indian National Congress at the time of
independence of India? Q54. Who among the following was not a
(a) Pattabhi Sitaramayya member of the Socialist group in the Congress
(b) Jawaharlal Nehru Party?
(c) Abul Kalam Azad (a) Jayaprakash Narayan
(d) J.B. Kripalani (b) Rajendra Prasad
(c) N.G. Ranga
Q48. During 1931, under whose leadership, did a (d) Narendra Dev
strong Kisan Sabha Movement develop in the
Gaya District of Bihar? Q55. Which one of the following was not
(a) Yadunandan Sharma recommended in Macaulay's 1835 Minute on
(b) Sahajanand Education?
(c) Sheetla Sahai (a) Teaching in English medium
(d) Tilka Manjhi (b) 'Liberal' literary training
(c) Teaching of Western texts alone at higher
Q49. Who produced the Pirpur Report (1938)? levels of education
(a) Muslim League (d) Teaching of both Western and Oriental texts at
(b) Unionist Party higher levels of education
(c) Ahrar Party
(d) Indian National Congress Q56. Who among the following did not represent
the militant nationalist school of thought in
Q50. Who among the following was the founder India?
of Swatantra Party? (a) Ashwini Kumar Dutt
(a) C. Rajagopalachari (b) Vishnushastri Chiplunkar
(b) Deen Dayal Upadhyaya (c) Krishna Kumar Mitra
(c) Shyama Prasad Mukherjee (d) Lala Lajpat Rai
(d) Acharya Narendra Dev
5|P a g e W W W . E D U T A P . C O . I N QUERY? HELLO@EDUTAP.CO.IN / 8146207241
Q57. Which one of the following statements (b) It scrutinizes appropriation and finance
about the situation in the Ryotwari areas is accounts of the Government.
correct? (c) It examines reports of the Comptroller and
(a) A large amount of land passed into the hands Auditor General and whether public
of the money lenders, merchants, and rich undertakings are run efficiently.
peasants, who usually utilised the services of (d) It examines bills on matters of general public
the tenants. interest.
(b) Landless labourers became the landowners.
(c) It stopped the practice of leasing out land to Q63. Which one of the following does not figure
tenants at high prices. in the list of languages in the Eighth Schedule of
(d) Landlordism was totally destroyed. the Constitution of India?
(a) Dogri
Q58. In the year 1911, who among the following (b) Bhoti
formed the Social Service League in Bombay? (c) Maithili
(a) G.K. Gokhale (d) Santhali
(b) S.A. Dange
(c) N.M. Joshi Q64. Who among the following Speakers of Lok
(d) M.R. Jayakar Sabha formally disassociated from the political
party after his election as the Speaker?
Q59. Who among the following was not part of (a) G.V. Mavalankar
the group of 'no-changers' in the Congress Party? (b) Sardar Hukam Singh
(a) Sardar Vallabhbhai Patel (c) Neelam Sanjiva Reddy
(b) Dr. Ansari (d) P.A. Sangma
(c) Babu Rajendra Prasad
(d) Motilal Nehru Q65. Which one of the following is not a female
reproductive organ in humans?
Q60. Who among the following was not a (a) Ovaries
member of the Cabinet Mission? (b) Oviducts
(a) Pethick-Lawrence (c) Cervix
(b) Stafford Cripps (d) Stamen
(c) A.V. Alexander
(d) Lord Wavell Q66. Which one of the following cannot be called
“Amphibian of the Plant Kingdom”?
Q61. Which one of the following facts pertaining (a) Spirogyra
to the National Green Tribunal (NGT) is not (b) Riccia
correct? (c) Funaria
(a) The NGT was set up in the year 2010. (d) Marchantia
(b) Its purpose is to ensure effective and
expeditious disposal of cases relating to Q67. Which one of the following is not a
environmental protection and conservation of consequence of deforestation?
forests and other natural resources. (a) Increased groundwater table
(c) It is bound by the procedure laid down under (b) Decreased biodiversity
the Code of Civil Procedure. (c) Increased soil erosion
(d) It is guided by the principles of natural justice. (d) Decreased rainfall

Q62. Which one of the following is the mandate Q68. Which one of the following chromosomes
of the Committee on Estimates? has a mismatched pair in a normal human male?
(a) It reports on what economies, improvements (a) Chromosome number 21
in organization, efficiency, or administrative (b) Chromosome number 18
reform consistent with the policy underlying (c) X-Chromosome
the estimates, may be effected. (d) Chromosome number 13

6|P a g e W W W . E D U T A P . C O . I N QUERY? HELLO@EDUTAP.CO.IN / 8146207241


Q69. The word ‘Vaccine’ has been derived from a (d) NaCl
Latin word having meaning
(a) Antibody Q76. Which one of the following, on adding to
(b) Immunity water, will not scatter a beam of light?
(c) Cow (a) Copper sulphate
(d) Guinea pig (b) Chalk powder
(c) Milk
Q70. Which one of the following statements is (d) Ink
true with regard to the image formation by two
eyes of a person? Q77. Which one of the following will not be
(a) Both the eyes see exactly the same image. reduced by metallic Zinc?
(b) One eye sees half portion of the object (a) Cu2+
(c) Both the eyes combine the two images seen by (b) H+
them (c) Ag+
(d) Each eye sees a slightly different image (d) Al3+

Q71. Which one of the following statements is Q78. Who among the following discovered
not correct? Proton?
Wavelength of microwaves ranges between (a) J.J. Thomson
(a) Infrared waves and radio waves. (b) E. Goldstein
(b) Visible waves and radio waves. (c) E. Rutherford
(c) Gamma-rays and X-rays (d) J. Chadwick
(d) X-rays and visible waves
Q79. Consider the following balanced equation:
Q72. Which one of the following is the size of CO(g) + 2H2(g) → CH3OH(l)
hydrogen atom? How many moles of CH3OH(l) can be obtained by
(a) 10-10 m reacting 2.0 mole of CO(g) with 2.0 mole of H2(g)
(b) 10 micro metres (a) 1
(c) 10 millimetres (b) 2
(d) 1000 Angstrom (c) 3
(d) 4
Q73. Which one of the following statements
regarding force is correct? Q80. Match List I with List II and select the correct.
(a) A positive force implies attractive nature. answer using the code given below the lists:
(b) A negative force implies repulsive nature. List I List II
(c) A positive force can be both attractive and (Term) (Explanation)
repulsive in nature. A. Closure 1. Permanent closing down
(d) A negative force implies attractive nature. of a place of employment
B. Workmen 2. Any person employed in
Q74. When a dielectric material is kept in an any industry to do skilled,
external electric field, which one of the following unskilled or manual work
phenomena may be realized? C. Strike 3. Temporary closing down
(a) Magnetization of workplace by the
(b) Polarization management
(c) Photoionization D. Lockout 4. Cessation of work by
(d) Circularization employees
Code:
Q75. Which one of the following substances, A B C D
when added to water, will not change the pH? (a) 1 2 4 3
(a) NaHCO3 (b) 3 4 2 1
(b) NH4Cl (c) 1 4 2 3
(c) Na2CO3 (d) 3 2 4 1
7|P a g e W W W . E D U T A P . C O . I N QUERY? HELLO@EDUTAP.CO.IN / 8146207241
(d) 48 hours
Q81. Who is an 'Adolescent' as per the Factories
Act, 1948? Q86. What is the maximum period in which the
(a) Who has completed 15 years of age but is less appropriate government shall review and revise
than 18 years the minimum rates of wages under the Minimum
(b) Who is less than 18 years Wages Act, 1948?
(c) Who has completed 14 years of age but is less (a) 2 years
than 18 years (b) 3 years
(d) Who has completed 16 years of age but is less (c) 4 years
than 18 years (d) 5 years

Q82. What is a controlled industry? Q87. In which one of the following places, was
(a) Any industry the control of which by the Union Asia’s first Export Processing Zone (EPZ) set up in
has been declared by any Central 1965?
Act to be expedient in the public interest (a) Ahmedabad
(b) Any industry the control of which by the State (b) Kandla
has been declared by any State Act to be (c) Mumbai
expedient in the public interest (d) Jaipur
(c) Any industry the control of which by the
Municipal bodies has been declared by any Q88. Which one of the following statements
Municipal Rules to be expedient in the public about Trial Balance is correct?
interest (a) It is a book containing different accounts of an
(d) Any industry the control of which by the State entity.
has been declared by any Central Act to be (b) It is a statement containing balances of debtors
expedient in the public interest of an entity.
(c) It is a statement containing balances of debtors
Q83. What is the minimum number of members and creditors of an entity.
required for registration of a Trade Union? (d) It is a statement containing the various ledger
(a) 2 members balances of an entity on a particular date
(b) 3 members
(c) 7 members Q89. Wages paid for installation of machinery is
(d) 10 members debited to which one of the following accounts?
(a) Wages Account
Q84. Which of the following disputes is/are (b) Machinery Account
considered as trade dispute(s) under the (c) Installation Account
provision of the Trade Union Act, 1926? (d) Profit and Loss Account
Any dispute of any person connected with
1.Employment Q90. When are current liabilities payable?
2.Non-Employment (a) Within a year
3.Conditions of Labour (b) After one year but within five years
Select the correct answer using the code given (c) Within five years
below : (d) Subject to a contingency
(a) 1 only
(b) 2 and 3 only Q91. Is the total of Debit and Credit side of Trial
(c) 1, 2 and 3 Balance the same?
(d) 1 and 3 only (a) No, there are some times good reason why
they differ.
Q85. What is the maximum number of hours in a (b) Yes, always.
week that an adult worker is allowed to work for? (c) Yes, except where the Trial Balance is extracted
(a) 35 hours at the year end.
(b) 40 hours (d) No, because it is not a Balance Sheet.
(c) 45 hours
8|P a g e W W W . E D U T A P . C O . I N QUERY? HELLO@EDUTAP.CO.IN / 8146207241
Q92. Which one of the following is the first book (b) 10,00,000
in which the transactions of a business unit are (c) 15,00,000
recorded? (d) 20,00,000
(a) Balance Sheet
(b) Cash Book Q99. According to the Economic Survey 2020 -
(c) Ledger 21," which one of the following will be India’s real
(d) Journal GDP growth in the year 2021 - 22?
(a) 9%
Q93. Which one of the following denotes Gross (b) 11 %
Profit? (c) 13%
(a) Cost of goods sold + Opening stock (d) 15%
(b) Sales less Cost of goods sold
Q100. Which one of the following is the acronym
(c) Sales less Purchases
for the word 'NITI' in NITI Aayog?
(d) Net profit less Expenses of the period
(a) National Integration and Transformation of
Q94. What is the underlying accounting concept India
that supports no anticipation of profits but (b) National Institution for Transforming India
provision for all possible losses? (c) National India Institution for Technological
(a) Matching (d) National Institution for Trust in India
(b) Materiality
(c) Consistency Q101. Which one of the following is not a correct
(d) Conservatism description of the Namami Gange Programme?
(a) It is an initiative of the Ministry of Jal Shakti.
Q95. Which one of the following accounting (b) It primarily covers villages in the States of Uttar
concepts is applied by an entity, when events Pradesh, Bihar and Odisha.
such as new competitor entering in the market (c) It aims to make villages on the banks of the
and rift between production and marketing river Ganga open defecation-free.
departments are not disclosed in the books of (d) It aims to transform some villages on the banks
accounts? of the river Ganga as Ganga Grams.
(a) Matching
(b) Money Measurement Q102. Which one of the following pairs of Zonal
(c) Revenue Recognition Railways and their Headquarters is not correctly
(d) Cost matched?
(a) Central - Nagpur
Q96. Which one of the following concerns (b) Eastern - Kolkata
prepares Receipts and Payments Account? (c) Western - Mumbai
(a) Trading concerns (d) Northern - New Delhi
(b) Non-trading concerns
(c) Manufacturing concerns Q103. Tri-service contingent of which one of the
(d) Companies registered under Companies Act following countries participated in India's
Republic Day parade in 2021?
Q97. Which one of the following statements is (a) Myanmar
correct about Income and Expenditure Account? (b) Nepal
(a) It is a Real Account. (c) Bhutan
(b) It is a Personal Account. (d) Bangladesh
(c) It is a Nominal Account.
(d) It is a Representative Personal Account. Q104. As a routine exercise, Operations `Garam
Hawa' and 'Bard Hawa' are conducted by
Q98. What is the maximum amount of gratuity (a) BSF
payable to the employees under the Payment of (b) ITBP
Gratuity Act, 1972? (c) CRPF
(a) 5,00,000 (d) CISF

9|P a g e W W W . E D U T A P . C O . I N QUERY? HELLO@EDUTAP.CO.IN / 8146207241


(b) 3rd
Q105. Which one of the following States will be (c) 4th
directly connected with Assam by the proposed (d) 5th
bridge between Dhubri and Phulbari over river
Brahmaputra? Q111. Naomi Osaka won the women’s singles
(a) Arunachal Pradesh title of the Australian Open Tennis Tournament,
(b) Meghalaya 2021. Who among the following was defeated by
(c) West Bengal her in the finals?
(d) Mizoram (a) Venus Williams
(b) Jennifer Brady
Q106. Which of the following statements is/are (c) Serena Williams
correct? (d) Simona Halep
1. India is a signatory to the Ramsar Convention.
2. The Ramsar Convention is about wetland Q112. UNESCO has been observing February 21 as
conservation. the International Mother Language Day since
3. At present there are 76 Ramsar Sites in India. 2000. The idea to celebrate the International
Select the correct answer using the code given Mother Language Day was the initiative of which
below: of the following nations?
(a) 2 and 3 only (a) Sri Lanka
(b) 1, 2 and 3 (b) Bangladesh
(c) 1 only (c) India
(d) 1 and 2 only (d) Nepal

Q107. Who among the following is the author of Q113. Dr. Ngozi Okonjo-Iweala was recently
the book “The Little Book of Encouragement”? selected as the first woman director general of
(a) Dalai Lama WTO. She hails from which of the following
(b) A.P.J. Abdul Kalam countries?
(c) Ravi Shankar (a) Japan
(d) Jagadish Vasudev (b) Ethiopia
(c) Nigeria
Q108. Which day has been declared by the (d) Egypt
Government of India to be celebrated every year
as “Parakram Diwas”? Q114. The Kaladan multimodal transit transport
(a) 23 January project well connect India with which one of the
(b) 14 April following countries?
(c) 28 May (a) Nepal
(d) 25 December (b) Myanmar
(c) Bhutan
Q109. A copy of the famous 16th century art work (d) Afghanistan
‘Salvator Mundi’, which was reportedly stolen,
has recently been recovered by the Police in Italy. Q115. Under Sukanya Samridhi Yojana, what is
The painting is attributed to whom among the the maximum amount that can be deposited
following? during a financial year?
(a) Vincent van Gogh (a) ₹1.5 lakh
(b) Pablo Picasso (b) ₹1 lakh
(c) Michelangelo (c) ₹2 lakh
(d) Leonardo da Vinci (d) ₹2.5 lakh

Q110. The Global Firepower Index-2021, a Q116. What is the minimum and maximum age at
military strength ranking, placed India at which which a subscriber can join the Atal Pension
rank? Yojana?
(a) 2nd (a) 21 years and 58 years respectively
10 | P a g e W W W . E D U T A P . C O . I N QUERY? HELLO@EDUTAP.CO.IN / 8146207241
(b) 18 years and 40 years respectively
(c) 18 years and 50 years respectively
(d) 21 years and 60 years respectively

Q117. What is the equivalent decimal value of


binary number 101110?
(a) 46
(b) 56
(c) 64
(d) 65

Q118. Which one of the following is the name of


the scheme introduced as a well-targeted system
of service delivery to LPG customers ?
(a) SAHAJ
(b) PAHAL
(c) UDAY
(d) UDAN

Q119. Which one of the following is not an


objective under Pradhan Mantri Krishi Sinchaye
yojana (PMKSY)?
(a) Providing subsidies to use fertilizer, high
yielding varieties (HYV) and pesticides
(b) To achieve convergence of investments in
irrigation at the field level
(c) To expand cultivable area under assured
irrigation
(d) Improving on-farm water use efficiency

Q120. Who among the following is the first player


in the history of Test Cricket to take 200 wickets
against left-handed batsmen?
(a) Muttiah Muralitharan
(b) James Michael Anderson
(c) Stuart Broad
(d) Ravichandran Ashwin

11 | P a g e W W W . E D U T A P . C O . I N QUERY? HELLO@EDUTAP.CO.IN / 8146207241


Visit: www.civilstap.com
For Any Query Mail us: hello@civilstap.com or call us at - (+91)-8146207241
0
(d) tactlessness - The quality of lacking tact.
SOLUTIONS – PREVIOUS YEAR
QUESTIONS 3. These regions are very fertile.
(a) sterile
(b) arid
UPSC EPFO EO/AO - 2021 (c) barren
(d) productive
PART- A
Directions : Each item in this section consists of a Answer – (C)
sentence with an underlined word followed by Explanation: Opposite of ‘fertile’ is ‘barren’.
your words. Select the option that is opposite in Fertile - (of land or soil) capable of growing
meaning to the underlined word and mark your abundant crops or plants.
response in your Answer Sheet accordingly. (a) sterile - Incapable of reproducing.
(b) arid - Lacking sufficient water or rainfall.
1. She always organises lavish parties on (c) barren - Having little or no vegetation;
birthdays. desolate and lifeless.
(a) frugal (d) productive - Producing or capable of
(b) decent producing (especially abundantly).
(c) cheap
(d) strange 4. Some sick immigrants were detained at the
airport.
Answer – (A) (a) settlers
Explanation: (b) natives
Opposite of ‘lavish’ is ‘frugal’. (c) vagabonds
Lavish - Characterized by extravagance and (d) foreigners
profusion.
(a) frugal - Avoiding waste. Answer – (B)
(b) decent - Socially or conventionally correct; Explanation: Opposite of ‘immigrants’ is
refined or virtuous. ‘natives’.
(c) cheap - Relatively low in price or charging low Immigrant - A person who moves to a foreign
prices or tastelessly showy. country to live there permanently.
(d) strange - Being definitely out of the ordinary (a) settler - A person who settles in a new colony
and unexpected; slightly odd or even a bit weird. or moves into new country.
(b) native - A person born in a particular place or
2. The soldier was greatly praised for his valour. country.
(a) clumsiness (c) vagabond - A wanderer who has no
(b) cowardice established residence or visible means of
(c) selfishness support.
(d) tactlessness (d) foreigner - A person who comes from a
foreign country; someone who does not owe
Answer – (B) allegiance to your country.
Explanation: Opposite of ‘valour’ is ‘cowardice’.
Valour - The qualities of a hero or heroine;
exceptional or heroic courage when facing 5. The questions were so elementary that anyone
danger (especially in battle). could pass the test.
(a) clumsiness - The carriage of someone whose (a) unusual
movements and posture are ungainly or (b) primary
inelegant. (c) naive
(b) cowardice - The trait of lacking courage. (d) advanced
(c) selfishness - Stinginess resulting from a
concern for your own welfare and a disregard of Answer – (D)
others.
1|P a g e W W W . E D U T A P . C O . I N QUERY? HELLO@EDUTAP.CO.IN / 8146207241
Explanation: Opposite of ‘elementary’ is Explanation: Clothes do not ‘bend’ or ‘expand’.
‘advanced’. ‘Squeeze’ is contextually incorrect. So, ‘stretch’
Elementary - Easy and not involved or is correct.
complicated.
(a) unusual - Not commonly encountered. 9. With his back to the old man, he stood at the
(b) primary – First or Most important element. window, fists clenched, and his shoulders ______
(c) naïve - Marked by or showing unaffected with his choked sobbing.
simplicity and lack of guile or worldly (a) shivered
experience. (b) shook
(d) advanced - At a higher level in training, (c) vibrated
knowledge or skill or Ahead in development; (d) moved
complex or intricate.
Answer – (B)
Fill in the blanks Explanation: to make a gesture by raising and
Directions : Each of the following sentences in dropping one's shoulders, often means that one
this section has a blank space and four words or does not know something or is indifferent to
group of words given after the sentence. Select something. Shoulders don’t ‘shiver’ or ‘vibrate.
the word or group of words you consider most ‘moved’ is contextually wrong.
appropriate for the blank space and indicate
your response on the Answer Sheet accordingly. 10. The bereaved mother proudly received the
6. It’s time you went out and ______ your own gallantry award ______ her brave son who had
living. laid down his life in the recent war.
(a) afforded (a) in the name of
(b) earned (b) in lieu of
(c) gained (c) on behalf of
(d) won (d) for the sake of

Answer – (B) Answer – (C)


Explanation: To earn livelihood means a means Explanation: ‘on behalf of’ means as a
of supporting one's existence, especially representative of.
financially or vocationally. ‘in the name of’ means bearing or using the
name of a specified person or organization.
7. The elephant fell into a _______ the hunters ‘in lieu of’ basically means ‘instead of’.
had set. ‘for the sake of’ means ‘because of; out of
(a) track consideration for; in the interest of’.
(b) trap So, option C is correct.
(c) trick
(d) trunk Spotting Errors
Directions : Each item in this section has a
Answer – (B) sentence with three underlined parts labelled
Explanation: To fall into trap means to be fooled (a), (b) and (c). Read each sentence to find out
and get into a difficult situation. whether there is any error in any underlined
part and indicate your response in the Answer
Sheet against the corresponding letter i.e., (a) or
8. The dress isn’t really tight. It’ll ______when (b) or (c). If you find no error, your response
you’ve worn it. should be indicated as (d).
(a) bend
(b) expand 11. Neither the servants/ nor the clerk/ has
(c) squeeze done this./ No error
(d) stretch
(a) Neither the servants
Answer – (D) (b) nor the clerk
2|P a g e W W W . E D U T A P . C O . I N QUERY? HELLO@EDUTAP.CO.IN / 8146207241
(c) has done this.
(d) No error (a) Psychology did not develop into a science
(b) based of careful observation
Answer – (D) (c) and experimentation until the late 1800s.
Explanation: The sentence is grammatically (d) No error.
correct.
Answer – (B)
Explanation: Replace ‘based of’ by ‘based on’.
12. The car which went past us/ when we were The correct complex preposition is ‘based on’.
driving on the highway/ must have been doing
at least a hundred miles an hour./ No error
Synonyms
(a) The car which went past us Directions : Each item in this section consists of a
(b) when we were driving on the highway sentence with an underlined word followed by
(c) must have been doing at least a hundred miles four words. Select the option that is nearest in
an hour. meaning to the underlined word and mark your
(d) No error response in your Answer Sheet accordingly.

Answer – (B) 16. Today the newspaper is full of carnage and


Explanation: Replace ‘when’ by ‘while’ as, for violence.
two simultaneous/ongoing actions, using ‘when’ (a) bloodshed
would be wrong. (b) lust
(c) concern
(d) satire
13. Raju will be back home/ in an year/ after his
stay abroad./ No error Answer – (A)
Explanation: Synonym of ‘carnage’ is
(a) Raju will be back home ‘bloodshed’.
(b) in an year Carnage - The savage and excessive killing of
(c) after his stay abroad. many people.
(d) No error (a) bloodshed - The shedding of blood resulting
in murder.
Answer – (B) (b) lust - A strong sexual desire.
Explanation: Replace ‘an’ by ‘a’ as ‘year’ starts (c) concern - Something that interests you
with a consonant sound so using ‘an’ would be because it is important or affects you.
wrong here. (d) satire - Witty language used to convey insults
or scorn, esp. saying one thing but implying the
opposite.
14. I have/ much work/ to do./ No error.

(a) I have 17. This religious gift, that makes men human, is
(b) much work still alive in Indian souls.
(c) to do. (a) mystical
(d) No error. (b) spiritual
(c) metaphysical
Answer – (D) (d) philosophical
Explanation: The sentence is grammatically
correct. Answer – (B)
Explanation: Synonym of ‘religious is ‘spiritual’.
15. Psychology did not develop into a science/ Religious - Having or showing belief in and
based of careful observation/ and reverence for a deity.
experimentation until the late 1800s./ No error.
3|P a g e W W W . E D U T A P . C O . I N QUERY? HELLO@EDUTAP.CO.IN / 8146207241
(a) mystical - Having an import not apparent to 20. He was in the early part of the twentieth
the senses nor obvious to the intelligence; century, a staunch nationalist of the liberal fold.
beyond ordinary understanding. (a) profuse
(b) spiritual - Concerned with sacred matters, (b) dogmatic
religion or the church. (c) plentiful
(c) metaphysical - Highly abstract and overly (b) tolerant
theoretical.
(d) philosophical - Of or relating to philosophy or Answer – (D)
philosophers. Explanation: Synonym of ‘liberal’ is ‘tolerant’.
Liberal - Showing or characterized by broad-
18. It is difficult for us to imagine how much this mindedness.
safety means to everyone. (a) profuse - Produced or growing in extreme
(a) feel abundance.
(b) understand (b) dogmatic - Of or pertaining to or
(c) think characteristic of a doctrine or code of beliefs
(d) see accepted as authoritative.
(c) plentiful - Existing in great number or
Answer – (B) quantity.
Explanation: Synonym of ‘imagine’ is (b) tolerant - Showing or characterized by broad-
‘understand’. mindedness.
Imagine - Expect to be true; believe. Part - B
(a) feel - Undergo an emotional sensation or be
in a particular state of mind. Q21. How many times does the digit 3 appear
(b) understand - Believe to be the case. between 1 and 100 such that the number where
(c) think - Use or exercise the mind or one's 3 appears is not divisible by 3?
power of reason in order to make inferences, (a) 11
decisions, or arrive at a solution or judgments. (b) 12
(d) see - Perceive by sight or have the power to (c) 13
perceive by sight. (d) 17
Answer: B
19. Public service is not merely something that Explanation:
occupies the hours you are doing it, but invades Numbers does have 3 appear between 1 and 100:
all your life and experience and affects them in 3, 13, 23, 30, 31, 32, 33, 34, 35, 36, 37, 38, 39, 43,
one way or the other. 53, 63, 73, 83, 93.
(a) preserves Numbers 3, 30, 33, 36, 39, 63, 93 are the
(b) demolishes numbers divisible by 3.
(c) covers Required numbers: 13, 23, 31, 32, 34, 35, 37, 38,
(d) spoils 43, 53, 73, 83.
Total 12 numbers.
Answer – (C)
Explanation: Synonym of ‘invade’ is ‘cover’. Q22. Which one of the following is the Average of
Invade - Occupy in large numbers or live on a all prime numbers between 21 & 55?
host. (a) 35.85
(a) preserves - Keep or maintain in unaltered (b) 36.71
condition; cause to remain or last. (c) 38.00
(b) demolishes - Destroy completely. (d) 39.00
(c) covers - Span an interval of distance, space or Answer: C
time. Explanation:
(d) spoils - Make a mess of, destroy or ruin. Prime numbers between 21 and 55: 23, 29, 31,
37, 41, 43, 47, 53.
Average = sum of observations/number of
observations
4|P a g e W W W . E D U T A P . C O . I N QUERY? HELLO@EDUTAP.CO.IN / 8146207241
Average = (23 + 29 + 31 + 37 + 41 + 43 + 47 + 53)/8 Answer: A
= 38 Explanation:
First 6 letters: F, E, D, C, B, A
Q23. If the ratio of Speed of ‘A’ & ‘B’ is 5:6 & ‘B’ Second 6 letters: L, K, J, I, H, G
allows ‘A’ a starts of 70 m in a 1.2km race, who Third 6 letters: R, Q, P, O, N, M
will win the race and by what distance? Fourth 6 letters: X, W, V, U, T, S
(a) ‘A’ wins by 30 m Last two letters: Z, Y
(b) ‘B’ wins by 200 m New alphabetical series: F, E, D, C, B, A, L, K, J, I,
(c) ‘B’ wins by 130 m H, G, R, Q, P, O, N, M, X, W, V, U, T, S, Z, Y
(d) The race finishes in a dead heat 13th letter from the left: R
Answer: C 4th letter to the right of R: N
Explanation: So, the correct answer is N.
B gives a start of 70m to A in a race of 1200m.
B has to cover 1200m Q26. If a varies as b then which of the following
A has to cover 1200 – 70 = 1130m statements is /are correct?
When A covers 5 m, B covers 6 m. 1. nth root of a²b varies as (2n)th root of a4b².
𝑎
When B covers 1200 m, A covers (5 / 6) × 1200 = 2. 𝑏2 varies inversely as b
1000 m
A had a headstart of 70m Select the correct answer using the code given
Total distance covered by A = 1000 + 70 = 1070m below:
B won the race by 1200 – 1070 = 130 m
(a) 1 only
Q24. ‘M’ is 60 year old, ‘R’ is 5 years junior to ‘M’ (b) 2 only
& 4 years senior to ‘V’. The youngest brother of (c) Both 1 and 2
‘V’ is ‘B’ & he is 6 years junior to ‘V’. What is the (d) Neither 1 nor 2
age difference between ‘M’ & ‘B’ ? Answer: C
(a) 18 years Explanation:
(b) 15 years As a varies as b then we can say that a = kb,
(c) 13 years where k is a constant.
(d) 11 years nth root of a²b = a2/nb1/n
Answer: B (2n)th root of a4b² = a4/2nb2/2n
Explanation: = a2/nb1/n So first statement is true.
Age of M = 60 years 𝑎 𝑘𝑏 𝑘
= 𝑏2 = 𝑏 Yes k/b varies inversely as b.
Age of R = 60 – 5 = 55 years 𝑏2
Both are true.
Age of V = 55 – 4 = 51 years
6 years junior to V.
Q27. Which one of the following is the remainder
Age of B = 51 – 6 = 45 years
when 74100 is divided by 9?
Required difference = 60 – 45 = 15 years
(a) 2
(b) 5
Q25. English alphabet is recorded in the following
(c) 3
manner:
(d) 7
Answer: D
The first 6 letters are written in opposite order,
Explanation:
the next 6 letters are written in opposite order
We have to find the remainder when 74100 is
and so on. At the end, ‘Y’ is interchanged with ‘Z’.
divided by 9.
Which one of the following is the fourth letter to
(74)100/9 = (2)100/9
the right of the 13th letter?
= 2(8)33/9 = 2(-1)33/9
(a) N = -2/9 which implies 9 – 2 = 7.
(b) O
Q28. Which one of the following is the arithmetic
(c) M
mean of √3 - √2 and its reciprocal?
(d) I
5|P a g e W W W . E D U T A P . C O . I N QUERY? HELLO@EDUTAP.CO.IN / 8146207241
(a) √3 Let n = 21 which has four factors 1, 3, 7, and 21.
(b) √2 The square root of 21 lies between 4 and 5.
(c) 2 3 is greater than 1 but less than the square root
(d) 1 of n so first statement Is true.
Answer: A 7 is a factor that is greater than the square root
Explanation: of n but less than n. So second statement is also
The reciprocal of √a - √b is always equal to √a + true. Both statements are true.
√b.
Reciprocal of: √3 - √2 is √3 + √2. Q31. The average age of husband and wife was 23
Arithmetic mean: (√3 - √2 + √3 + √2)/2 = √3 years when they were married 5 years ago. The
average age of a husband, his wife and child is 20
Q29. Suppose x and y are two positive numbers years now. How old is the child now?
such that when x is reduced by 2 and y is (a) 9 months
increased by 2, the ratio becomes 2:1; and when (b) 1 years
x is increased by 2 and y is reduced by 2, the ratio (c) 3 years
becomes 3:1. Which one of the following is equal (d) 4 years
to x-y? Answer: D
(a) 20 Explanation:
(b) 24 Let the current age of husband, wife and child be
(c) 18 x, y and z respectively.
(d) 22 (x – 5 + y - 5)/2 = 23
Answer: A (x + y - 10) = 46
Explanation: x + y = 56 ….(I)
First scenario, (x + y + z)/3 = 20
(x - 2)/(y + 2) = 2/1 x + y + z = 60 … (II)
x – 2 = 2y + 4 Subtracting (II) by (I)
x – 2y = 6 ….(1) z = 4 years
Second scenario,
(x + 2)/(y - 2) = 3/1 Q32. If W₁ and W₂ are weights of the two solid
x + 2 = 3y – 6 iron ball of radii ½ metre and ⅓ metre
x – 3y = -8 ….(2) respectively, then which one of the following is
Subtracting (1) by (2). equal to W₁: W₂?
x – 2y – x + 3y = 6 + 8 (a) 8: 27
y = 14, & x = 34 (b) 27: 8
x – y = 34 – 14 (c) 4: 16
= 20 (d) 16: 4
Answer: B
Q30. Let n(>1) be a composite natural number Explanation:
whose square root is not an integer. Consider the Volume of spherical balls will be directly
following statements: proportional to the cube of the radii of the balls.
1. n has a factor that is greater than 1 but less Ratio of volumes = (1/2)3/(1/3)3
than the square root of n. = (1/8)/(1/27)
2. n has a factor that is greater than the square = 27/8
root of n but less than n.
Which of the statements given above is/are Q33. 40% of the students in a class are from India
correct? and 50% are girls. If 25% of Indian students are
(a) 1 only girls, what percentage of non-Indian students are
(b) 2 only boys.
(c) Both 1 and 2 (a) 33.33%
(d) Neither 1 nor 2 (b) 40%
Answer: C (c) 25%
Explanation: (d) 20%
6|P a g e W W W . E D U T A P . C O . I N QUERY? HELLO@EDUTAP.CO.IN / 8146207241
Answer: A Q35. A hollow cube tube of side 10 cm each
Explanation: weighing 200 gm, is made. This cube is placed in
Let the total number of students in the class be water with a horizontal face. How many cm of its
100. height sink in water if the density of water is 1
gm/cm³?
Total number of girls be 50 so the number of boys (a) 1 cm
will be 100 – 50 = 50. (b) 1.5 cm
(c) 2 cm
As 40 students are from India rest 60 will be non- (d) 2.5 cm
indian.
Answer: C
25% of Indian students are girls: 25% of 40 = 10
Explanation:
Non-Indian girls will be equal to 50 – 10 = 40 Weight of the given cube = volume of water
displaced
Non-Indian boys will be equal to Total non-Indian Surface area of the cube face = 10 × 10 = 100 cm2
students minus total non-Indian girls. Let the depth be ‘x’.
Total volume = 100 × x = 100x
= 60 – 40 = 20 Density = Weight/Volume
Weight = 100 × 1 = 100 gm
Required percentage = 20/60 × 100 = 33.33% 100x = 200
x = 2 cm.
Boys Girls Total
Indian 30 10 40 Q36. Which one of the following basic operations
Non- 20 40 60 raw input data into for converting useful
Indian information is not performed by all computers?
Total 50 50 100 (a) Inputting
(b) Storing
Q34. The average weight of a hundred students (c) Switching
in a class is 46 kg. The average weight of boys and (d) Outputting
girls is 50 kg and 40 kg respectively. What is the
difference between the number of boys and girls? Answer: C
(a) 30 Explanation:
(b) 25 All computer systems perform the following five
(c) 20 basic operations for converting raw input data
(d) 10 into useful information-
Answer: C
Explanation: • Inputting: Process of entering data and
Average = sum of all observations/number of instructions into a computer system.
observation • Storing: Saving data and instructions to make
Total weight = 46 × 100 = 4600 kg them readily available for initial or additional
Let the number of boys be x and the number of processing as and when required.
girls will be equal to 100 – x. • Processing: Performing arithmetic operations
Total weight of boys = 50x (add, subtract, multiply, divide, etc.) or logical
Total weight of girls = 40(100 - x) operations (comparisons like equal to, less
50x + 40(100 - x) = 4600 than, greater than, etc.) on data to convert
4000 + 10x = 4600 them into useful information.
10x = 600, x = 60 • Outputting: Process of producing useful
Number of girls = 100 – 60 = 40 information or results for a user, such as
Required difference: 60 – 40 = 20 printed reports or visual displays.

7|P a g e W W W . E D U T A P . C O . I N QUERY? HELLO@EDUTAP.CO.IN / 8146207241


• Controlling: Directing the manner and • Rest all options are an example of Web-
sequence in which the above operations are browsers.
performed.
Q39. Which one of the following represents 1 GB
Q37. Which one of the following memories is of information?
extremely fast and acts as a high-speed buffer (a) 1024 KB
between the CPU and the main memory? (b) 1024 MB
(a) RAM (c) 1024 TB
(b) ROM (d) 1024 PB
(c) Flash Memory
(d) Cache Memory Answer: B
Explanation:
Answer: D • 1 Gigabyte is equal to 1024 megabytes
Explanation: (binary)
• Gigabyte (GB) is one of the most commonly
• Cache memory is a very high-speed used units of digital information which is
semiconductor memory that can speed up the equal to 1,000,000,000 bytes.
CPU. • However, in computer operating science, the
• Cache memory acts as a buffer between the value of 1 GB is considered to be equal to 230
main memory and the CPU. It holds the most or 10243 bytes which is equal to
frequently used part of the program and data. 1,073,741,824 bytes.
• Some advantages of cache memory are –
➢ For temporary use, it is helpful in storing Q40. Which one of the following is not a
data. language
➢ It is faster than the main memory. translator?
• Additional Information (a) Assembler
➢ Cache memory holds only those data and (b) Linker
instructions that are held by the primary (c) Interpreter
memory or main memory on which the (d) Compiler
computer is currently working.
➢ For storing data/information permanently Answer: B
secondary memory also known as Explanation:
external memory or non-volatile is used. • Language-translator: Program that is used to
translate information or instructions that are
Q38. Which one of the following is not a web written in source to object language. It means
browser? that can translate from high-level language or
(a) Internet Explorer assembly-language into machine-language.
(b) Firefox • There are generally three types of language-
(c) Fedora translator- Compiler, Assembler, Interpreter.
(d) Google Chrome • Complier is that kind of program which
translates one language into other target-
Answer: C language
Explanation: • inker is not language- translator. Hence Option
• First released in 2003, Fedora is a Linux B is the answer of this given question.
distribution sponsored by Red Hat.
• Fedora is a completely free and open-source Q41. Which one of the following statements is
operating system that anyone can install on correct?
their computers without ever paying a penny. A device driver of output devices
• A USB storage drive and a tiny little bit of (a) Interprets input provided by users into
enthusiasm for learning new things are all you computer usable form.
need to install Fedora on your PC. (b) Interprets computer output into user
understandable form.
8|P a g e W W W . E D U T A P . C O . I N QUERY? HELLO@EDUTAP.CO.IN / 8146207241
(c) Translates user inputs into output device. hold the program counter, a 32 bit will need
(d) Facilitates user to communicate with output 32, and so on.
device.
Q43. Which one of the following is not an audio
Answer: C file
Explanation: format?
• A device driver is a particular form of software (a) MIDI
application that allows one hardware device (b) WAV
(such as a personal computer) to interact with (c) SWF
another hardware device (such as a printer). (d) MPEG
• A device driver may also be called a software
driver. Answer: A
• Drivers facilitate communication between an Explanation:
operating system and a peripheral hardware • A file with the .MID or .MIDI file
device. extension(pronounced as “mid-ee”) is a
• Each driver contains knowledge about a Musical Instrument Digital Interface file.
particular hardware device or software • Unlike regular audio files like MP3sor WAVs,
interface that other programs — including the these don’t contain actual audio data and are
underlying operating system (OS) — does not therefore much smaller in size. They instead
have. explain what notes are played, when they’re
played, and how long or loud each note should
Q42. Which one of the following registers is used be.
to keep track of the next instruction to be • Files in this format are basically instructions
executed? that explain how the sound should be
(a) Memory address register produced once attached to a playback device
(b) Memory buffer register or loaded into a particular software program
(c) Program counter that knows how to interpret the data.
(d) Memory data register • This makes MIDI files perfect for sharing
musical information between similar
Answer: C applications and for transferring over low-
Explanation: bandwidth internet connections. The small
• Program Counter registers is used to keep size also allows for storing on small devices
track of address of the memory location where like floppy disks, a common practice in early
the next instruction is located. PC games.
• A program counter is a register in a computer
processor that contains the address (location)
of the instruction being executed at the Q44. Which one of the following denotes a
current time. sequential electronic circuit that is used to store
• As each instruction gets fetched, the program 1-bit of information?
counter increases its stored value by 1. The (a) Register
program counter (PC), commonly called the (b) Transistor
instruction pointer (IP) in Intel x86 and (c) Flip-flop
Itanium microprocessors, and sometimes (d) Capacitor
called the instruction address register (IAR), Answer: C
the instruction counter, or just part of the Explanation
instruction sequencer, is a processor register • In electronics, a flip-flop or latch is a circuit
that indicates where a computer is in its that has two stable states and can be used to
program. store state information.
• The program counter is simply the location of • A flip-flop stores a single bit (binary digit) of
the instruction being executed, and so it will data; one of its two states represents a “one”
change based on the processor architecture. and the other represents a “zero”.
That is, 64 bit architecture will need 64 bits to
9|P a g e W W W . E D U T A P . C O . I N QUERY? HELLO@EDUTAP.CO.IN / 8146207241
Q45. Where did Gandhiji put in his first major showing how this visit was understood by the
public appearance after returning from South local peasantry.
Africa in 1915?
(a) Bombay University Q47. Who among the following was the President
(b) Kheda of the Indian National Congress at the time of
(c) Champaran independence of India?
(d) Banaras Hindu University (a) Pattabhi Sitaramayya
Answer: D (b) Jawaharlal Nehru
Explanation: (c) Abul Kalam Azad
Key Points (d) J.B. Kripalani
• Gandhiji, the first major public appearance Answer: D
was at the opening of the Banaras Hindu Explanation:
University in February 1916. Hence option D is
Key Points
the correct answer.
• Banaras Hindu University is an internationally • J.B. Kripalani was the president of the Indian
reputed temple of learning, situated in the National Congress in 1947. Hence option D is
holy city of Varanasi. the correct answer.
• This Creative and innovative university was • Before Kripalani, the President of INC was
founded by the great nationalist leader, Jawaharlal Nehru in 1946.
Pandit Madan Mohan Malviya, in 1916 with • After Kripalani, Pattabhi Sitaraimayya was the
the cooperation of great personalities like Dr. president of INC in 1948 & 1949.
Annie Besant, who viewed it as the University
• In 1950, Purushottam Das Tandon was the
of India.
president of INC and after that in 1951,
• Banaras Hindu University was created under
Jawaharlal Nehru was the president of INC.
the Parliamentary legislation - B.H.U. Act
1915. Important Points
Important Points • The Indian National Congress was founded in
• Mahatma Gandhi, by name of Mohandas Bombay in December 1885.
Karamchand Gandhi, was born October 2, • A.O. Hume was the founder of the Indian
1869, in Porbandar, India & died January 30, National Congress.
1948, in Delhi. • Wyomesh Chandra Banerjee was the first
• Indian lawyer, politician, social activist, and President of INC in 1885.
writer who became the leader of the • Badruddin Tyabji was the First Muslim
nationalist movement against the British rule president of INC and overall, third president of
of India. INC in 1887.
• Annie Besant became the first female
president of INC.
Q46. Who among the following is the author of
`Gandhi as Mahatma’?
Q48. During 1931, under whose leadership, did a
(a) Mahadev Desai
strong Kisan Sabha Movement develop in the
(b) Shahid Amin
Gaya District of Bihar?
(c) Louis Fischer
(a) Yadunandan Sharma
(d) David Arnold
(b) Sahajanand
Answer: B
(c) Sheetla Sahai
Explanation:
(d) Tilka Manjhi
• Shahid Amin’s, “Gandhi as Mahatma:
Answer: A
Gorakhpur District, Eastern UP, 1921-2”.
Explanation:
Hence option B is the correct answer.
• In the northern and central regions of Bihar,
• In this, he examines Gandhi’s fleeting visit in
the Kisan Sabha (peasant movement) was an
February 1921 to Gorakhpur District, a largely
important consequence of the independence
rural area close to the border with Nepal –
movement.
10 | P a g e W W W . E D U T A P . C O . I N QUERY? HELLO@EDUTAP.CO.IN / 8146207241
• It began in 1929 under the leadership of Swami (d) Acharya Narendra Dev
Sahajanand Saraswati who formed the Bihar Answer: A
Provincial Kisan Sabha (BPKS) to mobilise Explanation:
peasant grievances against the zamindari • C. Rajagopalachari (1878 -1972) was a freedom
attacks on their occupancy rights. fighter, politician, associate of Gandhi, and the
• The movement intensified and spread from final governor-general of India.
Bihar across the rest of India. culminating in • Rajagopalachari parted ways with Congress in
the formation of the All-India Kisan Sabha 1957 after being disillusioned by the path it
(AlKS) at the Lucknow session of the Indian was taking.
National Congress in April 1936, where • He founded the Swatantra Party in 1959.
Saraswati was elected as its first president. which favoured classical liberal principles and
free enterprise. Hence Option A is the correct
Q49. Who produced the Pirpur Report (1938)? answer.
(a) Muslim League • In this speech made in April 1973, he
(b) Unionist Party highlighted the purpose and the need of the
(c) Ahrar Party party at the time.
(d) Indian National Congress • The Swatantra Party stands for the protection
Answer: A of the individual citizen against the increasing
Explanation: trespasses of the State.
• Pirpur Committee was established in 1938 by • The Swatantra Party is founded on the claim
the All-India Muslim League to prepare a that individual citizens should be free to hold
detailed report regarding the atrocities of the their property and carry on their professions
Congress Ministries (1937-1939) formed after freely and through binding mutual agreements
the elections under the 1935 Government of among themselves and that the State should
India Act in different provinces. Hence Option assist and encourage in every possible way the
A is the correct answer. individual in this freedom, but not seek to
• The 1938 Pirpur Report brought out by the replace him.
League listed cruelties suffered by Muslims in • The Swatantra party believes that going over
the Congress-ruled provinces. It tried to to the enemy is not defence, but surrender.
project a pro-Hindu stance and anti-Muslim
bias in them. This report became an important Q51. Which one of the following does `Nirguna
document used to demand and justify the Bhakti' refer to?
partition of the country. (a) Shaivite form of worship
• Its report, named Muslim sufferings under the (b) Vaishnavite form of worship
Congress rule by A.K.Fazlul Huq, charged the (c) Incarnation worship
Congress for interference with the religious (d) Abstract form of God worship
rites, suppression of Urdu and propaganda of Answer: A
Hindi, denial of legitimate representation, and Explanation:
suppression in the economy of the Muslims. • C. Rajagopalachari (1878 -1972) was a freedom
• Another such report was the Sharif Report fighter, politician, associate of Gandhi, and the
(Bihar Province) of 1938, documenting pro- final governor-general of India.
Hindu and anti-Muslim bias under Congress • Rajagopalachari parted ways with Congress in
governments. 1957 after being disillusioned by the path it
• Pirpur Report was severely criticised by was taking.
Congress. • He founded the Swatantra Party in 1959.
which favoured classical liberal principles and
Q50. Who among the following was the founder free enterprise. Hence Option A is the correct
of Swatantra Party? answer.
(a) C. Rajagopalachari • In this speech made in April 1973, he
(b) Deen Dayal Upadhyaya highlighted the purpose and the need of the
(c) Shyama Prasad Mukherjee party at the time.

11 | P a g e W W W . E D U T A P . C O . I N QUERY? HELLO@EDUTAP.CO.IN / 8146207241


• The Swatantra Party stands for the protection ➢ The first book dealt with Akbar's ancestors.
of the individual citizen against the increasing ➢ The second recorded the events of Akbar's
trespasses of the State. reign.
• The Swatantra Party is founded on the claim ➢ Ain-i Akbari is the third book.
that individual citizens should be free to hold ➢ The Akbar Nama was translated into
their property and carry on their professions English by Henry Beveridge in the early
freely and through binding mutual agreements twentieth century.
among themselves and that the State should
assist and encourage in every possible way the Q53. Gandhiji's Salt March to Dandi was started
individual in this freedom, but not seek to from which one of the following places?
replace him. (a) Kochrab Ashram
• The Swatantra party believes that going over (b) Sabarmati Ashram
to the enemy is not defence, but surrender. (c) Ahmedabad Textile Mill
(d) Ahmedabad Jail
Q52. What did Ain-i-Akbari seek to promote Answer: B
within the frontiers of the Mughal State? Explanation:
(a) Social disharmony influenced by the whims of
Important Points
the nobility
(b) Cessation of the provincial units from central • On March 12, 1930, Mahatma Gandhi
rule embarked a historic Salt March from
(c) Social harmony with the support of a strong Sabarmati Ashram in Gujarat's Ahmedabad to
ruling class the village of Dandi in the state's coastal area
(d) Establishment of a rule of rural republics by to protest against the steep tax the British
overthrowing the Mughal suzerainty levied on salt. Hence Option B is the correct
Answer: C answer.
Explanation: • The significant march came to be known as
• Ain-i-Akbari was authored by Akbar's court Dandi March or Salt March also referred to as
historian Abut Fazl. the Dandi Satyagraha.
• It is a 16th-century document written in the • The Salt March was a 24-day Salt March, which
Persian language. was non-violent in nature, and is historically
• This text meticulously recorded the significant as it led to the mass Civil
arrangements made by the state- to ensure Disobedience Movement.
cultivation, to enable the collection of revenue • The Salt March began on March 12, 1930, and
by the agencies of the state, to regulate the continued till April 6, 1930.
relationship between the state and rural • During that time, the British had prohibited
magnates, the zamindars. Indians from collecting or selling salt.
• The central purpose of the Ain-i-Akbari was to • Indians were also forced to buy the staple diet
present a vision of Akbar's empire where social ingredient from the British, who, not only
harmony was provided by a strong ruling class. exercised a monopoly over its manufacture
Hence Option C is the correct answer. and sale but also levied a heavy salt tax.
• The account of the Ain-i-Akbari can be • The Salt March was a collective beginning of a
supplemented by descriptions contained in mass resistance movement against British
sources available from regions away from the tyranny.
Mughal capital.
• Upon reaching the seashore in Dandi,
• These sources include detailed revenue Mahatma Gandhi broke the law by producing
records from Gujarat, Maharashtra, and illegal salt.
Rajasthan dating from the seventeenth and
• This later turned into a mass civil disobedience
eighteenth centuries.
movement throughout India as millions broke
Additional Information salt laws by either making salt or buying illegal
salt.
• The Akbar Nama is divided into three books: -
12 | P a g e W W W . E D U T A P . C O . I N QUERY? HELLO@EDUTAP.CO.IN / 8146207241
• The Salt March began with around 80 people, (c) Teaching of Western texts alone at higher
but as more and more people joined in for the levels of education
390 km-long journey, it grew into a strong (d) Teaching of both Western and Oriental texts at
force of 50,000 people. higher levels of education
Answer: D
Q54. Who among the following was not a Explanation:
member of the Socialist group in the Congress Macaulay’s Minute - Features
Party?
(a) Jayaprakash Narayan • Lord Macaulay arrived in India on June 10,
(b) Rajendra Prasad 1834, as a law member of the Governor
(c) N.G. Ranga General's Executive Council and was appointed
(d) Narendra Dev President of the Committee of Public
Answer: B Instruction.
Explanation: • In 1835, he was tasked with settling a dispute
• The Congress Socialist Party was founded in between orientalists and Anglicists.
1934 by Jai Prakash Narayan Ram Manohar • He presented his famous minutes to the
Lohia and Acharya Narendra Deva. council in February 1835, which Lord Bentick
• It was a Socialist group within the Congress approved, and a resolution was passed in
Party. March 1835.
The following points were emphasized by him:
• It was formed by members of the Congress
over ideological differences with Mahatma • The main goal of the British government
Gandhi. should be to promote European literature and
science among Indians, and that "all funds
• Independence and Socialism were its goals.
appropriated for the purpose of education
• Ram Manohar Lohia published a journal called,
would be best spent on English education
'Congress Socialist.'
alone." Hence option D is the answer.
• The party became defunct in 1948.
• All existing professors and students at all
• Most of the socialists were younger
institutions under the committee's supervision
generation. They established Congress
shall continue to receive stipends, but no
Socialist Party. Jaya Prakash Narayan became
stipend shall be given to any students who may
its General Secretary.
subsequently enter any of these institutions.
• Acharya Narendra Dev, Jawaharlal Nehru, and
• No funds from the government were to be
Subhas Chandra Bose were the other leaders.
spent on the printing of oriental works.
• Though Nehru identified himself as a Socialist,
• All funds available to the government would
he was with Gandhiji.
be spent in the future on imparting knowledge
• The Socialists organized the farmers and
of English literature and science to Indians.
workers to attain social justice in Indian
society.
Q56. Who among the following did not represent
• Jayaprakash Narayan, Ram Manohar Lohia and the militant nationalist school of thought in
Aruna Asf Ali and other Socialist leaders took India?
part in the Quit India movement and (a) Ashwini Kumar Dutt
performed major roles in its success and (b) Vishnushastri Chiplunkar
emerged as popular leaders. (c) Krishna Kumar Mitra
• Rajendra Prasad was not associated with the (d) Lala Lajpat Rai
Socialist group. Hence option B is the answer Answer: C
Explanation:
Q55. Which one of the following was not • The school of militant nationalism existed
recommended in Macaulay's 1835 Minute on from the beginning of the National Movement
Education? in India.
(a) Teaching in English medium
• This school was represented by leaders like Raj
(b) 'Liberal' literary training
Narain Bose and Ashwini Kumar Dutt in

13 | P a g e W W W . E D U T A P . C O . I N QUERY? HELLO@EDUTAP.CO.IN / 8146207241


Bengal and Vishnu Shastri Chiplunkar in • The rates were high and unlike the permanent
Maharashtra. system, they were open to being increased.
• The prominent face of this school was Bal • The principle was the direct collection of land
Gangadhar Tilak. revenue from each individual cultivator by
• At the end of the 20th century, this school had government agents.
found a favorable political climate and its
• The advantages of this system were the
adherents came forward to lead the second
elimination of middlemen, who often
stage of the national movement.
oppressed villagers, and an assessment of the
• Other leaders of this school were Bipin tax on land cultivated and not merely
Chandra Pal, Aurobindo Ghose, and Lala occupied.
Lajpat Rai.
➢ Here, no middlemen like zamindar and
• The leaders of this school believed that Indians
elitist groups were involved.
themselves must work for their own salvation
and make the required effort to rise from their • The tax contract was valid for 30 years. After
degraded position created by the British. which it needed to be restructured.
• They were also called extremists as they used • For this purpose, all holdings were measured
to believe in radical mass movements. and assessed according to crop potential and
• Hence option C is the correct answer. actual cultivation.
• Hence Option A is the correct answer.
Q57. Which one of the following statements
about the situation in the Ryotwari areas is
Q58. In the year 1911, who among the following
correct?
formed the Social Service League in Bombay?
(a) A large amount of land passed into the hands
(a) G.K. Gokhale
of the money lenders, merchants, and rich
(b) S.A. Dange
peasants, who usually utilised the services of
(c) N.M. Joshi
the tenants.
(d) M.R. Jayakar
(b) Landless labourers became the landowners.
Answer: C
(c) It stopped the practice of leasing out land to
Explanation:
tenants at high prices.
(d) Landlordism was totally destroyed. • The "Social Service League " was founded by
Answer: A Narayan Malhar Joshi in 1911. Hence Option
Explanation: C is the correct answer.
Key Points • Narayan Malhar Joshi:
➢ Narayan Malhar Joshi (5th June 1879 - 30th
• The Ryotwari system was one of the three
May 1955) was an Indian trade union
principal methods of revenue collection in
leader and follower of Gopal Krishna
British India.
Gokhale.
• It was prevalent in most of southern India,
➢ Joshi got involved in labour issues and
being the standard system of the Madras
started the All-India Trade Union Congress
Presidency.
(AITUC) in 1921 with extremist leader Lala
• The system was devised by Capt. Alexander Lajpat Rai.
Read and Thomas Munro at the end of the ➢ He was the general secretary of AITUC
18th century. from 1925 to 1929.
• This was practised in the Madras and Bombay ➢ He is the founder of the social service
areas, as well as Assam and Coorg provinces. league in 1911.
• In this system, the peasants or cultivators were
regarded as the owners of the land. They had • He was also associated with several Labour
ownership rights, and could sell, mortgage, or organizations and in 1921 he joined the All-
gift the land. India Trade Union Congress and in 1931, N.M.
• The rates were 50% in dryland and 60% in Joshi broke away from the AITUC and formed
wetlands. the All-India Trade Union Federation.

14 | P a g e W W W . E D U T A P . C O . I N QUERY? HELLO@EDUTAP.CO.IN / 8146207241


• Objectives of the Social Service League: • This school of thought led by Vallabhbhai
➢ To collect and study social facts and Patel, Rajendra Prasad, C. Rajagopalachari &
discuss, evaluating the social problems in M.A. Ansari came to be known as the ‘No-
forming public opinion on questions of changers’. Hence Option D is the correct
social service". answer.
➢ To secure for the masses a better quality of
life and work. Q60. Who among the following was not a
member of the Cabinet Mission?
(a) Pethick-Lawrence
Q59. Who among the following was not part of
(b) Stafford Cripps
the group of 'no-changers' in the Congress Party?
(c) A.V. Alexander
(a) Sardar Vallabhbhai Patel
(d) Lord Wavell
(b) Dr. Ansari
Answer: D
(c) Babu Rajendra Prasad
Explanation
(d) Motilal Nehru
Answer: D • The Cabinet Mission Plan was a statement
Explanation: made by the Cabinet Mission and the Viceroy,
• After Gandhi’s arrest (March 1922), there was Lord Wavell, on May 16, 1946, that contained
proposals regarding the constitutional future
disintegration, disorganisation, and
of India in the wake of Indian political parties
demoralisation among nationalist ranks. A
and representatives not coming to an
debate started among Congressmen on what
agreement.
to do during the transition period, i.e., the
passive phase of the movement. • The members of the Cabinet Mission were
Swarajists ➢ Lord Penthick-Lawrence, Secretary of
State for India,
• One section led by C R Das, Motilal Nehru, and
➢ Sir Stafford Cripps, President of the Board
Ajmal Khan wanted an end to the boycott of
of Trade, and
legislative councils so that the nationalists
➢ A.V Alexander, First Lord of Admiralty.
could enter them to expose the basic
➢ Hence Option D is the correct answer.
weaknesses of these assemblies and use these
councils as an arena of political struggle.
Q61. Which one of the following facts pertaining
• They wanted to ‘end or mend’ these councils, to the National Green Tribunal (NGT) is not
i.e., if the Government did not respond to the correct?
nationalists’ demands, then they would (a) The NGT was set up in the year 2010.
obstruct the working of these councils. (b) Its purpose is to ensure effective and
• Their only intention was to use the councils as expeditious disposal of cases relating to
arena of political struggle; they had no environmental protection and conservation of
intention to use the councils as organs for forests and other natural resources.
gradual transformation of colonial rule. (c) It is bound by the procedure laid down under
• Those advocating entry into legislative councils the Code of Civil Procedure.
came to be known as the Swarajists. (d) It is guided by the principles of natural justice.
Answer: C
No Changers Explanation:
• The ‘No-changers’ opposed council entry. Key Points
• They advocated concentration on constructive • National Green Tribunal (NGT):
work, and continuation of boycott and non- ➢ The National Green Tribunal was formed
cooperation. on 18 October 2010.
➢ It is a statutory body formed under the
• They also advocated quiet preparation for
National Green Tribunal Act 2010. Hence,
resumption of the suspended civil
NGT is a statutory body.
disobedience programme

15 | P a g e W W W . E D U T A P . C O . I N QUERY? HELLO@EDUTAP.CO.IN / 8146207241


➢ It is related to Article 21 of the Indian Constitution
Constitution. • The Committee on Estimates constituted for
➢ This Tribunal has the same status as the the first time in 1950, is a Parliamentary
High Court. Committee consisting of 30 members, elected
➢ Its headquarters is located in New Delhi. every year by the Lok Sabha from amongst its
➢ The main objective of the establishment of members.
the National Green Tribunal is to speedy • The Chairperson of the Committee is
disposal of environmental issues to reduce appointed by the Speaker from amongst its
the burden of pending litigations in the members.
country's courts. Hence, NGT is a Quasi- • A Minister cannot be elected as a member of
Judicial Body. the Committee and if a member after selection
➢ According to the National Green Tribunal to the Committee is appointed a Minister, the
Act NGT must deal with the environmental member ceases to be a Member of the
issues faced by them within 6 months. Committee from the date of such
appointment.
Important Point
• Composition: Chairman+ Members (Judicial Term of Office
and Expert members) The term of office of the Committee is one year.
• There are at least 10 and a maximum of 20 full-
time judicial members and expert members. Functions
• Tenure: 5 years The functions of the Estimates Committee are:
• Not eligible for re-appointment. 1. to report what economies, improvements in
• Appointment: The Chairman is appointed by organisation, efficiency, or administrative
the Central Government after consulting the reform, consistent with the policy underlying
Chief Justice of India. the estimates may be affected. Hence option
• A selection committee is formed by the central A is the correct answer
government to appoint judicial and expert 2. to suggest alternative policies in order to bring
members. about efficiency and economy in
• Salary and Allowances are determined by the administration.
Central Government. 3. to examine whether the money is well laid out
• Being a statutory body, the NGT has appellate within the limits of the policy implied in the
jurisdiction and under which it can conduct estimates. and
hearings. 4. to suggest the form in which the estimates
• NGT is not obliged to follow the judicial shall be presented to Parliament.
process mentioned in Civil Procedure Code,
1908. Hence option C is incorrect. The Committee does not exercise its functions in
relation to such Public Undertakings as are
Q62. Which one of the following is the mandate allotted to the Committee on Public
of the Committee on Estimates? Undertakings by the Rules of Procedure and
(a) It reports on what economies, improvements Conduct of Business of Lok Sabha or by the
in organization, efficiency, or administrative Speaker.
reform consistent with the policy underlying
the estimates, may be effected. Q63. Which one of the following does not figure
(b) It scrutinizes appropriation and finance in the list of languages in the Eighth Schedule of
accounts of the Government. the Constitution of India?
(c) It examines reports of the Comptroller and (a) Dogri
Auditor General and whether public (b) Bhoti
undertakings are run efficiently. (c) Maithili
(d) It examines bills on matters of general public (d) Santhali
interest. Answer: B
Answer: A Explanation:
Explanation:
16 | P a g e W W W . E D U T A P . C O . I N QUERY? HELLO@EDUTAP.CO.IN / 8146207241
• Languages recognized by the Constitution.
Originally, it had 14 languages but presently
there are 22 languages.
• They are Assamese, Bengali, Bodo, Dogri
(Dongri), Gujarati, Hindi, Kannada, Kashmiri,
Konkani, Mathili (Maithili), Malayalam,
Manipuri, Marathi, Nepali, Odia, Punjabi,
Sanskrit, Santhali, Sindhi, Tamil, Telugu, and
Urdu. Hence Option B is the correct answer.
• Sindhi was added by the 21st Amendment Act
of 1967.
• Konkani, Manipuri, and Nepali were added by
the 71st Amendment Act of 1992, and
• Bodo, Dongri, Maithili, and Santhali were
added by the 92nd Amendment Act of 2003.
• Oriya was renamed as ‘Odia’ by the 96th
Amendment Act of 2011.

Q64. Who among the following Speakers of Lok


Sabha formally disassociated from the political Q65. Which one of the following is not a female
party after his election as the Speaker? reproductive organ in humans?
(a) G.V. Mavalankar (a) Ovaries
(b) Sardar Hukam Singh (b) Oviducts
(c) Neelam Sanjiva Reddy (c) Cervix
(d) P.A. Sangma (d) Stamen
Answer: C Answer: D
Explanation: Explanation:
• There were two speakers, i.e. Neelam Sanjiva Female Reproductive system in humans
Reddy and Gurdial Sign Dhillon who resigned The female reproductive system consists of a pair
from Lok Sabha during his office tenure. Hence of ovaries alongwith a pair of oviducts, uterus,
option C is the correct answer. cervix, vagina and the external genitalia located in
• The election of the Speaker of Lok Sabha is pelvic region.
done in the first meeting just after the general • These parts of the system alongwith a pair of
elections. the mammary glands are integrated
• A Lok Sabha speaker has to serve a tenure of structurally and functionally to support the
5 years. processes of ovulation, fertilisation,
• The Lok Sabha Speaker is chosen from the pregnancy, birth and child care.
members of the House of the People. • The parts are shown in the figure below:
• Generally, the post of Lok Sabha Speaker is
given to the member of the ruling party.
• List of the Speakers of the Lok Sabha:

• In the figure below, the stamen is a male


reproductive part of the flower which

17 | P a g e W W W . E D U T A P . C O . I N QUERY? HELLO@EDUTAP.CO.IN / 8146207241


produces pollen grain to be transferred to widespread deforestation. The consequence of
stigma of female reproductive system. deforestation are:
• Soil Erosion: Trees help to stabilize soil, and
their roots help to hold soil in place. When
trees are removed, soil erosion can occur,
leading to nutrient depletion and reduced
agricultural productivity. Hence option (c)
represents a consequence of deforestation
and thus is not the correct answer.
• Loss of Biodiversity: Deforestation destroys
the habitat of many plant and animal species,
leading to a loss of biodiversity. This can result
Hence option D is the correct answer. in species extinction and a decline in
ecosystem services. Hence option (b)
Q66. Which one of the following cannot be called represents a consequence of deforestation
“Amphibian of the Plant Kingdom”? and thus is not the correct answer.
(a) Spirogyra • Water Cycle Disruption: Forests play an
(b) Riccia essential role in the water cycle by absorbing
(c) Funaria rainwater and releasing it slowly over time and
(d) Marchantia resulting in reduced rainfall. Deforestation can
Answer: A disrupt the water cycle, leading to water
Explanation: shortages, floods, and soil erosion. Hence
Bryophytes are also called amphibians of the option (d) represents a consequence of
plant kingdom because these plants can live in soil deforestation and thus is not the correct
but are dependent on water for sexual answer.
reproduction. • Economic Impacts: Deforestation can have
• They usually occur in damp, humid and shaded significant economic impacts, particularly in
localities. They play an important role in plant regions that rely on forest resources for their
succession on bare rocks/soil. livelihoods.
• E.g. Funaria, Marchantia, Sphagnum, Riccia
etc. Hence options (b), (c) and (d) represent Hence option A is the correct answer as ground
Amphibians of Plant Kingdom. water table actually decreases due to reduced
• While Spirogyra is an Algae and is a rainfall.
chlorophyll-bearing, simple, thalloid,
autotrophic and largely aquatic (both fresh Q68. Which one of the following chromosomes
water and marine) organism. Hence it cannot has a mismatched pair in a normal human male?
be called an amphibian of plant kingdom. (a) Chromosome number 21
(b) Chromosome number 18
Hence option A is correct. (c) X-Chromosome
(d) Chromosome number 13
Q67. Which one of the following is not a Answer: C
consequence of deforestation? Explanation:
(a) Increased groundwater table The chromosomes are cell organelles present in
(b) Decreased biodiversity the nuclei of human cells.
(c) Increased soil erosion • They are composed of DNA and carry the
(d) Decreased rainfall genetic information that determines an
Answer: A individual's traits and characteristics.
Explanation: • Humans typically have 23 pairs of
The growing population in today’s world calls for chromosomes, for a total of 46 chromosomes.
more and more space cleared for human • One set of chromosomes is inherited from the
settlements and related activities. This has led to mother and the other set from the father. Of
the 23 pairs, 22 pairs are known as
18 | P a g e W W W . E D U T A P . C O . I N QUERY? HELLO@EDUTAP.CO.IN / 8146207241
autosomes, and the 23rd pair determines the The human eye functions like a camera, with the
individual's sex. The autosomes are identical lens focusing light onto the retina, located at the
chromosomes. Hence options (a), (b) and (d) back of the eye.
are incorrect. • The image of the objects infront of the eye is
• The sex chromosomes in humans are labeled X formed on the retina. Each eye sees the entire
and Y, with females having two X potion of the object and forms a slightly
chromosomes (XX) and males having one X different image. Hence options A and B are
and one Y chromosome (XY). Thus the X- incorrect and option D is correct.
chromosome is mismatched in males • The signals from both the eyes and processes
Hence option C is correct. these signals to form an image that we
perceive as the things we see. Hence option C
Q69. The word ‘Vaccine’ has been derived from a is incorrect as it is the brain that combines the
Latin word having meaning two images.
(a) Antibody
(b) Immunity Q71. Which one of the following statements is
(c) Cow not correct?
(d) Guinea pig Wavelength of microwaves ranges between
Answer: C (a) Infrared waves and radio waves.
Explanation: (b) Visible waves and radio waves.
The word "vaccine" is derived from the Latin word (c) Gamma-rays and X-rays
"vacca," which means "cow." (d) X-rays and visible waves
• The term was coined by Edward Jenner, an Answer: A
English physician, in the late 18th century. Explanation:
• Jenner observed that milkmaids who had The electromagnetic spectrum refers to the full
contracted cowpox, a mild viral disease that range of all types of electromagnetic radiation,
affected cows, did not get infected with which includes both visible and invisible forms of
smallpox, a more severe and deadly disease radiation.
that affected humans. • The electromagnetic spectrum is organized
• Based on this observation, he developed the based on the wavelength and frequency of the
first vaccine by using cowpox virus to inoculate waves.
against smallpox. • At one end of the spectrum, we have the long-
• He called this procedure "vaccination," which wavelength, low-frequency radio waves used
later became the term for any method of for broadcasting, followed by microwaves,
inoculation against a disease. infrared radiation, visible light, ultraviolet
• Thus, the term "vaccine" originally referred to radiation, X-rays, and gamma rays at the other
the smallpox vaccine, but it is now used to end of the spectrum with short-wavelength,
refer to any preparation used to stimulate the high-frequency waves. Hence option A is
body's immune system to fight against correct as the wavelength and frequency of
infectious diseases. microwaves lie between infrared and radio
Hence option C is correct. waves.

Q70. Which one of the following statements is


true with regard to the image formation by two
eyes of a person?
(a) Both the eyes see exactly the same image.
(b) One eye sees half portion of the object
(c) Both the eyes combine the two images seen by
them
(d) Each eye sees a slightly different image
Answer: D
Q72. Which one of the following is the size of
Explanation:
hydrogen atom?
19 | P a g e W W W . E D U T A P . C O . I N QUERY? HELLO@EDUTAP.CO.IN / 8146207241
(a) 10-10 m Q75. Which one of the following substances,
(b) 10 micro metres when added to water, will not change the pH?
(c) 10 millimetres (a) NaHCO3
(d) 1000 Angstrom (b) NH4Cl
Answer: A (c) Na2CO3
Explanation: (d) NaCl
A hydrogen atom has a diameter of approximately Answer: D
1.06×10−10m, as defined by the diameter of the Explanation:
spherical electron cloud around the nucleus. • In water, sodium bicarbonate (NaHCO3)
Hence option A is correct. dissolves to form sodium and bicarbonate. This
makes the solution alkaline and thus changes
Q73. Which one of the following statements the pH. Hence option (a) is not the correct
regarding force is correct? answer.
(a) A positive force implies attractive nature. • Ammonium chloride (NH4Cl) when dissolved
(b) A negative force implies repulsive nature. in water it hydrolyses to form ammonium
(c) A positive force can be both attractive and hydroxide and hydrochloric acid. The formed
repulsive in nature. hydrochloric acid is strong acid and it ionizes to
(d) A negative force implies attractive nature. give H+ ions whereas ammonium hydroxide is
Answer: D weak base and give small amount of OH- ions
Explanation: on dissociation. It decreases the pH of water.
In the context of magnetic forces, like poles Hence option (b) is not the correct answer.
(positive with positive, or negative with negative) • When dissolved in water, sodium carbonate
repel each other, creating a positive force. forms carbonic acid and sodium hydroxide.
• Unlike poles attract each other and are Because sodium hydroxide is a strong base, the
associated with negative force. These are the resulting solution has an elevated pH value.
conventions used in physics. Hence option (c) is not the correct answer.
• In general, this depends upon the context and • Sodium chloride (NaCl) being a neutral salt,
the convention can vary. Hence option D is when dissolved in water does not vary the pH
correct. of water. Hence option (d) is the correct
answer.
Q74. When a dielectric material is kept in an
external electric field, which one of the following Q76. Which one of the following, on adding to
phenomena may be realized? water, will not scatter a beam of light?
(a) Magnetization (a) Copper sulphate
(b) Polarization (b) Chalk powder
(c) Photoionization (c) Milk
(d) Circularization (d) Ink
Answer: B Answer: A
Explanation: Explanation:
A dielectric material is a poor conductor of The Tyndall effect is a phenomenon in which light
electricity but an efficient supporter of is scattered by particles that are suspended in a
electrostatic fields. medium like water. Thus, substances that form a
• When an electric field is applied to a dielectric suspension or colloid scatter a beam of light.
material (or electric insulator) become • Copper Sulphate when dissolved in water
polarized, such that the negative charges in the creates a solution which has less chances of
material orient themselves toward the positive scattering light as true solutions do not show
electrode and the positive charges shift toward Tyndall effect. Hence option (a) is the correct
the negative electrode. This process is called answer
Polarization. • Chalk powder forms a suspension and thus
Hence option B is correct. scatters a beam of light. Hence option (b) is
not the correct answer

20 | P a g e W W W . E D U T A P . C O . I N QUERY? HELLO@EDUTAP.CO.IN / 8146207241


• Milk is a colloidal mixture that contains from its solution being less reactive than Zinc.
suspended particles of proteins and fat. When Hence options (a), (b) and (c) are not correct.
a beam of light passes through milk and water • While Aluminium (Al3+) is placed above Zinc
mixture, the suspended particles scatter the and is more reactive, thus it can replace Zinc
light in all directions. Hence option (c) is not from its solution. Hence option (d) is correct.
the correct answer
• When ink is added to water, the ink particles Q78. Who among the following discovered
become suspended in the water and create a Proton?
colloidal mixture. As a result, when a beam of (a) J.J. Thomson
light passes through the mixture, the ink (b) E. Goldstein
particles scatter the light in all directions, (c) E. Rutherford
causing the beam of light to become visible as (d) J. Chadwick
it illuminates the path of the scattered Answer: B
particles. Explanation:
Hence option (a) is the correct answer • J.J. Thomson won Nobel Prize in 1906 for
discovering the elementary particle electron.
Q77. Which one of the following will not be Hence option (a) is incorrect.
reduced by metallic Zinc? • E. Goldstein, while conducting experiments
(a) Cu2+ with the anode ray tube in 1886, Goldstein
(b) H+ observed a new type of positively charged
(c) Ag+ particle that had a higher mass than the
(d) Al3+ previously discovered canal rays. He called
Answer: D these particles "Kanalstrahlen," which were
Explanation: later identified as protons. Hence option (b) is
The reactivity series is a list of metals arranged in correct as he was the first to discover the
order of their reactivity towards chemical particle.
reactions. • Rutherford conducted a series of experiments
• The series is arranged with the most reactive in 1917, in which he bombarded thin gold foil
metals at the top and the least reactive metals with alpha particles (helium nuclei) and
at the bottom. observed the scattering pattern of the alpha
• The reactivity series is based on the tendency particles on a fluorescent screen. He observed
of metals to lose electrons and form positive the positively charged particle in the nucleus
ions when they react with other substances. and called it proton for the first time. As
The reactivity series is a s follows: E.Goldstein was the one who for the first time
discovered the particle he is regarded to have
discovered proton. Hence option (c) is
incorrect.
• In 1932, James Chadwick made a fundamental
discovery in the domain of nuclear science: he
proved the existence of neutrons – elementary
particles devoid of any charges present in the
nucleus of the atom. Hence option (d) is
incorrect.

Q79. Consider the following balanced equation:


• Metals that are more reactive are placed at the CO(g) + 2H2(g) → CH3OH(l)
top of the series. They can reduce the metal How many moles of CH3OH(l) can be obtained by
ions shown below them, from their solutions. reacting 2.0 mole of CO(g) with 2.0 mole of H2(g)
• It is evident from above that Copper (Cu2+), (a) 1
Hydrogen (H+) and Silver (Ag+) are below (b) 2
Zinc and thus can be reduced by metallic Zinc (c) 3
(d) 4
21 | P a g e W W W . E D U T A P . C O . I N QUERY? HELLO@EDUTAP.CO.IN / 8146207241
Answer: D (a) Who has completed 15 years of age but is less
Explanation: than 18 years
• In a reaction wherein, Carbon monoxide gas (b) Who is less than 18 years
reacts with hydrogen gas to form methanol. (c) Who has completed 14 years of age but is less
CO(g) + 2H2(g) → CH3OH(l) than 18 years
• As per the reaction, One mole of CO(g) reacts (d) Who has completed 16 years of age but is less
with 2.0 moles of H2(g) to give rise to One mole than 18 years
of Methanol.
• Hence, keeping the proportion same, in the Answer: A
presence of 2.0 mole of CO(g) with 2.0 mole of Explanation:
H2(g) only one mole of Methanol can be • As per the provisions of the Factories Act,
produced. 1948, “adolescent” means a person who
• The other mole of Carbon monoxide remains has completed his fifteenth year of age but
unreacted. has not completed his eighteenth year.
Hence option (a) is correct. • Hence, A is the right answer

Q80. Match List I with List II and select the correct. Q82. What is a controlled industry?
answer using the code given below the lists: (a) Any industry the control of which by the Union
List I List II has been declared by any Central
(Term) (Explanation) Act to be expedient in the public interest
A. Closure 1. Permanent closing down (b) Any industry the control of which by the State
of a place of employment has been declared by any State Act to be
B. Workmen 2. Any person employed in expedient in the public interest
any industry to do skilled, (c) Any industry the control of which by the
unskilled or manual work Municipal bodies has been declared by any
C. Strike 3. Temporary closing down Municipal Rules to be expedient in the public
of workplace by the interest
management (d) Any industry the control of which by the State
D. Lockout 4. Cessation of work by has been declared by any Central Act to be
employees expedient in the public interest
Code:
A B C D Answer: A
(a) 1 2 4 3 Explanation:
(b) 3 4 2 1 • As per section 2(ee) of the Industrial Disputes
(c) 1 4 2 3 Act, 1947, controlled industry” means any
(d) 3 2 4 1 industry the control of which by the Union has
been declared by any Central Act to be
expedient in the public interest
Answer: A • Hence, A is the right answer.
Explanation:
• Closure means Permanent closing down of a Q83. What is the minimum number of members
place of employment. required for registration of a Trade Union?
• Workmen means any person employed in any (a) 2 members
industry to do skilled, unskilled, or manual (b) 3 members
work. (c) 7 members
• Strike means Cessation of work by employees (d) 10 members
• Lockout means temporary closing down of
workplace by the management. Answer: C
• Hence, A is the right answer. Explanation:
As per the Trade Unions Act, 1926, Mode of
Q81. Who is an 'Adolescent' as per the Factories registration -
Act, 1948?
22 | P a g e W W W . E D U T A P . C O . I N QUERY? HELLO@EDUTAP.CO.IN / 8146207241
• Any seven or more members of a Trade Q85. What is the maximum number of hours in a
Union may, by subscribing their names to the week that an adult worker is allowed to work for?
rules of the Trade Union and by otherwise (a) 35 hours
complying with the provisions of this Act with (b) 40 hours
respect to registration, apply for registration (c) 45 hours
of the Trade Union under this Act. (d) 48 hours
• Where an application has been made for the
Answer: D
registration of a Trade Union, such application
Explanation:
shall not be deemed to have become invalid
• As per the provisions of Factories Act, no adult
merely by reason of the fact that, at any time
worker shall be required or allowed to work in
after the date of the application, but before
a factory for more than forty-eight hours in
the registration of the Trade Union, some of
any week.
the applicants, but not exceeding half of the
• Hence, D is the right answer.
total number of persons who made
the application, have ceased to be members
Q86. What is the maximum period in which the
of the Trade Union or have given notice in
appropriate government shall review and revise
writing to the Registrar dissociating
the minimum rates of wages under the Minimum
themselves from the applications.
Wages Act, 1948?
• Hence, C is the right answer. (a) 2 years
(b) 3 years
Q84. Which of the following disputes is/are (c) 4 years
considered as trade dispute(s) under the (d) 5 years
provision of the Trade Union Act, 1926?
Any dispute of any person connected with Answer: D
1.Employment Explanation:
2. Non-Employment • As per the Minimum Wages Act, 1948, the
3.Conditions of Labour appropriate Government shall review at such
Select the correct answer using the code given intervals as it may think fit, such intervals not
below: exceeding five years, the minimum rates of
(a) 1 only wages so fixed and revise the minimum rates.
(b) 2 and 3 only • Hence, D is the right answer.
(c) 1, 2 and 3
(d) 1 and 3 only Q87. In which one of the following places, was
Asia’s first Export Processing Zone (EPZ) set up in
Answer: C 1965?
Explanation: (a) Ahmedabad
• As per the Trade Unions Act, 1926, “trade (b) Kandla
dispute” means any dispute between (c) Mumbai
employers and workmen or (d) Jaipur
between workmen and workmen, or between Answer: B
employers and employers which is connected Explanation:
with the employment or non-employment, or India was one of the first in Asia to recognize the
the terms of employment or the conditions of effectiveness of the Export Processing Zone (EPZ)
labour, of any person, and “workmen” means model in promoting exports, with Asia's first EPZ
all persons employed in trade or industry set up in Kandla in 1965. Therefore, (b) is the right
whether or not in the employment of the answer.
employer with whom the trade dispute arises.
• Hence, C is the right answer. Q88. Which one of the following statements
about Trial Balance is correct?
(a) It is a book containing different accounts of an
entity.
23 | P a g e W W W . E D U T A P . C O . I N QUERY? HELLO@EDUTAP.CO.IN / 8146207241
(b) It is a statement containing balances of debtors • Current liabilities are a company’s short-term
of an entity. financial obligations that are due within one
(c) It is a statement containing balances of debtors year or within a normal operating cycle.
and creditors of an entity. • Current liabilities are typically settled using
(d) It is a statement containing the various ledger current assets, which are assets that are used
balances of an entity on a particular date up within one year.
• Examples of current liabilities include accounts
Answer: D payable, short-term debt, dividends, and notes
Explanation: payable as well as income taxes owed.
• Trial Balance may be defined as a statement • The analysis of current liabilities is important
which contains balances of all ledger accounts to investors and creditors. This can give a
on a particular date. picture of a company’s financial solvency and
• Trial Balance consists of a debit column with all management of its current liabilities.
debit balances of accounts and credit column
with all credit balances of accounts. Q91. Is the total of Debit and Credit side of Trial
• The totals of these columns if tally it is Balance the same?
presumed that ledger has been maintained (a) No, there are sometimes good reason why they
correctly. differ.
• However, Trial Balance proves only the (b) Yes, always.
arithmetical accuracy of posting in the ledger. (c) Yes, except where the Trial Balance is extracted
at the year end.
(d) No, because it is not a Balance Sheet
Q89. Wages paid for installation of machinery is
debited to which one of the following accounts? Answer: A
(a) Wages Account Explanation:
(b) Machinery Account If there any difference in the trial balance, it
(c) Installation Account signals that journal or ledger posting is not
(d) Profit and Loss Account carried out efficiently. It clearly implies that there
are errors and it is high time for accountants to
Answer: B find and correct it. The error may have occurred
Explanation: at any of the following stages of accounting.
• Wages paid for the installation of Machinery ➢ Posting journal entries to the ledger account
should be debited to the Machinery Account. ➢ Totalling of subsidiary books
• Such wages should not be debited to the ➢ Calculation errors
Trading Account because these wages ➢ Posting of Balance from Ledger account to
represent capital expenditure. trial balance
• Therefore the wages for the ‘installation of ➢ Error in totalling Trial balance.
machinery’ are added to the cost of
machinery since without spending the Q92. Which one of the following is the first book
amount for the installation, the machine will in which the transactions of a business unit are
not be operational. recorded?
• So, it must be added to the cost of machine (a) Balance Sheet
(b) Cash Book
Q90. When are current liabilities payable? (c) Ledger
(a) Within a year (d) Journal
(b) After one year but within five years
(c) Within five years Answer: D
(d) Subject to a contingency Explanation:
• A journal is known as the original book of
Answer: A entry and is also called the prime entry.
Explanation:

24 | P a g e W W W . E D U T A P . C O . I N QUERY? HELLO@EDUTAP.CO.IN / 8146207241


• All the accounting transactions are first 'anticipated loss' but if the market price has
recorded in journal in the order of the gone up then ignore the 'anticipated profits'.
transactions taking place.
• Such order is called a chronological order. Q95. Which one of the following accounting
• Therefore, journal is the original book of concepts is applied by an entity, when events
recording transactions in the chronological such as new competitor entering in the market
order. and rift between production and marketing
departments are not disclosed in the books of
Q93. Which one of the following denotes Gross accounts?
Profit? (a) Matching
(a) Cost of goods sold + Opening stock (b) Money Measurement
(b) Sales less Cost of goods sold (c) Revenue Recognition
(c) Sales less Purchases (d) Cost
(d) Net profit less Expenses of the period Answer: B
Explanation
Answer: B This accounting concept states that only financial
Explanation: transactions will find a place in accounting.
• Gross profit, also called gross income, is • So only those business activities that can
calculated by subtracting the cost of goods be expressed in monetary terms will be
sold from revenue. recorded in accounting. Any other
• Gross profit assesses a company's efficiency at transaction, no matter how significant,
using its labour and supplies in producing will not find a place in the financial
goods or services. accounts.
• Gross profit, which only reflects the cost of • So for example, if the company underwent
goods sold, is different than net profit which a major management overhaul this would
factors in all company-wide expenses. have no effect on the accounting records.
• A derivative of gross profit is gross margin, a This concept is actually one of the major
margin that indicates what percent of revenue drawbacks of accounting.
a company earns can be applied towards
company operating costs.
Q96. Which one of the following concerns
Q94. What is the underlying accounting concept prepares Receipts and Payments Account?
that supports no anticipation of profits but (a) Trading concerns
provision for all possible losses? (b) Non-trading concerns
(a) Matching (c) Manufacturing concerns
(b) Materiality (d) Companies registered under Companies Act
(c) Consistency Answer: B
(d) Conservatism Explanation:
• Receipts and payments Account is prepared
Answer: D by non-Trading concerns.
Explanation: • It acts like a cash account for the Non-Trading
• Conservatism concept emphasizes that profit Concerns
should never be overstated or anticipated • It helps in the making of income and
• Traditionally, accounting follows the rule expenditure of account which will show the
"anticipate no profit and provide for all deficit and surplus of the concern.
possible losses.
• For example, the closing stock is valued at cost Q97. Which one of the following statements is
price or market price, whichever is lower. correct about Income and Expenditure Account?
• The effect of the above is that in case market (a) It is a Real Account.
price has come down then provide for the (b) It is a Personal Account.
(c) It is a Nominal Account.
(d) It is a Representative Personal Account.
25 | P a g e W W W . E D U T A P . C O . I N QUERY? HELLO@EDUTAP.CO.IN / 8146207241
(a) It is an initiative of the Ministry of Jal Shakti.
Answer: C (b) It primarily covers villages in the States of Uttar
Explanation: Pradesh, Bihar and Odisha.
• Nominal account is related to expenses or (c) It aims to make villages on the banks of the
losses and incomes or gains. river Ganga open defecation-free.
• Income and Expenditure account is an (d) It aims to transform some villages on the banks
account prepared by non trading concerns to of the river Ganga as Ganga Grams.
ascertain surplus or deficit of income over Answer: B
expenditure for a particular period. Explanation
• So Income and Expenditure account is a About Namami Gange Program
Nominal account. • It is an initiative of Ministry of Jal Shakti
• It has been completed in the states of
Uttarakhand, Uttar Pradesh, Bihar,
Q99. According to the Economic Survey 2020 - Jharkhand, West Bengal, Delhi, Himachal
21," which one of the following will be India’s real Pradesh, Haryana, and Rajasthan.
GDP growth in the year 2021 - 22? • It aims to make villages on the banks of the
(a) 9% river Ganga open defecation-free.
(b) 11 % • It aims to transform some villages on the
(c) 13% banks of the river Ganga as Ganga Grams.
(d) 15%
Answer: B Q102. Which one of the following pairs of Zonal
Explanation Railways and their Headquarters is not correctly
• The Survey says, India’s GDP is estimated matched?
to contract by 7.7 per cent in FY2020-21, (a) Central - Nagpur
composed of a sharp 15.7 per cent decline (b) Eastern - Kolkata
in first half and a modest 0.1 per cent fall (c) Western - Mumbai
in the second half. (d) Northern - New Delhi
Answer: A
Explanation
• Central Railway Mumbai
• Eastern Railway Kolkata
• Western Railway Mumbai
• Northern Railway New Delhi

Q103. Tri-service contingent of which one of the


following countries participated in India's
Republic Day parade in 2021?
(a) Myanmar
Q100. Which one of the following is the acronym (b) Nepal
for the word 'NITI' in NITI Aayog? (c) Bhutan
(a) National Integration and Transformation of (d) Bangladesh
India Answer: D
(b) National Institution for Transforming India Explanation
(c) National India Institution for Technological • In Republic Day Prade of 2021, a marching
(d) National Institution for Trust in India contingent and band of Bangladesh tri-
Answer: B services participated in the Republic Day
Explanation Parade for the first time to commemorate
• NITI in NITI Aayog stands for “National 50 years of its historic liberation.
Institution for Transforming India”
Q104. As a routine exercise, Operations `Garam
Q101. Which one of the following is not a correct Hawa' and 'Bard Hawa' are conducted by
description of the Namami Gange Programme? (a) BSF
26 | P a g e W W W . E D U T A P . C O . I N QUERY? HELLO@EDUTAP.CO.IN / 8146207241
(b) ITBP (a) Dalai Lama
(c) CRPF (b) A.P.J. Abdul Kalam
(d) CISF (c) Ravi Shankar
Answer: A (d) Jagadish Vasudev
Explanation Answer: A
• BSF conducts “Operation Garam Hawa” Explanation
in summer and “Operation Sard Hawa” in Edited by Renuka Singh and published by Penguin
winter every year. Random House, Dalai Lama's 'The Little Book of
Encouragement' has 130 quotes with words of
Q105. Which one of the following States will be wisdom amid pandemic, promotes human values,
directly connected with Assam by the proposed spills the beans on key to happiness and
bridge between Dhubri and Phulbari over river comments on Sino-India ties
Brahmaputra?
(a) Arunachal Pradesh Q108. Which day has been declared by the
(b) Meghalaya Government of India to be celebrated every year
(c) West Bengal as “Parakram Diwas”?
(d) Mizoram (a) 23 January
Answer: B (b) 14 April
Explanation (c) 28 May
• The Dhubri - Phulbari Bridge is an under (d) 25 December
construction bridge over the Brahmaputra Answer: A
River between Assam and Meghalaya in Explanation
North-East India. Parakram Diwas (also called Parakram Divas) is
celebrated on January 23 every year. It is an Indian
Q106. Which of the following statements is/are national holiday marking the birthday of Netaji
correct? Subhas Chandra Bose
1. India is a signatory to the Ramsar Convention.
2. The Ramsar Convention is about wetland Q109. A copy of the famous 16th century art work
conservation. ‘Salvator Mundi’, which was reportedly stolen,
3. At present there are 76 Ramsar Sites in India. has recently been recovered by the Police in Italy.
Select the correct answer using the code given The painting is attributed to whom among the
below: following?
(a) 2 and 3 only (a) Vincent van Gogh
(b) 1, 2 and 3 (b) Pablo Picasso
(c) 1 only (c) Michelangelo
(d) 1 and 2 only (d) Leonardo da Vinci
Answer: D
Explanation Answer: D
• The Convention on Wetlands (Ramsar
Convention) is an intergovernmental Explanation
treaty that provides the framework for Italian police have recovered a 500-year-old copy
national action and international of Leonardo da Vinci’s 16th century “Salvator
cooperation for the conservation and Mundi” painting of Jesus Christ that was stolen
wise use of wetlands and their resources from a Naples church during the pandemic
• India is one of the Contracting Parties to without the priests even realizing it was gone.
Ramsar Convention, signed in Ramsar,
Iran, in 1971. Q110. The Global Firepower Index-2021, a
• India signed it on 1st Feb 1982. military strength ranking, placed India at which
• Number of Ramsar sites in India are 46. rank?
(a) 2nd
Q107. Who among the following is the author of (b) 3rd
the book “The Little Book of Encouragement”? (c) 4th

27 | P a g e W W W . E D U T A P . C O . I N QUERY? HELLO@EDUTAP.CO.IN / 8146207241


(d) 5th from 2021. She was the first woman and the first
African to head the WTO.
Answer: C
Q114. The Kaladan multimodal transit transport
Explanation project well connect India with which one of the
following countries?
India was ranked fourth in the Global (a) Nepal
Firepower Index 2021. (b) Myanmar
(c) Bhutan
(d) Afghanistan
Q111. Naomi Osaka won the women’s singles Answer: B
title of the Australian Open Tennis Tournament, Explanation
2021. Who among the following was defeated by The Kaladan Multi Modal Transit Transport Project
her in the finals? was jointly identified by the India and Myanmar to
(a) Venus Williams create a multi-modal mode of transport for
(b) Jennifer Brady shipment of cargo from the eastern ports of India
(c) Serena Williams to Myanmar as well as to the North-Eastern part
(d) Simona Halep of India through Myanmar.
Answer: B
Explanation Q115. Under Sukanya Samridhi Yojana, what is
Naomi Osaka defeated Jennifer Brady in the final, the maximum amount that can be deposited
6–4, 6–3 to win the women's singles tennis title at during a financial year?
the 2021 Australian Open. (a) ₹1.5 lakh
(b) ₹1 lakh
Q112. UNESCO has been observing February 21 as (c) ₹2 lakh
the International Mother Language Day since (d) ₹2.5 lakh
2000. The idea to celebrate the International Answer: A
Mother Language Day was the initiative of which Explanation
of the following nations? Sukanya Samriddhi Account Scheme
(a) Sri Lanka • Minimum deposit ₹ 250/- Maximum
(b) Bangladesh deposit ₹ 1.5 Lakh in a financial year.
(c) India • Account can be opened in the name of a
(d) Nepal girl child till she attains the age of 10 years.
Answer: B • Only one account can be opened in the
Explanation name of a girl child.
According to the UNESCO website, the day was • Account can be opened in Post offices and
established on the initiative of Bangladesh and has in authorised banks.
been observed worldwide since 1999. • Withdrawal shall be allowed for the
purpose of higher education of the
Q113. Dr. Ngozi Okonjo-Iweala was recently Account holder to meet education
selected as the first woman director general of expenses.
WTO. She hails from which of the following • The account can be prematurely closed in
countries? case of marriage of girl child after her
(a) Japan attaining the age of 18 years.
(b) Ethiopia
• The account can be transferred anywhere
(c) Nigeria
in India from one Post office/Bank to
(d) Egypt
another.
Answer: C
• The account shall mature on completion of
Explanation
a period of 21 years from the date of
Ngozi Okonjo-Iweala, is a Nigerian-American
opening of account.
economist who served as the seventh director
general of the World Trade Organization (WTO)
28 | P a g e W W W . E D U T A P . C O . I N QUERY? HELLO@EDUTAP.CO.IN / 8146207241
Q116. What is the minimum and maximum age at • The scheme has helped in identifying ‘ghost’
which a subscriber can join the Atal Pension accounts, multiple accounts and inactive
Yojana? accounts.
(a) 21 years and 58 years respectively • It has helped in curbing diversion of subsidised
(b) 18 years and 40 years respectively LPG to commercial purposes.
(c) 18 years and 50 years respectively
(d) 21 years and 60 years respectively Q119. Which one of the following is not an
Answer: B objective under Pradhan Mantri Krishi Sinchaye
Explanation yojana (PMKSY)?
(a) Providing subsidies to use fertilizer, high
• Any Citizen of India can join Atal Pension yielding varieties (HYV) and pesticides
Yojana. (b) To achieve convergence of investments in
• The age of the subscriber should be between irrigation at the field level
18 - 40 years. (c) To expand cultivable area under assured
irrigation
Q117. What is the equivalent decimal value of (d) Improving on-farm water use efficiency
binary number 101110? Answer: A
(a) 46 Explanation:
(b) 56
(c) 64
(d) 65
Answer: A
Explanation
The decimal equivalent of a binary number can be
Q120. Who among the following is the first player
obtained by multiplying powers of 2 to the binary
in the history of Test Cricket to take 200 wickets
number from right side as given below:
against left-handed batsmen?
(1x25) + (0x24) + (1x23) + (1x22) + (1x21) + (0x20)
(a) Muttiah Muralitharan
= 32 + 0 + 8 + 4 + 2 + 0
(b) James Michael Anderson
= 46
(c) Stuart Broad
Hence, (101110)2 = (46)10
(d) Ravichandran Ashwin
Hence option A is correct
Answer: D
Q118. Which one of the following is the name of Explanation
the scheme introduced as a well-targeted system • Ravichandran Ashwin became the first player
of service delivery to LPG customers ? in the history of Test cricket to take 200
(a) SAHAJ wickets against left-handed batsmen. He
(b) PAHAL achieved the feat against England.
(c) UDAY
(d) UDAN
Answer: B
Explanation
About PAHAL Scheme
• Government has introduced well targeted
system of subsidy delivery to LPG consumers
through PAHAL.
• It was aimed at rationalizing subsidies based
on approach to cut subsidy leakages, but not
subsidy per se.
• Applicable subsidy is directly transferred into
the bank account of the beneficiaries.

29 | P a g e W W W . E D U T A P . C O . I N QUERY? HELLO@EDUTAP.CO.IN / 8146207241


Visit: www.civilstap.com
For Any Query Mail us: hello@civilstap.com or call us at - (+91)-8146207241
0
(a) Strange
UPSC EPFO EO/AO - 2017 (b) Nimble
PART- A (c) Everyday
(d) Customary
Q1. Which one of the following is the synonym of
the word written in capital letters in the sentence,
Q7. Which one of the following parts of the
“The FUNDAMENTAL character of an individual
sentence, “As you know that the ignorant are
defines his entire life”?
easily duped”, has an error?
(a) Productive
(a) As you know
(b) Essential
(b) that the ignorant
(c) Successful
(c) are easily duped
(d) Effective
(d) No error
Q2. Which one of the following is the synonym of
Q8. Which one of the following parts of the
the word written in capital letters in the sentence,
sentence, “After a lot of argument I brought him
“She has the habit of using HACKNEYED and
forward to my point of view”, has an error?
redundant examples from her past even while
(a) After a lot of argument
dealing with the future generation”?
(b) I brought him forward
(a) Indifferent
(c) to my point of view
(b) Imaginative
(d) No error
(c) Clichéd
(d) Impressive
Q9. Which one of the following parts of the
sentence, ‘Don’t ask him how he is because if he
Q3. Which one of following is the synonym of the
starts talking about his health you’ll never get off
word written in capital letters in the sentence,
from him”, has an error?
“She is an IMMACULATE soul whose presence
(a) Don’t ask him how he is because
make all the difference”?
(b) If he starts talking about his health you’ll
(a) Spotless
never
(b) Excited
(c) get off from him
(c) Extraordinary
(d) No error
(d) Unparalleled
Q10. Which one of the following words can be
Q4. Which one of the following is the antonym of
used as a substitute for the phrase, “A nursery
the word written in capital letters in the sentence,
where children of working parents are cared for
“there is a great deal of ENMITY”?
while their parents are at work”?
(a) Amity
(a) Sculler
(b) Cooperation
(b) Crèche
(c) Dispute
(c) Dormitory
(d) Hostility
(d) Refectory
Q5. Which one of the following is the antonym of
Q11. Which one of the following words can be
the word written in capital letters in the sentence,
used as a substitute for the phrase, “That which
“She was RELUCTANT to sell her car”?
cannot be taken by force”?
(a) Anxious
(a) Impossible
(b) Happy
(b) Improbable
(c) Eager
(c) Impeccable
(d) Unwilling
(d) Impregnable
Q6. Which one of the following is the antonym of
Q12. Which one of the following is the appropriate
the word written in capital letters in the sentence,
question tag in the sentence, ‘Take a seat, ____”?
“I cannot comply with the QUEER regulations of
(a) shall you?
your land”?
(b) can’t you?
1|P a g e W W W . E D U T A P . C O . I N QUERY? HELLO@EDUTAP.CO.IN / 8146207241
(c) won’t you? (c) for
(d) would you? (d) inside

Q13. Which one of the following is the appropriate Q19. Which one of the following can be used as a
question tag in the sentence, “Anyone can make substitute for the words written in capital letters
mistakes, ______”? in the sentence, “There is no meaning to CRY
(a) isn’t it? OVER THE SPILT MILK since the matter is over”?
(b) shall they? (a) No use of worrying about
(c) can’t they? (b) No use of spilling now
(d) is it? (c) No use of celebration
(d) No use of asking for
Q14. Which one of the following phrasal verbs may
be used in the blank space in the sentence, “He is Q20. Which one of the following can be used as a
fond of art and seems _______ (suited) to be an substitute for the words written in capital letters
artist”? in the sentence, “Serving the sufferers is true
(a) fit out YEOMAN’S SERVICE”?
(b) bring out (a) A risky job
(c) cut out (b) An amusing job
(d) suit out (c) An excellent work
(d) An unwanted job
Q15. Which one of the following phrasal verbs may
be used in the blank space in the sentence, “We Part - B
were happy in the beginning, but now all the Q21. In case of gold, revenue is recognized in the
troubles have _______ (appeared)”? accounting period in which the gold is
(a) cropped up (a) delivered
(b) come about (b) sold
(c) come out (c) mined
(d) rushed in (d) identified to be mined

Q16. Which one the following phrasal verbs may Q22. As per the traditional approach, the expense
be used in the blank space in the sentence, “No to be matched with revenue is based on
one knows how it all _______ (happened)”? (a) original cost
(a) came across (b) opportunity cost
(b) came about (c) replacement cost
(c) came into (d) cash cost
(d) came forward
Q23. Preliminary expenses are the examples of
Q17. Which one of the following phrasal verbs may (a) capital expenditure
be used in the blank space in the sentence, “I have (b) capital gain
to _______ (finish) this work before I leave (c) deferred revenue expenditure
tomorrow morning”? (d) revenue expenditure/expense
(a) clear up
(b) clean up Q24. Depreciation of fixed assets is an example of
(c) finish up (a) deferred revenue expenditure
(d) do away with (b) capital expenditure
(c) capital gain
Q18. Which one of the following prepositions may (d) revenue expenditure/expense.
be used in the blank space in the sentence, “He
lived a hand ____ mouth existence, surviving Q25. In the context of accounting, the term IFRS
on just a few rupees a week”? stands for
(a) in (a) International Financial Reporting Standards
(b) to (b) Indian Standards
2|P a g e W W W . E D U T A P . C O . I N QUERY? HELLO@EDUTAP.CO.IN / 8146207241
Financial Reporting (c) 3,75,000
(c) Indian Financial Reporting System (d) 4,00,000
(d) International Financial Reporting System
Q32. Branch Account under Debtors System is
Q26. From the information given below, calculate (a) Real Account
the sum insurable: (b) Personal Account
• Date of fire-01.03.2016 (c) Nominal Account
• Turnover from 01.03.2015 to 29.02.2016 (d) Liability Account
88,00,000
• Agreed GP ratio-20% Q33. The cost of electric power should be
• Special circumstances clause provided for the apportioned over different departments
increase of turnover by 10% according to
(a) 19,36,000 (a) horsepower of motors
(b) 48,40,000 (b) number of light points
(c) 10,32,000 (c) horsepower multiplied by machine
(d) 24,20,000 hours
(d) machine hours
Q27. Income and Expenditure Account is
(a) Real Account Q34. Under which Schedule of the Companies
(b) Personal Account Act, 2013, the formats of financial statements are
(c) Nominal Account prescribed?
(d) Capital Account (a) Schedule I
(b) Schedule II
Q28. Legacies are generally (c) Schedule III
(a) capitalized and taken to Balance Sheet (d) Schedule IV
(b) treated as income
(c) treated as expenditure Q35. In the absence of any provision in the
(d) capitalized and taken to Suspense. partnership agreement, profits and losses are
Account shared by the partners
(a) in the ratio of the capital of partners
Q29. The abnormal loss on consignment is (b) equally
credited to (c) in the ratio of loans given by them to the
(a) Profit and Loss Account partnership firm
(b) Consignee's Account (d) in the ratio of the initial capital introduced by
(c) Consignment Account the partners
(d) Income and Expenditure Account
Q36. CD-ROM is a
Q30. When goods are purchased for the Joint (a) secondary memory
Venture, the amount is debited to (b) magnetic memory
(a) Purchase Account (c) memory register
(b) Joint Venture Account (d) semiconductor memory
(c) Venturer's Capital Account
(d) Profit and Loss Account Q37. WAP stands for
(a) Wireless Addition Protocol
Q31. Consider the following information: (b) Wireless Automation Protocol
• Rate of gross profit-25% on (c) Wireless Adaption Protocol
• cost of goods sold (d) Wireless Application Protocol
• Sales 20,00,000
Which one of the following is the Q38. Bluetooth technology allows
amount of gross profit? (a) sending of files within the range of
(a) 5,00,000 10 km
(b) 6,25,000 (b) sending an e-mail
3|P a g e W W W . E D U T A P . C O . I N QUERY? HELLO@EDUTAP.CO.IN / 8146207241
(c) wireless connection between 2. Dipa Karmakar
various devices/equipment’s over
short distances 3. P. V. Sindhu
(d) downloading Internet movies
4. Sakshi Malik
Q39. Which one among the following is not a
basic function of a computer? Select the correct answer using the code given
(a) Accept and process data below.
(b) Store data (a) 2 and 3 only
(c) Scan text (b) 2, 3 and 4 only
(d) Accept input (c) 1 and 4 only
(d) 1, 2, 3 and 4
Q40. Which one of the following is hardware?
(a) Power point Q44. ISRO in August 2016 successfully test
(b) Control unit launched Supersonic Combustion Ramjet
(c) Printer driver (Scramjet) Engine. Which of the following
(d) Operating system statements with regard to Scramjet Engine is/are
correct?
Q.41 Which one of the following is not a feature
of the Aadhaar (Targeted Delivery of Financial 1. It can efficiently operate both in subsonic and
and other Subsidies, Benefits and Services) Act, supersonic combustor modes.
2016?
2. India is the first country to demonstrate the
(a) Only citizens of India are entitled for enrolment flight testing of a Scramjet Engine.
under this Act.
Select the correct answer using the code given
(b) Both demographic and biometric information below.
is to be submitted for enrolment.
(a) 1 only
(c) The Unique Identification Authority of India is (b) 2 only
responsible for enrolment and authentication (c) Both 1 and 2
under this Act. (d) Neither 1 nor 2

(d) It is the responsibility of the Unique Q45. Startup Hubs are agreed to be set up in
Identification Authority to ensure the security of
identity of individuals. 1. IIITs

Q.42 Which one of the following bodies has 2. IISERs


formulated the 'National Student Startup Policy',
launched by the President of India in November 3. NITS
2016?
(a) UGC 4. Central Universities
(b) NCERT
(c) AIU Select the correct answer using the code given
(d) AICTE below.
(a) 1, 2 and 3
Q43. Who among the following were conferred (b) 1 and 4 only
with Rajiv Gandhi Khel Ratna Award for the year (c) 3 and 4 only
2016? (d) 1, 3 and 4

I. Jitu Rai Q46. Two vehicles which are 100 km apart are
running towards each other in a straight line. In

4|P a g e W W W . E D U T A P . C O . I N QUERY? HELLO@EDUTAP.CO.IN / 8146207241


how much time will they met each other Q51. What is the maximal number of spherical
provided they follow a uniform speed of 45 km balls of radius 1 cm each that can be placed inside
per hour and 80 km per hour respectively? a cubical box of height 10 cm?
(a) 60 minutes (a) 25
(b) 55 minutes (b) 125
(c) 48 minutes (c) 250
(d) 45 minutes (d) 1000

Q47. The price of an article is increased by 20% Q52. In a city, 80% population eat rice and 90% of
further, there is a tax of 5% on the increment. If the rice eaters are non-vegetarians. Then what
the article costs Rs. 1,331 to the customer, then per cent of the population are vegetarian rice
what was the price of the article before the eaters?
increase in price? (a) 7.2
(a) Rs 1,000 (b) 8
(b) Rs 1,064 (c) 9
(c) Rs 1,100 (d) 10
(d) Rs 1,200
Q53. Two vehicles A and B travel with uniform
Q48. A cricket bat is purchased at a 20% discount. speed 30 km per hour and 60 km per hour
If the selling price of the bat is Rs. 1,000, what respectively in the same direction. They start at
was the original price of the bat? the same time and from the same place for a
(a) Rs 1,100 distance of 120 km. The faster vehicle B reaches
(b) Rs 1,200 the destination and travels back with the same
(c) Rs 1,225 speed. Assume that the loss of time to change the
(d) Rs 1,250 direction is negligible. If x is the distance travelled
by the slower vehicle A before the tow vehicles
Q49. Which one of the following diagrams is most cross each other, then x is
appropriate to the statement “Tea-producing (a) 70 km
places are either in Assam or in Bengal”? (b) 75 km
(c) 80 km
(d) 90 km

(a) Q54. The price of a bottle of cold drink is Rs. 10.


One bottle of cold drink can also be bought by
returning 10 empty bottles. A person has Rs.
(b) 1,000 and 19 empty bottles. Assuming that the
person can consume any number of bottles he
buys, what will be the number of empty bottles
(c)
he possesses at the end if he buys the maximum
number of bottles of cold drink and consumes all?
(d) (a) 0
(b) 1
Q50. Suppose, the remainder obtained while (c) 2
dividing x by 61 is 2. What is the remainder (d) 3
obtained while dividing x7 by 61?
Q55. If the radius of the new spherical container
(a) 2 is double the radius of the old spherical
(b) 4 container, then the ratio of the volume of the
(c) 5 new container and of the old container is
(d) 6 (a) 2: 1
(b) 4: 1
(c) 8: 1
5|P a g e W W W . E D U T A P . C O . I N QUERY? HELLO@EDUTAP.CO.IN / 8146207241
(d) 2π: 1 Q60. The circumference of a circle is 2π cm. Then
the area of a square inscribed in the circle is
Q56. A container is filled with 300 litres of (a) π/2 cm²
hydrogen gas. The day it loses 100 litres of (b) 1 cm²
hydrogen gas and every day it loses one-third of (c) 2π cm²
the volume it lost in the previous day. Then the (d) 2 cm²
container
(a) loses entire hydrogen gas in 3 days Directions: The following two (2) items consist of
(b) loses entire hydrogen gas in 10 days two statements, Statement I and Statement II.
(c) loses 150 litres of hydrogen gas in 10 days Examine these two statements carefully and select
(d) possesses at least 150 litres of hydrogen gas on the correct answer using the code given below.
100
Codes:
Q57. The area of the smallest circle which (a) Both the statements are individually true and
contains a square of area 4 cm²inside is Statement II is the correct explanation of
(a) π cm² Statement I
(b) 2π cm² (b) Both the statements are individually true but
(c) 3π cm² Statement II is not the correct explanation of
(d) 4π cm² Statement I
(c) Statement I is true but Statement II is false
Q58. Consider the following figure:
(d) Statement I is false but Statement II is true

Q61. Statement I:
The force on Moon due to Earth is the action, while
the force on Earth due to Moon is the reaction.
Statement II:
Which one of the following is the image of the
To every action, there is an equal and opposite
object in the mirror?
reaction.
(a)
Q62.
Statement I:
(b) It is a common observation that if we place a glass
of ice-water on a table at room temperature, the
(c) ice-water will get warmer.
Statement II:
(d) Heat is energy that flows between a system and its
environment because of temperature difference
between them.
Q59. Three persons A, B and C run a business
together and their shares are 17%, 37% and 46% Q63. Which one of the following materials is not
respectively. Any profit they earn is distributed diamagnetic at Standard Temperature and
according to the proportion of their shares. If the Pressure (STP)?
difference of the profits of B and A on a given (a) Nitrogen
date is Rs. 1,000, what is the profit of C on that (b) Sodium chloride
day? (c) Water
(d) Iron
(a) Rs 2,300
(b) Rs 2,350
Q64. Which one of the following gases has the
(c) Rs 2,450
highest solubility in water?
(d) Rs 4,600
(a) Chlorine
(b) Ammonia
(c) Carbon dioxide
6|P a g e W W W . E D U T A P . C O . I N QUERY? HELLO@EDUTAP.CO.IN / 8146207241
(d) Bromine (c) M. N. Roy
(d) Jawaharlal Nehru
Q65. Bleaching powder contains
(a) Nitrogen Q72. In the context of international affairs, which
(b) Iodine one of the following is correct about Lord Curzon?
(c) Chlorine (a) He advocated a pacifist policy.
(d) Bromine (b) He wanted to establish a British sphere of
influence over the Persian Gulf and Seistan.
Q66. What is the causal agent of Chikungunya? (c) He wanted England and Russia to become
(a) Non-chlorophyllous bacterium friends against China.
(b) Nematode (d) He wanted to lead a flag-waving mission to the
(c) Virus USA.
(d) Fungus
Q73. Which one of the following is correct about
Q67. Bio-remediation is a technology which is Assam in British India?
being extensively utilized in controlling (a) It was a part of the North-East Frontier Agency.
(a) Global warming (b) It was made a province in 1865.
(b) Melting of glaciers (c) It was separated from Bengal in 1874, and
(c) Ozone depletion along with Sylhet made into a Chief
(d) Heavy metal pollutions Commissioner's province.
(d) It was a Princely State ruled by Ahom kings.
Q68. Beauty of some historical monuments is
greatly affected by the growth of certain living Q74. What were the peasant associations set up
organisms. These living organisms belong to in Kerala in the 1930s called?
which one of the following groups? (a) Kisan Sabha
(a) Amphibious plants (b) Kirti Kisan
(b) Lichens (c) Karshaka Sangam
(c) Bacteria (d) Kisan Morcha
(d) Viruses
Q75. Why did the Congress declare 9th May to be
'Ethiopia Day'?
Q69. Which one of the following has the
characteristics of both an animal as well as a (a) Because the Ethiopians declared their support
plant? for the cause of Indian independence from
(a) Fern British rule.
(b) Moss (b) Because Ethiopia became independent from
(c) Earthworm Britain on that day.
(d) Euglena (c) Because Ethiopia was attacked by Italy in early
1936 and their resistance was considered
Q70. In order to save the stored food grains from freedom struggle.
insects, farmers usually mix with them (d) Because Ethiopian leaders denounced
(a) Neem leaves Mussolini.
(b) Mango leaves Q76. Who set up the Bharat Stree Mahamandal
(c) Peepal leaves which first met in Allahabad in 1910?
(d) Orange leaves
(a) Annie Besant
(b) Meherbai Tata
Q71. Who is the author of the work, The (c) Saraladevi Chaudhurani
Evolution of Provincial Finance in British India: A (d) Tarabai Shinde
Study in the Provincial Decentralization of
Q77. Which one of the following is not a feature
Imperial Finance?
of the Non-Cooperation Movement?
(a) Dadabhai Naoroji
(a) Economic boycott was intense and successful.
(b) Dr. B. R. Ambedkar
7|P a g e W W W . E D U T A P . C O . I N QUERY? HELLO@EDUTAP.CO.IN / 8146207241
(b) The middle class participated in very large Q82. Which social reformer's autobiography,
numbers in the movement. entitled Looking Back, describes his experiences
(c) It was marked by uneven geographical spread in setting up schools for women in Poona in the
and regional variations. 1890s?
(d) Along with Non-Cooperation, other Gandhian (a) Dhondo Keshav Karve
social reform movements like the anti-liquor (b) K. T. Telang
campaign achieved some success. (c) Jyotirao Phule
Q78. Which one of the following publications was (d) Dr. B. R. Ambedkar
started by Khan Abdul Ghaffar Khan in 1928? Q83. Who was the first Director General of the
(a) Pakhtun Archaeological Survey of India?
(b) Khudat Khidmatgar (a) John Marshall
(c) Young India (b) Alexander Cunningham
(d) India Awakens (c) Mortimer Wheeler
Q79. Who among the following was the founder (d) Francis Buchanan
of the Arya Mahila Samaj in the early 1880s? Q84. Which of the following works was not
(a) Swami Dayananda Saraswati contributed by the famous social reformer
(b) Swami Vivekananda Begum Rokeya?
(c) Pandita Ramabai (a) Ardhangi
(d) Ramabai Ranade (b) Griha
Q80. Who among the following is considered to (c) Mother India
be the first Indian to go to jail in performance of (d) Sultana's Dream
his duty as a journalist? Q85. Which one of the following is not a
(a) Mahatma Gandhi characteristic of the Mughal chronicles?
(b) Rabindranath Tagore (a) They present a continuous chronological
(c) Lokmanya Tilak record of events.
(d) Surendranath Banerjee (b) They were a repository of factual information
Q81. Which of the following was/were the about the Mughals.
reason/reasons for the lack of economic (c) They allow us to understand how imperial
development in India in the 19th century? ideologies were created and disseminated.
(d) They contain brief biographies of the authors
I. Officially the British Government was
of the chronicles.
committed to a policy of laissez-faire, but
it was actually a policy of discriminatory Q86. Which one of the following is not a feature
intervention. of monopolistic competition?
II. European entrepreneurs had connections (a) Large number of buyers and sellers in the
to banks and agency houses, while Indians market
had to rely on kin, family, and caste men. (b) Differentiated products constitute the market
III. When plantations were transferred to (c) Product in the market is homogeneous
individual capitalist ownership, native (d) Selling costs are used for sale promotion
investors were deliberately ignored.
Select the correct answer using the code given Q87. Social cost is higher than economic cost
below. because
(a) Society is bigger than economy
(a) I only
(b) Society includes polity, while economy does
(b) II and III only
not include it
(c) I and III only
(c) Cost borne by bystanders is positive
(d) I, II, and III
(d) Society includes both consumers and
producers

8|P a g e W W W . E D U T A P . C O . I N QUERY? HELLO@EDUTAP.CO.IN / 8146207241


Q88. Cess on coal at Rs. 100 per ton is a type of Q94. Which one of the following is an exception
(a) carbon tax from the five functional types of unionism
(b) carbon subsidy identified by Robert Hoxie?
(c) carbon incentive for technology (a) Business Unionism
(d) carbon incentive for selling carbon permit (b) Predatory Unionism
(c) Revolutionary Unionism
Q89. Core inflation is different from headline (d) Evolutionary Unionism
inflation because the former
(a) Ignores articles of volatile nature in the price Q95. Which one of the following is the process in
index which representatives of workmen and employer
(b) Considers articles of volatile nature in the price involved in an industrial dispute are brought
index together before a third person or group of
(c) Is not based on commodity price index persons who facilitates/ facilitate through
(d) Considers only core items of consumption in mediation to reach a mutually satisfactory
the price index agreement?
(a) Arbitration
Q90. Who among the following was the Chairman (b) Adjudication
of the Working Group for determining the (c) Conciliation
methodology for construction of Producer Price (d) Collective negotiation
Index (PPI) in India (2014)?
(a) Saumitra Chaudhury Q96. Questions relating to the application or
(b) D.V. Subbarao interpretation of a standing order certified under
(c) Abhijit Sen the Industrial Employment (Standing Orders) Act,
(d) B.N. Goldar 1946 may be referred to
(a) Industrial Tribunal
Q91. Which one of the following countries is not (b) Labour Commissioner
a member of WTO? (c) Labour Court
(a) Japan (d) Industrial Employment Court
(b) China
(c) Iran Q97. A union whose membership may cover
(d) Russia workers employed in many industries,
employment and crafts is known as
Q92. Works Committee, Safety Committee and (a) industrial union
Canteen Management Committee are the (b) general union
examples of (c) craft union
(a) workers' participation in management (d) region-cum-industry level union
(b) workers' education schemes
(c) workers' cooperatives Q98. Which one of the following perspectives of
industrial relations is based on the assumption
(d) workers' suggestion schemes
that both the parties strive (and have
opportunity) to exercise economic (wages and
Q93. Which one of the following is not part of benefits) as well as political (control) power?
the aims and purposes of the ILO as per (a) Pluralistic perspective
Philadelphia Declaration? (b) Unitary perspective
(a) Labour is not a commodity (c) Radical perspective
(b) Freedom of expression and of association are (d) Trusteeship perspective
essential to sustained progress
(c) Poverty anywhere constitutes danger to Q99. Which one of the following is not a trade
prosperity everywhere union security measure?
(d) The war against want requires to be carried on (a) Closed shop system
with unrelenting vigour within each nation and is (b) Agency shop system
solely the responsibility of the government (c) Open shop system
9|P a g e W W W . E D U T A P . C O . I N QUERY? HELLO@EDUTAP.CO.IN / 8146207241
(d) Union shop system Q105. ''Everyone as a member of the society has
the right to social security, and is entitled to
Q100. Which one of the following is statutory realization through national efforts and
machinery functioning at the central level? international cooperation and in accordance
(a) Central Implementation and with the organisation and resources of each
Evaluation Committee state of economic, social and cultural rights
(b) Central Board for Workers' Education indispensable for his dignity and free
(c) Standing Labour Committee development of his personality." This statement
(d) Employee's State Insurance Corporation which is emphasizing the importance of social
security has been expressed in which of the
Q101. Which one of the following explains the following?
'citizen concept' of labour? (a) Universal Declaration of Human Rights
(a) Labour is largely regarded by the employers as (b) Philadelphia Declaration of the ILO
operating organizations in industry (c) Report of the First National
(b) Labour is affected by the law of demand and Commission on labour
supply. (d) Directive Principles of State Policy of the Indian
(c) Labour has a right to be consulted in regard to Constitution
the terms and conditions under which they are
supposed to work
(d) Labour is a cog in the machine Q106. For the first time in India, medical benefit
as a non-cash benefit was provided under
Q102. Who among the following can be (a) the Employees’ State Insurance Act, 1948
appointed as the Chairman of the Central (b) the Factories Act, 1948
Advisory Board constituted by the Central (c) the Maternity Benefit Act, 1961
Government under the Minimum Wages Act, (d) the Mines Act, 1952
1948?
(a) One of the independent members of the Board Q107. Which one of the following statements
(b) One of the employer’s representatives of the regarding an Overseas Citizen of India (OCI) is not
Board correct?
(c) One of the employee’s representatives of the
(a) An OCI is a citizen of another country.
Board
(b) An OCI possesses a multiple-entry long-term
(d) A functionary of the Central Government
nominated by the Government visa for visiting India.
(c) An OCI is at par with NRIs in all matters.
(d) An OCI is not entitled to the fundamental right
Q103. The assumption that ''man is selfish and to equality of opportunity in public
self-centred, and always tries to achieve his own employment.
ends even at the cost of others" explains which
theory of labour welfare? Q108. Forced labour or beggar is a violation of
(a) Placating theory which one of the following Articles of the
(b) Police theory Constitution of India?
(c) Religious theory (a) Article 16
(d) Philanthropic theory (b) Article 17
(c) Article 19
Q104. Dr. Aykroyd’s formula is associated with (d) Article 23
determination of
(a) fair wage Q109. Which one of the following statements
(b) minimum wage regarding the Constituent Assembly of India is
(c) living wage not correct?
(d) real wage (a) The Constituent Assembly was composed of
members who had been elected indirectly by
the Members of the Provincial Legislative
Assembly.
10 | P a g e W W W . E D U T A P . C O . I N QUERY? HELLO@EDUTAP.CO.IN / 8146207241
(b) The Constituent Assembly held its first sitting Q115. A Stand up enterprise can be established in
on 9th December 1946.
(c) The seats in each province were distributed 1. Farming Sector
among the three main communities—Muslim, 2. Manufacturing Sector
Sikh, and General, in proportion to their 3. Service Sector
respective populations. 4. Trading Sector
(d) The method of selection in the case of
representatives of Indian States was to be (a) 1, 2 and 4
determined by the Governor-General of India. (b) 1, 3 and 4
(c) 1, 2 and 3
Q110. Which one of the following is not a (d) 2, 3 and 4
constitutional body?
(a) The Election Commission of India Q116. StandUp India Programme envisages each
(b) The Finance Commission bank branch to give loan between Rs.10 lakh to
(c) The Official Languages Commission Rs. 100 lakh
(d) The National Commission for Women
1. To at least one SC/ST borrower
Q111. Panchayats (Extension to the Scheduled
Areas) Act, 1996 permits self-governance of 2. To at least one woman borrower
natural resources by
(a) Gram Sabha 3. To at least one rural unemployed youth
(b) Gram Panchayat borrower
(c) the Chairman of Gram Panchayat
(d) forest dwellers of a village situated in the areas Select the correct answer using the code given
mentioned in the Fifth Schedule of the below.
Constitution of India
(a) 1 only
Q112. The provision of workers' participation in
the management of industries is provided under (b) 2 and 3
(a) Article 39A of the Constitution of India
(b) Article 13A of the Constitution of India (c) 1 and 3
(c) Article 42 of the Constitution of India
(d) Article 43B of the Constitution of India (d) 1 and 2

Q113. Which one of the following comes under Q117. Which one of the following statements is
the 'State List' under the Seventh Schedule of the not correct for Atal Pension Yojana?
Constitution of India?
(a) Relief of the disabled and unemployable (a) There is guaranteed minimum monthly
(b) Regulation of labour and safety in mines pension for the subscribers ranging between
(c) Regulation and control of manufacture, Rs 1,000 and Rs 5,000 per month.
supply, and distribution of salt (b) The benefit of minimum pension would be
(d) Social security and social insurance guaranteed by the Government of India.
(c) Government of India co-contributes 50% of
Q114. Which one of the following schemes is the subscriber's contribution or Rs 1,000 per
aimed at all-round development of adolescent annum, whichever is lower.
girls in the age group of 11-18 years and making (d) It is applicable to all citizens of India aged
them self-reliant? above 40 years.

(a) RGSEAG Q118. Which one of the following is the amount


(b) IGMSY of annual premium of the Pradhan Mantri
(c) NMEW Suraksha Bima Yojana (PMSBY) for accident and
(d) RMK disability cover up to Rs 2,00,000?

11 | P a g e W W W . E D U T A P . C O . I N QUERY? HELLO@EDUTAP.CO.IN / 8146207241


(a) Rs 100
(b) Rs 50
(c) Rs 20
(d) Rs 12

Q119. Who among the following is the author of


the book, Choices of Techniques?
(a) A.K. Sen
(b) K.N. Raj
(c) V.K.R.V. Rao
(d) Sukhamoy Chakravarty

Q120. Which one of the following is the correct


set of contingencies identified by William
Beveridge in his comprehensive social security
schemes?
(a) Want, disease, ignorance, squalor and idleness
(b) Want, sickness, disability, squalor and idleness
(c) Want, disease, old age, squalor and
unemployment
(d) Disease, invalidity, old age, unemployment
and ignorance

12 | P a g e W W W . E D U T A P . C O . I N QUERY? HELLO@EDUTAP.CO.IN / 8146207241


Visit: www.civilstap.com
For Any Query Mail us: hello@civilstap.com or call us at - (+91)-8146207241
0
SOLUTIONS – PREVIOUS YEAR Q3. Which one of following is the synonym of the
QUESTIONS word written in capital letters in the sentence,
“She is an IMMACULATE soul whose presence
make all the difference”?
UPSC EPFO EO/AO - 2017 (a) Spotless
(b) Excited
PART- A (c) Extraordinary
Q1. Which one of the following is the synonym of (d) Unparalleled
the word written in capital letters in the sentence,
“The FUNDAMENTAL character of an individual Answer – (A)
defines his entire life”? Explanation: Synonym of ‘immaculate’ is
(a) Productive ‘spotless’.
(b) Essential Immaculate - Without fault or error.
(c) Successful (a) Spotless - Completely neat and clean.
(d) Effective (b) Excited - In an aroused state.
(c) Extraordinary - Beyond what is ordinary
Answer – (B) or usual; highly unusual or exceptional or
Explanation: Synonym of ‘fundamental’ is remarkable; special.
‘essential’. (d) Unparalleled - Radically distinctive and
Fundamental - Serving as an essential without equal.
component.
(a) Productive - Yielding positive results. Q4. Which one of the following is the antonym of
(b) Essential - Absolutely necessary; vitally the word written in capital letters in the sentence,
necessary. “there is a great deal of ENMITY”?
(c) Successful - Having succeeded or being (a) Amity
marked by a favourable outcome. (b) Cooperation
(d) Effective - Producing or capable of producing (c) Dispute
an intended result or having a striking effect. (d) Hostility

Answer – (A)
Q2. Which one of the following is the synonym of Explanation: Antonym of ‘enmity’ is ‘amity’.
the word written in capital letters in the sentence, Enmity - The feeling of a hostile person.
“She has the habit of using HACKNEYED and (a) Amity - A state of friendship and
redundant examples from her past even while cordiality.
dealing with the future generation”? (b) Cooperation - Active help from a person,
(a) Indifferent organization, etc.
(b) Imaginative (c) Dispute - Have a disagreement over
(c) Clichéd something.
(d) Impressive (d) Hostility - A state of deep-seated ill-will.

Answer – (C) Q5. Which one of the following is the antonym of


Explanation: Synonym of ‘hackneyed’ is ‘clichéd’. the word written in capital letters in the sentence,
Hackneyed - Repeated too often; overfamiliar “She was RELUCTANT to sell her car”?
through overuse. (a) Anxious
(a) Indifferent - Marked by a lack of interest. (b) Happy
(b) Imaginative - marked by independence (c) Eager
and creativity in thought or action. (d) Unwilling
(c) Clichéd - Repeated regularly without
thought or originality. Answer – (C)
(d) Impressive - Making a strong or vivid Explanation: Antonym of ‘reluctant’ is ‘eager’.
impression.
1|P a g e W W W . E D U T A P . C O . I N QUERY? HELLO@EDUTAP.CO.IN / 8146207241
Reluctant - Unwillingness to do something (c) to my point of view
contrary to your custom. (d) No error
(a) Anxious - Causing or fraught with or
showing anxiety. Answer – (B)
(b) Happy - Enjoying or showing or marked by Explanation: ‘bring something forward’ means to
joy or pleasure. propose a plan or a new idea which is incorrect in
(c) Eager - Having a strong or impatient wish this context. It should be ‘brought him around’
to do something. which means to ‘convince him’.
(d) Unwilling - Not disposed or inclined
toward.
Q9. Which one of the following parts of the
Q6. Which one of the following is the antonym of sentence, ‘Don’t ask him how he is because if he
the word written in capital letters in the sentence, starts talking about his health you’ll never get off
“I cannot comply with the QUEER regulations of from him”, has an error?
your land”? (a) Don’t ask him how he is because
(a) Strange (b) If he starts talking about his health you’ll
(b) Nimble never
(c) Everyday (c) get off from him
(d) Customary (d) No error

Answer – (D) Answer – (C)


Explanation: Antonym of ‘queer’ is ‘customary’. Explanation: ‘get off’ means to leave work which
Queer - Beyond or deviating from the usual or is incorrect in this context. It should be ‘get away
expected. from him’ which means to ‘leave’.
(a) Strange - Being definitely out of the
ordinary and unexpected; slightly odd or even a
bit weird. Q10. Which one of the following words can be
(b) Nimble - Moving quickly and lightly. used as a substitute for the phrase, “A nursery
(c) Everyday - Found in the ordinary course of where children of working parents are cared for
events. while their parents are at work”?
(d) Customary - Commonly used or practised; (a) Sculler
usual. (b) Crèche
(c) Dormitory
Q7. Which one of the following parts of the (d) Refectory
sentence, “As you know that the ignorant are
easily duped”, has an error? Answer – (B)
(a) As you know Explanation: Crèche: A nursery for the
(b) that the ignorant supervision of preschool children while the
(c) are easily duped parents work.
(d) No error

Answer – (B) Q11. Which one of the following words can be


Explanation: Using ‘that’ after ‘As you know’ is used as a substitute for the phrase, “That which
incorrect because using two conjunctions cannot be taken by force”?
together is superfluous. So, remove ‘that’ from (a) Impossible
the sentence. (b) Improbable
(c) Impeccable
Q8. Which one of the following parts of the (d) Impregnable
sentence, “After a lot of argument I brought him
forward to my point of view”, has an error? Answer – (D)
(a) After a lot of argument Explanation: Impregnable: Incapable of being
(b) I brought him forward overcome, challenged or refuted.
2|P a g e W W W . E D U T A P . C O . I N QUERY? HELLO@EDUTAP.CO.IN / 8146207241
Explanation: ‘Cropped up’ means to appear
suddenly or unexpectedly; happen unexpectedly.
Q12. Which one of the following is the appropriate
question tag in the sentence, ‘Take a seat, ____”? Q16. Which one the following phrasal verbs may
(a) shall you? be used in the blank space in the sentence, “No
(b) can’t you? one knows how it all _______ (happened)”?
(c) won’t you? (a) came across
(d) would you? (b) came about
(c) came into
Answer – (C) (d) came forward
Explanation: The question tag for affirmative
Imperative sentences always ‘won’t you?’. Answer – (B)
Explanation: ‘come about’ means (of an event)
come to pass (in time, so that it is real and actual
Q13. Which one of the following is the appropriate at some time).
question tag in the sentence, “Anyone can make ‘come across’ - Find unexpectedly.
mistakes, ______”? ‘come into’ - Obtain, especially accidentally.
(a) isn’t it? ‘come forward’ - Make oneself visible; take
(b) shall they? action.
(c) can’t they?
(d) is it? Q17. Which one of the following phrasal verbs may
be used in the blank space in the sentence, “I have
Answer – (C) to _______ (finish) this work before I leave
Explanation: The question tag for distributive tomorrow morning”?
pronouns each/ everyone/ everybody/ anyone/ (a) clear up
anybody always takes a plural pronoun so ‘can’t (b) clean up
they’ is correct. (c) finish up
(d) do away with
Q14. Which one of the following phrasal verbs may
be used in the blank space in the sentence, “He is Answer – (A)
fond of art and seems _______ (suited) to be an Explanation: ‘clear up’ means to finish a pending
artist”? task completely.
(a) fit out ‘clean up’ - Make oneself clean, presentable or
(b) bring out neat.
(c) cut out ‘finish up’ - Finally be or do something.
(d) suit out ‘do away with’ - Terminate, end, or take out.

Answer – (C) Q18. Which one of the following prepositions may


Explanation: To ‘cut out’ for something means to be used in the blank space in the sentence, “He
be able or suited to do it. lived a hand ____ mouth existence, surviving
on just a few rupees a week”?
(a) in
Q15. Which one of the following phrasal verbs may (b) to
be used in the blank space in the sentence, “We (c) for
were happy in the beginning, but now all the (d) inside
troubles have _______ (appeared)”?
(a) cropped up Answer – (B)
(b) come about Explanation: hand to mouth means with barely
(c) come out enough money for immediate needs.
(d) rushed in

Answer – (A)
3|P a g e W W W . E D U T A P . C O . I N QUERY? HELLO@EDUTAP.CO.IN / 8146207241
Q19. Which one of the following can be used as a • The revenue must have been realised in the
substitute for the words written in capital letters form of an asset or assets (usually cash or
in the sentence, “There is no meaning to CRY receivables) or by the extinction of a liability.
OVER THE SPILT MILK since the matter is over”? • In case of gold mining revenue is recognised in
(a) No use of worrying about the accounting period in which the gold is
(b) No use of spilling now mined and not in the period in which it is sold.
(c) No use of celebration • Hence the correct answer is option C
(d) No use of asking for
Q22. As per the traditional approach, the expense
Answer – (A) to be matched with revenue is based on
Explanation: To ‘cry over spilt milk’ means to feel (a) original cost
sorry or sad about something that has already (b) opportunity cost
happened; used to emphasize that this is not (c) replacement cost
helpful. (d) cash cost

Answer: A
Q20. Which one of the following can be used as a Explanation:
substitute for the words written in capital letters • Financial accounts can be classified into two
in the sentence, “Serving the sufferers is true types of approaches.
YEOMAN’S SERVICE”? • Firstly, according to the Traditional approach
(a) A risky job or the British approach.
(b) An amusing job • The other way is the Modern approach or the
(c) An excellent work American approach.
(d) An unwanted job • Traditional approach classifies the accounts
while Modern approach uses the Accounting
Answer – (C) equation for accounting.
Explanation: ‘Yeoman’s service’ means very • Under the Traditional approach, all the ledger
good, hard, and valuable work that someone accounts are classified as “Personal” and
does especially to support a cause, to help a “Impersonal accounts”.
team, etc. • In this approach expenses to be matched with
Part - B revenue on original cost.
• Hence the correct answer will be option A
Q21. In case of gold, revenue is recognized in the
accounting period in which the gold is
Q23. Preliminary expenses are the examples of
(a) delivered
(a) capital expenditure
(b) sold
(b) capital gain
(c) mined
(c) deferred revenue expenditure
(d) identified to be mined
(d) revenue expenditure/expense
Answer: C
Answer: C
Explanation:
Explanation:
• Deferred Revenue Expenditure is an
• Revenue is the monetary expression of the
expenditure that is revenue in nature and
aggregate of products or services, transferred
incurred during an accounting period,
by an enterprise to its customers during a
however, related benefits are to be derived in
period of time.
multiple future accounting periods.
• In determining whether a particular revenue
• These expenses are unusually large in amount
should be recognised at a particular point of
and, essentially, the benefits are not
time, regard must be made to:
consumed within the same accounting period.
• The amount involved should be capable of
• Hence the correct answer is option C
being measured objectively. It must earn
revenue.
4|P a g e W W W . E D U T A P . C O . I N QUERY? HELLO@EDUTAP.CO.IN / 8146207241
• Agreed GP ratio-20%
Q24. Depreciation of fixed assets is an example of • Special circumstances clause provided for the
(a) Deferred revenue expenditure increase of turnover by 10%
(b) Capital expenditure (a) 19,36,000
(c) Capital gain (b) 48,40,000
(d) Revenue expenditure/expense. (c) 10,32,000
(d) 24,20,000
Answer: D
Explanation: Answer: A
Explanation:
• Revenue expense is any expense incurred as • Sum insurable is an amount for which
part of normal business operations. insurance policy must be taken.
Depreciation represents the periodic, • An entrepreneur is concerned with loss of
scheduled conversion of a fixed asset into an profit due to unforeseen event.
expense as fixed asset is used during normal • So sum insurable is the amount of profit which
business transactions. is expected to be earned during the period.
• Since the asset is part of normal business • Turnover during the period 1.3.15 to 29.2.16 =
operations, depreciation is considered as Rs. 88,00,000.
revenue expense. • This turnover will increase by 10%, therefore
• Hence the correct answer will be option D. adjusted turnover = 88,00,000 × (100% + 10%)
= Rs 96,80,000.
• Gross profit % given = 20%
Q25. In the context of accounting, the term IFRS • Sum insurable = Adjusted turnover (×) Gross
stands for profit %
(a) International Financial Reporting Standards • Therefore, sum insurable = 96,80,000 × 20% =
(b) Indian Standards Rs19,36,000.
Financial Reporting • Hence the correct answer will be option A
(c) Indian Financial Reporting System
(d) International Financial Reporting System Q27. Income and Expenditure Account is
(a) Real Account
Answer: D (b) Personal Account
Explanation: (c) Nominal Account
• International Financial Reporting Standards (d) Capital Account
(IFRS) are a set of accounting rules for the
financial statements of public companies that Answer: C
are intended to make them consistent, Explanation:
transparent, and easily comparable around the • Nominal Accounts are accounts related to and
world. associated with losses, expenses, income, or
• The IFRS is issued by the International gains.
Accounting Standards Board (IASB). • Examples include a purchase account, sales
• The IFRS system is sometimes confused with account, salary A/C, commission A/C, etc
International Accounting Standards (IAS), • The nominal account is an income statement
which are the older standards that IFRS account (expenses, income, loss, profit).
replaced in 2001. • It is also known as a temporary account,
• Hence the correct answer will be option D unlike the balance sheet account (Asset,
Liability, owner’s equity), which are
permanent accounts.
Q26. From the information given below, calculate • Hence the correct answer will be option C
the sum insurable:
• Date of fire-01.03.2016 Q28. Legacies are generally
• Turnover from 01.03.2015 to 29.02.2016 (a) capitalized and taken to Balance Sheet
88,00,000
5|P a g e W W W . E D U T A P . C O . I N QUERY? HELLO@EDUTAP.CO.IN / 8146207241
(b) treated as income • When purchases are made for the joint
(c) treated as expenditure venture out of joint bank account.
(d) capitalized and taken to Suspense. • The joint venture account is debited as it is an
Account expenditure incurred by the joint venture
through its bank account.
Answer: Option A • So, the purchases will be directly charged to
Explanation: the joint venture account.
• Legacies are assets and liabilities transferred • Hence the correct answer will be option B
as a result of the operation of a will.
• This means legacies being assets or liabilities is Q31. Consider the following information:
not in the nature of a nominal account, they • Rate of gross profit-25% on
can either be in the nature of a personal • cost of goods sold
account or a real account. • Sales 20,00,000
• All accounts in the nature of real and personal Which one of the following is the
are taken to balance sheet. amount of gross profit?
• Therefore, closing balances in legacies account (a) 5,00,000
are transferred to the balance sheet only and (b) 6,25,000
NOT transferred to income and expenditure. (c) 3,75,000
• Hence the correct answer option A. (d) 4,00,000

Q29. The abnormal loss on consignment is Answer: Option D


credited to Explanation:
(a) Profit and Loss Account • Sales = Cost of goods sold (COGS) + Profit
(b) Consignee's Account • Let us assume that COGS = 100
(c) Consignment Account • It means profit = 25 (being 25% of 100)
(d) Income and Expenditure Account • Therefore sales =125 (100+25)
• We are given sales=Rs 20,00,000.
Answer: Option A • If sales of Rs. 20,00,000 = 125
Explanation: • Then the value of 25 should be
• Profit = (20,00,000/125) × 25 = Rs. 4,00,000
• The abnormal loss is avoidable in nature and
• Hence the correct answer will be option D
generally arises due to reasons like fire, theft,
accident or flood etc.
• In consignment, the value of abnormal loss is Q32. Branch Account under Debtors System is
charged to profit and loss account and not to (a) Real Account
consignment account. (b) Personal Account
• The consignment account, in fact, is given a (c) Nominal Account
credit for the value of abnormally lost units so (d) Liability Account
that true profit or loss of consignment can be
computed. Answer: Option C
• Hence the correct answer will be option A Explanation:

Q30. When goods are purchased for the Joint • Debtor System of accounting is suitable for
Venture, the amount is debited to the small-size branches.
(a) Purchase Account • Under this, a Branch Account is opened for
(b) Joint Venture Account each branch in the head office ledger.
(c) Venturer's Capital Account
• All transactions are recorded in this account.
(d) Profit and Loss Account
• The Branch Account is prepared in such a way
that it discloses the profit or loss of the
Answer: Option B
branch.
Explanation:

6|P a g e W W W . E D U T A P . C O . I N QUERY? HELLO@EDUTAP.CO.IN / 8146207241


• A branch Account is a Nominal Account in (d) in the ratio of the initial capital introduced by
nature. the partners
• Head office may send goods to branch either
at “Cost price” or “Selling price”. Journal Answer: Option B
Entries Under Debtor Method. Explanation:
• Hence the correct answer will be option C • A partnership agreement is a contract
between partners in a partnership which sets
Q33. The cost of electric power should be out the terms and conditions of the
apportioned over different departments relationship between the partners.
according to • It includes Percentages of ownership and
(a) horsepower of motors distribution of profits and losses.
(b) number of light points • Description of management powers and duties
(c) horsepower multiplied by machine of each partner.
hours • But In the absence of any provision in the
(d) machine hours partnership agreement, profit and losses are
shared equally. Hence option 2 is correct.
Answer: Option C
Explanation: Q36. CD-ROM is a
• Cost is apportioned based on the factor which (a) Secondary memory
is most related to the cost. (b) Magnetic memory
• The cost of electric power should be (c) Memory register
apportioned in the proportion of electricity (d) Semiconductor memory
used.
• Therefore, Electricity used = Power required Answer: Option A
by the machine × the time for which the Explanation
machine was operated.
• Hence the correct answer will be option C • CD-ROM stands for Compact Disc-Read Only
Memory.
• It is used for storage of Secondary memory or
Q34. Under which Schedule of the Companies secondary storage. Other secondary storage
Act, 2013, the formats of financial statements are devices include magnetic disks, magnetic
prescribed? tapes etc.
(a) Schedule I • Secondary storage cannot be processed
(b) Schedule II directly by the CPU as it must first be copied
(c) Schedule III into primary storage.
(d) Schedule IV • Magnetic memory is the main way to store
data on a magnetic medium.
Answer: Option C • Memory Register that either store the
Explanation: memory address from which data will be
• Schedule III of the Companies Act 2013, collected to the CPU, or the address to which
provides the format of financial statements of data will be sent and stored.
companies complying with Accounting • Semiconductor memory is a class of computer
Standards (AS) and Ind AS memory devices consisting of one or more
• Hence the correct answer will be option C integrated circuits.
• Hence the correct answer will be option A
Q35. In the absence of any provision in the
partnership agreement, profits and losses are Q37. WAP stands for
shared by the partners (a) Wireless Addition Protocol
(a) in the ratio of the capital of partners (b) Wireless Automation Protocol
(b) equally (c) Wireless Adaption Protocol
(c) in the ratio of loans given by them to the (d) Wireless Application Protocol
partnership firm
7|P a g e W W W . E D U T A P . C O . I N QUERY? HELLO@EDUTAP.CO.IN / 8146207241
(b) Control unit
Answer: Option D (c) Printer driver
Explanation (d) Operating system
• Wireless Application Protocol (WAP) is a
specification for a set of communication Answer: C
protocols to standardize the way wireless Explanation
devices, such as mobile phones and radio
transceivers, can be used for internet access, • Hardware: The physical part of the computer
including email, the web, newsgroups and which we touch and feel is called hardware.
instant messaging. ➢ Input Device: The device that is used to
• Hence the correct answer will be option D take input data. Example- Keyboard,
Mouse, Microphones, etc.
Q38. Bluetooth technology allows ➢ Output Devices: The device that is used to
(a) sending of files within the range of 10 km output the result. Example- Printer,
(b) sending an e-mail Speakers, Monitors, etc.
(c) wireless connection between various • Software: Software is a set of
services/equipment’s over short distances instructions/tasks that the computer has to
(d) downloading Internet movies perform.
➢ Application Software: It is software used
Answer: Option C to perform tasks requested by users such
Explanation as word processing and web browsers.
• The correct answer is Wireless connection ➢ System Software: It is software designed
between various devices/equipment over to run computer hardware and
short distances. applications programs. It includes
• Bluetooth waves don't travel very far and keep programs like-Operating Systems,
switching frequencies constantly. Language Translators, etc.
• The maximum connectivity range of a
Bluetooth device is of about 30 feet. Q41. Which one of the following is not a feature
• Hence the correct answer will be option C of the Aadhaar (Targeted Delivery of Financial
and other Subsidies, Benefits and Services) Act,
Q39. Which one among the following is not a 2016?
basic function of a computer?
(a) Accept and process data (a) Only citizens of India are entitled for enrolment
(b) Store data under this Act.
(c) Scan text
(d) Accept input (b) Both demographic and biometric information
is to be submitted for enrolment.
Answer: C
Explanation (c) The Unique Identification Authority of India is
• The basic function of the computer involves - responsible for enrolment and authentication
➢ Accepting input of data through input under this Act.
devices,
➢ Processing of data, (d) It is the responsibility of the Unique
➢ storing of data through memory units, Identification Authority to ensure the security of
➢ displaying the output of the information. identity of individuals.
Answer: A
• Scanning of text is not a basic function of the
Explanation
computer, scanning of text is done through
• Aadhaar (Targeted Delivery of Financial
hardware called a scanner.
and other Subsidies, Benefits and
• Hence the correct answer will be option C
Services) Act, 2016
• Eligibility:- Every resident shall be entitled
Q40. Which one of the following is hardware?
to obtain an Aadhaar number. A resident is
(a) Power point
8|P a g e W W W . E D U T A P . C O . I N QUERY? HELLO@EDUTAP.CO.IN / 8146207241
a person who has resided in India for 182 2. Dipa Karmakar
days, in the one year preceding the date of
application for enrolment for Aadhaar. 3. P. V. Sindhu
• The key functions of the UID authority
include, 4. Sakshi Malik
o specifying demographic and biometric
information to be collected during Select the correct answer using the code given
enrolment below.
o assigning Aadhaar numbers to (a) 2 and 3 only
individuals (b) 2, 3 and 4 only
o authenticating Aadhaar numbers, and (c) 1 and 4 only
o specifying the usage of Aadhaar (d) 1, 2, 3 and 4
numbers for delivery of subsidies and Answer: D
services. Explanation
• Authentication: Unique Identification • Winners of Rajiv Gandhi Khel Ratan
Authority will authenticate the Aadhar Award 2016
number of an individual • Ms. P.V. Sindhu - Badminton
• It is the responsibility of the Unique • Ms. Dipa Karmakar - Gymnastics
Identification Authority to ensure the • Shri Jitu Rai – Shooting
security of identity of individuals. • Ms. Sakshi Malik - Wrestling

Q42. Which one of the following bodies has


formulated the 'National Student Startup Policy', Q44. ISRO in August 2016 successfully test
launched by the President of India in November launched Supersonic Combustion Ramjet
2016? (Scramjet) Engine. Which of the following
(a) UGC statements with regard to Scramjet Engine is/are
(b) NCERT correct?
(c) AIU
(d) AICTE 1. It can efficiently operate both in subsonic and
Answer: D supersonic combustor modes.
Explanation
• The National Student Startup Policy, 2. India is the first country to demonstrate the
formulated by AICTE. flight testing of a Scramjet Engine.
• It aims to create 100,000 technology based
student start-ups and a million Select the correct answer using the code given
employment opportunities within the next below.
10 years.
• The policy plans on achieving this by (a) 1 only
developing an ideal entrepreneurial (b) 2 only
ecosystem and promoting strong inter- (c) Both 1 and 2
institutional partnerships among technical (d) Neither 1 nor 2
institutions. Answer: A
• It emphasizes the much-desired need for Explanation
an appropriate startup policy to propel the • A scramjet engine is an improvement over
youth of India through and beyond the the ramjet engine as it efficiently operates
21st century. at hypersonic speeds and allows
supersonic combustion. Thus it is known
Q43. Who among the following were conferred as Supersonic Combustion Ramjet, or
with Rajiv Gandhi Khel Ratna Award for the year Scramjet.
2016? • India is the fourth country to demonstrate
the flight testing of a Scramjet Engine.
I. Jitu Rai
9|P a g e W W W . E D U T A P . C O . I N QUERY? HELLO@EDUTAP.CO.IN / 8146207241
Q45. Startup Hubs are agreed to be set up in ⇒ x = 100/125
⇒ x = 4/5 hrs
1. IIITs ⇒ x = 4/5 × 60
⇒ x = 48 min
2. IISERs ∴ Both vehicles meet each other after 48 min.

3. NITS Q47. The price of an article is increased by 20%


further, there is a tax of 5% on the increment. If
4. Central Universities the article costs Rs. 1,331 to the customer, then
what was the price of the article before the
Select the correct answer using the code given increase in price?
below. (a) Rs 1,000
(a) 1, 2 and 3 (b) Rs 1,064
(b) 1 and 4 only (c) Rs 1,100
(c) 3 and 4 only (d) Rs 1,200
(d) 1, 3 and 4 Answer: C
Answer: A Explanation:
Explanation Price of article increase by = 20%
The Ministry of Human Resource Development Rate of tax on-increment = 5%
and the Department of Science and Technology Customer buys the article = Rs. 1331
have agreed to partner in an initiative to set up If a number increased by x%, then
startup support hubs in the Indian Institutes of Increase number = Actual number × (100 + x)/100
Information Technology (I IIT's), the Indian Percentage of tax = 20% × 5/100 = 1%
Institutes of Science Education and Research So percentage of price increase by = 20% + 1% =
(IISER's), the National Institutes ofTechnology 21%
(NIT's),and National Institutes of Pharmaceutical Let price of the article before be x,
Education and Research (NIPER's). According to the question
x × 121/100 = 1331
Q46. Two vehicles which are 100 km apart are ⇒ x = 1331 × 100/121
running towards each other in a straight line. In ⇒ x = 1100
how much time will they met each other ∴ Price of the article before was Rs. 1100.
provided they follow a uniform speed of 45 km
per hour and 80 km per hour respectively? Q48. A cricket bat is purchased at a 20% discount.
(a) 60 minutes If the selling price of the bat is Rs. 1,000, what
(b) 55 minutes was the original price of the bat?
(c) 48 minutes (a) Rs 1,100
(d) 45 minutes (b) Rs 1,200
Answer: C (c) Rs 1,225
Explanation: (d) Rs 1,250
Two vehicles which are apart from each other = Answer: D
100 km Explanation:
Speed of first vehicle = 45 km/hr Discount percentage = 20%
Speed of second vehicle = 80 km/hr The selling price of the bat = Rs. 1,000
If the speed of two trains are x km/hr and y km/hr MP = SP × 100/(100 – Discount%)
respectively, then ∴ MP of the article = 1000 × 100/80 = Rs. 1250
Relative speed, if both train running opposite
directions = (x + y) km/hr Q49. Which one of the following diagrams is most
Speed = Distance/Time appropriate to the statement “Tea-producing
Relative speed if both vehicles running opposite places are either in Assam or in Bengal”?
directions = 45 + 80 = 125 km/hr
Let both vehicles meet each other after x hrs.
125 = 100/x

10 | P a g e W W W . E D U T A P . C O . I N QUERY? HELLO@EDUTAP.CO.IN / 8146207241


(d) 1000
Answer: B
Explanation:
(a) Side of cubical box = 10 cm
Radius of each ball = 1 cm
Diameter of each ball = 2 × 1 = 2 cm
(b) Number of balls arranged with respect to the
length of box = 10/2 = 5
Number of balls arranged with respect to the
(c)
breadth of box = 10/2 = 5
Number of balls arranged with respect to the
(d) height of box = 10/2 = 5
Answer: D ∴ Required number of balls = 5 × 5 × 5 = 125
Explanation:
Statement “Tea-producing places are either in Q52. In a city, 80% population eat rice and 90% of
Assam or in Bengal” implies that there is no state the rice eaters are non-vegetarians. Then what
other than Assam and Bengal that are producing per cent of the population are vegetarian rice
Tea. eaters?
Here rectangle is representing the states and oval (a) 7.2
represents tea producing places. (b) 8
(c) 9
(d) 10
Answer: B
Explanation:
Percentage of population who eat rice = 90%
x% of a number = Actual number × x/100
Let total number of population of city be 100, then
Q50. Suppose, the remainder obtained while Number of people who eat rice = 100 × 80/100 =
dividing x by 61 is 2. What is the remainder 80
obtained while dividing x7 by 61? Number of people who are non-vegetarian = 80 ×
90/100 = 72
(a) 2 Number of vegetarian people who eat rice = 80 –
(b) 4 72 = 8
(c) 5 ∴ Required percentage = 8/100 × 100 = 8%
(d) 6
Answer: D Q53. Two vehicles A and B travel with uniform
Explanation: speed 30 km per hour and 60 km per hour
The remainder obtained while dividing x by 61 is 2. respectively in the same direction. They start at
Let the number be 63, the same time and from the same place for a
According to the question distance of 120 km. The faster vehicle B reaches
637/61 the destination and travels back with the same
As we know, 63 – 61 = 2, speed. Assume that the loss of time to change the
27/61 direction is negligible. If x is the distance travelled
⇒ 128/61 by the slower vehicle A before the tow vehicles
When we divide 128 by 61 we get remainder 6. cross each other, then x is
(a) 70 km
Q51. What is the maximal number of spherical (b) 75 km
balls of radius 1 cm each that can be placed inside (c) 80 km
a cubical box of height 10 cm? (d) 90 km
(a) 25 Answer: C
(b) 125 Explanation:
(c) 250 Speed of slower vehicle = 30 km/hr
11 | P a g e W W W . E D U T A P . C O . I N QUERY? HELLO@EDUTAP.CO.IN / 8146207241
Speed of faster vehicle = 60 km/hr (c) 8: 1
Speed between two places = 120 km (d) 2π: 1
If the speed of two vehicles is x km/hr and y km/hr Answer: C
respectively, then in 1 hour distance covered by Explanation:
two vehicles is (x + y) km Volume of sphere = (4/3) πr³
Speed = Distance/Time Let the radius of old spherical container be 1 cm
Total distance covered by both vehicles until they Radius of new spherical container = 2 cm
meet each other = 2 × 120 = 240 km Volume of old spherical container = (4/3) π × 1³=
Total distance covered by both vehicles in 1 hour (4/3) π × 1
= 30 + 60 = 90 km Volume of new spherical container = (4/3) π × 2³=
Let both vehicles meet each other after x hrs (4/3) π × 8
According to the question ∴ Required ratio = (4/3) π × 8 : (4/3) π × 1 = 8 : 1
90 = 240/x
⇒ x = 240/90 Q56. A container is filled with 300 litres of
⇒ x = 8/3 hrs hydrogen gas. The day it loses 100 litres of
∴ Distance covered by slower vehicles in 8/3 hrs = hydrogen gas and every day it loses one-third of
8/3 × 30 = 80 km. the volume it lost in the previous day. Then the
container
Q54. The price of a bottle of cold drink is Rs. 10. (a) loses entire hydrogen gas in 3 days
One bottle of cold drink can also be bought by (b) loses entire hydrogen gas in 10 days
returning 10 empty bottles. A person has Rs. (c) loses 150 litres of hydrogen gas in 10 days
1,000 and 19 empty bottles. Assuming that the (d) possesses at least 150 litres of hydrogen gas on
person can consume any number of bottles he 100
buys, what will be the number of empty bottles Answer: D
he possesses at the end if he buys the maximum Explanation:
number of bottles of cold drink and consumes all? The total capacity of gas = 300 litres
(a) 0 First-day gas lost = 100 L
(b) 1 Second day, gas lost = 100/3 = 33.33L
(c) 2 Third day, gas lost = 33.33/3 = 11.11 L = 100/3² L
(d) 3 Fourth day, gas lost = 11.11/3 = 3.7L = 100/3³ L
Answer: C Fifth day, gas lost = 3.7/3 = 1.23L
Explanation: Sixth day, gas lost = 1.23/4 = 0.41L
Price of one bottle of cold drink = Rs. 10 Seventh-day, gas lost = 0.41/3 = 0.137L
Persons has Rs. 1000 and 19 bottles. Eighth day, gas lost = 0.137/3 = 0.045L
The number of bottles of cold drink he bought for Ninth day, gas lost = 0.045/3 = 0.015L
Rs. 1000 = 1000/10 = 100 Tenth day, gas lost = 0.015/3 = 0.005L
Number of bottles of cold drink he bought from Total gas loss is equal to 149.982L
returning 100 empty bottles = 100/10 = 10 On 100Th day = 100/399 L
Number of bottles of cold drink he bought from Total lost in 100 day = 100 + 100/3 + 100/3² +....
returning 10 empty bottles = 10/10 = 1 + 100/399
Number he has total number of empty bottles = 19 = 100 [(1+ 1/3 + 1/3² + 1/3³)/(1-(1/3))] = 150 [1 -
+ 1 = 20 (1/3100)]
He bought number of bottles from returning 20 ∴ possesses at least 150 litres of hydrogen gas on
empty bottles = 20/10 = 2 100th day
Now he has only 2 empty bottles.
Q57. The area of the smallest circle which
Q55. If the radius of the new spherical container contains a square of area 4 cm²inside is
is double the radius of the old spherical (a) π cm²
container, then the ratio of the volume of the (b) 2π cm²
new container and of the old container is (c) 3π cm²
(a) 2: 1 (d) 4π cm²
(b) 4: 1 Answer: B
12 | P a g e W W W . E D U T A P . C O . I N QUERY? HELLO@EDUTAP.CO.IN / 8146207241
Explanation: Three persons A, B and C run a business together
Area of the square = 4 cm² and their shares are 17%, 37% and 46%
Area of the square = a² respectively
Diagonal of the square = √2 a The difference between the profits of B and A = Rs.
Area of the circle = πr² 1,000
Side of the square = √4 = 2 cm Profit = Time × Money
Diagonal of the square = 2 √2 Profit ratio of A, B and C = 17x: 37x: 46x
The diameter of the circle = Diagonal of the square According to the question
= 2√2 37x – 17x = 1000
Radius of the circle = 2√2/2 = √2 ⇒ 20x = 1000
∴ Area of the circle = π × (√2)² = 2π ⇒ x = 1000/20
⇒ x = 50
Q58. Consider the following figure: ∴ Profit share of C = 50 × 46 = Rs. 2300

Q60. The circumference of a circle is 2π cm. Then


the area of a square inscribed in the circle is
(a) π/2 cm²
Which one of the following is the image of the (b) 1 cm²
object in the mirror? (c) 2π cm²
(d) 2 cm²
(a)
Answer: D
(b) Explanation:
The circumference of a circle = 2π
(c) Circumference of the circle = 2 πr
Area of the square = a²
(d) Area of the square = (Diagonal)² /2
Answer: A 2πr = 2 π
Explanation: ⇒ r = 2π/2π
Mirror image of the given figure is as follows: ⇒ r = 1 cm
Diameter of the circle = 2 cm
Diameter of the circle = Diagonal of the square = 2
∴ Area of the square = 2²/2 = 4/2 = 2 cm²

Directions: The following two (2) items consist of


two statements, Statement I and Statement II.
Examine these two statements carefully and select
Q59. Three persons A, B and C run a business
the correct answer using the code given below.
together and their shares are 17%, 37% and 46%
respectively. Any profit they earn is distributed
Codes:
according to the proportion of their shares. If the
(a) Both the statements are individually true and
difference of the profits of B and A on a given
Statement II is the correct explanation of
date is Rs. 1,000, what is the profit of C on that
Statement I
day?
(b) Both the statements are individually true but
(a) Rs 2,300 Statement II is not the correct explanation of
(b) Rs 2,350 Statement I
(c) Rs 2,450 (c) Statement I is true but Statement II is false
(d) Rs 4,600 (d) Statement I is false but Statement II is true
Answer: A
Explanation: Q61. Statement I:

13 | P a g e W W W . E D U T A P . C O . I N QUERY? HELLO@EDUTAP.CO.IN / 8146207241


The force on Moon due to Earth is the action, while which warms up. Hence Statement I is correct
the force on Earth due to Moon is the reaction. and Statement II is the correct explanation of
Statement II: Statement I
To every action, there is an equal and opposite
reaction. Q63. Which one of the following materials is not
Answer: A diamagnetic at Standard Temperature and
Explanation: Pressure (STP)?
Newton’s Law of Universal Gravitation states that (a) Nitrogen
every particle attracts every other particle in the (b) Sodium chloride
universe with force directly proportional to the (c) Water
product of the masses and inversely proportional (d) Iron
to the square of the distance between them. Answer: D
• Also, Newton’s third Law of Motion states that Explanation:
to every action, there is an equal and opposite The origin of magnetism lies in the orbital and spin
reaction. Hence Statement II is correct. motions of electrons and how the electrons
• Similarly, the earth attracts the moon and the interact with one another. The best way to
moon attracts the earth with the same amount introduce the different types of magnetism is to
of force as per the Newton’s Law of describe how materials respond to magnetic
Gravitation. fields. There are various types of magnetic
• Now, if earth’s gravitational force is materials which are described below:
considered the action force, moon’s • Paramagnetic materials - The materials which
gravitational pull is the reaction force. Hence are not strongly attracted to a magnet are
Statement I is correct. known as paramagnetic material. For example:
• These two forces are equal in magnitude and Aluminium, Tin, Magnesium etc.
act in opposite direction as per Newton’s third • Diamagnetic materials - The materials which
law of motion. are repelled by a magnet such as Zinc.
• Thus, the Earth’s action force is balanced by Mercury, Lead, sulphur, Nitrogen, Copper,
moon’s reaction force. Hence Statement II is Silver, Bismuth, Wood, Sodium chloride and
the correct explanation of Statement I most salts, Water etc., are known as
Hence option A is correct diamagnetic materials. Hence options (a), (b)
and (c) represent diamagnetic materials.
Q62. • Ferromagnetic materials - The materials which
Statement I: are strongly attracted by a magnetic field or
It is a common observation that if we place a glass magnet is known as ferromagnetic material for
of ice-water on a table at room temperature, the eg: Iron, Steel, Nickel, Cobalt etc. Hence
ice-water will get warmer. option (d) is a ferromagnetic material and not
Statement II: a diamagnetic material. Hance option D is
Heat is energy that flows between a system and its correct.
environment because of temperature difference
between them. Q64. Which one of the following gases has the
Answer: A highest solubility in water?
Explanation: (a) Chlorine
Heat is the form of energy that is transferred from (b) Ammonia
one body to another as the result of a difference (c) Carbon dioxide
in temperature. Hence Statement II is correct. (d) Bromine
• If two bodies at different temperatures are Answer: B
brought together, energy is transferred—i.e., Explanation:
heat flows from the hotter body to the colder. Ammonia is a colorless gas with a chemical
• Thus, if ice-water is placed on a table at room formula. Due to hydrogen bonding with water, it is
temperature (which is higher than the most soluble in water and is polar as well. The
temperature of ice-water), heat energy flows solubility (Grams of gas dissolved in 100g of water
from the objects in the room into ice-water at STP) is given below:
14 | P a g e W W W . E D U T A P . C O . I N QUERY? HELLO@EDUTAP.CO.IN / 8146207241
• Chikungunya is a disease transmitted to
humans by mosquitoes in Africa, Asia, and the
Americas; sporadic outbreaks have been
reported in other regions.
• Dengue and Zika have similar symptoms to
chikungunya, making chikungunya easy to
misdiagnose.
• Chikungunya causes fever and severe joint
pain, which is often debilitating and varies in
duration; other symptoms include joint
swelling, muscle pain, headache, nausea,
fatigue and rash.
• There is currently no approved vaccine or
Hence option B is correct. specific treatment for chikungunya virus
infections.
Q65. Bleaching powder contains
(a) Nitrogen Q67. Bio-remediation is a technology which is
(b) Iodine being extensively utilized in controlling
(c) Chlorine (a) Global warming
(d) Bromine (b) Melting of glaciers
Answer: C (c) Ozone depletion
Explanation: (d) Heavy metal pollutions
Bleaching powder Answer: D
It is produced by the action of chlorine on dry Explanation:
slaked lime [Ca(OH)2]. Bleaching powder is Bioremediation is a technology that employs the
represented as CaOCl2. Thus it consists of Calcium. use of living organisms, like microbes and bacteria,
in the removal of contaminants, pollutants, and
toxins from soil, water, and other environments.

Bleaching powder is used • Bioremediation may be used to clean up


• for bleaching cotton and linen in the textile contaminated groundwater or environmental
industry, for bleaching wood pulp in paper problems, such as oil spills, heavy metal
factories and for bleaching washed clothes in pollution etc.
laundry; • Bioremediation can have indirect role in
• as an oxidising agent in many chemical controlling Global warming and melting of
industries; and glaciers, and Ozone depletion, by maintaining
• for disinfecting drinking water to make it free environment clean but has no direct role.
of germs. Hence options (a), (b) and (c) are incorrect.
Hence option C is correct.
Hence option D is correct.
Q66. What is the causal agent of Chikungunya?
(a) Non-chlorophyllous bacterium Q68. Beauty of some historical monuments is
(b) Nematode greatly affected by the growth of certain living
(c) Virus organisms. These living organisms belong to
(d) Fungus which one of the following groups?
Answer: D (a) Amphibious plants
Explanation: (b) Lichens
Chikungunya is a mosquito-borne viral disease (c) Bacteria
caused by the chikungunya virus (CHIKV), an RNA (d) Viruses
virus in the alphavirus genus of the family Answer: B
Togaviridae. Hence option C is correct. Explanation:

15 | P a g e W W W . E D U T A P . C O . I N QUERY? HELLO@EDUTAP.CO.IN / 8146207241


Lichens are a complex life form that is a symbiotic (b) Mango leaves
partnership of two separate organisms, a fungus (c) Peepal leaves
and an algae. A lichen is not an independent (d) Orange leaves
organism, rather it co-exists with a fungi or algae Answer: A
partner Explanation:
• Lichens grow naturally on all substrates, The neem is a tropical evergreen tree traditionally
including very nutrient-poor ones. The reason well known for its medicinal value.
for this is that they do not take their nutrients • Stored grains are prone to attack by rodents,
from the substrate, but mostly from the air and insects and pathogens like fungi. Neem leaves
ambient water. Therefore, they can survive on have a bitter fragrance and some alkaloids
nutrient-poor substrates including surfaces of that repel them. Thus, they are used to save
monuments. the stored food grains from insects. Hence
• Some lichens excrete acids which may affect option A is correct.
the monuments and spoil them. • Mango leaves have antibacterial properties
Hence option B is correct. that help treat bacterial skin infections such as
staph Infections and skin burns. They have
Q69. Which one of the following has the religious importance and are part of rituals of
characteristics of both an animal as well as a most Hindu festivals.
plant? • Traditionally, the leaf juice of the peepal tree
(a) Fern may be helpful for cough, asthma, diarrhoea,
(b) Moss ear pain, toothache, haematuria (blood in
(c) Earthworm urine), migraine, scabies, eye troubles, and
(d) Euglena gastric problems.
Answer: D • Orange leaves have high levels of vitamin C
Explanation: and other antioxidants, which help boost the
Certain organisms of the Kingdom Protista have immune system and fortify the body against
the characteristics of both an animal as well as a diseases.
plant.
• All single-celled eukaryotes are placed under
Protista, but the boundaries of this kingdom Q71. Who is the author of the work, The
are not well defined. Evolution of Provincial Finance in British India: A
• Majority of Euglenoids are fresh water Study in the Provincial Decentralization of
organisms found in stagnant water. Instead of Imperial Finance?
a cell wall, they have a protein rich layer called (a) Dadabhai Naoroji
pellicle which makes their body flexible. They (b) Dr. B. R. Ambedkar
have two flagella, a short and a long one. (c) M. N. Roy
Though they are photosynthetic in the (d) Jawaharlal Nehru
presence of sunlight (which is a characteristic Answer: B
of a plant), when deprived of sunlight they Explanation:
behave like heterotrophs by predating on • The book named "The Evolution of Provincial
other smaller organisms (which is a Finance in British India: A Study in the
characteristic of an animal). Interestingly, the Provincial Decentralization of Imperial
pigments of euglenoids are identical to those Finance" was written by B.R Ambedkar.
present in higher plants. Example: Euglena. Hence option B is the correct answer.
Hence option D is correct. • It was originally published in 1924. This book
• Mosses and Ferns belong to plant kingdom deals with the following Chapters: Provincial
while earthworm belongs to animal kingdom. Finance: Its Origin, Development, Mechanism
Hence options A, B and C are incorrect. and Under the Government of India Act of
1919.
Q70. In order to save the stored food grains from • Bhimrao Ambedkar was born on 14 April 1891
insects, farmers usually mix with them in Madhya Pradesh. He was the architect of
(a) Neem leaves the Indian Constitution. He was a well-known
16 | P a g e W W W . E D U T A P . C O . I N QUERY? HELLO@EDUTAP.CO.IN / 8146207241
politician and an eminent jurist. Ambedkar's • He encouraged British-India trade with
efforts to eradicate social evils like Persia.
untouchability and caste restrictions were • He started the Quetta-Seistan trade route and
remarkable. He was posthumously awarded appointed officials as "Resident" to watch
the Bharat Ratna, India's highest civilian British interest in Persia. Hence Option B is
honour in 1990. the correct answer.
• Some other notable books and their authors: • Later, he also adopted an aggressive foreign
➢ "Poverty and Un-British Rule in India" by policy against countries such as Persia, Tibet,
Dadabhai Naoroji in 1901 in which he had and Afghanistan.
given the Drain Theory. According to this • He sent a military expedition under Major
theory, Imperial Britain was draining away Douglas to Persia to mitigate the ever-
India's wealth through exploitative growing presence of Russia, France, Germany,
economic policies. and Turkey in the Persian Gulf region.
➢ "The Discovery of India" was written by
Jawaharlal Nehru during his Q73. Which one of the following is correct about
imprisonment at Ahmednagar fort for Assam in British India?
participating in the Quit India Movement (a) It was a part of the North-East Frontier
(1942 - 1946). The book started from Agency.
ancient history, Nehru wrote at length of (b) It was made a province in 1865.
Vedas, Upanishads, and textbooks on (c) It was separated from Bengal in 1874, and
ancient times and ended during the along with Sylhet made into a Chief
British raj. The book is a broad view of Commissioner's province.
Indian history, culture, and philosophy. (d) It was a Princely State ruled by Ahom kings.
➢ M. N. Roy was a twentieth-century Indian Answer: C
philosopher. He began his career as a Explanation:
militant political activist and left India in • In 1824 Assam was occupied by British forces
1915 in search of arms for organizing an following the First Anglo-Burmese War and
insurrection against British rule in India. on 24 February 1826, it was ceded to Britain
According to M. N. Roy, his books by Burma under the Yandaboo Treaty of 1826.
"Scientific Politics" (1942) along with • Between 1826 and 1832 Assam was made
"New Orientation" (1946), and "Beyond part of Bengal under the Bengal Presidency.
Communism" (1947) constitute the • From 1832 to October 1838 the Assam
history of the development of radical princely state was restored in Upper Assam
humanism. The final ideas are, of course, while the British ruled in Lower Assam.
contained in "New Humanism and • Purandar Singha was allowed to rule as king
Materialism". of Upper Assam in 1833, but after that brief
period, Assam was annexed to Bengal by the
Q72. In the context of international affairs,
British.
which one of the following is correct about Lord
• Purandar Singha who became the king of the
Curzon?
Ahom Kingdom twice (1818-19,1833-1838)
(a) He advocated a pacifist policy. was the last king of Ahom kingdom in Assam.
(b) He wanted to establish a British sphere of • Ahom kingdom was a late medieval kingdom
influence over the Persian Gulf and Seistan. in the Brahmaputra Valley in Assam. It is well
(c) He wanted England and Russia to become known for maintaining its sovereignty for
friends against China. nearly 600 years and successfully resisted the
(d) He wanted to lead a flag-waving mission to Mughal expansion in Northeast India.
the USA. • On 6 February 1874 Assam, including Sylhet,
Answer: B was severed from Bengal to form the Assam
Explanation: Chief-Commissionership, also known as the
• During his official tenure as a Viceroy in India, 'North-East Frontier. Hence Option C is the
Lord Curzon had special attention to the correct answer.
Persian Gulf.
17 | P a g e W W W . E D U T A P . C O . I N QUERY? HELLO@EDUTAP.CO.IN / 8146207241
• Shillong was chosen as the capital of the Non- (a) Because the Ethiopians declared their
Regulation Province of Assam in September support for the cause of Indian independence
1874. from British rule.
• From 16 October 1905 Assam became part of (b) Because Ethiopia became independent from
the Province of East Bengal and Assam. Britain on that day.
• The province was annulled in 1911 following (c) Because Ethiopia was attacked by Italy in
a sustained mass protest campaign and on 1 early 1936 and their resistance was
April 1912 the two parts of Bengal were considered freedom struggle.
reunited, and a new partition based on (d) Because Ethiopian leaders denounced
language followed, Oriya and Assamese Mussolini.
areas were separated to form new Answer: C
administrative units: Bihar and Orissa Explanation:
Province were created to the west and Assam • The Second Italo—Ethiopian War, also
Province to the east. referred to as the Second Italo—Abyssinian
War, was a colonial war that started in
Q74. What were the peasant associations set up October 1935 and ended in May 1936.
in Kerala in the 1930s called? • The war was fought between the armed
(a) Kisan Sabha forces of the Kingdom of Italy (Regno d'Italia)
(b) Kirti Kisan and the armed forces of the Ethiopian Empire
(c) Karshaka Sangam (also known as Abyssinia).
(d) Kisan Morcha • The war resulted in the military occupation of
Answer: C Ethiopia and its annexation into the newly
Explanation: created colony of Italian East Africa.
Key Points • When Ethiopia was attacked by fascist Italy in
early 1936, Congress declared the Ethiopian
• In the Malabar region of Kerala, a powerful people's struggle to be part of all exploited
peasant movement developed as the result of people's struggle for freedom.
the efforts mainly of Congress Socialist Party • The Congress declared 9 May to be Ethiopia
activists, who had been working among the Day on which demonstrations and meetings
peasants since 1934, touring villages and were held all over India expressing sympathy
setting up Karshaka Sanghams (peasant and solidarity with the Ethiopians. Hence
associations). Hence Option C is the correct option C is the correct answer.
answer.
• The main demands, around which the Q76. Who set up the Bharat Stree Mahamandal
movement cohered, were: which first met in Allahabad in 1910?
➢ the abolition of (a) Annie Besant
▪ feudal levies or akramapirivukal, (b) Meherbai Tata
▪ renewal fees or the practice of (c) Saraladevi Chaudhurani
policceluthu, (d) Tarabai Shinde
▪ advance rent. Answer: C
➢ stopping of eviction of tenants by Explanation:
landlords on the ground of personal • Bharat Stree Mahamandal (the Large Circle of
cultivation. Indian Women) was the first all India women's
➢ reduction in the tax, rent, and debt burden. association.
➢ use of proper measures by landlords when • It was set up in 1910 by Sarala Devi
measuring the grain rent. Chaudhurani. Hence option C is the correct
➢ end to the corrupt practices of the answer.
landlords' managers. • The centre was formed in Lahore in November
Q75. Why did the Congress declare 9th May to 1910 at a personal meeting of women.
be 'Ethiopia Day'? • It was decided to hold the opening meeting in
Allahabad in December 1910 when many from

18 | P a g e W W W . E D U T A P . C O . I N QUERY? HELLO@EDUTAP.CO.IN / 8146207241


different parts of India were expected to very limited. Hence option B is the correct
assemble there for the Congress session. answer.
• The most important feature of the inaugural • Peasants' participation was massive.
session of the Bharat Stree Mahamandal: Although Congress was against the class war,
➢ it was presided over by Her Highness the the masses broke this restraint.
Begum Sahiba of Janjira, a Muslim woman, ➢ In Bihar, the confrontation between the
and 'lower and upper castes' on the issue of
➢ was graced by the presence of Her the former taking the sacred thread got
Highness the Nawab Begum of Bhopal, the merged with the Non-Cooperation
only ruling prince of India and a Muslim Movement.
woman as well.
➢ In general, the peasants turned against
• The objective of Bharat Stree Mahamandal
the landlords and the traders.
was the creation of an organization by means
of which women of every race, creed, class, • The movement had shown wide geographical
and party in India may be brought together on spread with regional variations due to
the basis of their common interest in the differences in the cause of struggles.
moral and material progress of women in • This regional variation led to other social
India; and in and through which organization reform movements in different parts of India.
they may work in association and in a spirit of Some of the are as follows:
mutual helpfulness for the progress of ➢ A movement against Union board taxes
humanity that through her sex. has been launched in the Midnapore
Q77. Which one of the following is not a feature district of Bengal.
of the Non-Cooperation Movement? ➢ No-tax movements were also organized
(a) Economic boycott was intense and successful. in the Andhra district of Guntur in Chirala
- Perala and Pedanandipadu taluka.
(b) The middle class participated in very large
numbers in the movement. ➢ In U.P, where a powerful Kisan Sabha
movement was underway, Jawaharlal
(c) It was marked by uneven geographical spread
Nehru led the non - cooperation
and regional variations.
movement, among others.
(d) Along with Non-Cooperation, other Gandhian
➢ The Non - Cooperation and Khilafat
social reform movements like the anti-liquor
propaganda in the Malabar region of
campaign achieved some success.
Kerala helped to arouse Muslim tenants,
Answer: B called the Moplahs, against their renters,
Explanation: but the movement sometimes took on a
Key Points common colour.
➢ In Assam, tea plantation labourers went
• The Non - Cooperation Movement launched
on strike.
by Mahatma Gandhi on August 1, 1920, was
the first mass movement organized ➢ Andhra became popular with defiance of
nationwide during India's struggle for forest laws.
freedom. ➢ The Akali movement took place in Punjab
• The economic boycott received support from as part of the Non - Cooperation
the Indian business groups because they had Movement to wrest control of the
benefited from the nationalists' emphasis on gurudwaras from the corrupt mahants
the use of swadeshi. (priests).
• People from the middle classes led the Q78. Which one of the following publications
movement in the beginning but later they was started by Khan Abdul Ghaffar Khan in
showed a lot of reservations about Gandhi's 1928?
program. In places like Calcutta, Bombay, and (a) Pakhtun
Madras, which were centers of elite (b) Khudat Khidmatgar
politicians, the response to Gandhi's call was (c) Young India
19 | P a g e W W W . E D U T A P . C O . I N QUERY? HELLO@EDUTAP.CO.IN / 8146207241
(d) India Awakens marriage. Hence option C is the correct
Answer: A answer.
Explanation: • The Arya Mahila Samaj was founded on
• Khan Abdul Ghaffar Khan was a Pashtun November 30, 1882. It was founded by Pandita
independence activist who worked to end the Ramabai with the aim of ‘empowering and
rule of the British Raj in colonial India. educating each woman to lead a dignified
• He founded several reform movements such life’.
as the Anjuman-e Isiah ul-Afghan in 1921, the • She was the first woman to be accorded the
farmers' organisation Anjuman-e Zamidaran titles of Pandita as a Sanskrit scholar and
in 1927, and the youth movement Pashtun Sarasvati after being examined by the faculty
Jirga in 1927. of the University of Calcutta.
• Trying to further spread awareness of Pashtun
Q80. Who among the following is considered to
issues Abdul Ghaffar Khan founded the
be the first Indian to go to jail in performance of
magazine Pakhtun in May 1928. Hence option
his duty as a journalist?
A is the correct answer.
• Finally in March 1930, almost on the eve of the (a) Mahatma Gandhi
Qissa Khwani Bazaar Massacre the Khudai (b) Rabindranath Tagore
Khidmatgar were formed. (c) Lokmanya Tilak
• In 1929, the Khudai Khidmatgars ("Servants of (d) Surendranath Banerjee
God") movement, led by Khan Abdul Ghaffar Answer: D
Khan, non-violently mobilized to oppose the Explanation:
British in India's Northwest Frontier Province. • Surendranath Banerjee, one of the founding
It was also known as the "Red Shirts fathers of the Indian national movement, was
Movement". the first Indian to go to jail in the performance
• Young India was a weekly paper or journal in of his duty as a journalist. Hence option D is
English published by Mahatma Gandhi from the correct answer.
1919 to 1931. Gandhi wrote various • A dispute concerning a family idol, a saligram,
quotations in this journal that inspired many. had come up before Justice Norris of the
He used Young India to spread his unique Calcutta High Court.
ideology and thoughts regarding the use of • To decide the age of the idol, Norris ordered it
nonviolence in organising movements and to to be brought to the Court and pronounced
urge readers to consider, organize, and plan that it could not be a hundred years old.
for India's eventual independence from • This action deeply hurt the sentiments of the
Britain. Bengali Hindus.
• Banerjee wrote an angry editorial in the
Q79. Who among the following was the founder Bengalee of 2 April 1883.
of the Arya Mahila Samaj in the early 1880s? • Comparing Norris with the notorious Jeffreys
(a) Swami Dayananda Saraswati and Seroggs (British judges in the 17th century
(b) Swami Vivekananda notorious for infamous conduct as judges), he
(c) Pandita Ramabai said that Norris had done enough 'to show
(d) Ramabai Ranade how unworthy he is of his high office'.
Answer: C • Banerjee suggested that some public steps
Explanation: should be taken to put a quietus to the wild
• Pandita Ramabai Sarasvati was an Indian social eccentricities of this young and raw Dispenser
reformer. Ramabai was born in 1859 in of Justice.
Maharashtra to a Chitpavan Brahmin family. • Immediately, the High Court hauled him up for
Her parents, her brother. and eventually, her contempt of court before a bench of five
husband died within a four-year span (1876- judges, four of them Europeans.
1880). • With the Indian judge, Romesh Chandra Mitra,
• After which Ramabai became a lecturer and dissenting, the bench convicted and
founded the Arya Mahila Samaj to promote sentenced him to two months imprisonment.
women's education and prevent child
20 | P a g e W W W . E D U T A P . C O . I N QUERY? HELLO@EDUTAP.CO.IN / 8146207241
Q81. Which of the following was/were the metropolitan manufacturers and a field for
reason/reasons for the lack of economic the investment of British capital.
development in India in the 19th century?
82. Which social reformer's autobiography,
I. Officially the British Government was entitled Looking Back, describes his experiences
committed to a policy of laissez-faire, but in setting up schools for women in Poona in the
it was actually a policy of discriminatory 1890s?
intervention.
II. European entrepreneurs had connections (a) Dhondo Keshav Karve
to banks and agency houses, while Indians (b) K. T. Telang
had to rely on kin, family, and caste men. (c) Jyotirao Phule
III. When plantations were transferred to (d) Dr. B. R. Ambedkar
individual capitalist ownership, native
Answer: A
investors were deliberately ignored.
Explanation:
Select the correct answer using the code given • Dhondo Keshav Karve was popularly known
below. as Maharshi Karve, was a social reformer in
(a) I only India in the field of women's welfare.
(b) II and III only • He advocated widow remarriage and he
(c) I and III only himself married a widow.
(d) I, II, and III • Karve wrote two autobiographical works
Answer: D ➢ Atmawrutta (1928) in Marathi, and
Explanation: ➢ Looking Back (1936) in English
• The early intellectuals of the first half of the • In these books, he described his experiences
nineteenth century supported British rule in setting up schools for women in Poona in
under the impression that it would modernise the 1890s.
the country based on the latest technology and 83. Who was the first Director General of the
capitalist economic organisation. Archaeological Survey of India?
• After the 1860s, disillusionment started to set
(a) John Marshall
in among the politically conscious and they
(b) Alexander Cunningham
began to probe into the reality of British rule in
(c) Mortimer Wheeler
India.
(d) Francis Buchanan
• Following were the reasons for the lack of
Answer: B
economic development in India in the 19th
Explanation
century:
• Alexander Cunningham was the first
➢ Officially the British Government was
Director-General of the Archaeological Survey
committed to a policy of laissez-faire, but
of India. Hence option B is the correct
it was a policy of discriminatory
answer.
intervention. Hence statement 1 is
Important Points
correct.
• Alexander Cunningham was a British
➢ European entrepreneurs had connections
Engineer.
to banks and agency houses, while Indian
had to rely on kin, family, and caste men. • In 1861 he was appointed to the newly
Hence statement 2 is correct. created position of archeological surveyor of
➢ When plantations were transferred to the government of India.
individual capitalist ownership, native • He was the founder and organizer of the
investors were deliberately ignored. Archaeological Survey of India.
Hence statement 3 is correct. • Archaeological Survey of India is an Indian
➢ Therefore, option D is the correct answer. Government Agency attached to the Ministry
• Hence, the essence of nineteenth-century of Culture.
colonialism lay in the transformation of India
into a supplier of foodstuffs and raw Additional Information
materials to the metropolis, a market for
21 | P a g e W W W . E D U T A P . C O . I N QUERY? HELLO@EDUTAP.CO.IN / 8146207241
• John Marshall was the director-general of the d) Abarodhbasini (The Confined Women,
Archaeological Survey of India from 1902 to 1931), a spirited attack on the extreme
1928. forms of purdah that endangered
• Lord Curzon is the British governor-general women's lives and self-image.
known for the partition of Bengal. • Bangladesh observes Rokeya Day on 9
• Sir Robert Eric Mortimer Wheeler was a December every year to commemorate her
British archaeologist and officer in the British works and legacy. On that day, the
Army. Bangladesh government also confers Begum
Rokeya Padak on individual women for their
84. Which of the following works was not exceptional achievements.
contributed by the famous social reformer • Mother India (1927) is a polemical book by
Begum Rokeya? American historian Katherine Mayo that
(a) Ardhangi attacks Indian society, religion, and culture.
(b) Griha Hence option C is the correct answer.
(c) Mother India ➢ It was written in opposition to the Indian
(d) Sultana's Dream demands for self-rule and independence
Answer: C from British rule.
Explanation ➢ The book pointed to the treatment of
• Rokeya Sakhawat Hossain commonly known India's women, the untouchables,
as Begum Rokeya, was a Bengali feminist animals, dirt, and the character of its
thinker, writer, educator, and political activist nationalistic politicians.
from British India (present-day Bangladesh). ➢ A large part of the book dealt with the
• She is widely regarded as a pioneer of problems resulting from the marriage of
women's liberation in South Asia. young girls.
• She advocated for men and women to be ➢ This was considered to be one of the main
treated equally as rational beings, noting that causes that led to an uproar across India
the lack of education for women was after many newspapers declared it
responsible for their inferior economic scurrilous against Hinduism.
position.
Q85. Which one of the following is not a
• Rokeya writes about the degraded status of
characteristic of the Mughal chronicles?
women in Stree Jatir Abanati, about the
evolution of man into a being who (a) They present a continuous chronological
subordinates women through intellectual and record of events.
physical prowess. (b) They were a repository of factual information
• In a series of reports, Rokeya also depicted about the Mughals.
the degraded status of women in Ardhangi, (c) They allow us to understand how imperial
Muktipal, Bhrata bhagni and Nari Sristi. ideologies were created and disseminated.
• The homelessness of women in Indian society (d) They contain brief biographies of the authors
irrespective of religion is evident in one of of the chronicles
Rokeya's essays Griha (The Home). Answer: D
• Her major works include: Explanation
a) Matichur (A String of Sweet Pearls, 1904 • The Mughal chronicles were the mouthpiece
and 1922), a collection of essays in two of Mughal dynasty with distinctive
volumes expressing her feminist characteristics.
thoughts. • The characteristic features of Mughal
b) Sultana's Dream (1908), a feminist Chronicles are as follows:
science fiction novella set in Ladyland I. Chronicles commissioned by the Mughal
ruled by women. emperors are an important source for
c) Padmarag ("Essence of the Lotus", 1924) studying the empire and its court.
depicting the difficulties faced by Bengali II. They were written to project a vision of an
wives: enlightened kingdom to all those who
came under its umbrella.
22 | P a g e W W W . E D U T A P . C O . I N QUERY? HELLO@EDUTAP.CO.IN / 8146207241
III. At the same time, they were meant to prices and sacrifice a higher profit margin,
convey to those who resisted the rule of hoping for higher sales. Another may raise its
the Mughals that all resistance was price and use packaging or marketing that
destined to fail. Also, the rulers wanted to suggests better quality or sophistication.
ensure that there was an account of their Hence option (d) is a feature and thus the not
rule for posterity. correct answer.
IV. The authors of Mughal chronicles were • Limited barriers to entry and exit: Barriers to
invariably courtiers. The histories they entry, or the costs or other obstacles that
wrote focused on events centred on the prevent new competitors from entering an
ruler, his family, the court and nobles, industry, are low in monopolistic competition.
wars, and administrative arrangements.
V. Their titles, such as the Akbar Nama, Therefore, option (c) is the right answer.
Shahjahan Nama, Alamgir Nama, that is,
the story of Akbar, Shah Jahan and Alamgir Q87. Social cost is higher than economic cost
(a title of the Mughal ruler Aurangzeb), because
suggest that in the eyes of their authors the (a) Society is bigger than economy
history of the empire and the court was (b) Society includes polity, while economy does
synonymous with that of the emperor. not include it
VI. Hence, the Mughal chronicles do not (c) Cost borne by bystanders is positive
contain the biographies of the authors. (d) Society includes both consumers and
Hence option D is answer. producers
Answer: C
Explanation:
Q86. Which one of the following is not a feature
• Option (a) is not correct: Social cost can be
of monopolistic competition?
regarded as being higher than economic cost
(a) Large number of buyers and sellers in the not because society is bigger than economy. It
market may happen that economy is bigger in certain
(b) Differentiated products constitute the market cases. Also, the sizes of economy and society
(c) Product in the market is homogeneous may not be comparable as the parameters
(d) Selling costs are used for sale promotion determining their sizes are different.
Answer: A • Option (b) is not correct: Society as well as
Explanation: include polity.
Monopolistic competition exists when many • Option (c) is correct: Economic cost can be
companies offer competing products or services mostly determined in terms of quantitative
that are similar, but not perfect, substitutes. terms and the social cost of a wrong decision
Hence option (a) is a feature and thus not the (say) may be higher than the actual economic
correct answer. cost and also a bystander (a person who has
not taken the decision) can be paying due to
The following are the important features of repercussions of the wrong decision.
monopolistic competition: o For example, if the government of a
• Differentiated products constitute the country decides to go on a war, the
market: Firms in monopolistic competition economic cost of supplies and
differentiate their products through pricing losses is huge.
and marketing strategies. Hence option (b) is o Higher will be the social cost, cost
a feature and thus not the correct answer. to the society and a citizen and
Hence option (c) is not a feature and thus the even a resident who is a bystander
correct answer. and was not part of the decision
• Selling costs are used for sale promotion: made, directly bears the cost.
Demand is highly elastic for goods and • Option (d) is not correct: Society as well as
services of the competing companies and economy includes both consumers and
pricing is often a key strategy for these producers and thus it is not an appropriate
competitors. One company may opt to lower logic.
23 | P a g e W W W . E D U T A P . C O . I N QUERY? HELLO@EDUTAP.CO.IN / 8146207241
Therefore, option (c) is the right answer. • A commodity price index is not much related
to Core inflation. Hence option (c) is
Q88. Cess on coal at Rs. 100 per ton is a type of
incorrect.
(a) carbon tax Therefore, option (a) is the right answer.
(b) carbon subsidy
(c) carbon incentive for technology Q90. Who among the following was the Chairman
(d) carbon incentive for selling carbon permit of the Working Group for determining the
Answer: A methodology for construction of Producer Price
Explanation: Index (PPI) in India (2014)?
• Carbon tax - A carbon tax directly sets a price (a) Saumitra Chaudhury
on carbon by defining a tax rate on (b) D.V. Subbarao
greenhouse gas emissions or – more (c) Abhijit Sen
commonly – on the carbon content of fossil (d) B.N. Goldar
fuels. Hence cess on coal per ton is a carbon Answer: D
tax. Hence option (a) is correct. Explanation:
• Indian Economic Survey 2014-15 • Producer Price Index measures the average
acknowledged the green actions taken by change in the price a producer receives for his
India, including imposing significantly higher goods/services sold in the domestic
taxation of petroleum products and thereby market/exports.
reenergizing the renewable energy sector. • A Working Group on PPI was constituted on
o India increased the coal cess from 21st August 2014 under the chairmanship of
Rs. 50 per ton to Rs. 100 per ton in Dr. B N Goldar, Ex-member, National
2015, which is equivalent to a Statistical Commission along with the
carbon tax of about US$ 1 per ton. representatives from Ministry of Finance,
• Carbon subsidy - A carbon subsidy is given by CSO, Agriculture, Consumer Affairs,
the government to keep the prices of fossil Commerce, CBEC, RBI and other field experts.
fuels low. It acts as an incentive attracting • It was constituted to suggest the
more production and use of fossil fuels. Hence methodology for introducing Producer Price
option (b) is not the correct answer. Index (PPI) in India.
• Hence options (c) and (d) are incorrect as cess Therefore, option (d) is the right answer.
is not an incentive.
Therefore, option (a) is the right answer. Q91. Which one of the following countries is not
a member of WTO?
Q89. Core inflation is different from headline (a) Japan
inflation because the former (b) China
(a) Ignores articles of volatile nature in the price (c) Iran
index (d) Russia
(b) Considers articles of volatile nature in the price Answer: C
index Explanation:
(c) Is not based on commodity price index The World Trade Organisation (WTO) is the
(d) Considers only core items of consumption in world's largest international economic
the price index organization, with 164 member states
Answer: A representing over 98% of global trade and global
Explanation: GDP. The members of WTO are shown below:
Core inflation is the change in the costs of goods
and services, but it does not include those from
the food and energy sectors.
• This measure of inflation excludes these items
because their prices are much more volatile.
Hence option (a) is correct while options (b)
and (c) are incorrect.

24 | P a g e W W W . E D U T A P . C O . I N QUERY? HELLO@EDUTAP.CO.IN / 8146207241


Explanation:
International Labour Organisation (ILO)
Declaration of 1944 -
• The ILO Declaration of 1944 is also known as
the Philadelphia Declaration
• The Philadelphia Declaration is concerned with
the aims and purpose of the ILO.
❖ Labour is not a commodity,
❖ Freedom of expression and of the
association are essential to sustained
progress and
It can be seen that Japan, China and Russia are ❖ Poverty anywhere constitutes a danger to
members of WTO while Iran has an observer prosperity everywhere, are among three
statues. fundamental principles on which
organisation is based.
Therefore, option (c) is the right answer. • The fourth option is incorrect as it states a
continuous and concerted international effort
Q92. Works Committee, Safety Committee and in which the representative of workers and
Canteen Management Committee are the employees enjoy equal status with those of
examples of the government. it is important for the
(a) workers' participation in management promotion of common welfare of the workers
(b) workers' education schemes • Hence, D is the right answer.
(c) workers' cooperatives
(d) workers' suggestion schemes Q94. Which one of the following is an exception
from the five functional types of unionism
Answer: A identified by Robert Hoxie?
Explanation: (a) Business Unionism
• The committee under Factories Act - (b) Predatory Unionism
• There are three main types of committees i.e., (c) Revolutionary Unionism
standing, select or special, and joint. (d) Evolutionary Unionism
• The Works Committee, Safety Committee and
Canteen Management Committee are the Answer: D
examples of the joint committee where it Explanation:
provides a forum for employees and • Types of unionism identified by Robert Hoxie -
management to work together. • Robert F. Hoxie was born in 1868.
• Hence, A is the right answer. • He was the Prof. of Economics at the University
of Chicago.
Q93. Which one of the following is not part of • He was very much influenced by Veblen for
the aims and purposes of the ILO as per which he became a tedious investigator and
Philadelphia Declaration? tireless hard-work researcher on economics,
(a) Labour is not a commodity particularly on "The Theory of Trade
(b) Freedom of expression and of association are Unionism".
essential to sustained progress • Robert mainly describes the establishment of
(c) Poverty anywhere constitutes danger to the idea of functional types of trade unions
prosperity everywhere among the labour organization and to find out
(d) The war against want requires to be carried on the principal types existing in this country.
with unrelenting vigour within each nation and is • The formation of a union is based on a set of
solely the responsibility of the government agreed aims, policies and methods which
distinguish a union or a set of unions from
Answer: D others and diagnose the social situation

25 | P a g e W W W . E D U T A P . C O . I N QUERY? HELLO@EDUTAP.CO.IN / 8146207241


differently. Q96. Questions relating to the application or
• Hoxie mentions different categories of the interpretation of a standing order certified under
union on the basis of their structure and the Industrial Employment (Standing Orders) Act,
functional Operation i.e., Business unionism, 1946 may be referred to
Friendly, or uplift unionism, Revolutionary (a) Industrial Tribunal
unionism, Predatory Unionism and (b) Labour Commissioner
Dependent unionism. (c) Labour Court
• It does not include Evolutionary unionism. (d) Industrial Employment Court
• Hence, D is the right answer.
Answer: C
Q95. Which one of the following is the process in Explanation:
which representatives of workmen and employer Industrial Employment (Standing Orders) Act,
involved in an industrial dispute are brought 1946 -
together before a third person or group of • An Act to require employers in industrial
persons who facilitates/ facilitate through establishments formally to define conditions
mediation to reach a mutually satisfactory of employment under them.
agreement? • Whereas it is expedient to require
(a) Arbitration employers in industrial establishments to
(b) Adjudication define with sufficient precision the
(c) Conciliation
conditions of employment under them and
(d) Collective negotiation
to make the said conditions known to
Answer: C workmen.
Explanation: • Standing Orders' means rules of conduct for
• ADR is a dispute resolution mechanism that is workmen employed in industrial
non-adversarial i.e., working together co- establishments.
operatively to reach the best resolution for • The object of the Act is to require employers
everyone. in industrial establishments to formally
• ADR can be instrumental in reducing the define conditions of employment under
burden of litigation on courts while delivering a them.
well-rounded and satisfying experience for the • Any questions relating to the application or
parties involved. interpretation of a standing order certified
• It provides the opportunity to "expand the pie" under this Act, 1946 may be referred to Labour
through creative, collaborative bargaining, and Court constituted for this purpose, whose
fulfil the interests driving their demands. decision shall be final and binding on all
• ADR is generally classified into the following parties.
types: Arbitration, conciliation, Mediation and • Hence, C is the right answer.
negotiations.
• Conciliation: A non-binding procedure in which Q97. A union whose membership may cover
an impartial third party, the conciliator, assists workers employed in many industries,
the parties to a dispute in reaching a mutually employment and crafts is known as
satisfactory agreed settlement of the dispute. (a) industrial union
• Conciliation is a less formal form of arbitration. (b) general union
• The parties are free to accept or reject the (c) craft union
recommendations of the conciliator. (d) region-cum-industry level union
• However, if both parties accept the
settlement document drawn by the Answer: B
conciliator, it shall be final and binding on Explanation:
both. • A trade union is there to represent and
• Hence, C is the right answer. protect the interests of its members. It is
there to protect the economic, political and
26 | P a g e W W W . E D U T A P . C O . I N QUERY? HELLO@EDUTAP.CO.IN / 8146207241
social interests of its members. relationship between workers and business
• The principal aim is to negotiate with the leaders. Unions are often a central
employer over pay, job security, working component of the pluralistic approach that
hours, etc. putting the collective power of seeks a balance of power between leadership
its members to the best advantage. and employees
• Hence, A is the right answer.
• There are various types of a trade union that
is Craft Union, Industrial Union, General Q99. Which one of the following is not a trade
Union, Labour Unions, Blue-Collar Workers' union security measure?
Unions, White-Collar Workers' Unions and (a) Closed shop system
Reformist Unions etc. (b) Agency shop system
• General Union: This type is open to all (c) Open shop system
members irrespective of their craft and (d) Union shop system
industry within a particular city or region.
• For example, Jamshedpur Labour Union, Answer: C
whose membership includes workers engaged Explanation:
in different industries and crafts of Trade Union Security Measure -
Jamshedpur. In this case, there is no • The Closed Shops system, Agency shop
distinction between skilled and unskilled system and union shop system are the types
workers of the trade union security measure.
• Hence, B is the right answer. • The open shop system is not the trade union
security measure.
Q98. Which one of the following perspectives of • In an open shop system, a company that may
industrial relations is based on the assumption have a union but hires both union and non-
that both the parties strive (and have union employees for that union membership is
opportunity) to exercise economic (wages and not a requirement for continued employment.
benefits) as well as political (control) power? • Closed Shops system: Ones in which the
(a) Pluralistic perspective employer and the union agree that the
(b) Unitary perspective employer will only hire union members.
(c) Radical perspective • Union Shops system: Businesses in which
(d) Trusteeship perspective employers are free to hire non-union
members, but union membership is required
Answer: A within a specified period of type (often 30
Explanation: days) as a condition of continued employment.
• Industrial relations is a set of phenomena • Agency shop system: In terms of the
operating both within and outside the agreement between management and the
workplace, concerned with determining and union, a non-union member has to pay the
regulating the employment relationship. union a sum equivalent to a member's
• Scholars have described three major subscription in order to continue employment
theoretical perspectives or frameworks, that with the employer.
contrast in their understanding and analysis of
workplace relations. • Hence, C is the right answer.
• The three important perspectives on industrial
relations are generally known as Unitarian, Q100. Which one of the following is statutory
Pluralism and Marxism or radical. machinery functioning at the central level?
• Each offers a particular perception of (a) Central Implementation and
workplace relations and will, therefore, Evaluation Committee
interpret such events as workplace conflict, (b) Central Board for Workers' Education
the role of unions and job regulation (c) Standing Labour Committee
differently. (d) Employee's State Insurance Corporation
• The Pluralistic approach suggests that there is
more than one source of power in the Answer: D
Explanation:
27 | P a g e W W W . E D U T A P . C O . I N QUERY? HELLO@EDUTAP.CO.IN / 8146207241
Employee’s State Insurance Corporation - Advisory Board constituted by the Central
• Employee's State Insurance Corporation is a Government under the Minimum Wages Act,
statutory machinery functioning at the central 1948?
level. (a) One of the independent members of the Board
• The corporation was the result of the (b) One of the employer’s representatives of the
Employees' State Insurance Act 1948. Board
• Employee's State Insurance is a self-financing (c) One of the employee’s representatives of the
social security and health insurance scheme Board
for India workers. (d) A functionary of the Central Government
• It is contributory in nature. nominated by the Government
• It was the first major legislation on social
Security for workers in independent India. Answer: A
• The fund is managed by the Employees' State Explanation:
Insurance Corporation according to rules and Central Advisory Board -
regulation stipulated in ESI act 1948. • For the purpose of advising the Central and
• Hence, D is the right answer. state governments in the matters of the
fixation and revision of minimum rates of
Q101. Which one of the following explains the wages and other matters under section 8 of
'citizen concept' of labour? Minimum wage Act and for coordinating
(a) Labour is largely regarded by the employers as the work of the Advisory Board, the Central
operating organizations in industry Government shall appoint a Central
(b) Labour is affected by the law of demand and Advisory Board.
supply. • The Central Advisory Board shall consist of
(c) Labour has a right to be consulted in regard to persons to be nominated by the Central
the terms and conditions under which they are Government representing employers and
supposed to work employees in SCHEDULE employment, who
(d) Labour is a cog in the machine shall be equal in number.
• An independent person not exceeding one-
Answer: C
third of its total number of members; one
Explanation:
of such independent persons shall be
• As a citizen in a democracy has certain
appointed the Chairman of the Board by the
inalienable rights and voice in determining and
Central Government.
exercising these rights.
• Hence, A is the right answer.
• Industrial Labour has also the same right as
industrial citizens.
• Labour has a right to be consulted in regard to Q103. The assumption that ''man is selfish and
the terms and conditions under which they: self-centred, and always tries to achieve his own
are supposed to work. ends even at the cost of others" explains which
theory of labour welfare?
• Labour welfare is also mandatory in an
(a) Placating theory
Industry, which is an extension of the term
(b) Police theory
welfare and its application to labour.
(c) Religious theory
• The term labour, labourer, workers, workman
(d) Philanthropic theory
or employee are all used to refer to the wage-
earning human agents in the industry.
Answer: B
• The concept of labour welfare has received
Explanation:
inspiration from the concepts of democracy
Police Theory of Labour Welfare -
and welfare state.
• Labour welfare relates to taking care of the
• Hence, C is the right answer.
well-being of workers by employers, trade
unions, governmental and non-governmental
Q102. Who among the following can be
institutions and agencies.
appointed as the Chairman of the Central • Welfare includes anything that is done for the
28 | P a g e W W W . E D U T A P . C O . I N QUERY? HELLO@EDUTAP.CO.IN / 8146207241
comfort and improvement of employees and • Hence, A is the right answer.
is provided over and above the wages.
• The police theory is based on the assumption
that Human Beings are selfish and always try Q106. For the first time in India, medical benefit
to gain their own benefits often at the cost of as a non-cash benefit was provided under
others welfare. (a) the Employees’ State Insurance Act, 1948
• Hence, B is the right answer. (b) the Factories Act, 1948
(c) the Maternity Benefit Act, 1961
Q104. Dr. Aykroyd’s formula is associated with (d) the Mines Act, 1952
determination of
(a) fair wage Answer: A
(b) minimum wage Explanation:
(c) living wage
• Employees' State Insurance is a social security
(d) real wage
and health insurance scheme for Indian
Answer: B workers.
Explanation: • The rules and regulations regarding this
• Dr. Aykroyd’s formula is associated with the scheme are organised under the Employees'
determination of minimum wage.
• Hence, B is the right answer. State Insurance Act, 1948.
• The fund for this scheme is managed by the
Q105. ''Everyone as a member of the society has Employees' State Insurance Corporation
the right to social security, and is entitled to (ESIC).
realization through national efforts and ❖ It is a statutory body under the Ministry of
international cooperation and in accordance Labour and Employment.
with the organisation and resources of each ❖ It was founded in February 1952.
state of economic, social and cultural rights
• It is a scheme that protects workers in
indispensable for his dignity and free sickness, maternity, permanent or temporary
development of his personality." This statement disablement, injury or death.
which is emphasizing the importance of social
• It also provides medical care to workers and
security has been expressed in which of the
their families.
following?
(a) Universal Declaration of Human Rights • Hence, A is the right answer.
(b) Philadelphia Declaration of the ILO
(c) Report of the First National Q107. Which one of the following statements
Commission on labour regarding an Overseas Citizen of India (OCI) is not
(d) Directive Principles of State Policy of the Indian correct?
Constitution
(a) An OCI is a citizen of another country.
(b) An OCI possesses a multiple-entry long-term
Answer: A
visa for visiting India.
Explanation:
(c) An OCI is at par with NRIs in all matters.
• The above-given statement is a Universal
(d) An OCI is not entitled to the fundamental right
Declaration of Human Rights.
to equality of opportunity in public
• The Universal Declaration of Human Rights employment.
(UDHR) is a milestone achievement in the Answer: C
history of human rights. Explanation:
• The Declaration was proclaimed in the United
Nations General Assembly in Paris in The correct answer is An OCI is at par with NRIs in
December 1948 i.e., General Assembly all matters.
resolution 217 A.
• Overseas Citizens of India (OCI) refers to all
• There are a total of 30 Articles provided in the the Persons of Indian Origin (PIOs) who were
declaration. citizens of India on 26th January 1950 or
29 | P a g e W W W . E D U T A P . C O . I N QUERY? HELLO@EDUTAP.CO.IN / 8146207241
thereafter or were eligible to become citizens Explanation:
of India on 26th January 1950 except who is • The beggar was a system in which British
or had been a citizen of Pakistan, Bangladesh Government officers and Zamindars used to
or such other country as the Central compel the persons to carry their goods when
Government may, by notification in the they moved from one place to another place,
Official Gazette, specify. and this was forced labour in which no
• A registered Overseas Citizen of India enjoys remuneration was paid.
the following benefits: • Begar is prohibited under Article 23.
➢ he/she is granted multiple entries, multi- • According to Article 23, traffic in human
purpose, life-long visa for visiting India, beings and the beggar and other similar forms
➢ he/she is exempted from registration with of forced labour are prohibited and any
Foreign Regional Registration Officer or contravention of this provision shall be an
Foreign Registration Officer for any length offence punishable in accordance with the
of stay in India, and law. Hence option D is the correct answer.
➢ is entitled to general 'parity with Non-
Resident Indians in respect of all facilities The other important Articles of the Indian
available to them in economic, financial Constitution are as follows:
and educational fields except in matters 1. Article 16 - It talks about the right of equal
relating to the acquisition of agricultural opportunity in matters of public
or plantation properties'. employment. It states that there shall be
➢ However, OCI is not at par with NRIs in all equality of opportunity for all citizens in
the matters. Hence option C is the matters relating to employment or
answer. appointment to any office under the State.
2. Article 17 - It talks about the abolition of
Additional Information
untouchability and states that
• NRI means Non-Resident Indian. NRI is a "untouchability" is abolished and its
residential status given to a citizen of India practice in any form is forbidden. The
with an Indian Passport who resides in a enforcement of any disability arising out of
foreign country for the purpose of "untouchability' shall be an offence
work/business, or education. punishable in accordance with the law.
• The eligibility criteria for NRI status is that the 3. Article 19 - It talks about the Freedom of
person should be an Indian citizen with a valid speech and expression guaranteed to
Indian Passport and should reside outside of citizens of India. It states that all citizens
India for at least 183 days or more in a shall have the right
financial year (April 1st — March 31st). a. to freedom of speech and
• Until the year 2010, NRIs were not allowed to expression.
vote in the elections. b. to assemble peaceably and without
• However, post-2010 an amendment arms.
permitted NRIs to vote at their particular c. to form associations or unions.
constituency in India, but the voting required d. to move freely throughout the
the direct presence of the NRI in India at the territory of India.
particular polling booth where the name was e. to reside and settle in any part of
registered. the territory of India: and
f. to practise any profession, or to
Q108. Forced labour or beggar is a violation of
carry on any occupation, trade, or
which one of the following Articles of the
business.
Constitution of India?
(a) Article 16 Q109. Which one of the following statements
(b) Article 17 regarding the Constituent Assembly of India is
(c) Article 19 not correct?
(d) Article 23 (a) The Constituent Assembly was composed of
Answer: D members who had been elected indirectly by
30 | P a g e W W W . E D U T A P . C O . I N QUERY? HELLO@EDUTAP.CO.IN / 8146207241
the Members of the Provincial Legislative President of the Assembly, following the
Assembly. French practice.
(b) The Constituent Assembly held its first sitting
on 9th December 1946.
(c) The seats in each province were distributed Q110. Which one of the following is not a
among the three main communities—Muslim, constitutional body?
Sikh, and General, in proportion to their (a) The Election Commission of India
respective populations. (b) The Finance Commission
(d) The method of selection in the case of (c) The Official Languages Commission
representatives of Indian States was to be (d) The National Commission for Women
determined by the Governor-General of India. Answer: D
Answer: D Explanation:
Explanation: Key Points
• The Constituent Assembly was constituted in • Constitutional bodies derive their powers and
November 1946 under the scheme formulated authorities from the Constitution of India. They
by the Cabinet Mission Plan. are mentioned in the Constitution. Since they
• The features of the scheme were: get their power from the Indian Constitution,
1. The total strength of the Constituent Assembly any change in the mechanism of the
was to be 389. Of these, 296 seats were to be constitutional bodies would require a
allotted to British India and 93 seats to the constitutional amendment.
princely states. • Some of the examples of constitutional bodies
2. Each province and princely state (or group of are as follows:
states in the case of small states) was to be 1. The Election Commission of India was
allotted seats in proportion to their respective established in 1950. It is an autonomous
population. Roughly, one seat was to be constitutional body under Article 324,
allotted for every million population. which oversees the conduct of free and fair
3. Seats allocated to each British province were elections in the country.
to be divided among the three principal 2. The Finance Commission is a constitutional
communities—Muslims, Sikhs, and General body under Article 280, that determines
(all except Muslims and Sikhs), in proportion the method and formula for distributing
to their population. the tax proceeds between the Centre and
4. The representatives of each community were states, and among the states as per the
to be elected by members of that community constitutional arrangement and present
in the provincial legislative assembly and requirements.
voting was to be by the method of 3. Official Languages Commission of India is a
proportional representation by means of a commission which was constituted by the
single transferable vote. Hence option D is President of India in pursuance to the
incorrect and is the answer. provisions stated in Article 344 of the
5. The representatives of the princely states were Indian Constitution. This commission was
to be nominated by the heads of the princely constituted on June 7, 1955, vide
states. notification of the Ministry of Home Affairs,
• It is, thus, clear that the Constituent Assembly Government of India.
was to be a partly elected and partly • The National Commission for Women (NCW) is
nominated body. the statutory body of the Government of India
• The Constituent Assembly held its first generally concerned with advising the
meeting on December 9, 1946. government on all policy matters affecting
• The Muslim League boycotted the meeting women. It was established on 31 January 1992
and insisted on a separate state of Pakistan. under the provisions of the Indian Constitution,
The meeting was, thus, attended by only 211 as defined in the 1990 National Commission for
members. Women Act. Hence option D is the correct
• Dr. Sachchidananda Sinha, the oldest answer.
member, was elected as the temporary
31 | P a g e W W W . E D U T A P . C O . I N QUERY? HELLO@EDUTAP.CO.IN / 8146207241
Q111. Panchayats (Extension to the Scheduled • Article 43 A of the Indian constitution deals
Areas) Act, 1996 permits self-governance of with the workers’ participation in
natural resources by management. Hence option B is the correct
(a) Gram Sabha answer.
(b) Gram Panchayat • It states that "The state shall take steps, by
(c) the Chairman of Gram Panchayat suitable legislation or in any other way, to
(d) forest dwellers of a village situated in the areas secure the participation of workers in the
mentioned in the Fifth Schedule of the management of undertakings, establishments
Constitution of India or any other organisation engaged in an
Answer: A industry'.
Explanation: • Worker’s participation in management means
• The Provisions of the Panchayats (Extension to sharing the decision-making power with
Scheduled Areas) Act, 1996 or PESA is a law lower-level workers in the organization.
enacted by the Government of India for • The concept of Workers’ participation in
ensuring self-governance through traditional management originated from the word
Gram Sabhas for people living in the Scheduled "democracy".
Areas of India. Hence option A is the correct • Worker’s participation in management is the
answer. process of management of the people, for the
• Gram Sabha has the power to safeguard and people and by the people.
preserve the traditions and customs of people, • It gives a sense of belongingness among the
their cultural identity, community resources workers towards the organization.
and customary mode of dispute resolution.
• It also permits self-governance of natural Q113. Which one of the following comes under
resources by the Gram Sabha. the 'State List' under the Seventh Schedule of the
Constitution of India?
Powers of the Gram Sabha - (a) Relief of the disabled and unemployable
• It also has the power to approve plans, (b) Regulation of labour and safety in mines
programmes, and projects for social and (c) Regulation and control of manufacture,
economic development. supply, and distribution of salt
• To identify persons as beneficiaries under (d) Social security and social insurance
poverty alleviation and other programs. Answer: A
• To give the certificate of the utilisation of Explanation:
funds for various plans and programmes. State List –
• If there is an acquisition of land in these areas, • The seventh schedule under Article 246 of the
Gram Sabha must be consulted. constitution deals with the division of powers
• However, actual planning and implementation between the union and the states.
of the projects shall be coordinated at the state • It contains three lists: Union List, State List, and
level. Concurrent List.
• So, in land acquisition, the role of Panchayats • The union list details the subjects on which
in these areas is advisory only. Parliament may make laws while the state list
details those under the purview of state
Q112. The provision of workers' participation in legislatures.
the management of industries is provided under • The concurrent list, on the other hand, has
(a) Article 39A of the Constitution of India subjects in which both Parliament and state
(b) Article 13A of the Constitution of India legislatures have jurisdiction. However, the
(c) Article 42 of the Constitution of India Constitution provides federal supremacy to
(d) Article 43B of the Constitution of India Parliament on concurrent list items in case of a
Answer: B conflict.
Explanation: • The State List is a list of 61 (Originally 66) items
Article 43A of the Constitution of India - in Schedule Seven to the Constitution of India.

32 | P a g e W W W . E D U T A P . C O . I N QUERY? HELLO@EDUTAP.CO.IN / 8146207241


• The respective state governments have • Enterprise may be in manufacturing,
exclusive power to legislate on matters services or the trading sector.
relating to these items. • In case of non-individual enterprises at
• 'Relief of the disabled and unemployable* least 51% of the shareholding and
comes under the state list of the seventh controlling stake should be held by either
schedule. Hence option A is the correct an SC/ST or woman entrepreneur.
answer.

Q114. Which one of the following schemes is Q116. StandUp India Programme envisages each
aimed at all-round development of adolescent bank branch to give loan between Rs.10 lakh to
girls in the age group of 11-18 years and making Rs. 100 lakh
them self-reliant?
1. To at least one SC/ST borrower
(a) RGSEAG
(b) IGMSY 2. To at least one woman borrower
(c) NMEW
(d) RMK 3. To at least one rural unemployed youth
borrower
Answer: A
Select the correct answer using the code given
Explanation below.
RGSEAG refers to Rajiv Gandhi Scheme for
Empowerment of Adolescent Girls. (a) 1 only
• It was launched in 2010.
• This scheme is aimed at all-round (b) 2 and 3
development of adolescent girls between
the age group of 11-18 years to make (c) 1 and 3
them self-reliant.
• The scheme has been implemented in 200 (d) 1 and 2
districts across the country.
Answer: D
Q115. A Stand up enterprise can be established in Explanation
• The objective of the Stand-Up India
1. Farming Sector scheme is to facilitate bank loans between
2. Manufacturing Sector 10 lakh and 1 Crore to at least one
3. Service Sector Scheduled Caste (SC) or Scheduled Tribe
4. Trading Sector (ST) borrower and at least one woman
borrower per bank branch for setting up a
(a) 1, 2 and 4 greenfield enterprise. This enterprise may
(b) 1, 3 and 4 be in manufacturing, services, agri-allied
(c) 1, 2 and 3 activities or the trading sector In case of
(d) 2, 3 and 4 non-individual enterprises at least 51% of
the shareholding and controlling stake
Answer: D should be held by either an SC/ST or
Explanation Woman entrepreneur.
Stand-Up India Scheme
• It facilitates bank loans between Rs 10 lakh Q117. Which one of the following statements is
and Rs 1 Crore to at least one Scheduled not correct for Atal Pension Yojana?
Caste (SC) or Scheduled Tribe (ST)
borrower and at least one woman (a) There is guaranteed minimum monthly
borrower per bank branch for setting up a pension for the subscribers ranging between
greenfield enterprise. Rs 1,000 and Rs 5,000 per month.

33 | P a g e W W W . E D U T A P . C O . I N QUERY? HELLO@EDUTAP.CO.IN / 8146207241


(b) The benefit of minimum pension would be Answer: D
guaranteed by the Government of India.
(c) Government of India co-contributes 50% of Explanation
the subscriber's contribution or Rs 1,000 per • Under Pradhan Mantri Suraksha Bima
annum, whichever is lower. Yojana a beneficiary has to pay an annual
(d) It is applicable to all citizens of India aged premium of RS 12 for availing accident
above 40 years. and disability cover up to Rs 2,00,000.
Note: Annual Premium under Pradhan Mantri
Answer: D Suraksha Bima Yojana has been enhanced from
Explanation Rs 12 to Rs 20 in 2022.
• Atal Pension Yojana
• Administered By: Pension Fund Regulatory Q119. Who among the following is the author of
and Development Authority through the book, Choices of Techniques?
National Pension System (NPS). (a) A.K. Sen
• Eligibility: (b) K.N. Raj
o The age of the subscriber should be (c) V.K.R.V. Rao
between 18 and 40 years. (d) Sukhamoy Chakravarty
o He / She should have a savings Answer: A
bank account/ post office savings Explanation:
• The author of the book ‘Choices of
bank account.
Techniques’ is Amartya Kumar Sen or A.K. Sen.
o The scheme is open to all bank
Hence option A is the correct answer.
account holders who are not
• Choice of Techniques is Sen's first publication
members of any statutory social and is, thus, representative of the beginning of
security scheme. an illustrious career.
o The scheme is open for Indian • It talks about the correct choices of techniques
citizen only. associated with the maximisation of social
• Key Features welfare.
o The Scheme guarantees minimum
monthly pension for the Q120. Which one of the following is the correct
subscribers ranging between Rs set of contingencies identified by William
1,000 and Rs 5,000 per month. Beveridge in his comprehensive social security
o Government of India co- schemes?
contributes 50% of the subscriber's (a) Want, disease, ignorance, squalor and idleness
contribution or Rs 1,000 per (b) Want, sickness, disability, squalor and idleness
annum, whichever is lower. (c) Want, disease, old age, squalor and
o Contributions to the Atal Pension unemployment
Yojana (APY) are eligible for tax (d) Disease, invalidity, old age, unemployment
benefits similar to the National and ignorance
Pension System (NPS). Answer: A
Explanation:
• William Beveridge (1879-1963) was a social
Q118. Which one of the following is the amount economist who in November 1942 published a
of annual premium of the Pradhan Mantri report titled, 'Social Insurance and Allied
Suraksha Bima Yojana (PMSBY) for accident and Services' that would provide the blueprint for
disability cover up to Rs 2,00,000? social policy in post-war Britain.
• Beveridge had been drawn to the idea of
(a) Rs 100 remedying social inequality while working for
(b) Rs 50 the Toynbee Hall charitable organisation in
(c) Rs 20 East London.
(d) Rs 12 • He saw that philanthropy was simply not
sufficient in such circumstances and a
34 | P a g e W W W . E D U T A P . C O . I N QUERY? HELLO@EDUTAP.CO.IN / 8146207241
coherent government plan would be the only
sufficient action.
• By the outbreak of World war II, Beveridge
found himself working in Whitehall where he
was commissioned to lead an inquiry into
social services.
• His vision was to battle against what he called
the five giants; idleness, ignorance, disease,
squalor and want.

35 | P a g e W W W . E D U T A P . C O . I N QUERY? HELLO@EDUTAP.CO.IN / 8146207241

You might also like